You are on page 1of 681

Set 01: Seating Arrangement

Directions (1-4): Study the following information and answer the questions given below.
There are eight people viz. P, O, I, U, Y, T, R and E are sitting in two circles in equal numbers facing each other
as shown in the fig. below. The people sitting in the inner circle faces outside the centre of the circle and the
people sitting in the outer circle faces towards the centre of the circle. Each of them likes different colours viz.
Red, Yellow, Brown, Black, Purple, Green, Pink and Blue. All the information is not necessary to be in the same
order. Also, they were made to sit in a straight line, the people sitting in the inner circle faces south and likes
same colour. The persons sitting in the outer circle faces north and likes different colours.

E sits third to left of both I and the one who likes Blue colour in the straight line. I is an immediate neighbour of
R and sits in the extreme end of the line. The person, who faces T in the circle, likes Green colour in the straight
line. U sits immediate right of T in the circle. R sits second from the extreme end of the line and likes the colour
liked by E in the circle. The person who likes Yellow faces the person who sits immediate left of P in the circle. E
faces the person who sits to the immediate left in the straight line. The person who likes Blue colour faces the
person who likes Black colour. R is not an immediate neighbour of E, who likes Brown colour in the circle. R and
P doesn‟t face each other or sits in the same circle. I faces the person who sits immediate left of Y, who likes
Purple colour. T and O face each other; neither of them likes Yellow colour. Y sits second to the left of the one
who likes Blue colour in the straight line. P neither likes Blue or Pink colour while sitting in the circle. The
person, who likes Red colour, sits second to the left of the one who likes Green colour in the straight line. E
doesn‟t like Yellow colour in the straight line.

1) What is the position of T with respect to U in the a) Second to the right


straight line? b) Fourth to the right
c) Third to the left a) P
d) Fourth to the left b) The one who sits third to the right of E
e) None of these c) R
d) E
2) Which of the following statement is true e) Cannot be determined
regarding the circular arrangement?
a) P and E face each other 4) Four of the following five are alike in a certain
b) The one who likes Brown colour is an immediate way thus form a group with respect to circular
neighbour of one who likes Yellow colour arrangement. Find the one which doesn‟t belong
c) U likes Pink colour and sits to the immediate left of to the group?
P a) P – Green
d) O likes Green colour and sits to the immediate left b) E – Purple
of R c) U – Yellow
e) All are true d) O – Black
e) I – Red
3) Who among the following likes Black colour in
the straight line?

Set 02: Seating Arrangement


Direction (5-8): Study the following information and answer the questions given below.
Eight friends – P, Q, R, S, T, U, V and W are seated in anticlockwise direction in the same order around a circular
table. All of them were facing towards the centre of the table. They all played a game called “Passing the Ball”
where in every round the ball moves 15 positions in clockwise direction. At the end of each round, the person
who started the round and the person who receives the ball last interchange their positions and the next round
starts from the final position of the ball in the previous round. It is known that W starts the game.
5) Who among the following sits second to the left a) Q
of P in 33rd Round? b) R
a) Q b) R c) U d) V e) Cannot be determined c) U
d) P
6) What is the position of R with respect to W in e) Cannot be determined
123rd round?
a) Immediate left 8) Four of the following five are alike in a certain
b) Second to the left way based on the group. Find the one which does
c) Immediate right not belongs to the group?
d) Third to the right a) Round 112
e) None of these b) Round 147
c) Round 84
7) Who among the following faces S in 99th d) Round 98
round? e) Round 179

Set 03 : Puzzle
Directions (9-12): Study the following information and answer the questions given below.
Virgo Corporation conducts Food Tour at four different months at various countries. It allows their customers to
customize their own tour package by choosing a country in each month with certain conditions. The tour package
consists of 4 countries. The customer can‟t choose more than one country in the same month.
The Food Tour commences in the following four months- January, March, May and July. In January it has 4
countries- India, china, Germany and Mexico. In March it has 4 countries- Japan, Nigeria, Brazil and Peru. In
May, it has 4 countries-Turkey, Ghana, Poland and Spain. In July, it has 4 countries- Thailand, Ethiopia, Italy and
Cuba. The countries also can be grouped into 4 based on the continents as Asia, Africa, Europe and America. In
Asia it includes- India, China, Japan, Turkey and Thailand. In Africa it includes- Nigeria, Ghana and Ethiopia. In
Europe it includes- Germany, Poland, Spain and Italy. In America it includes- Mexico, Brazil, Peru and Cuba.

The conditions for choosing the countries in the package were,


1) The African countries and European countries can‟t be opted in the same package.
2) The Asian countries and American countries can‟t be opted in the same package.
3) Germany and Thailand can‟t be opted in the same package.
4) Japan and Cuba can‟t be opted in the same package.
5) If Ghana is opted then Brazil is automatically included in the package.
6) If Cuba is opted then Germany is automatically included in the package.

9) Robert plans to customize the package within a


single continent; then which of the following might 11) Patrick has already opted to travel Germany
be the suitable package for him? in January and Japan in March; which of the
a) China, Nigeria, Turkey, Thailand following might be the valid countries for the next
b) India, China, turkey, Thailand months based on the above conditions of Virgo
c) Germany, Turkey, China, Cuba Corporation?
d) India, Japan, Turkey, Thailand a) Turkey, Thailand b) Japan, Cuba c) Poland, Cuba
e) Mexico, Brazil, Peru, Cuba d) Turkey, Italy e) Ghana, Italy

10) Anderson plans to customize his tour package 12) If the Virgo Corporation relaxes the
with maximum American countries. Which of the Condition (4) and (2), then which of the following
following might be the valid package? package is still not valid?
a) Mexico, Brazil, Turkey, Cuba a) India, Japan, Poland, Cuba
b) Germany, Peru, Ghana, Cuba b) China, Nigeria, Ghana, Cuba
c) Germany, Brazil, Ghana, Cuba c) Germany, Peru, Poland, Italy
d) Mexico, Brazil, Peru, Cuba d) Mexico, Nigeria, Turkey, Thailand
e) Germany, Brazil, Spain, Cuba e) Both (b) and (d)
Set 04 : Blood Relation
Directions (13-14): Study the following information and answer the questions given below.
There are nine members in a family. Each has different amount of chocolates. A has 250 chocolates, he takes 12
chocolates and gives the remaining to his only son L. L takes 7 chocolates and gives the rest of the chocolates to
his mother F. F takes 9 chocolates and gives the remaining chocolates to his only Nephew S. S takes 5 chocolates
and gives the remaining chocolates to his father D. D takes 8 chocolates and gives the remaining chocolates to G,
who is his only niece. G takes 14 chocolates and gives the rest of the chocolates to her only sister-in-law H. J‟s
mother H, takes 7 chocolates and gives the remaining chocolates to her only son K. K takes 19 chocolates and
gives the rest to his only sister J. G is sister of L.

13) What is the square root of the number of (i) H x L + A x S; H gets 190 Chocolates from G
Chocolates J is having and how is J related to F? (ii) L + F x D % S; S gets 222 Chocolates from F
a) 13 – Grandson (iii) G – A % L x H; G gets 217 Chocolates from
b) 13 – Granddaughter D
c) √173 – Daughter (iv) J x K + L + A; K gets 188 Chocolates from H
d) √184 – Grandfather a) Only (ii) is true
e) Cannot be determined b) Both (i) and (iii) are true
c) Both (ii) and (iv) are true
14) If A x B = A is sister of B, A + B = A is the son d) Only (iv) is true
of B, A – B = A is the daughter of B, A % B = A is e) None of these
the father of B. Then which of the following
statement is true regarding the given relation?

Circular Arrangement:
Directions (15-18): Study the following information and answer the questions given below.
Eight friends – P, Q, R, S, T, U, V and W are seated around a circular table such that they are at equal distances
from each other and facing inwards. Four of them are true persons (always speaking the truth) and the remaining
four are false persons (always speaking a lie). The true and false persons are seated alternatively. They record
their statements as follows:
S: U is sitting second to my left.
U: P is sitting opposite to me.
R: T is sitting second to my left.
V: Q is sitting opposite to me.
W: V is a true person and he faces U.
Q: W is sitting third to my left.
T: S is a false person.
P: R, who is an immediate neighbor of V, is sitting second to my left.

15) Who among the following is sitting fourth to 17) Which of the following statement is true?
the right of W? a) V is a true person
a) R b) R sits third to the right of S
b) V c) V faces the immediate neighbour of U
c) U d) W faces U
d) Q e) None is true
e) P
18) If all the persons are made to sit in
16) Which among the following pair of people is alphabetical order from the right of P, then the
an immediate neighbor of T? position of how many persons remains unchanged
a) Q – R excluding P?
b) U – V a) One
c) S – W b) Two
d) Q – P c) Three
e) None of these d) More than Three
e) None
Directions (19-20): Study the following information and answer the questions given below.
There is a network of ten cities (P – Y) connected by roads (represented by straight lines) as shown. Joseph plans
to tour all cities in such a manner that he drives along all of the roads exactly once, neither repeating nor skipping
any stretch of road.
Note: Joseph never minds visiting a city more than once in the process. Also, his ending point need not be the
same as his starting point.

19) From which of the following city should Joseph 20) Four of the following five are alike in a
starts his journey? similar way thus form a group. Find the one
a) U which does not belongs to the group?
b) Q a) Q
c) P b) U
d) T c) X
e)X d) S
e) V

Circular Arrangement
Directions (21-24): Study the following information and answer the questions given below.
“Wheel of Fortune” is a game that involves turning three concentric wheels simultaneously and it is in the form as
shown in the figure below. The eight colours are Red (R), White (W), Orange (O), Yellow (Y), Pink (P), Green
(G), Blue (B) and Violet (V) but not in the same order.
Note: Each wheel has 8 equal parts with eight different colours in the same circular order. For example, In
outermost wheel if white part is to the immediate right of Pink part, Similarly the position of white part is to the
immediate right of pink part in the innermost and middle wheel.
Rule of the Game:
All the three wheels are rotated simultaneously. After the wheelshave stopped rotating, if all the three wheels
have each colour at the same position; then the player is said to be „won the fortune‟ or else he lost the game.
Robert took his turn and he lost the game. The following things were observed in his turn. All the directions
mentioned are with respect to the view of opposite to the centre.
(i) In the outermost wheel, the blue part is at Position 2.
(ii) In the innermost wheel, the orange part is at Position 4.
(iii) The Pink part in the middle wheel is to the immediate Right of Green part in the outer mostwheel.
(iv) In the middle wheel, the Red part is at Position 6.
(v) The white part in the innermost wheel is to the immediate right of the Blue part in the outermost wheel.
(vi) The Violet part in the innermost wheel is to the immediate left of Orange part in the middle wheel.
(vii) The Red part in the innermost wheel, the pink part in the middle wheel and the yellow part in the outermost
wheel were at the same position.
(viii) Any coloured part in one wheel is neither to the immediate right or left of the same coloured part in another
wheel.
(ix) The Green part in one of the wheels is at Position 1.
(x) The white part in the middle wheel is to the immediate right of Yellow part in the outermost wheel.

21) What is the position of Pink part in the middle c) Third to the right
wheel with respect to Yellow part in the same d) Immediate right
wheel? e) None of these
a) Immediate left
b) Second to the left
22) Which of the following is in straight line (inner e) None of these
wheel – middle wheel – outer wheel) with respect to
the arrangement? 24) If the Position 1 is interchanged with
a) Pink – Yellow – Red Position 8; Position 2 is interchanged with
b) Yellow – Pink – Blue Position 7; Position 3 is interchanged with
c) White – Orange – Green Position 6;Position 4 is interchanged with
d) Pink – Blue – Yellow Position 5. Then what is the position of
e) Green – Red – Violet Orange part in the middle wheel with
respect to Green part in the inner wheel?
23) What is the position of Blue part in a) Second to the right
the middle wheel? b) Third to the right
a) Position 1 c) Fourth to the left
b) Position 3 d) Immediate right
c) Position 5 e) Second to the left
d) Position 7

Set 06 : Seating Arrangement with Blood Relation


Directions (25-28): Study the following information and answer the questions given below.
Robert‟s family plans for a trip. He takes his family in a van having 3 rows of 3 seats each, the seats in the first
row are numbered 1, 2 and 3. The seats in the 2nd row are numbered 4, 5 and 6 and those in the last row are
numbered 7, 8 and 9. The seats are arranged in a grid form with 3 rows and 3 columns. The seats were occupied
by Q, W, E, R, T, Y,A and S but not necessarily in the same order. Also it is known that,
(i) Since, Robert drives the van, he occupies the seat numbered 3 having R and Y in his column.
(ii) Robert and his father-in-law are not seated in the same row.
(iii) E, the son of S occupies the seats in the same row as T and Y, who is his only sister.
(iv) Q occupies seat numbered 5 and is in same row as his husband A and is not seated immediately in front of
Robert‟s son E.
(v) Q, the mother of Robert and his daughter are not seated in the same column.
(vi) E and his sister areseated with his maternal grandmother.

25) If the persons sitting in the seats swap their


positionsin the following order (1,4) (2,6) (4,6) (5, 27) If the persons sitting in the seats swap their
7), (4, 5),(7,2), then which of the following persons positionsin the following order (1,5) (2,7) (4,7) (5,
will beseated beforeT? 8), (4, 6),(7,1), then how is Robert related to the
a) A person sitting in the 7th seat?
b) Q a) Wife
c) W b) Sister
d) S c) Mother
e) R d) Son
e) Father-in-Law
26) If swapping of seats is done in the order
(1,2)(2,8) (9,2) (4,9), (7,5) (5,8) (8,4), then which 28) Which of the following statement is true?
ofthe following statement is true? a) A is the uncle of Robert
a) S is in 9th seat b) W sits in 7thseat
b) Q is in 4th seat c) R is the father of Y
c) R is in 7th seat d) All are true
d) E is in 4th seat e) None is true
e) None is true
Set 7 : Puzzle
Directions (29-32): Study the following information and answer the questions given below.
Mr. Dan Brown has applied for an entrance exam and the following points are known about his roll number
which is an eight digit number.
Note: The roll number doesn‟t starts with zero. Zero is not considered as an even digit at any instance. Also, zero
doesn‟t come more than once in the roll number.
(i) The Two digit number formed by 3rd and 4th digits respectively is square of the number formed by 5th and 6th
digits.
(ii) 8th and 1st digits are an even numbers.
(iii)The Number formed by 7rd and 8th digits is a product of two consecutive natural numbers.
(iv) The Two digit number formed by 4th and 7th digits respectively is not a perfect square.
(v) Sum of the two-digit numbers formed by 7rd and 8th digits, and 1st and 2nd digits is a product of two
consecutive natural numbers.
(vi)TheNumbers formed by 7th and 8th digits is six less than the number formed by 1st and 2nd digit.
(vii) In the number, three digits have been used two times while other two digits have been used only once.

29) What is the sum of the digits of Mr. Dan Brown‟s roll are there in the odd numbered positions
number? respectively?
a) 28 b) 32 c) 30 d) 34 e) None of these a) Three – Two b) Two – Three c) Four – Two
d) Four – Four e) None of these
30) Which of the following is the sixth digit in
the roll number? 32) What is the product of the digits which are at
a)8 b) 3 c) 4 d) 5 e) 2 the prime numbered positions?
a) 249 b) 160 c) 0 d) 320 e) None of these
31) How many odd digits are there in the even
numbered positions and how many even digits
Set 08 : Puzzle
Directions (33-36): Study the following information and answer the questions given below.
There are nine countries – Africa (A), Bangladesh (B), China (C), Denmark (D), Egypt (E), Fiji (F), Greece (G),
Hungary (H) and Italy (I) which are neighbour to each other as shown in the fig. below.

Note: H has three neighbouring countries (D, I and F); C has 5 neighbouring countries (D, I, B, E and A) and I
has eight neighbouring countries (H, F, G, B, D, E, C and A) and so on.
A, B, C, D, E, F, G, H and I are nine airlines belong to nine different countries which are mentioned above. Each
airlines belonging to a country goes to one of its neighbouring country. Not more than one airline goes to the
same country. All the information is not necessary to be in the same order.
Also it is known that,
(i) Airline-B goes to Country E
(ii) Country H receives airline from Country F
(iii) Airline-H belongs to Country A
(iv) Only one Airline whose name and the country it belongs to are same.
(v)Airline-D belongs to Country H
(vi)Airline-I belongs to Country C
(vii) Airline from Country G goes to Country F
(viii) Airline-F goes to Country I
(ix) Airline from Country B goes to Country G
(x) Only one Airline whose name and the country it goes to are same.
(xi) Airline-E goes to Country A

33) Which of the following Country does Airline-D 34) Which of the following Airline goes to
goes to? Country B?
a) Country H a) Airline-A
b) Country F b) Airline-I
c) Country D c) Airline-C
d) Country C d) Airline-D
e) Country E e) None of these
35) Which of the following statement is d) Both II and III
true? e) Both I and III
I) Airline-C belongs to Country B and goes
to Country G 36) Four of the following five is related in a
II) The Country to which Airline-G goes has certain way thus form a group. Find the one
5 neighbouring countries which does not belongs to the group?
III) The Country to which Airline-G belongs a) Airline-D
has 3 neighbouring countries b) Airline-G
a) Only I c) Airline-C
b) Only II d) Airline-F
c) Only III e) Airline-H

Direction Based Puzzle:


Directions (37-38): Study the following information and answer the questions given below.
The following network gives the bus routes of new Public transport in Delhi between various cities. Any
passenger boarding a bus is charged Rs.3 as a service charge and Rs.9 as a fixed charge in addition to a charge of
Rs.5 per km.
Further it is also known that

Note: In the route shown, the values in bracketsdenote the distance in kilometres.
(1) A bus does not visit the same city more thanonce.
(2) Between any two cities only one mode oftransport is available, i.e. the new bus.

c) Rs.92
37) Which of the following might be the minimum bus
charge to travel from P to R? d) Rs.78
a)Rs. 58 e) None of these
b)Rs. 102
38) If all the routes connected to City U are b) Rs. 177
blocked, then find the second maximum charge to c) Rs.168
travel from S to P? d) Rs.198
a)Rs. 157 e) None of these

Set 09 : Critical Reasoning


Directions (39-40): Study the following passage and answer the questions based on the given statements:
Infosys Ltd has decided to appoint BhartiAirtelLtd‟s global CFO Nilanjan Roy as chief financial officer, effective
1 March 2019. He replaces interim CFO JayeshSanghrajka, who will resume his responsibilities as deputy CFO.
Roy spent 13 years with Airtel, before which he worked for 15 years with Unilever across their global operations.
JayeshSanghrajka has been feeling sick at work and won‟t be able to get back to work.

39) Which of the following is a conclusion, (IV) Person should be well experienced to take on
Assumption and Inference? the job of a CFO
(I) Nilanjan Roy is not working anywhere currently (V) CFO is required by Infosys Ltd
(II) Nilanjan Roy is a proper replacement for a) Statement I is inference, Statement III is
JayeshSanghrajka assumption and Statement II is conclusion.
(III) Nilanjan Roy is working as a CFO at competitor b) Statement II is inference, Statement III is
firm assumption and Statement V is conclusion.
c) Statement III is inference, Statement V is 40)Which of the following is a conclusion,
assumption and Statement II is conclusion. Assumption and Inference?
d) Statement IV is inference, Statement II is (I) The fuel prices are increasing in the alarming rate
assumption and Statement V is conclusion. (II) Reduction in the price of the petrol might reduce
e) Statement V is inference, Statement IV is the quality of Petrol and oils in large scale
assumption and Statement II is conclusion. (III) The price of certain essential commodities might
It is good news for Delhi residents heading to work get reduced with respect to the slash in the oil prices
on Friday morning after Christmas Eve, the fuel (IV) As it was the Christmas Eve, it is necessary for
prices in the national capital have been decreased the government to reduce petrol prices so as the
further. While petrol price has been decreased by Rs common man can enjoy their vacation
0.19, diesel rates have been slashed by Rs 0.14. After (V) The cost of living in Mumbai is high when
the cut, petrol will now be available at 79.18 per litre compared with National Capital
and diesel at Rs 73.64 per litre. a) Statement I is inference and Statement V is
The fuel prices have also been reduced in Mumbai. assumption.
Commuters can now get a litre of petrol for b) Statement IV is assumption, Statement III is
Rs 88.68 in the financial capital, registering a inference and Statement V is conclusion.
decrease of Rs 0.18. Diesel will cost c) Statement III is inference and Statement V is
Mumbaikars 79.18 per litre, a reduction of Rs assumption.
0.14. The fuel prices had earlier witnessed d) Statement IV is conclusion, Statement III is
a relentless hike in the country, burning a hole assumption and Statement V is inference.
in the common commuter's pocket. e) Statement V is inference and Statement IV is
assumption.

Directions (41-44): Study the following passage and answer the questions based on the given statements:
Bumblebees exposed to the insecticide imidacloprid tend to contribute less to the welfare of their colony than
untreated bees, according to a report in Science today. Inside their nests, bees chronically eating this common
pesticide were less active, didn‟t care for larvae as much or pitch in on maintaining the nest, and had fewer social
interactions, the researchers found.
“It‟s very fascinating and gives us a much greater understanding of the mechanism behind the patterns that we‟ve
seen of reduced [bee] colony growth and reproduction,” says biologist MajRundlof of Lund University in Sweden
who was not involved in the research.
Imidacloprid is one of a class of neonicotinoid neurotoxic insecticides that were developed during the 1980s and
1990s and are now “absolutely pervasive globally,” says biologist James Crall, a postdoc in the lab of Benjamin
de Bivort at Harvard University, who led the research. In an agricultural context, neonicotinoids are most
commonly applied to seeds in high concentrations, so that they pervade and protect the whole plant, explains
Crall. “The problem with that is that it also gets into nectar and pollen on which bees and other beneficial insects
feed,” he says.
41) Which of the following is an inference which 43) What we can say about the following
can be made from the facts stated in the above statement?
paragraph? “It‟s very fascinating and gives us a much greater
a) Neonicotinoid is a kind of neurotoxic that triggers understanding of the mechanism behind the
the crop production and reproduction of Bumblebees patterns that we‟ve seen of reduced (bee) colony
in a positive scale. growth and reproduction”
b) Imidacloprid alters the pattern of Bumblebees and a) Really impressive and the technology has proven
it leads to less colony growth. ahead of lives
c) Bumblebees exposed to Imidacloprid are more b) Definitely false and it is not necessary to find the
active than untreated bees. mechanism behind bee colony by sparing the lives
d) Reduction in the crop production due to use of c) Probably true
Imidacloprid. d) The technology driven changes are much more
e) None of these needed to reduce the growth of bee colonies.
e) Nothing can be said
42) Which of the following conclusions can be
drawn from the facts stated in the above 44) Which of the following could be the course of
paragraph? actiondrawn from the facts stated in the above
a) Now honeyproduction will never face a crisis like paragraph?
the crisis of 1980s due to the use of Imidacloprid. a) Govt should increase the subsidy on cash crops
b) Pesticide exposure alters bumblebees‟ behaviour including apiculture
in their nests. b) The Studies should be carried out to examine how
c)Neonicotinoids are most commonly applied to nicotinoids might positively induce the growth of
seeds in high concentrations, so that they pervade crops.
and protect the whole plant. c) Banning the use of certain neonicotinoids,
d) Maize growers are no more interested in the including imidacloprid, on all outdoor crops.
production of maize. d) Govt should plan to import more crops from
e) None of these abroad rather than sparing the ecosystem.
e) None of these
Day 10 : Computer Aptitude
Directions (45-48): Study the following information and answer the questions given below.

For example:

When rectangle is in unit digit its value is 2 and when it is in tens digit its value is thrice of 2 which is 6; when it
is in hundredth place, thrice of 6 which is 18 and so on.

45) How will you represent „208‟ in this code a) 1000


language? b) 2500
a) c) 3000
d) 2457
b)
e) 1250
c)
d) 48) What will be the resultant of the following
e) None of these code given below?
46) What will be the sum of digit of numerical

value for the code ?


a) 21
b) 13 a)
c) 23 b)
d) 16
c)
e) 14
d)
47) What will be the product of numerical value e) None of these

for the code and ?


Day 11 : Linear Arrangement
Direction (49-53): study the following information carefully and answer the below questions:
The distance between K and M is not even numbered value in line 1 but both the persons are immediate
neighbours. N is not an immediate neighbour of K. Persons name with Consecutive alphabets does not sit
together. All the person position in line 1 is same as in line 2. In line 1, the sum of the distance between N and I
and the distance between O and J is same as K and M. N sits third to the right of O. The length of the line 1 is
smaller than line 2. The distance between P and I in line 1 is same as the sum of the distance between J and O and
the distance between N and I in line 2. Two persons sit between N and M.

There are eight persons; they sit in each line such that the person sitting in line 1 sits at the same position in line 2.
Their names are consecutive alphabets but not necessarily in the same order. All of them are facing north
direction.
In line 1: Distance between second and first person is a multiple of 11m and increasing consecutively from left to
right.
In line 2: Distance between second and first person is a multiple of 13m and increasing consecutively from left to
right.

49) What is the length of line 2?


a) 533 b) 520 c) 650 d) 637 e) None of these
50) What is the distance between N and P in line a) 23
2? b) 17
a) 77 b) 66 c) 91 d) 117 e) None of these c) 13
d) 15
51) Which of the following pairs sit at the extreme e) 10
end of the line?
a) O and M 53) Who among the following sits to the
b) M and J immediate right of N?
c) K and O a) P
d) J and K b) M
e) None of these c) K
d) I
52) What is the difference between e) L
distance of O and L in line 2 and L and I
in line 1?

Set 12 : Direction Based Linear Arrangement


Directions (54-58): Study the following information carefully and answer the questions given below:
Eight students Ranjith, Ram, Rajesh, Rahul, Prisha, Priya, Preethi and Prema are waiting in a different railway
station viz., A, B, C, D, E, F, G and H but not necessarily in the same order. Each of the railway stations is
located in different directions and places. The Metro train faces north direction and starts from the first railway
station A to reach the eighth railway station F. The eight students are sitting in a straight line inside the metro and
facing Metro train engine‟s left side. Eight students are studying different department viz., CS, EEE, ECE, IT,
Civil, Chemical, Mechanical and Automobile. The above information is not necessarily in the same order. Not
more than one person waiting in a railway station.
Note: The train starts at 7a.m. and the stoppage time is not considered. The train takes 3min to cover 2km.
The one who is studying in ECE department sits third from the left extreme end of the line. The train reaches the
second station D at 7:12a.m., where, Ranjith boards the train but does not sits to the immediate left of the one who
is studying in IT department. Now the train turn right from the station D to reach the third station H at 7:36a.m.,
where the one who is studying in EEE department get into the train. And the train turn right from the station H to
reach the fourth station C at 7:48a.m., where Priya get into the train but not sits any extreme ends of the line. And
the train turn left from the station C to reach the fifth station B. Again turn left from the station B to reach the
sixth station G at 8:16:30a.m. Rajesh studying in the Chemical department and waiting in the station B and sits
third to the right of Rahul who is not studying in ECE department. The distance between fifth and sixth station is
12km. Preethi and the one who studying in EEE department are sit together but both are not sits immediate
neighbour of either Rahul or Rajesh. The train turns left from E. The one who is studying in Mechanical
department sits immediate left of Rahul. The train turn left from the station G to reach the seventh station E at
8:46:30 a.m., where Preethi get into the train and sit second to the left of Priya. Three persons sit between the one
who waits in the station E and the one who waits in the station D. The one who is studying in Automobile
department sits third to the left of the one who is studying in IT department. The one who is studying in Civil
department waits in the station F. Prisha and Ram does not wait either in station G or F. Prisha does not waits in
the starting point. The train reach the last station at 8.57a.m.

54) What is the shortest distance between from a) Rajesh


the starting station to the end station? b) The one who is studying in the CS department
a) 6km b) 9km c) 8km d) 10km e) 7km c) The one who waits in station G
d) The one who waits in station C
55) Who among the following is studying in the e) The one who studying in the Mechanical
Mechanical department? department
a) Prisha b) Preethi c) Prema d) Ram e) Priya
58) Who among the following sits to the
56) What is the actual distance between the immediate left of the one who is studying in the
stations of the persons sitting at the extreme civil department?
ends? a) Rajesh
a) 32km b) 30km c) 15km d) 24km e) 19km b) Rahul
c) The one who studying IT department
57) Who among the following sits third to the d) The one who waits in the station H
right of the one who is studying in the Automobile e) Ranjith
department?
Set 13 : Input Output
(Directions 59–62): A string of numbers is given as input. The further steps given are obtained by applying
certain logic. Each step is a resultant of previous step only. Study the following information carefully and
answer the questions given below it.

As per above applied logic in above steps, find appropriate step for given input:

59) What will be the difference of highest to lowest a) 54


number in step II? b) 23
a) 21 b) 28 c) 39 d) 32 e) None of these c) 46
d) 28
60) What will be the difference of square of digits e) None of these
of lowest number in step III?
a) 15 b) 55 c) 21 d) 9 e) None of these 62) Which of the following is the final output in
step IV?
61) What will be the sum of digits of all a) 45 b) 36 c) 74 d) 81 e) None of these
numbers in step I?
Set 14 : Coding Decoding
(Directions 63–66): Study the following information carefully and answer the questions given below it.
In a certain code language some statements are coded as follow:
“Player Comedy World Victory Final” is coded as “K%W C#F H#N G%P I$A”.
“Prime Trailer Video Genius Mode” is coded as “K@C M$T E#G P%Q D#Q”.
“Service Token Smart Fear Society” is coded as “B$G E#V D@P E$A I#P”.
“List Risk Under Valid Project” is coded as “J@I O$V I#T J@R H#F”.

63) How “Smart Project” is coded in given code 65) How “Genius Mind Power” is coded in
language? given code language?
a) I#T D@P a) J%T C@D M$T
b) B$G H#F b) E@B J#T P%Q
c) E#V O$V c) E#G J@Q J@E
d) J@I E$A d) K@E P%Q D$B
e) None of these e) None of these

64) Which of the following statement is coded as 66) How “Monthly Policy” is coded in
“A#V I#P I$A” in same code language? given code language?
a) Mount Victory Token a) L%W C$A
b) Victory Build Fear b)B#G D$T
c) Token Count Victory c) Y%C D%P
d) Maker Victory Fear d) Either A or C
e) Either A or C e) None of these

Direction Sense:
(Directions 67–68): Study the following information carefully and answer the questions given below it.
P@Q means – P is north of Q.
P^Q means – P is east of Q.
P%Q means – P is south of Q.
P&Q means – P is west of Q.
If,
P@^Q means P is North-East of Q and so on.

67) In the given expression below what is the (vi) D%^A


position of A with respect to H? (vii) F@^A
(i) A%B&C (viii) F%&C
(ii) F&G@H a) East
(iii) C@D^E b) North-East
(iv) F@E c) South-East
(v) D@&H d) North-West
e) Can‟t be determined. (vi) F&I
(vii) D@H
68) In the given expression below what is the (viii) A@&F
position of I with respect to B? a) East
(i) A&B%C b) North-East
(ii) H^G%F c) South-East
(iii) D@E^F d) North-West
(iv) I@^H e) Can‟t be determined.
(v) D^C

Set 15 : Seating Arrangement


Directions (69-73):Study the following information carefully and answer the questions given below:
Eight persons A, B, C, D, E, F, G and H are sitting around a rectangular table. Four persons sit at middle of the
table and they are facing the centre and four persons sit at corner side of the table and they are facing away from
the centre. Each one of them were in different age viz. 19, 15, 18, 37, 21, 42, 17 and 36. Person name starts with
consecutive alphabet does not sit next to each other. All the above information is not necessarily in the same
order.
Two persons sit between E and the one whose age is 18. C sits second to the left of the one who sits opposite to
the one whose age is 18. D sits second to the left of F. A sits second to the right of the one who is the eldest.
Person those sit at corner are only in odd number age. Sum of H and E is 10 more than thrice the age of A. Sum
of B and F is two less than D. Neither F nor D age is 18.

69) What is the age difference between total sum 70) How many persons sit between C and the one
of G and E and total sum of B and C (in years)? whose age is second eldest, when counted from
a) 15 b) 13 c) 10 d) 12 e) None of these left of C?
a) Three b) Two c) Four d) Five e) None of these
71) If G is related to 17, C is related to 36, in the e) None of these
same way which of the following is related to 15?
a) G b) H c) D d) B e) None of these 73) Which of the following persons sits second to
the left of fifth to the right of the one whose age is
72) Which of the following statement is true? 21?
a) Three persons sit between G and the one who is a) The one who is third eldest
second youngest b) Immediate neighbour of E
b) B and H are immediate neighbour of the one who c) Second to the right of B
is the second eldest d) Both b) and c)
c) Sum of A and B is three less than age of H e) None of these
d) All the above are true

Set 16 : Input & Output


Directions (74-78):Study the following information carefully and answer the questions given below:
A machine rearrangement given an input line having both words and numbers in a particular set of rules in step
by step. The following is an illustration of input and its rearrangement.
Input: match 63 cricket 47 17 bat 96 squads 24 division
Step I:cbu match 63 cricket 47 17 squads 24 division 95
Step II: 18cbu match 63 cricket 47 squads 24 95 chuhrhnm
Step III: nbudi 18cbucricket 47 squads 24 95 chuhrhnm 62
Step IV: 25 nbudi 18cbu 47 squads 95 chuhrhnm 62 bqhbjds
Step V: trvbef 25 nbudi 18cbu 95 chuhrhnm 62 bqhbjds 46
As per rules followed in the given steps, find appropriate step for given Input.

74) Which of the following element is fourth to 75) How many elements are there between „14‟
the left of second from the right end in step IV? and „present‟ in step III?
a) 34 b) Squats c) cbdl d) 93 e) None of these a) Four b) One c) Three d) Two e) None of these
76) What is the difference between second 78) Which of the following step represent the
element from the left end in step V and second following output
element from the right end in step II? „14 cbdl squats 81 34 present 67 start 93
a) 56 b) 49 c) 47 d) 58 e) None of these dwodbsdc‟?
a) Step V
77) Which of the following is fourth element from b) Step I
the left in second last step? c) Step III
a) cbdl b) squats c) tubsu d) dwodbsdc d) Step II
e) None of these e) None of these

Set 17 : Puzzles
(Directions 79–80): Study the following information carefully and answer the questions given below it.
There is a 5x5 matrix which can produce signals which in turn help in illumination of some bulbs. The rows of
the matrix are denoted by @, %, &, £ and ¥ from bottom to top in the same order and the columns are denoted by
A, B, C, D and E from left to right:
≦ row contains numbers which are consecutive multiple of „9‟ starting from „36‟.
£ row contains numbers which are consecutive multiple of „13‟.
& row contains numbers which are consecutive multiple of „8‟ starting from „32‟.
% row contains numbers which are consecutive multiple of „7‟.
@ row contains numbers which are consecutive multiple of „11‟.
Note: If the starting number of a multiple is not given; then start the row from the same number.
E.g.: If a row contains numbers which are consecutive multiple of „5‟. Then the row might be as follows;
The matrix helps in producing signals which is a string of numbers. There are four bulbs P, Q, R and S. Based on
the outcome of the string one of the bulb blinks.
Condition for blink:-
I. If outcome of the string is below 80, the bulb Q blinks.
II. If outcome of the string is between 90 and 110, the bulb S blinks.
III. If outcome of the string is between 125 and 150, the bulb P blinks.
IV. If outcome of the string is between 175 and 200, the bulb R blinks.
V. If none of the above condition follows then, no bulb blinks.
For outcome of the string:
I. If all the numbers of the string is an even number then, outcome is obtained by multiplying unit digit of all the
two digit numbers.
II. If a prime number is followed by another prime number then, outcome is obtained by sum of all two digits
number.
III. If a prime number is preceded by a perfect square then, outcome is obtained by multiplying tenth place of all
the numbers.

79) If Y = £B &E £A @D, then which of the 80) If X = ¥C @D £B @B, then which of the
following bulb blinks? following bulb blinks?
a) R a) R
b) P b) Q
c) Q c) P
d) S d) S
e) None blinks e) None blinks

Matrix Based Puzzle:


(Directions 81–83): Study the following information carefully and answer the questions given below it.
Twelve boxes marked A to L as in alphabetical order are placed in the form of a 3 x 4 matrix. The rows of the
matrix are denoted as 1, 2 and 3 from top to bottom and the columns are denoted as #, &, % and @ from right to
left in the same order. The blocks contains twelve different articles viz. Pen, Watch, LED, Toy, Bat, Book, Ring,
Banana, Cheery, Ball, Rose and Mango. All the information is not necessary in same order.
Note: When a box is said to be „beside /adjacent /between or at a gap of‟ it can be either horizontally or vertically.
 The box having Rose is kept second to left of box L. The box having Ring is kept just below box L. Box J is
kept besides the box having Ring. The box having Mango is kept exactly between the box C and the box B.
Box D is kept second to left of box E.
 The box J has five neighboring boxes. Box having Banana is kept adjacent to box B. The box having Banana
is kept exactly below box D. Box I is kept at a gap of one from box G. Box H is kept adjacent to box A.
 Box J having Pen is kept exactly between box having Ball and box C. Box B neither contains Ball nor Rose.
The box having Bat is kept adjacent to box H, which neither contains Mango nor kept adjacent to box having
Ball.
 The box having LED is kept exactly between box having Toy and box B. The box having Watch is kept at a
gap of one from box G. Three boxes are neighbors of box I. Box K is kept exactly between the box having
Book and box A.

81) Which of the following position correctly II. The box having Pen is kept second to left of the
represent the box having Rose? box which contain Watch.
a) 1% b) 2& c) 1@ d) 3% e) None of these III. The box which contains Book and the box
having Banana are kept together.
82) Which of the following box contains LED? IV. The box having Ball is kept exactly between box
a) L b) F c) A d) G e) None of these H and the one contains Pen.
a) Only I and IV
83) Which of the following statements is/are true? b) Only III
I. Box G contains Mango is kept immediate left of c) Only III and IV
the one which contains Cheery. d) Only II and IV
e) None of these
Set 18 : Numerical Series
Directions (84 -86): Study the following information to answer the given questions.
There are two rows given. To find out the resultant of a particular row we need to follow the following condition:-
Condition 1: If an even number is followed by an odd number (prime number) then the resultant will be the
addition of both the numbers.
Condition 2: If an odd number (prime number) is followed by a perfect square then the resultant will be the
difference between the numbers.
Condition 3: If an odd number (prime number) is followed by another odd number (non-prime) then the resultant
will be the product of both the numbers.
Condition 4: If an odd number (non-prime) is followed by an even number then the resultant will be the product
of the numbers.
Condition 5: If an odd number (non-prime number) is followed by a cube number then the resultant will be the
addition of both the numbers.
Condition 6: If an even number is followed by a perfect square then the resultant will be the product of both the
numbers.
Note: Give preference to Condition (5) when both Condition (4) and (5) follows.

84) If the resultant of row 2 is 362 more than the 15 27 36


resultant of row 1. Then find the value of X? a) 131 b) 127 c) 141 d) 124 e) None of the above
13 64 X 64 X
th
17 21 64 86) If X is 1/5 the value of resultant of the first
a) 13 b) 8 c) 16 d) 9 e) None of the above row then what will be the resultant of the second
row?
85) Which of the following is the difference of the 28 9 53
resultant of first and second row? 21 36 X
32 49 73 a) 915 b) 817 c) 493 d) 978 e) None of these 49 7

Alphabet Series:
(Directions 87–88): The given questions are based on five, 4-letter words which are given below. Study the
following information carefully and answer the questions given below it.

CREW RISK MART COLD FARM

87) If all the letters within given words are 88) If all the letters within given words are
arranged in reverse alphabetical order and thus written together in an alphabetical order without
newly formed words are rearranged as they any space then, how many letters are there in
appear in dictionary then, which of the following alphabetical series between sixth letter from left
word appears third? end and fifth letter from right end?
a) RISK b) FARM c) CREW d) COLD e) MART a) Seven b) Eleven c) Five d) Fifteen e) Nine
Set-19: Puzzles
Direction (89-93): Read the following information carefully and answer the questions given below.
Eight boxes A, B, C, D, E, F, G and H are kept in the stack one above another which has eight racks numbered
1-8. The bottom most rack is numbered 1, the one above is numbered 2 and so on till the topmost rack is
numbered 8. The boxes were in different colours namely, Violet, Indigo, Blue, Green, Yellow, Orange, Red and
White but not necessary in the same order. Each box consist different number of balls, which is either prime or
square number between 10 to 55.
Two boxes are kept between box E and the Yellow coloured box, which is kept at one of the odd numbered rack
but not in bottom most rack. Yellow coloured box is kept below the box E.There are 20 balls difference between
the boxes was kept at the rack 5 and rack 2, which has square number of balls.Three boxes are kept between the
box F and the Red coloured box, which is kept immediate above the box which has the balls in multiple of 12.
The box F is kept in one of the even numbered rack.Two boxes are kept between Red coloured box and box A.
Box A is kept immediately above the box which contains square number of balls in odd number.The White
coloured box is kept immediately below the box B. As many boxes are kept below the White coloured box is
same as above the box C. The White coloured box has only four balls less than the box which has maximum
number of balls.H has the maximum number of balls. The Orange coloured box has 10 balls less than box A and
kept one of the even numbered racks. The Blue coloured box is immediately above the Violet coloured box. As
many boxes kept between box H and the Orange coloured box is same as between box B and the Green coloured
box.The difference between the balls in the Yellow coloured box and box D is equal to the number of balls in the
box C. Box D is kept above the box which contains 47 balls.

89) How many boxes are kept between box D and 90) Which of the following boxes has lowest
the Red coloured box? number of balls?
a) Three b) Two c) Four d) Five e) One a) The Orange coloured box
b) A d) Box B
c) The Yellow coloured box e) The Indigo coloured box
d) G
e) The Violet coloured box 92) If Violet is related to 19 and White is related
to 25 in a certain way. Then, Blue is related to
91) Four of the following five are alike in a which of the following?
certain way and hence form a group. Which of a) 31 b) 36 c) 17 d) 47 e) 49
the following one that does not belong to the
group? 93)What is the difference between the balls in the
a) The Violet coloured box box D and the Indigo coloured box?
b) Box G a) 36 b) 26 c) 13 d) 14 e) 22
c) The Blue coloured box

Set-20: Direction Sense


Direction (94-96): Read the following information carefully and answer the questions given below.
„P@Q‟ means „P is to the north of Q‟,
„P#Q‟ means „P is to the south of Q‟,
„P%Q‟ means „P is to the west of Q‟,
„P&Q‟ means „P is to the east of Q‟,
„P$Q‟ means „P is in the distance of 5m from Q‟
„P*Q‟ means „P is in the distance of 7m from Q‟ and
„P#*Q‟ means „P is at 7m to the south of Q‟
A%$E#*B%*F@$C&$G@*D
94) A is in which direction with respect to D?
a) Northeast 96) What is the direction of D from F and the
b) North shortest distance?
c) South a) Southwest, 13m
d) Northwest b) Northeast, √13m
e) Southwest c) Southeast, 12m
d) Northwest, 13m
95) If X is 2m to the west of G, then what is the e) Northwest, 2√13m
distance between B and X?
a) 9m b) 5m c) 2m d) 7m e) 6m

Blood Relation:
Direction (97-98): Read the following information carefully and answer the questions given below.
„A$B‟ means „A is the father of B‟,
„A%B‟ means „A is the spouse of B‟,
„A&B‟ means „A is the sister of B‟,
„A@B‟ means „A is the brother of B‟,
„A#B‟ means „A is the parent of B‟,
„A*B‟ means „A is the child of B‟,
„A^B‟ means „A is the mother of B‟,
P@T^D%G&M*V^G*K
98) If N is the wife of M, then how is N related to
97) How is K related to D? G?
a) Son-in-law a) Aunty
b)Daughter b) Sister-in-law
c) Mother-in-law c)Mother
d) Son d) Sister
e) Father-in-law e) Cannot be determined
Set -21: Seating Arrangement
Direction (99-103): Read the following information carefully and answer the questions given below.
Eight persons A, B, C, D, E, F, G and H are sitting in a square table in the hotel given below and four of them
are sitting in the corners of the table while four of them are sitting middle of the four sides. The four corners are
numbered 1, 2, 3 and 4. The persons who are sitting in the corners are facing away from the centre and the
persons who are sitting in the middle of the sides facing inside of the table. (Note: The sides are denoted as 12,
24, 34 and 31)

B sits second to the right of D. Only two persons aresitting between A and H (Either from left or right side), who
sits in the even numbered corner but not 4.F sits second to the left of G and both of them are not an immediate
neighbour of A.E is not an immediate neighbour of G.As many persons sitting between F and D is same as
between C and D.
After they are seated in the square table, they are moved to sit in the circular table of the hotel due to
inconvenience. They all are sitting with equal distance from each other and facing each other. The arrangement in
the circular table is based on the seating position of square table.
The person who was sitting in middle of the side-34 sits third to the right of the person who was sitting at the
corner-1.Only one person sits between the persons who are sitting in corner-1 and corner-2. The person who was
sitting at the corner-3 sits second to the left of the person who was sitting at corner-4, who is an immediate
neighbour of the one who was sitting at corner-2. The one who was sitting at middle of the side-31 sits third to the
left of the person who was sitting at opposite to the person who sitting at middle of the side-31.

99)Who among the following persons sits third to c) The one who was sitting at corner-2 in the square
the right of G? table
a) The one who is sitting second to the left of F in d) The one who sits immediate neighbour of E
the square table e) None of them
b) E
100) How many persons are sitting between E and arrangement and hence form a group. Which of
F in the final arrangement? the following one that does not belong to the
a) One b) Two c) Three d) Four group?
e) Cannot be determined a) BG b) AD c) HC d) DE e) EF

101) In the circular table, who among the 103) If all the persons are made to sit as in the
following persons sits opposite to the one who was English alphabetical series from A in clockwise
sitting at middle of the side-24? direction in the final arrangement, then how
a) E b)A c) B d) H e) C many persons are remains their original position
(Excluding A)?
102) Four of the following five are alike in a a)One b) Two c) Three d) Four e) No one
certain way based on their position in the final

Set -22: Coding & Decoding


Direction (104-108): Read the following information carefully and answer the questions given below.
In a certain code language,
„Making good things‟ is coded as „@T3 @N6 #G1‟,
„Fun bearing happy‟ is coded as „#S17 %Y5 #U8‟ and
„Pollution creates warm‟ is coded as „$K2 #D10 %X16‟.

104)What will be the code for „Advancing‟? 106) What will be the code for „Decrying‟?
a) $Z8 b) %C6 c) #Y12 d) %Z6 e) @Z9 a) @D8 b) #W3 c) @W8 d) %M3 e) @W3

105) What will be the code for „Brought 107) Which of the following will be coded as
together‟? „%K4‟?
a) %G4 @T16 b) #V18 $H12 c) @Y18 %G2 a) Princes
d) %Y2 @G18 e) $R8 @Y18 b) Principal
c) Practicals
d) Descriptive b) @F3 $R3
e) Paper c) %R4 @F4
d) #F5 @R3
108) What will be the code for „Under ideal‟? e) None of these
a) %R3 @F3

Set -23: Circular & Linear Arrangement


Direction (109-113): Read the following information carefully and answer the questions given below.
Eight persons A, B, C, D, E, F, G and H are sitting in two circles which are named inner circle and outer circle.
Four of them are sitting in the each circle, some are facing inside while some are facing outside from the centre
and each one likes different colours namely, Violet, Blue, Green, Yellow, Orange, Red, Black and White but not
necessary in the same order. Also that they all are sitting with equidistance in the straight row facing north. If
anyone likes any one of the colours, that person will not change the colour in both arrangements. The persons
who are in the inner circle sitting equidistance between each other and sits exactly opposite to the persons sitting
in the outercircle.
E sits opposite to the person who sits second to the left of the one who likes Green, who sits one of the extreme
ends of the row. H faces the person who likes Red in the inner circle but none of them sits opposite to E, who
does not sit in the same circle of H sits. G sits third to the right of the one who likes Green in the linear row. G sits
opposite to the person who sits immediate right of H in thecircle and both of them faces opposite directions. A
faces the one who likes Violet in the circle and both of them are does not sit in the same circle. The one who sits
opposite to A in the same circle sits third to the left of the one who likes Blue. The person who likes Blue sits
immediate right of G in the linear row. B likes Yellow and does not sit opposite to the one who likes Red in the
circle.In the linear row, only one person sits between G and the one who sits immediate right of G in the
circle.The one who likes Red sits fourth to the right of E, who is an immediate neighbour of G in the linear row.
The one who likes White sits to the immediate right of F in the circle. The one who likes Orange sits third to the
left of F in the linear row.Only three persons are sitting between H and C in the linear row.A sits to the left of F in
the linear row but not immediately.The persons sitting in the extreme ends in the linear row facing the same
direction as A in the circle. The one who likes Yellow does not faces the same direction as D.

109) Who among the following sits second to the d) The one who likes Violet
right of Bin the circle? e) B
a) C
b) The one who sits opposite to D 112) If A is related to Violet and B is related to
c) G Blue in a certain way based on the linear row.
d) The one who likes Black Then, G is related to which of the following?
e) A a) Green
b) White
110) Which of the pairs represents the immediate c) Black
neighbours of the one who likes White in the d) Blue
circle? e) Orange
a) F, H b) H, C c) E, F d) G, B e) B, D
113) How many persons are sitting between H
111) Four of the following five are alike in a and the one who likes Black as per the linear
certain way and hence form a group. Which of row?
the following one that does not belong to the a) One
group according to the circle? b) None
a) The one who likes Blue c) Three
b) F d) More than three
c) The one who likes Black e) Two
Set -24: Coding & Decoding
Direction (114-118): Read the following information carefully and answer the questions given below.
In alphabet series A-Z, each letter except vowels is coded as 2 to 9 from A till Z (For ex. B is coded as 2 and C is
coded as 3 and so on till K is coded as 9. Again L is coded as 2 and so on till Z).
The vowels are coded by using the symbols #, @, %, $ and &.
In a certain code language,
„Something are happen‟ is coded as „8$3%97#46 @77 7@55%4‟,
„Taken for inquiry‟ is coded as „9@9%4 5$7 546&#7#‟ and
„Measure by weight‟ is coded as „*%@8&7* 25 3%#679‟.
i) If the first letter is a vowel and the last letter is a consonant, then both are to be interchanged.
ii) If the first letter is a consonant and the last letter is a vowel, then both are to be coded as „*‟.
iii) If both the first and last letters are vowels, then the last letter is coded as the code for second letter.

114) What will be the code of „Home Ministry‟? b) *#4* 3#4 846#3%#
a) *$3* *#4#897* c) @34 #44#3%8 *#4*
b) 3#4#8975 7$3% d) 844#3%# 3@4 3#4%
c) 3#4#8975 *$3* e) None of these
d) *$3* 3#498#75
e) None of these 117) Which of the following will represent by the
code „5%@7 759%@‟?
115) What will be the code of „Escalate Module‟? a) year before b) after year c) yeast back
a) 3$4&2% %83@2@98 d) pack helps e) None of these
b) %83@2@9% *$4&2*
c) *$4&2* 883@2@9% 118) What will be the code of „Distributed‟?
d) %83@2@98 *&4$2* a) *#897#2&9%*
e) %83@2@98 *$4&2* b) 4#897#2&9%4
c) 4%897#2&9&4
116) What will be the code of „Indices Can d) 4#897#29&%*
Wide‟? e) None of these
a) 3@4 844#3%# *#4*
Set -25: Seating Arrangement
Direction (119-123): Read the following information carefully and answer the questions given below.
Ten persons A, B, C, D, E, P, Q, R, S and T are sitting in two parallel rows and all of them are facing each other
sitting in opposite rows but not necessary in the same order. In Row-1, P, Q, R, S and T are sitting and all of them
are facing north while in Row-2, A, B, C, D and E are sitting and all of them are facing south. The persons who
are sitting in the Row-1 having their ages in multiple of four between 10 to 50. The persons who are sitting in
Row-2 having their ages in multiple of seven between 15 to 80.
Note: The elder person is sitting to the right of the younger person in each row. No younger one is sitting to the
right of elder.
The one whose age is multiple of 11 sits second to the right of T and either one of them sits at the extreme ends
of the row. The immediate neighbour of T faces the person who has the age in multiple of 5. Only one person sits
between B and C, neither of them sits at the extreme ends of the row. The one whose age is multiple of 10 sits
second to the left of P and faces the immediate neighbour of D. The one whose age is thrice the age of S sits third
to the right of S. The one whose age is multiple of 9 sits to the immediate right of D. The difference between the
ages of T and E is 3 years. R is elder than D. A‟s age is not an odd number. D‟s age is an even number but not in
multiple of 4. S does not faces the immediate neighbour of B.

119) Who among the following is the youngest of the following that does not belong to the
person in the group? group?
a) E b) Q c) B d) T e) S a) A
b) The one whose age is 44
120) Who among the following persons sits second c) The one who sits second to the left of T
to the left of the one who faces Q? d) The one who sits opposite to Q
a) The one whose age is 49 e) E
b) The one whose age is 35
c) E 122) What is the difference between the ages of D
d) The one who sits opposite to T and S (in years)?
e) None of them a) 32 b) 18 c) 30 d) 12 e) 42

121) Four of the following five are alike in a 123) Which of the following pairs represent the
certain way and hence form a group. Which one persons who are oldest in the both rows?
a) AS b) RA c) SE d) PA e) EQ
Set: 26 Coding Decoding
Direction (124-128): Study the information carefully answers the questions given below.
* Means either hour hand or minute hand is at 6
^ Means either hour hand or minute hand is at 3
+ Means either hour hand or minute hand is at 8
@ Means either hour hand or minute hand is at 9
# means either hour hand or minute hand is at 7
! Means either hour hand or minute hand is at 5
& Means either hour hand or minute hand is at 10
Note: if two symbols are given than by default first symbol is consider as hour hand and second one is consider as
minute hand. And all time are consider at AM.
Example: +* represents 8 hours 30 minutes

124) If a train departed from a station at ** it


takes 190 minutes to reach the destination, then at 127) If a boy leave from his House to school at
what time the train will reach the destination? „@^‟. Usually he takes 20 minutes to reach the
a) @+ b) ^* c) Data inadequate d)+@ school, but he takes 10 minutes more to reach the
e) None of these school, then at what time he will reach the school?
a) %^ b) *@ c) &@ d) Cannot be determined
125) A man reached the office at @+, if he gets e) @@
late by 35 minutes, then at what time he reached
the office? 128) A person takes „^*hours to reach the
a) @* b) !+ c)!@ d) &^ e) None of these Railway station from his home and his train is
scheduled to depart at „&&‟, so at what time
126) Find the time taken by the train to cover a should he leave from his home to the railway
distance of 495 km, if the speed of the train is 48 station to reach the station at 10 minutes earlier?
km/h. a) +* b) @+ c) #@ d) @@ e) None of these
a) +@ b) @+ c) &^ d) &# e) +!
Set -27: Puzzles
Directions (129-133): Refer to the data below and answer the questions that follow.
Five friends – P, Q, R, S and T - decide to read seven books numbered from 1 to 7. Books 1, 3, 4, 5 and 7 can be
read in one week. However, the time required to read books 2 and 6 is 2 weeks. Each one of them exchanges the
book in library at each weekend, only after he finishes reading it. Book 3 is a second part of book 2 so one can
read Book 3 only if book 2 is already read by him.
In the first week, P is reading book 1; Q is reading book 2 and so on as in the alphabetical order. They decide to
exchange the books in the numerical order of Books.

Note: If the book is not available as per numerical order, then they got the book with next serial number and so
on. If a Book has second part, then reading the both part of the book is considered as completing a book.
Book 7 will be followed by book 1 for each of them. They decide to meet till 10 weeks. Each friend reads a book
only once. If, for a person, no book is available for a particular week (either because he has already read all the
available books or he cannot read the available book or the book that he has to read is taken by the person who is
before as in alphabetical order), then that person does not read any book during that week.

129) Who among the following reads Book 1 in 130) Who among the following reads minimum
the Week 6? number of books at the end of Week 4?
a) P b) Q c) R d) S e) T a) P b) Q c) R d) S e) T
131) Which of the following Book does T reads in b) S – Week 5
week 7? c) T – Week 6
a) Book 1 d) P – Week 3
b) Book 3 e) Q – Week 5
c) Book 4
d) No Book Available 133) Which of the following Book does Q reads in
e) None of the above Books week 8?
a) Book 1
132) Four of the following five are alike in a b) Book 6
certain way thus form a group. Find the one c) Book 4
which does not belongs to the group? d) Book 5
a) R – Week 2 e) None of the above Books

Set -28: Numerical Series


(Directions 134–138): Following questions are based on the 5 given combination of numbers and words,
each consisting of a 5-digit number. Study the following information to answer the given questions.
34265, 37596, 48367, 65284, 43827

134) If all digits within 5-digit number are a) Two b) Four c) Three d) None e) One
arranged in ascending order then, what is the
difference of sum of digits of 2nd lowest to 2nd 136) If 1st and 3rd& 2nd and 5th digit in the given 5-
highest number after rearrangement? digit numbers are interchanged then, what is
a) 2 b) 4 c) 1 d) 3 e) None of the above difference of 2nd lowest to 2nd highest number
after rearrangement?
135) If „2‟ is subtracted from 1st, 3rd& 5th digits a) 31033 b) 31026 c) 31712 d) 18929
and „1‟ is subtracted from 2nd& 4th digits in the 5- e) None of these
digit numbers then, how many newly formed
digits are perfect square?
137) If all 5-digits numbers are divided by its 138) If all 5-digits in a number is added and
highest digit within the same number then, for arranged in descending order. Then which of the
how many numbers obtained remainder are non- following is third from the right end?
prime digit? a) 65284
a) 1 b) 4 c) 2 d) 3 e) None b) 37596
c) 34265
d) 48367
e) 43827

Set-29: Seating Arrangement


Directions (139-143): Study the following information carefully and answer the below questions.
Ten person namely – Mayank, Rinku, Bittu, Payal, Ishu, Dev, Kavita, Hari, Sam and Anuj are sitting around a
rectangular table in such a way that one person sits on each smaller side, two persons on each bigger side and one
person at each corners. The persons at the four sides sits facing center while persons at the four corners sits facing
away from center. Weight of each person is different viz. 42, 48, 52, 56, 64, 68, 72, 76, 78 and 84 kg. All the
information is not necessary to be in the same order.
Numbers in the given figure represents their respective position.
The one whose weight is 84 kg sits at longer side at a gap of two places from Anuj, whose weight is neither 72 kg
nor 64 kg. The one whose weight is 52 kg sits second to left of Rinku. One person sits between the one whose
weight is 48 kg and the one whose weight is 68 kg. Three people sits between Anuj and the one whose weight is
48 kg, who neither sits adjacent to Payal nor sits adjacent to the one whose weight is 84 kg. The one whose
weight is 76kg sits facing Ishu, who neither sits adjacent to Sam nor the one whose weight is 82kg. The one
whose weight is 68 kg and Kavita sits together at longer side. One person sits between Ishu and the one whose
weight is 72kg. Either Bittu or Hari sits at corner of the table. Payal doesn‟t sit adjacent to the one whose weight
is 48 kg. Two people sit between Bittu and Hari, who sits adjacent to the one whose weight is 64 kg. Two people
sit between Kavita and Dev whose weight is 56 kg. The one whose weight is 52 kg sits third to the left of Payal,
whose weight is 42 Kg.
Now they start playing card game. The shuffled a pack of card, each person draws one card and change their
position according to given conditions:
Condition 1: If the card drawn is a face card of „Diamond‟ then, the person who draws card interchange his/her
position with the one who sits at position 2, else interchange his/her position with the one who sits at position 5.
Condition 2: If the card drawn is a number card of „Club‟ then, the person who draws card interchange his/her
position with the one who sits at position 6, else interchange his/her position with the one who sits at position 4.
Condition 3: If the card drawn is a face card of „Heart‟ then, the person who draws card interchange his/her
position with the one who sits at position 7, else interchange his/her position with the one who sits at position 10.
Condition 4: If the card drawn is a number card of „Spade‟ then, the person who draws card interchange his/her
position with the one who sits at position 1, else interchange his/her position with the one who sits at position 3.
Note:- All the person draws cards in same sequence as given below:
i) Mayank draws 7 of diamond.
ii) Rinku draws King of heart.
iii) Payal draws Queen of club.
iv) Anuj draws 10 of club.
v) Kavita draws ace of spade.
vi) Hari draws King of club.
vii) Sam draws 5 of spade.
viii) Dev draws ace of diamond.
ix) Ishu draws 4 of heart.
x) Bittu draws 9 of spade.

139) What is the position of Bittu with respect to 140) How many people remain unchanged after
Sam in the final arrangement? final arrangement?
a) Third to left a) One
b) Fourth to right b) Three
c) Immediate left c) Two
d) Second to right d) None
e) None of the above. e) Four
141) How many person sits between the one b) Hari
whose age is 64 years and the one whose age is 52 c) Payal
years when counted right of the one whose age is d) Anuj
64 years after final arrangement? e) None of the above
a) Four
b) Two 143) What is the position of Dev with respect to
c) One Mayank before playing cards?
d) Five a) Third to left
e) None b) Fourth to right
c) Immediate left
142) Who sits to the immediate right of Rinku d) Second to right
before playing cards? e) None of the above.
a) Ishu

Set -30: Direction Sense


Directions (144-148): Study the following information to answer the given questions.
A, B, C and D are the corner points of the rectangular field in such a way that A is west of point B. C is south of
point A. Side CD is the largest side of the field. Point D is 18m north of point I. E, F, G and H are the corner
points of another rectangular field having area of 2304 m2 in such a way that Line EF intersects Line AB at M and
Line CD at N respectively. Point E is North of F. C is 12m west of point N. The length of largest side of field
ABCD is 80m.Point I is 54m south of point B. H is 32m east of point E. Line GH intersects the rectangular field
at L and P. D, I, J and K form another rectangular field in such a way that point J is 8m east of point P. Area of
the rectangular field DIJK is 504m2. Point H is 8m from point L.
Note: Area of rectangle= length * breadth (m2)
144) What is position of K with respect to G? c) 642m2.
a) North-East, 2√41m d) 392m2.
b) South-West, 11√2m e) None of these
c) East, 12m
d) North-East, 4√13m 147) What is the shortest distance between M and
e) East, 8m H?
a) 2√58m
145) If Sam standing at point F, wants to reach b) 3√62m
point B, then which of the following is the correct c) 4√68m
shortest direction to reach his destination? d) 3√68 m
a) FGPDB e) 5√60m
b) FNCMB
c) FGLB 148) In which direction is Point B with respect to
d) FEHLB N?
e) Either A or C a) South-east
b) North-east
146) What is the difference of area formed by c) East
MLPN to EHLM? d) North-west
a) 768m2. e) North
b) 752m2.
Set -31: Puzzles
Directions (149-153): Study the following information to answer the given questions.
The table below shows the result of the IPL T20 Championships. (For E.g. KKR won match against CSK, DD
won match against MI and so on).

The first IPL match was played on Monday and the last was on Friday. On each day the matches were conducted
in two sessions – Day match and Day-night match. No team played more than one match in a day.
Further it is also known that:

 DD won match against MI on Monday.


 KKR did not play on Thursday.
 DD won the Tuesday Day match.
 CSK played two Day matches and two Day-night matches.
 MI lost its Day-night match on Wednesday.
 RR lost its Day-night match against MI.
 CSK did not play on Monday.
 The least winner in the series had a Day match on Monday.

149) When did the match between CSK and RR 151) Who won the Day-night match on
been held? Thursday?
a) Day Match - Wednesday a) RR b) CSK c) DD d) MI e) KKR
b) Day-night Match – Thursday
c) Day Match – Friday 152) Four of the following five are alike in a
d) Day-night match – Tuesday certain way thus form a group. Find the one
e) None of the above which does not belongs to the group?
a) Monday – DD
150) How many Day matches were played by RR? b) Friday – CSK
a) One b) Three c) Two d) None e) Four c) Thursday – MI
d) Wednesday – CSK
e) Tuesday – RR
b) MI
153) Which teams among the following has played c) RR and DD
the highest number of Day matches? d) KKR and CSK
a) KKR e) None of these

Set -32: Puzzles


Directions (154-158): Study the following information carefully and answer the below questions.
A, B, C, D and E are five friends. They live in a 10 storey building. The building has a ground floor+10 floors.
Each of them lives in different floors and none of them lives in the ground floor. All of them entered the lift on
the ground floor and each pressed two floor buttons such that all the 10 buttons (1-10) are pressed. None of them
presses the buttons of the floor in which they lives.
E lives below D but above B. There are two floors between A and B. C lives above A but below D. A pressed the
floor number of C. The difference between the floor numbers pressed by D is a perfect square. B pressed the floor
number of D. C pressed the floor number of E. There is exactly one floor between C and E where none of the five
lives. E pressed the floor number of B. The floor numbers pressed by B are two perfect square numbers. B does
not live in floor numbered 1. A pressed consecutive floor numbers. B pressed the buttons of the floors which are
above the floor in which he lives. The difference between the floor numbers pressed by E is the floor number of
C.

154) In which of the following floor does C lives? 156) What is the sum of the floor numbers
a) Floor numbered 5 pressed by D?
b) Floor numbered 6 a) 12 b) 6 c) 13 d) 9 e) 17
c) Floor numbered 8
d) Floor numbered 7 157) Which of the following statement is true?
e) None of these a) E lives immediately below D
b) C pressed 8 and 2
155) How many floors are there above the floor in c) Sum of numbers pressed by A is 11
which B lives? d) All are true
a) Two b) Five c) Six d) Eight e) None of these e) None is true
158) How many floors are there between D and a) One b) Two c) Three d) Four e) None of these
A?

Set-33: Puzzles
(Directions 159-163): Study the following information carefully and answer the questions given below it.
Nine person namely – P, Q, R, S, T, U, V, W and X lives in nine floored building marked 1 to 9 in such a way
that lowermost floor is marked as 1, floor above it is marked as 2 and so on till topmost floor is marked as 9. Each
person likes different bird viz. Parrot, Owl, Duck, Peacock, Dove, Crow, Eagle, Sparrow and Pigeon. Each person
also likes different animal viz. Cat, Lion, Elephant, Rat, Dog, Monkey, Horse, Rabbit and Cow. All the given
information is not necessary in same order.
One who likes Horse lives on even numbered floor at a gap of three floor from one who likes Peacock. S, who
doesn‟t like Eagle but likes Rat, lives on floor just below the one who likes Peacock, who neither lives on floor
marked as 6 nor on floor marked as 8. V, who doesn‟t lives adjacent floor of W, lives on adjacent floor of one
who likes Crow. T, who doesn‟t lives on floor adjacent to one who likes Sparrow, lives on odd numbered floor at
a gap of one floor from one who likes Elephant. Only one person lives between one who likes Parrot and one who
likes Horse. U who likes Rabbit lives on any floor below one who likes Eagle. Only three person lives between
one who likes Parrot and W, who likes Cow. R who likes Owl neither lives on adjacent floor of one who likes
Horse nor one who likes Cow. One who likes Rat and Cat doesn‟t lives on adjacent floor. Only two person lives
between one who likes Owl and one who likes Pigeon. At least four person lives between one who likes Duck and
Crow, who lives on any floor above one who likes Horse. Only three person lives between one who likes Eagle
and one who likes Elephant, who lives on adjacent floor of one who likes Cat. Only two person lives between P
and one who likes Monkey. One who likes Lion lives on odd numbered floor just below one who likes Sparrow.
Q, who neither likes Dove nor Horse, lives at a gap of one floor from one who likes Dog.

159) Which of the following combination is not a) Only three person lives above one who likes
true? Horse.
a) V – Horse – 6 – Pigeon b) One who likes Cat lives just below one who likes
b) Crow – T – 7 – Dog Owl.
c) Q – Monkey – 5 – Sparrow c) One who likes Rabbit and Lion lives on adjacent
d) R – Elephant – 9 – Owl floor.
e) All the given combination are true. d) Only three person lives below one who likes Cow
and Parrot.
160) Who among the following likes Dove? e) All the given statements are true.
a) W b) S c) Q d) U e) None of these
163) If all the persons are rearranged in
161) How many person lives above one who likes alphabetical order of their name from top to
Sparrow? bottom, then position of how many persons
a) Five b) Two c) Four d) One e) None remains unchanged?
162) Which of the following statement is not true? a) Three b) Two c) Four d) One e) None

Set-34: Puzzles
(Directions 164-168): Study the following information carefully and answer the questions given below it.
Nine boxes namely – A, B, C, D, E, P, Q, R and S are kept one above other. Each box contains different articles
viz. Pen, Ring, Cup, Ball, Book, Laptop, Toy, Watch and Calculator. Each box is wrapped with different colored
paper viz. Black, Blue, Green, Yellow, Brown, White, Red, Pink and Orange. All the given information is not
necessary in same order.
Box Q which contains Watch is kept at a gap of three boxes from one which is wrapped with Blue paper. Only
one box is kept between box that contains Watch and box C. Only three boxes are kept between box C and the
one that contains Ring, which is kept at top. Box which contains Toy is neither kept adjacent to the one which
contains Laptop nor kept at bottom. The box which contains Book and box C, which doesn‟t contain Pen are kept
adjacent to each other. Box A is wrapped with Yellow paper is kept at a gap of three boxes from the one which is
wrapped with Green paper. Box S, which is neither kept adjacent to box Q nor with the one that contains Book,
but it is kept at a gap of three boxes from the one which contains Pen. Box that contains Pen and Calculator are
kept adjacent to each other. Only four boxes are kept between the one which contains Pen and the one which is
wrapped with Red paper. Only two boxes are kept between the one that contains Ball and the box which is
wrapped with Black paper. The box wrapped with Green paper is kept just above box D which contains Laptop.
Box that wrapped with Brown paper is not kept at bottom. Box that contains Cup kept just below one wrapped
with White paper. Box E, which is neither wrapped with Pink nor white paper is kept at any place above box R,
which is kept at a gap of one box from box B. Number of boxes between E and R is same as number of boxes
between box P and one which wrapped with Orange paper. Box P neither wrapped with Brown paper nor contains
Books. Box which wrapped with Pink paper is kept at any place above box wrapped with Orange paper. Box
which contains Toy and one wrapped with Brown paper is kept adjacent to each other.

164) Which of the following box is wrapped with b) Only two boxes are kept between the one that
Pink paper? contains Cup and the one wrapped with Green Paper.
a) The one which contains Toy. c) The one which is wrapped with Orange paper
b) The one which is kept just above one contains contains Watch.
Pen. d) The one which contains Ring is kept just above
c) The one which is kept at a gap of three from one the one that contains Toy.
contains Ring. e) All the given statements are true.
d) The one which is kept fourth from top.
e) None of these 167) If box R is related to S and box Q is related
to C, then in same way which of the following box
165) How many boxes are kept above the one that is related to B?
contains Laptop? a) E b) A c) D d) T e) R
a) Three b) Two c) Four d) One
e) None of these 168) How many boxes are kept between one that
contains Ball and one wrapped with Blue paper?
166) Which of the following statement is not true? a) Three b) Two c) Four d) One e) None
a) Only four boxes are kept below the one which is
wrapped with Black paper.
Set-35: Direct Sense
(Directions 169–170): Study the following information carefully and answer the questions given below it.
^ - South
% - East
@ - North
# - West
$ - Either 2 metre or 3 metre.
£ - Either 4 metre or 5 metre.
i.e., P ^ Q means P is south of Q.
P ^% Q means P is South-East of Q.
P ^$ Q means P is either 2 metre or 3 metre south of Q.
(i) H ^$ G
(ii) F @# E
(iii) B #$ C
(iv) A # H
(v) F %$ G
(iv) E %£ D
(vii) D @£ C
(viii) G @ C
(ix) E £ F
(x) A @ B
(xi)Distance between EF is more than CD.
(xii) Distance between BC is more than FG.
(xiii) Distance between CG is equal to 8m.
(xiv) Distance between (AH + GH) = ED.

169) What is the position of A with respect B? 170) What is the position of F with respect A?
a) 7m, North a) North-west
b) 5m, North b) North-East
c) 4m, North c) South-East
d) 6m, North d) East
e) Can‟t be determined. e) Can‟t be determined.
Input Output:
(Directions 171-173): A string of numbers is given as input. The further steps given are obtained by
applying certain logic. Each step is a resultant of previous step only. Study the following information
carefully and answer the questions given below it.

As per above applied logic in above steps, find appropriate step for given input:

171) What is sum of digits of lowest numbers a) 37 b) 23 c) 18 d) 27 e) None of these


obtained in step III?
a) 12 b) 7 c) 9 d) 13 e) None of these 173) What is the difference of square of number
in step II?
172) Which of the following number is second a) 440 b) 388 c) 240 d) 360 e) None of these
lowest in step I?
Set-36: Matrix Based Puzzles
Directions (174-178): Study the following information carefully and answer the below questions.
There is a board in the form of 5x7 matrix in which rows are labeled as A, B, C, D & E from top to bottom and
columns are labeled as @, %, &, #, ¥, ^ and £ from right to left. All consecutive odd numbers are written in each
cell from left to right in each row such that „A£‟ has smallest odd number and „E@‟ has the largest possible odd
number. Then the cells in the matrix are filled with respect to the below statements:
Note: The letters of the word „JOURNEY‟ is filled first before filling the letters of the word „PLASTIC‟. Each
cell should either contains one number and letter or one number and symbol
 All the letters of the word “JOURNEY” are written in alphabetical order to each cell having multiple of
5.
 All the letters of the word “PLASTIC” are written in reverse alphabetical order to each of the remaining cell
having number multiple of 3.
 „%‟ is written to each remaining cells which are preceded by a vowel in a row.
 „K‟ is immediately preceded by „&‟. One element is written between „K‟ & „Y‟.
 Two elements are written between „@‟ and „#‟ in the same row. „@‟ is written left of „#‟, which is not
written in same column having „U‟.
 „*‟ and „G‟ are written in the same row. „β‟ is preceded by „D‟. „%‟ and „F‟ are written in same column.
 „Z‟ and „Q‟ are not written in same column. Also „Z‟ and „Y‟ are not written in the same row.
 „G‟ and „M‟ are written in the same column next to each other.
 „M‟ is followed by „≦‟.
 At least four and at most eight elements are written between „B‟ and „Z‟. B is written before Z.
 „B‟ is preceded by „$‟. „G‟ is preceded by „£‟.
 „Q‟ is written before „A‟ but not in any cell having prime number. Q and A are not written in the same row.
 „V‟ and „W‟ are written in same column.
 „G‟ is written after „M‟. „D‟ and „V‟ are written in same row.
174) What is the correct cell position of „V‟? 177) Which of the following element is present in
a) E^ b) D¥ c) E& d) C£ e) None of these the „E%‟ cell of the matrix?
a) 67K b) 53% c) 35O d) 67& e) None of these
175) Four out of five are related to each other in
some way and thus form a group, then which of 178) Which of the following statement is/are true?
the following doesn‟t belongs to the group? a) The cell „B#‟ of the matrix is written as „21Z‟
a) MZJ b) W%R c) Vβ& d) E$T e) G%£ b) „31G‟ is written in the „D^‟ cell of the matrix.
c) The cell „E≦‟ of the matrix is written as „61V‟
176) Which of the following element is in the d) „65F‟ is written in the „E&‟ cell of the matrix.
adjacent cell of „W‟? e) All are true
a) # b) @ c) A d) F e) None of these

Set-37: Seating Arrangement


(Direction 179-183): Study the information given below and answer the questions based on it.
Twelve Children namely – Mike, Jake, Tom, Benny, Danny, Dave, Hugh, Jack, Chris, Martin, Ben and Lee
belongs to different Classes viz. Class I, Class III and Class V and they were sitting in different position in each
class as given below. Each likes different chocolates viz. Dairy milk, Kitkat, Munch and Milkybar not necessarily
in same order.
Note: Not more than one in same class likes same chocolate. The Children who likes Milkybar sits in Position 1
in each Class.
Lee and Jack belong to same class and there is one children sitting between them. The one who likes Kitkat sits
immediate right of Jack. Dave is junior to Lee and both likes Milkybar. Both Martin and Chris like same
Chocolate. Martin and Benny exactly face each other and neither of them sits with Jack nor Jake. Jake likes
munch and is senior to Jack. Mike sits third to the right of the one who likes Milkybar. Mike faces Hugh and both
are immediate neighbours of one who likes Dairy milk. Ben sits between Jake and the one who likes Dairy milk.
The one who is sitting between Dave and Danny likes Kitkat. Both Tom and Danny were sitting immediate right
of one who likes Dairy milk. Tom doesn‟t like Milkybar.

179) Who among the following likes Dairy Milk? c) Jake – Jack – Danny
a) Benny b) Mike c) Jack d) Ben e) Hugh d) Danny – Hugh – Jake
e) Jake – Hugh – Mike
180) What is the position of Dave with respect to
Martin? 182) Who among the following belongs to Class I?
a) Immediate right a) Danny b) Tom c) Mike d) Jake e) Jack
b) Second to the left
c) Second to the right 183) Which class and what chocolate does Tom
d) Third to the right likes?
e) Third to the left a) Class III – Kitkat
b) Class I – Munch
181) Which group among the following likes c) Class V – Kitkat
Munch? d) Class I – Dairy Milk
a) Hugh – Tom – Benny e) Class III – Milkybar
b) Martin – Chris – Hugh
Set-38: Direction Based Puzzles
(Direction 184-188): Study the information given below and answer the questions based on it.
There are seven members in a family – A, B, C, D, E, F and G which has three generations. Out of which 5
members - A, B, C, E and F works in different cities viz. Chennai, Bangalore, Mumbai, Kochi and Hyderabad but
not necessarily in same order. Among the working people two owns Benz Car, two owns Audi Car and one has
Jaguar Car. Among the cars, two were in White colour and three were in Black colour. Two of them got salary in
60k, two of them in 80k and only one in 70k. Each planned to meet at a meeting point X; and their directions
were as shown in the fig. below.

 The difference between the salary of A and his Father-in-law is 10k. C is the grandson of the one who works
in North direction from Meeting point X. Both the Audi Car was in same colour.
 The person from Mumbai and the one who works exactly in the difference of 90 0 from him owns same Car. B
is the son in law of one who keeps Audi Car. E is the niece of C‟s mother and earns 60k.
 C earns 80k which is as same as the person who works in west direction to him. The one who works in
Hyderabad earns less salary than his son and owns Black coloured Car. People with Black Car don‟t earn
same salaries.
 The eldest of the family earns 70k. A earns as same as the salary of his wife‟s Nephew who works neither in
North nor North-east to him. G‟s brother-in-law owns a white Car. The person who is south-west direction
from meeting point X owns neither Benz nor Audi Car.

184) What car does E owns and how is she related 185) In which direction is A with respect to B and
to F? what Car does B owns?
a) Benz – Daughter a) South east – Jaguar
b) Audi – Mother b) South west – Benz
c) Jaguar – Niece c) North east – Audi
d) Benz – Grand daughter d) North west – Benz
e) Audi – Grandson e) South east – Benz

186) Who among the following keeps Black Car?


a) The person who is working in Kochi d) G is the father of E
b) A e) F is the mother of D
c) The one who get salary of 80k in Chennai
d) The person who is working in Mumbai 188) How many females are there in the family?
e) E a) One
b) Two
187) Which among the following is true? c) Three
a) E earns 60k and is working in Mumbai d) Four
b) The person from Chennai owns Benz Car e) More than Four
c) The people who gets salary of 60k owns Black
Car

Set-39: Seating Arrangement


(Direction 189-193) Study the information given below and answer the questions based on it.
Eight persons namely Zylah, Amyrah, Victoria, Tessa, Gwen, Nylah, Sasha and Cynthia were travelling in a train
towards different destinations viz. Surat, Vadodara, Kota, Mathura and Delhi but not necessarily in same order.
Not more than two and less than one travel towards the same destination. Each has different age and their age was
either a prime number or cube number ranging from 20-65 years. Not more than one has same age. Each sits in
different berth of a same compartment. Their seat details and seat number were as shown in the fig. below
 Age of Tessa is a cube number and she is elder to Victoria. Amyrah is seated in Upper berth and the person
who gets down with Amyrah sits diagonal to her. The age of Gwen is as same as her seat number and doesn‟t
get down at Surat.
 The age difference between Amyrah and Nylah is a square of a number less than 4 and Amyrah is younger to
Nylah. Zylah and Nylah both sits at Lower berth of different sides. Neither Amyrah nor Tessa goes to
Vadodara.
 The person belonging to same destination doesn‟t faces or sits adjacent to each other. Tessa and the person
whose age is 20 years less than Victoria are sitting in same berth of the same side.
 Tessa is an immediate neighbour of Victoria who goes to Delhi. Only one person gets down at Kota and her
age is one less than the twice the age of Sasha. Cynthia goes to Surat in upper berth and her seat number is the
age of the one who goes to Delhi.
 The person who goes to Vadodara and Sasha faces each other and the difference between their seat numbers is
1. The difference between the seat numbers of Victoria and the one who goes to Kota is 2. The age of Cynthia
is 20 more than her seat number.

189) What is the age of Sasha and where does she


goes? 191) Who are the eldest and youngest people in
a) 29 – Delhi the compartment?
b) 61 – Mathura a) Gwen – Tessa
c) 55 – Vadodara b) Sasha – Victoria
d) 27 – Delhi c) Tessa – Sasha
e) 23 – Mathura d) Amyrah – Nylah
e) Victoria – Nylah
190) Who among the following faces Gwen?
a) Tessa 192) Who among the following is sitting in the
b) Victoria seat number-26?
c) Amyrah a) Victoria
d) Sasha b) Cynthia
e) Zylah c) Sasha
d) Zylah b) Victoria is 4 years younger to Tessa
e) Nylah c) Sasha sits in the lower berth
d) Zylah goes to Kota and sits at seat number-28
193) Which of the following statement is true? e) Only one person gets down at Mathura
a) Gwen is younger to Sasha and sits at seat number-
27

Set-40: Seating Arrangement


Directions (194-198): Study the information carefully and find out the seating position for circle A and by
applying the conditions given below to find out the seating position for circle B.

Eight famous NASA Astronauts Eric, Acaba, Boe, Robert, Serena, Daniel, Joseph and Peggy are sitting in a
circular table A. Some of them are facing centre and remaining are facing opposite to the centre. Each of them has
working for either Falcon 9 satellite or Falcon heavy satellite. The persons who working for same satellite are not
sit together but they are facing same direction. The persons who working for different satellite are always sit
together but they are facing opposite direction. They are all have different years of experience 20, 25, 30, 32, 27,
16, 18 and 40 in the NASA space centre. All the above information is not necessarily in the same order.
The one who have 30 years of experience sits second to the left of Eric. Peggy is the most experienced among
them. Serena sits in an even numbered seat and sits second to the left of the one whose experience is square of a
number. Acaba and Serena are facing same direction and Acaba‟s experience is twice the experience of Serena.
The one who has experience of 32 years sits second to the left of the one who has experience of 20 years.
Joseph‟s experience is four years less than that of average experience of Robert and Peggy. Boe who doesn‟t
work for falcon 9 satellite, have 3 years more experience than Eric and never sits in seat number 3. Robert doesn‟t
sits opposite to Boe. Daniel sits in seat number 6. Eric sits third to the right of Daniel.
According to the conditions given below, the persons shifts their position from CIRCLE A to CIRCLE B.
CONDITIONS:
 If the experience of Astronauts is multiple of 3 but not a multiple of 5, then, he moves to the same numbered
position in circle B and doesn‟t change his direction.
 If the experience of Astronauts is multiple of 4 and also 5, then he moves to a position that is opposite to his
current numbered position in circle B (i.e. Position 2 is opposite to position 6; and Position 4 is opposite to
Position 8 and so on.) and change his direction to opposite direction. (if he faces outside the centre in circle
A now he face the centre in the circle B and vice versa.)
 If the experience of Astronauts is multiple of 4 but not a multiple of 5, then he moves to the same numbered
position in circle B and change his direction to opposite direction.(if he faces outside the centre in circle A
now he face the centre in the circle B and vice versa.)
 If the experience of Astronauts is either a Square of an odd number or a multiple of both 3 and 5, then he
moves a position that is opposite to his current numbered position in circle B (i.e. Position 2 is opposite to
position 6; and Position 4 is opposite to Position 8 and so on.) and doesn‟t change his direction.
 Note: Final arrangement is obtained after all the conditions are applied. All the Questions are based on final
arrangement.

194) Who among them is sits immediate right of D. ii and iii only
the one who has least experience ? E. All i, ii and iii
A. Joseph B. Peggy C. Boe D. Acaba
E. None of these 197) Four of the five among the following are
similar in such a way to form a group, which one
195) How many people have more experience doesn‟t belongs to the group?
than Joseph? A. Eric and Daniel B. Boe and serena
A. None B. 1 C. 2 D. 3 E. More than three C. Daniel and Acaba D. Peggy and Acaba
E. Robert and Boe
196) Which of the following statement(s) is/are
correct? 198) Who sits second to the right of the one who is
i) Boe and Daniel are facing same direction opposite to 30 years of experienced person among
ii) Eric has more experience than Joseph the group?
iii) Acaba is not an immediate neighbour of the A. The one who is immediate right of Acaba
persons who works in falcon heavy satellite B. The one who is second to the right of Boe
A. i only C. The one who is works in falcon heavy satellite
B. ii only D. Both b and c
C. i and ii only E. None of these
Set-41: Puzzles
(Directions 199–203): Study the following information carefully and answer the questions given below it.
A student from a central university develops a scientific calculator for specific encrypted calculation. Digits from
0 to 9 are coded as different letters, and perform a 3x3 multiplication as shown below in the figure.

Note: No two digits are coded as with same letter.


Carefully observe above calculation and answer the following question based on same multiplication.

199) What is the possible value of “P2 + 2M”? c) 65


a) 55 d) 52
b) 76 e) None of these
200) What is the possible value of “D3 + 4Q – 202) What is the possible value of “4P + 3G –
2Z”? 3B”?
a) 226 a) 27
b) 312 b) 28
c) 117 c) 45
d) 200 d) 34
e) None of these e) None of these

201) If “GKB” is multiplied by “DEM” then what 203) If “PZ” is multiplied by “BQ” then what is
is the possible resultant of the multiplication? the possible resultant of the multiplication?
a) EGBPBM a) MKED
b) MZKQEM b) QPZQ
c) GMPKQZ c) DGQD
d) GEPMBM d) EKEG
e) Can‟t be determined e) Can‟t be determined
Set-42: Puzzles
Directions (204-208): Study the following information carefully and answer the below questions.

Four Teams – Team Red, Team Blue, Team Green and Team Orange – of three members each, have to be created
such that they satisfy the following conditions:
 S and U have to be together.
 A Team cannot have all three members having a common domain of experience.
 Team Red includes W and Y.
 The total “Web development” experience in Team Blue is 8 years.
 The total experience of one team is 27, which is highest among all teams and Team Orange has total
experience of 17 years, which is least among all teams. The other two teams have different total years of
experience.
 None of the members in Team Green have any experience in Data Science.
 If P is in a team, then U cannot be in the same team.

204) What is the total experience of all employees b) 23 years


in Healthcare management of Team Orange? c) 20 years
a) 10 years d) 27 years
b) 5 years e) None of these
c) 8 years
d) 7 years 206) Who among the following does not belongs
e) None of these to Team Blue?
a) T
205) What is the total experience of all employees b) Q
in Team Green? c) Z
a) 19 years d) R
e) Both Z and Q e) Z

207) If P is related to Z in a certain way; and X is 208) Which among the following team has people
related to W in the same way. Then who among with maximum experience in Internet security?
the following is T related to? a) Team Orange
a) Q b) Team Blue
b) R c) Team Red
c) V d) Both Team Blue and Team Red
d) W e) Both Team Green and Team Red

Set-43: Direction Based Puzzles


(Directions 209–213): Study the following information carefully and answer the questions given below it.
Ten friends namely – N, O, P, Q, R, S, T, U, V and W are sitting in an examination hall in such a way that all of
them sit facing north but not necessarily in same row.
 W sits east of O. R and O sits in same line. W sits at any place south of T. V sits 12m north of the one who is
9m east of Q.
 P is to the north of Q. S is 13m south-west of T.N sits north of W and is east of Q.
 V and T sit at a gap of 5m. R sits somewhere in-front of V but not necessarily north of V.
 P is 10m west of the one who is 5m north-west of T. U sits 20m south of V. W and N sits at a gap of 6m. O
sits at a gap of 4m west of the one who is south of T.
209) What is position of U with respect to O? d) 10√3m
a) West e) None of these
b) North-East
c) South-West 212)Which of the following statement is/are true?
d) North-West I) QS + OW > TV + PR.
e) None of these II) PQ > (US + RT) ≥ (WN + QS).
III) NS + OW + TR < NW + VT.
210) Which of the following statement is true? IV) VS + SQ >PQ + TR
a) P is south-west of R. a) Only III
b) Q is 12m west of T. b) Only II and IV
c) R is 5m north-west of the one who is 6m east of c) Only I and III
V. d) Only II and III
d) Q is 16m south of the one who is 10m west of R. e) None
e) All the above statement is false.
213) What is position of P with respect to W?
211) What is the shortest distance between O to a) West
T? b) North-East
a) 15m c) South-West
b) 2√85m d) North-West
c) 16m e) None of these

Set-44: Puzzles
Directions (214-218): Study the following information carefully and answer the below questions.
The tribes of North Sentinel Island have recently developed a language for themselves, which has a very limited
vocabulary.
Also, the words can be classified into only three types: the Tongs, the Yongs and the Pongs.
The Tongs types of words are: Coop, Sloop and Whoop
The Yongs types of words are: Whoop, Droop and Hoop
The Pongs types of words are: Knoop, Roop and Stroup
Also they have formulated some rules of grammar namely-
 Each sentence should have only five words but not repeated words even if a word is classified under two
types.
 Each sentence should have two Tongs, one Yongs and two Pongs.
 If Coop is used in a sentence, then Whoop must be used and vice versa.
 Hoop can be used in a sentence only if Knoop is used.
 Sloop and Droop cannot be used in a single sentence.

214) Which of the following choice of words in a b) Sloop Coop Hoop Roop Stroup
sentence doesn‟t violate the grammar? c) Sloop Whoop Droop Stroup Knoop
a) Coops and Sloop as the Tongs and Droop as the d) Coop Roop Sloop Whoop Knoop
Yongs e) Roop Knoop Hoop Droop Sloop
b) Sloop and Coop as the Tongs and Hoop as the
Yongs 217) If Coop and Sloop is the Tongs in a sentence,
c) Knoop, Droop and Whoop as the Yongs used in a and no rule of grammar is violated, then which of
sentence the following is/are true?
d) Knoop and Stroup as the Pongs and Sloop as the I. Hoop is the Yongs.
Tongs II. Whoop is the Yongs.
e) All violates the grammar rules III. Knoop and Stroop are the Pongs.
a) Only II
215) If in a sentence Coop is the Tongs and no b) Only I
rule of grammar is violated, then which of the c) Both II and III
following could be other part of the sentence? d) Both I and III
a) Whoop Hoop Droop Stroup e) None of these
b) Sloop Hoop Whoop Stroup
c) Whoop Hoop Knoop Stroup 218) Which of the following is a possible sentence
d) Whoop Sloop Roop Droop if certain grammar rule is violated?
e) None of these a) Knoop Coop Whoop Roop Sloop
b) Coop Sloop Whoop Stroup Knoop
216) Which of the following is a possible sentence c) Sloop Whoop Hoop Roop Stroup
if no grammar rule is violated? d) Knoop Coop Hoop Roop Whoop
a) Hoop Stoop Coop Roop Knoop e) None of these
Set-45: Numerical Series
Directions (219-223): Study the following information to answer the given questions.
There are three rows given. To find out the resultant of a particular row we need to follow the following
conditions: -
Condition 1: If an odd number (non-prime) is followed by another odd number (prime number) then the resultant
will be the addition of both the numbers.
Condition 2: If an even number is followed by a perfect square then the resultant will be the difference between
the numbers.
Condition 3: If an odd number (prime number) is followed by another odd number then the resultant will be the
product of both the numbers.
Condition 4: If an even number is followed by another even number then the resultant will be the product of the
numbers.
Condition 5: If an odd number (non-prime number) is followed by a perfect square number then the resultant will
be the difference of both the numbers.
Condition 6: If an even number is followed by anodd number (prime) then the resultant will be the sum of both
the numbers.

219) If the sum of the resultants of all three rows b) 8


is 462. Then find the value of X? c) 16
13 15 X d) 9
21 64 5 e) None of the above
36 81 23
a) 13
220) What will be the product of digits at unit 222) Find the sum of second digits from left of the
place from resultant of all three rows? resultants of all three rows?
24 31 9 15 9 12
23 27 49 20 36 17
32 17 13 15 23 13
a) 24 a) 13
b) 16 b) 8
c) 21 c) 7
d) 27 d) 6
e) None of the above e) None of the above

221) If the sum of the resultants of all three rows 223) If the sum of the resultants of all three rows
is 1149. Then find the square root of Y? is 89. Then find the value of X?
21 36 53 24 31 X
48 9 49 48 9 49
64 18 Y 20 36 17
a) 8 a) 9
b) 9 b) 32
c) 11 c) 16
d) 10 d) 12
e) None of the above e) None of the above

Set-46: Puzzles
Directions (224-228): Study the following information to answer the given questions.
There was a Cricket match between Team-A and Team-B. Team -A scored 354 runs for 7 wickets. Team-A squad
consists of Jack, John, Lee, Max, Mark, Ben, Mike, Paul, Sam, Peter and Tom. Each player scored different runs
from 0-150 such that their score is either a square number or cube number other than zero. Not more than one
player scored same runs and two players were yet to bat. Each have different brands of bat viz. Reebok, Spartan
CG, Sunridges, Puma, SG Cobra and GM Icon and not less than one and more than two have same branded bat.
In scorecard, the player who scored the highest runs is numbered 1 and the player who scored second highest runs
is numbered 2 and so on.
Note: Sum of runs scored by all players = Total score of Team A. The players who were yet to bat also own
different brands of bat and they were arranged in the scorecard in alphabetical order.
 Run scored by Jack is a square of the position of Tom in the scorecard. The run scored by the 4 th least scorer
(excluding the players yet to bat) is cube of
 a number. Reebok brand bats are used by the player who is at one of the position less than 5 and another
player (who uses the same brand bat) is double the position of this player.
 There are two players between Ben and the one who scored runs in a square of a number. The persons who
scored runs in square and cube of a number are placed above and below of Ben respectively in scorecard.
Two people owns Spartan CG bat and sum of their score is 27 runs.
 Paul owns Puma bat and he is yet to bat. Neither of the players who were yet to bat owns Spartan CG nor
Reebok branded bats. The runs scored by using Puma branded bat is the second highest. Lee got one of the
positions in top 3 least scorers.
 Lee scored runs as same as the position of Ben in scorecard and Ben scored as same as the square of position
of Lee in the scorecard. Peter scored 7 runs less than the double the score of the only one who uses Sunridges
brand bat.
 Tom got 8th position in scorecard without scoring any runs. Only Peter scored runs in three digits. The person
who uses GM Icon brand bat is the second least among the other branded bat.
 Mike is yet to bat. Number of players between Mark and Mike in the scorecard is as same the number of
players between Mark and Max. Mark who is in one of the positions below John and the one who scored runs
in square number as well as the one who scored runs in cube number uses SG Cobra branded bat.
 Not more than one player uses Sunridges brand bat. Lee does not use SG cobra branded bat.

224) Who among the following uses Reebok d) Spartan CG


brand bat? d) SG Cobra
a) Ben – Sam b) John – Tom c) Ben – Tom
d) Max – Lee e) Peter – Mike 227) How many players are there between Mike
and John?
225) Who scored the 5th least runs excluding the a) One b) Three c) Two d) Four e) More than Four
batsman who were yet to bat?
a) Lee b) John c) Ben d) Mark e) Max 228) Which of the following statement is true?
a) Lee scored 8 runs and owns GM icon brand bat
226) Which among the following Bat brand b) Mark is three positions below Max in the
holders hold the third highest runs? scorecard
a) Sunridges c) Tom and Ben owns same brand of bats
b) Reebok d) Score of Max is a cube number
c) Puma e) Reebok bat holders scored second least runs.
Set-47: Puzzles
Directions (229-233): Study the following information to answer the given questions.
The given grid (8x8) represents the road map of 7 cities – Jaipur, Haryana, Lucknow, Rajasthan, Goa, Gurugram
and Kolkata. Each city is located at different points as shown in the fig. below – A, B, C, D, E, F and G but not
necessarily in the same order.
Note: All the distance calculated were the minimum distance between them. The only mode to travel between
cities is along the gridlines.
Each grid (1x1) represents 100km along each side. The cities can be reached by travelling along the grid lines
shown in the diagram. The shaded area represents a restricted areas i.e., one cannot travel along the vertices
(corners) and gridlines bordering/passing through these areas.
Also it is known that,
 Shortest route between Jaipur and Rajasthan is 200 km
 Rajasthan is equidistant from Haryana and Goa
 Shortest route between Lucknow and Kolkata is 600 km
 Shortest route between Gurugram and Goa is 400 km
 Point E is not Rajasthan
 Shortest route between Lucknow and Haryana is 800 km

229) Which of the following city is at Point C? 232) If it takes 6 hours to travel from Point C to
a) Rajasthan b) Kolkata c) Haryana d) Lucknow Kolkata; then what is the time required for
e) None of these travelling from Gurugram to Point F?
a) 10 hours
230) What is the minimum distance between b) 6 hours
Point B and Haryana? c) 4 hours
a) 800 km b) 1200 km c) 1000 km d) 1100 km d) 8 hours
e) None of these e) Cannot be determined

231) If Ramu, has to travel from Goa to Haryana 233) Which of the following City is at Point G?
via Kolkata; then what is the minimum distance a) Haryana
to be travelled? b) Kolkata
a) 1000 km b) 800 km c) 1300 km d) 1100 km c) Gurugram
e) 1400 km d) Lucknow
e) None of these
Set-48: Puzzles
Directions (234-238): Study the following information to answer the given questions.
There are 15 boxes of three different sizes – large (L), small (S) and medium (M) and are of five different colours
– Peach (Ph), Cyan (Cy), Magenta (Mg), Brown (Br) and Grey (Gy). They are arranged in certain pattern facing
north in a straight line numbered 1-15 (i.e., 1 being the box at the extreme left and 15 being the box at the extreme
right).
 Peach boxes are not at prime places and also not at the place that is cube of a natural number.
 Every medium sized box has one small box and one large box adjacent to it on its either side.
 No two boxes of the same size and same colour are kept together.
 There are five boxes of each size. Each size has all the five coloured boxes.
 Also Peach box is always placed at immediate right of Grey box.
 The first box is a large Cyan box and the last box is a small Magenta box.
 A small box is never kept to the immediate right side of the large box.
 A box is not placed adjacent to same coloured boxes on both sides.
 The sum of position of all three Magenta coloured boxes is an odd number.
 All three Brown coloured boxes are placed at even numbered positions.

234) What is the sum of position of all Cyan a) 40 b) 38 c) 43 d) 38 e) 45


coloured boxes?
a) 15 b) 17 c) 18 d) 22 e) None of these 237) Which of the following box is placed exactly
at the middle of the arrangement?
235) Which of the following box is placed at 9th a) Large – Cyan b) Small – Peach
position? c) Medium – Grey d) Large – Peach
a) Medium – Grey b) Small – Peach e) None of these
c) Large – Cyan d) Large – Magenta
e) None of these 238) What is the position of small sized Brown
box?
236) What is the sum of all medium size box a) Position 2 b) Position 4 c) Position 8
positions? d) Position 6 e) Position 10
Set-49: Puzzles
Directions (239-243): Study the following information to answer the given questions.
There are 5 containers viz. C1, C2, C3, C4 and C5 loaded in a ship one above another, such that C2 is above C1
and C3 is above C2 and so on. Each container is in different heights. Also, all the containers contain different
number of Bikes and Cars in it. Each Container should be unloaded in different ports viz. Mumbai, Kolkata,
Kochi, Kandla and Tuticorin but not necessarily in same order. Total height of all five containers is 666ft.
Note: Total height of a container = (Total number of Bikes + Total number of Cars) in a container.

 Number of Cars to be unloaded in Kochi is as same as number of cars in C2. Container C4 is 44ft taller than
one of the containers placed below it. Height of the container which is headed towards Mumbai port is 141ft
and number of Cars in it is a prime number.
 Number of Cars and bikes in C2 is a multiple of 11. No height of a container is above 150ft and below 100ft.
Number of cars in a container which is headed towards Tuticorin is as same as number of bikes in C2. The
container which is 106ft tall has 29 cars in it.
 Number of bikes in C1 is even and the height of the same container is an odd number. Difference between
number of bikes in the container which is headed towards Kandla and Mumbai port is as same as the
difference between bikes in the container which is headed towards Mumbai and Kochi port.
 There is 88 Cars in one of the containers and the container which is headed towards Kochi is 17ft shorter
than the container which is headed towards Kolkata port. There are two containers between the one which is
headed towards Tuticorin and Mumbai port.

239) How many bikes were shipped towards E. 18 Cars


Kochi port?
A. 38 bikes 241) Which Container is shortest and tallest
B. 71 bikes among the following?
C. 77 bikes A. C5 – C3
D. 62 bikes B. C4 – C1
E. 55 bikes C. C3 – C4
D. C2 – C1
240) What is the difference between the number E. C3 – C2
of Cars in C5 and C1?
A. 21 Cars 242) Which of the following is true?
B. 36 Cars A. Container C1 belongs to Kochi port
C. 26 Cars B. There are 77 bikes in C4 container
D. 12 Cars C. All are true
D. Container C1 contains 67 Cars A. Container C1
E. There is a height difference of 15ft between B. Container C2
container C3 and C5 C. Container C3
D. Container C4
243) Which of the following Containers is headed E. Container C5
towards Kandla port?

Set-50: Seating Arrangement


Directions (244-248): Study the following information to answer the given questions.
Eight cricket team captains viz. P, Q, R, S, T, U, V and W representing different teams Australia, Pakistan, India,
England, Bangladesh, Afghanistan, Kenya and Zimbabwe were sitting in a straight line for a press meet but not
necessarily in same order. All are facing north. Each team got different ODI rankings from 1-9; not more than one
got same rankings. After the press meet two team captains left the place and rest were made to sit in a table as
shown in the fig. Below
Note: 1 is considered as a perfect square and perfect cube number.

 Immediate neighbours of V are facing towards the centre in the table. The Kenya team captain sits at one of
the position right to T in the press meet. The difference between ODI ranking of U and T‟s team is a prime
number.
 There are 3 team captains sitting between T and Afghanistan team captain, who sits at extreme end. Only the
team which got ODI ranking in perfect square faces outside the table. The product of ODI ranks of U and V‟s
team is a perfect cube number.
 P attends press meet at one of the position right to W; the difference between their ODI ranks is the ODI rank
of R. R sits at extreme end and he is not the captain of Afghanistan team. The ODI rank of Afghanistan is 2
positions below W.
 Bangladesh got ODI ranking of 3 and is an immediate neighbour of India and Zimbabwe in Press meet. Only
one person sits between Afghanistan and Zimbabwe‟s team captain in both press meet and table.
 T faces towards centre and Kenya‟s team captain sits right to the T in the table. There are 2 team captains
sitting between T and V in either side of the table. Kenya got the highest ODI rank and the ODI rank of India
is an even number but not 4.
 Zimbabwe got ODI rank of 5 and is an immediate neighbour of Kenya in the table. There is only one cube
number in ODI rankings, which is the rank of P. Zimbabwe team captain W sits third to the right of R, who
represents Pakistan in the table and sits at one of the extreme end during press meet.
 England team captain sits second from the left end and got ODI ranking of 2.In press meet, Australia team
captain sits immediate right of T and there is three people sitting between S and Australia team captain.

244) Who sits second to the right of Australia 246) What is the difference between ODI ranks of
team captain in the press meet? Australia and Afghanistan team?
a) S a) One b) Three c) Five d) Two e) Four
b) The captain of the team which got ODI rank of 5
c) Pakistan team Captain 247) Who sits immediate right of R in the table?
d) T a) T
e) The captain of the team which got ODI rank of 4 b) Kenya team Captain
c) The captain of the team which got ODI rank of 3
245) What is the ODI rank of India and who d) W
represents team India? e) The captain of the team which got ODI rank of 7
a) 6th – T b) 4th – S c) 2nd – P d) 8th – Q
e) 4th – V 248) Who among the following faces outside?
a) Q b) P c) S d) T e) W
Set-51: Input Output
Direction (249-253): Study the following information carefully and answer the questions given below.
An input-output is given in different steps. Some operations are done in each step. No operation is repeated in
next step but it can be repeated with some other operation (as multiplication can be used with addition in step I
and same can be used with subtraction in step II)

As per the rules followed in the steps given above, find out in each of the following questions the appropriate
steps for the given input.
Input:

249) Which of the following letter is obtained in a) A


last step? b) D
c) E e) None of these
d) H
e) None of these 252) How many letters are there between first
block in step IV and second letter in third block
250) How many vowels are there in step II? of step I as in the English alphabet series?
a) None a) One
b) One b) Two
c) Two c) Three
d) Three d) Four
e) None of these e) Five

251) Which of the following letter is immediately 253) How many letters are repeated in step II?
followed by the second letter in third block of step a) One
I? b) Two
a) E c) Three
b) H d) Four
c) G e) None of these
d) F

Set-52: Blood Relation Based Puzzle


Directions (254-258): Study the following information carefully and answer the below questions.
In family of nine members each member works in different bank viz. ICICI, PNB, HDFC, BOB, CBI, SBI, BOI,
BOM and RBI. Each person also likes Different fruits viz. Cherry, Orange, Papaya, Mango, Grapes, Litchi,
Apple, Banana and Guava. Each person was born in different month from January to December. All the
information is not necessary in same order.
There is not more than one month gap between births of two people. Only three married couples are there in the
family. One person was born between the one who likes Cherry and M, who works in ICICI. The one works in
RBI was born in February and is sister of the one who works in PNB. Two people were born between E and the
one who works in BOM, who was born in adjacent month of the one who likes Papaya. P who works in SBI was
born in one of the month having 30 days and was born at a gap of two month from his husband. The one who
likes Guava is sister of the one who likes Apple. D is unmarried member of the family and is sister-in-law of the
one who likes Papaya. The one who works in CBI also likes Cherry. Q is the only daughter of the one who likes
Orange and is married to the one who works in SBI. H was born at a gap of two month from the one who works
in BOB. P‟s wife likes Banana but was not born on adjacent month of the one who works in RBI. The one who
works in HDFC is mother of the one who works in BOM. K is married to P‟s father-in-law and neither likes
Grapes nor works in PNB. The one who works in HDFC was born in May. The one who likes Cherry is only son
of H, who likes Guava. U‟s father works in ICICI and was born in November. The one who works in BOB was
born in adjacent month of the one who works in RBI. One person was born between the one who works in PNB
and U. M is father of the one who likes Cherry. K is sister-in-law of the one who likes Litchi. U‟s mother works
in PNB. One person was born between the one who likes Grapes and his mother. L is only child of the one who
likes Litchi and is grandson of the one who was born in November. The one who likes Papaya is married to H and
is brother-in-law of the one who likes Apple. No person was born between the one who works in BOI and D. Two
people were born between K and the one who works in BOI, who doesn‟t like Guava. L was neither born after
October nor works in BOB. The one who works in RBI likes neither Litchi nor Orange. Three people were born
between L‟s mother and the one who likes Mango.

254) How the one who works in RBI is related to d) E works in CBI was born just before P.
E? e) More than one statement is false.
a) Uncle b) Sister c) Brother-in-law
d) Sister-in-law e) None of these 257) In which of the following month the one who
likes Guava was born?
255) How many person was/were born between a) February
the one who likes Grapes and the one who works b) August
in PNB? c) June
a) One b) Three c) Four d) Two e) None d) December
e) None of these
256) Which of the following statement is not true?
a) The one who likes Orange is married to Q‟s 258) Which of the following combination is true?
mother. a) Apple – February – L – RBI
b) The one who works in BOM is married to the one b) BOI – January – U – Grapes
who was born in May. c) E – August – Orange – SBI
c) The one works in BOB was born in the month just d) CBI – Cherry – August – H
before the one who works in HDFC. e) All the above combination is not true.
Set-53: Input & Output
(Directions 259–263): A string of numbers is given as input. The further steps given are obtained by
applying certain logic. Each step is a resultant of previous step only. Study the following information
carefully and answer the questions given below it.

As per above applied logic in above steps, find appropriate step for given input:
259) What is the difference of sum of digits of 261) What is the difference of second lowest and
highest to lowest number in step II? second highest number obtained in step I?
a) 4 b) 1 c) 7 d) 3 e) None of these a) 13 b) 21 c) 46 d) 35 e) None of these

260) Which of the following number represents 262) What is the lowest number obtained in step
final output? IV?
a) 45 b) 84 c) 36 d) 28 e) None of these a) 7 b) 3 c) 8 d) 4 e) None of these

Data Sufficiency:
Directions: Each of the questions below consists of a question and two statements numbered I and II given
below it. You have to decide whether the data provided in the statements are sufficient to answer the
question:
a) If the data in Statement I alone is sufficient to answer the question, while the data in Statement II alone is not
sufficient to answer the question.
b) If the data in Statement II alone is sufficient to answer the question, while the data in Statement I alone is not
sufficient to answer the question.
c) If the data either in Statement I alone or in Statement II alone is sufficient to answer the question.
d) If the data in both the Statements I and II together are not sufficient to answer the question.
e) If the data in both the Statements I and II together are necessary to answer the question.

263) Seven books of different subjects viz. Math, second position from top. Book having Red cover is
English, Science, Hindi, History, GK and kept at the top. English having Blue cover is above
Geography having different cover says Red, Hindi. History book is kept just below math book.
Green, Blue, Black, White, Yellow and Pink are II. Science is kept third from bottom just below
kept one above other on the self then, then how English having Blue cover. GK book is kept on top
many books is/are kept between Yellow covered at a gap of one place from History book. Science
book and Science book? book having Black cover is kept at any place below
I. Geography is kept at bottom having pink cover. Math book. Geography book having Pink cover is
There is a gap of one place between English and kept at any place below Hindi book.
Hindi, who has cover of Green color. Math is at
Set-54: Alphanumeric Series
Directions (264-268): Study the following information carefully and answer the below questions.
8%DF4@H#1µA>6&U^5M£3ΩQB2≤LI$0R¥9E₹W7
Step 1 – Each such special character that is immediately preceded by a number and immediately followed by a
consonant is to be placed immediate left of 1st, 5th, 9th and 13th element from left end and so till all possible
special character are placed.
Step 2 – After completing step–1, each such digits immediately preceded by a special symbol and immediately
followed by a Consonant should be placed immediate right of 3rd, 7th, 11th 15th and 19th element from the left end
and so till all possible digits are placed in descending order from left to right.
Step 3 – After completing step–2, each such alphabets immediately preceded by a special character and
immediately followed by a number should be placed immediate right of „≤‟.
Step 3 is the final output for the given string.

264) How many such special characters are there 266) Four out of five follows a creation common
in the final series which is/are immediately rule and thus formed a group in final
preceded by a consonant and immediately arrangement. Which of the following combination
followed by a number? doesn‟t belong to same group?
a) Three b) One c) Two d) Four e) None a) F1W b) >&2 c) 6MI d) L^≤ e) AΩ8

265) If all the elements preceded by prime digits 267) How many such alphabet are there in final
are dropped in the final series then which of the arrangement which is/are immediately preceded
following element is eighth to left of thirteenth by a special character and immediately followed
element from right end? by a consonant?
a) W b) % c) A d) Ω e) None of these a) Two b) Four c) One d) Three e) None

Data Sufficiency:
Directions: Each of the questions below consists of a question and two statements numbered I and II given
below it. You have to decide whether the data provided in the statements are sufficient to answer the
question:
a) If the data in Statement I alone is sufficient to answer the question, while the data in Statement II alone is not
sufficient to answer the question.
b) If the data in Statement II alone is sufficient to answer the question, while the data in Statement I alone is not
sufficient to answer the question.
c) If the data either in Statement I alone or in Statement II alone is sufficient to answer the question.
d) If the data in both the Statements I and II together are not sufficient to answer the question.
e) If the data in both the Statements I and II together are necessary to answer the question.
268) Eight friends P, Q, R, S, T, U, V and W like Drawing. S sits immediate left of U. The one
sitting in a circular table facing center each who likes Football sit in-front of the one who likes
having different hobby viz. Painting, Boxing, Badminton, who sits third to left of T. P sits third to
Singing, Badminton, Drawing, Singing, Cricket left of Q.
and Football then, what is the hobby of the II. The one likes Boxing sits immediate right of W.
person who sit second to right of the one having Q sits third to left of S, who sits second to right of
Cricket as hobby? the one who likes Singing. U sits second to left of R,
I. U sits third to right of T, who likes Singing. Q who sits second to left of the one who likes Boxing.
having Boxing as hobby sits second to right of R, The one who likes Dancing sits third to right of the
who sits third to right of S. V sits third to right of one likes Painting, who sits second to right of V. P
one likes Dancing, who sits second to right of P. The sits third to left Q, who like Boxing. T likes Singing
one who likes singing sits second to left of the one and sits third to left of the one who likes Cricket.

Set-55: Seating Arrangement


Directions (269-273): Study the following information carefully and answer the given question:
Eight scientists Ashoke, Berner, Shinya, Jane, Noam, Alan, James, and Blackburn are sitting in a square table
facing center, such that four of them are sitting at the corner of the square table and remaining four persons are
sitting in the middle of the sides in the square table. All the persons are sitting along the perimeter of the square
table at an equal distance between them. The perimeter of the square is 320m and the perimeter of the square is
4a, where „a‟ is the side of the square. Distances are considered only on the perimeter of the square table.
Berner sits 80m away from Jane who is not sits at any corner of the table. Three person sits between Jane and
Ashoke. Noam sits at corner of the table and Alan sits 40m to the right of Noam. Blackburn and Shinya sits
opposite to each other. Alan is not an immediate neighbour of Shinya. Jane doesn‟t sit immediate left of James.

269. Who sits immediate right of James? i) Berner and Blackburn are immediate
a) Berner b) Ashoke c) Alan d) Jane neighbour
e) None of these ii)Ashoke sits third to the left of Noam
iii) James sits second to the right of Shinya
270. How many person sits between Noam and a) Only (i)
Blackburn when counting right of Noam? b) Only (i) and (ii)
a) 2 b) 3 c) 1 d) more than three e) None c) Only (ii) and (iii)
d) Only (iii)
271. Who sits second to the right of the one who e) All (i), (ii) and (iii)
sits opposite to Alan?
a) Shinya 273. Who sits 120m to the left of Berner?
b) James a) Alan
c) Jane b) The one who is immediate left Ashoke
d) Berner c) The one who is second to the right of Noam
e) None of these d) Both (b) and (c)
e) None of these
272. Which of the following statement(s) is/are
not true?

Set-56: Puzzles
Directions (274-278): Study the following information carefully and answer the below questions.
Five friends – Jammy, Jackie, Jubbie, Jaakko and Jasbir ordered a pair of shoes each from online retailer –
„FootBoot‟. Their feet sizes are 7.5, 8, 8.5, 9 and 9.5 but not necessarily in the same order. Among them, Jubbie
and Jasbir have Narrow feet whereas the remaining three have Broad feet. The recommended shoe size (the size
that fits the person) for Narrow feet is either same size as the foot size or half a size greater than the foot size. The
recommended shoe size (the size that fits the person) for Broad feet is either half a size greater than the foot size
or one size greater than the foot size. Each of the five friends ordered a shoe size as recommended for themselves.
While delivering the five pairs of shoes, „FootBoot‟ messed up with their addresses and each one received a pair
that was neither the one that they ordered nor one that was recommended for their feet size.
Also it is known that,
 Jaakko didn‟t order the shoes size received by Jammy.
 Jammy received shoes of size 9.5.
 Jasbir received shoes of size 9.5.
 Jubbie received shoes of size 9.
 Jaakko received shoes of size 8.5.
 Jackie received shoes of size 7.5.

274) For whom among the following the a) 9 b) 8 c) 8.5 d) 7.5 e) Cannot be determined
difference between the sizes of the shoes ordered
and shoes received is Maximum? 277) Who among the following received the shoe
a) Jammy which was ordered by Jaakko?
b) Both Jaakko and Jasbir a) Jammy b) Jaakko c) Jasbir d) Jackie e) Jubbie
c) Both Jackie and Jasbir
d) Jackie 278) Four out of five given combination follows
e) None of these certain rule and thus formed a group. Which of
the following combination doesn‟t belong to same
275) Who among the following ordered Shoes size group?
9? a) Jubbie – Jammy b) Jammy – Jackie
a) Jammy b) Jaakko c) Jasbir d) Jackie e) Jubbie c) Jaakko – Jubbie d) Jasbir – Jackie
e) Jubbie – Jasbir
276) What is the feet size of Jackie?
Set-57: Seating Arrangement
Directions (279-283): Study the following information carefully and answer the below questions.
Eight friends - P, Q, R, S, T, U, V and W plays a game by sitting around a circular table such that the distance
between any two adjacent players is the same and all are facing the centre of the table. The game consists of 2
rounds. In round 2, P & Q, R & S, T & U and V & W interchanged their positions with each other. Also it is
known that,
 In round 1, Q is sitting opposite R.
 In round 2, V is sitting opposite R.
 In round 1, S is sitting 2 places to the left of U.
 In round 2, W is sitting 3 places to the right of S.

279) Who among the following sits third to the a) One b) Two c) None d) More than three
right of P in Round 2? e) Three
a) U b) T c) W d) Q e) None of these
282) If P sits second to the left of R in Round 1,
280) What is the position of U with respect to T, then the position of how many persons remains
in Round 1? unchanged if they were made to sit in
a) Immediate right alphabetical order from right of P (excluding P)?
b) Second to the left a) One b) Two c) None d) More than three
c) Second to the right e) Three
d) None of these
e) Cannot be determined 283) Who is sitting two places to the right of W in
Round 2?
281) How many people are sitting between S and a) V b) T c) R d) Q e) None of these
V, when counted from left of V in Round 1?
Set-58: Puzzles
Directions (284-288): Study the following information carefully and answer the below questions.
Five teams from five different IITs – IIT Bombay (IIT-B), IIT Delhi (IIT-D), IIT Madras (IIT-M), IIT Goa (IIT-
G) and IIT Kanpur (IIT-K) participated in Indian Institute of Tech Champions League (IITCL) played in the year
2018. Each team played exactly one match against every other team. Each win provides two points to the winning
team, each No-Result (i.e. N/R) gains one point to both teams and a loss gains zero. If two teams score same
points, the team with higher run rate gets higher rank. Also it is known that,
 The performance chart (number of won-Lost-N/R matches) for only IIT-M and IIT-K was identical.
 IIT-B didn‟t win any match.
 IIT-D won two matches and lost one in the whole series.
 There was only one match which didn‟t produce any result. Every team scored at least one point.
 IIT-G beat only IIT-B.
 IIT-M topped the list because of its fabulous victory against IIT-K in the last match at higher net run rate.

284) Who among the following has got third rank 287) Which of the following match has No-Result
in the points table? (N/R)?
a) IIT-G b) IIT-D c) IIT-K d) IIT-M a) IIT-G vs IIT-M
e) None of these b) IIT-D vs IIT-G
c) IIT-B vs IIT-D
285) How much points did IIT-M gained in the d) IIT-M vs IIT-B
IITCL series? e) None of these
a) 8 b) 7 c) 12 d) 6 e) None of these
288) Which of the following team has won against
286) IIT-D has lost match against which of the IIT-M?
following team? a) IIT-G
a) IIT-G b) IIT-D c) IIT-K d) IIT-M b) IIT-D
e) None of these c) IIT-K
d) IIT-M
e) None of these
Set-59: Seating Arrangement
(Directions 289–293): Study the following information carefully and answer the questions given below it.
Nine students namely – P, Q, R, S, T, U, V, W and X are sitting in a row in such a way that some are facing north
while other sits facing south. Each student belongs to different grade viz. 4th, 5th, 6th, 7th, 8th, 9th, 10th, 11th and
12th. All the information is not necessary in same order. At least four people sit facing south. One who belongs to
9th grade sits third from either end of the row. P, who doesn‟t belong to 9 th grade, sits second to right of one who
belongs to 4th grade. R, who neither sits adjacent to U nor belongs to 12th grade, sits third to right of one belongs
to 7th grade who neither sit adjacent to U nor sits adjacent to W. W, who belongs to 10 th grade, sits immediate
right of U. T, who doesn‟t sit adjacent to one who belongs to 5th grade, sits second to left of V. X and W sits
facing in opposite same direction. Only two people sit between U and one who belongs to 9th grade. One who
belongs to 8th grade, who doesn‟t sit adjacent to one who belongs to 9th grade, sits second to left of U. One who
belongs to 6th grade sits third to right of Q, who sits facing north. Only two people sit between W and X, who
neither sits adjacent to R nor adjacent to one who belongs to 4th grade. Person adjacent to V sits facing in same
direction but opposite in direction to V. Only three people sit between one who belongs to 6 th grade and one who
belongs to 11th grade. Person sitting at end of the row sits facing in opposite direction. S, who neither sits at end
nor belongs to 9th grade, sits fourth to left of one who belongs to 5th grade.

289) Who sits third to left of one who belongs to c) One who belongs to 6th grade sits second lo left of
11th grade? S.
a) U d) One who belongs to 5th grade sits immediate right
b) W of V.
c) R e) All the given statements are true.
d) P
e) None of these 291) If all the students are rearrange in ascending
order of grade from left to right then how many
290) Which of the following statement is not true? person remains unchanged?
a) One who belongs to 8th grade sits second to left of a) Three
one who belongs to 6th grade. b) One
b) Only four person sits between one who belongs to c) Four
4th grade and R. d) Two
e) None
293) If Q sits second to left of P, then what is the
292) Who sits second to right of one who belongs position of U with respect to one who belongs to
to 12th grade? 8th grade?
a) Q a) Third to left
b) W b) Second to left
c) R c) Immediate right
d) S d) Second to right
e) Can‟t be determined. e) Can‟t be determined

Set-60: Seating Arrangement


(Directions 294–298): Study the following information carefully and answer the questions given below it.
Ten friends P, Q, R, S, T, U, V, W, Y and Z are sitting in a row facing north but not necessary in the same order.
Each person has different profession as Writer, Poet, HR, MR, Doctor, Teacher, Engineer, PO, Lawyer and Clerk.
The distance between any two adjacent persons is not same and person sitting adjacent to each other is at a
distance among 2, 3, 4, 5, 6, 7, 8, 9 and 10 meters but not necessary in the same order.
Distance between Clerk and Lawyer is 26m. S sits third to left of T, who sits third to left of PO. Distance between
Z and MR is 9m. Neither PO nor S sits at end of the row. W sits third to left of V, who is a Teacher and Teacher
sits immediate neighbor of T. The one who is a Writer sits immediate left of W. P sits second to left of Doctor,
who neither sits immediate neighbor of HR nor immediate neighbor of U. Distance between V and T is the
highest while that of HR and Poet is 25m. Only three persons sits between HR and Poet, who sits immediate
neighbor of R. U sits immediate neighbor of the one who is PO and sits third to right of Teacher. Distance
between S and W is 5 meter. The one who is a HR sits third to left of Q, who does not sit at extreme ends of the
line. Q is not a PO. Distance between Poet and Doctor is 13m and distance between Q and R is 16m. Number of
persons sitting between Z and P is same as that between Clerk and Y.

294) What is the position of Poet with respect to b) Fifth to the right
the MR? c) Immediate left
a) Third to the left d) Second to the left
e) Third to the right 297) If the positions of P and R are interchanged,
then what will be the profession of the one who
295) What is the distance between Y and the one sits immediate right of P after rearrangement?
who is a Poet? a) Engineer b) Doctor c) Teacher d) Clerk
a) 15 meters b) 12 meters c) 17 meters e) Poet
d) 18 meters e) 21 meters
298) What is the difference of the distance
296) How many persons are sitting between the between W and T and the one who is Writer and
one who is a Lawyer and Doctor? HR?
a) Five b) Two c) One d) Four e) Three a) 6 meters b) 1 meters c) 8 meters d) 5 meters
e) 4 meters

Set-61: Seating Arrangement


(Directions 299–303): Study the following information carefully and answer the questions given below it.
Eight persons from three different generations of a family A, B, C, D, E, F, G and H are sitting in a row. Some of
them are facing north and others are facing south. Not more than two people facing same direction are sitting
together. There are three married couples. They were born on the same day of the same month of different year
i.e.2001, 1983, 1985, 1953, 1957, 1982, 1987 and 2005 but not necessarily in the same order. Their ages are
considered as on the same month and day of 2017 as their date of births. The distance (in meters) between two
people is a successive multiple of 12. And the distance between two people increased from left to right.
E is the daughter of C. A is the sister of F. The distance between immediate neighbours of any two people should
not be greater than 144m and less than 48m. D is two years elder than her sister-in-law. A is 312m away from F
who is the third eldest person in a family. H faces north and sits third to the right of her son in law. Persons sit at
the two extreme ends face same directions. Number of persons sits to the left of H is two less than that of number
of person sits to the right of B. B who is not a married person, sits 288m away from the eldest person. Only one
person sits to the left of H‟s son in law. D is wife of F and Mother of B. G has two children. G is married to H. A
faces south direction and sits second to the right of B. E was born in 2005 and sits at one of the extreme ends. H‟s
daughter in law is an immediate neighbour of F and B. G and C are immediate neighbours. A‟s father sits second
to the right of E and A‟s father is not C. A is not born in 1985. Immediate neighbours of D belong to same gender.
A‟s age is half the age of D‟s mother-in-law.

299) Who is immediate right of the one who is 301) What is the total distance between B‟s
second to left of E‟s Mother? mother and B‟s Cousin?
a) One who is eldest person in a group a) 84m b) 612m c) 616m d) 638m
b) F‟s father e) None of these
c) One who is third to left of B
d) Both A and B 302) Who are the immediate neighbours of C‟s
e) F Wife?
a) G, H b) A, G c) H, A d) D, B e) D, H
300) What is the sum of ages of H‟s grandson and
G‟s Daughter-in-law? 303) Four of the following five are similar in a
a) 32 years b) 42 years c) 48 years d) 46 years certain way and form a group. Which one doesn‟t
e) None of these belong to the group?
a) F b) H c) E d) C e) D
Answers with Details Explanation:
Set 1:
Directions (1-4):

1) Answer: B  E sits third to left of both I and the one who likes
2) Answer: C Blue colour in the straight line.
3) Answer: D  I is an immediate neighbour of R and sits in the
4) Answer: D extreme end of the line.
 R sits second from the extreme end of the line and
Explanation: likes the colour liked by E in the circle. From this
Note : statement, R and E sits in the outer circle because of
 They were made to sit in a straight line, the variation of colours and faces north in the straight
people sitting in the inner circle faces south and line.
likes same colour.  R is not an immediate neighbour of E, who likes
 The persons sitting in the outer circle faces north Brown colour in the circle.
and likes different colours.
 From the above statements we get the following  The person who likes Yellow faces the person who
possibilities in the straight line. sits immediate left of P in the circle.
 E faces the person who sits to the immediate left in
the straight line.

By Taking Case-I:

Since in Case-I, I faces south; we can conclude that I


sits in the inner circle in Case-I.

 U sits immediate right of T in the circle.


 R and P doesn‟t face each other or sits in the same
circle.
 From the above statements we get the following
possibilities in the circle.

Since the statement- „I faces the person who sits


immediate left of Y, who likes Purple colour. E faces
the person who sits to the immediate left in the
straight line.‟ doesn‟t satisfies the Case-I. This case
 T and O face each other; neither of them likes
gets eliminated.
Yellow colour.
By Taking Case-II:
 Y sits second to the left of the one who likes Blue
colour in the straight line.
 I faces the person who sits immediate left of Y, who
likes Purple colour.
 The person who likes Yellow faces the person who
sits immediate left of P in the circle.
 E faces the person who sits to the immediate left in
the straight line.
 P neither likes Blue or Pink colour while sitting in
the circle.
 The person who likes Blue colour faces the person
Since in Case-II, I faces north; we can conclude that
who likes Black colour.
I sits in the outer circle in Case-II.
From the above statement, we can conclude that Y
likes Purple colour. Since, Y sits immediate left of E
in the straight line; we can conclude that E faces Y in
the circle. Also, we can conclude the person who
likes Yellow colour.
Since, the person who likes Blue colour faces south
in the straight line; we can conclude that the person
sitting in the inner circle likes Blue colour. Also,
from the statement that P doesn‟t likes Blue colour
and The person who likes Blue colour faces the
person who likes Black colour; we can conclude that
T likes Blue colour.

 I faces the person who sits immediate left of Y, who


likes Purple colour.
 The person, who likes Red colour, sits second to the  E doesn‟t like Yellow colour in the straight line.
left of the one who likes Green colour in the straight  Since O likes Green colour in the straight line; we
line. can conclude that the person sitting in the outer
 The person, who faces T in the circle, likes Green circle likes Green colour. Also, we can conclude that
colour in the straight line. P likes Red as it doesn‟t like Pink or Blue colour.
So, we can conclude the Position of P and U in the straight line; we can conclude E likes Black colour in
straight line. As, E doesn‟t likes Yellow colour in the the straight line.
So, the final arrangement is,
Set 2: 123 = (7 x 17 + 4).
Direction (5-8): So, 4th round resembles the position of people in
5) Answer: C 123rd round. R sits second to the left of W.
Since, the position of Round 7 resembles the initial
position; 7) Answer: D
33 = (7 x 4 + 5). Since, the position of Round 7 resembles the initial
So, 5th round resembles the position of people in 33rd position;
round. U sits second to the left of P in Round 5. 99 = (7 x 14 + 1).
So, 1st round resembles the position of people in 99th
6) Answer: B round. P faces S in 99th round.
Since, the position of Round 7 resembles the initial
position;
8) Answer: E
All other rounds except option (e) resemble Round 7.

Explanation:
Let the initial arrangement be,

As per the rules of the game, we found each person


changes their position as follows in each round,

Since, the position of Round 7 resembles the initial


position; Round 8 resembles the Round 1 and so on.

Set 3:
Directions (9-12):
9) Answer: D
10) Answer: E
11) Answer: D
12) Answer: B

Explanation:
From the above information we get,
13) Answer: B
J has 169 Chocolates.
√169  13.
Conditions: J is the Granddaughter of F.
1) Africa x Europe
2) Asia x America 14) Answer: C
3) Germany x Thailand From the above blood relation diagram, we get only
4) Japan x Cuba statement (ii) and (iv) to be true.
5) Ghana + Brazil
6) Cuba + Germany Directions (15-18):

Set 4:
Directions (13-14):

15) Answer: C
16) Answer: B
17) Answer: D
18) Answer: B

Explanation:
The seating arrangement of true and false persons
will be as follows:

U: P is sitting opposite to me.


P: R, who is an immediate neighbor of V is sitting
second to my left.
To consider who is a true person and who is a false R: T is sitting second to my left.(Doesn‟t satisfy)
person, Let us assume the below: Since, U is a true person; P is also a true person. As
per statement of P we find that R is also a true
(i) Considering the Statement of S as True: person. But the statement of R- „T is sitting second
to my left.‟ doesn‟t satisfy.
So, we can conclude that „U is a False Person‟.

(iii) Considering the Statement of R as True:

S: U is sitting second to my left.


U: P is sitting opposite to me.
P: R, who is an immediate neighbor of V is sitting
second to my left. (Doesn‟t satisfy)
Since, S is a true person; U is also a true person. As R: T is sitting second to my left.
per statement of U we find that P is also a true T: S is a false person. (Not sufficient)
person. But the statement of P- „R, who is an Since, R is a true person; T is also a true person. But
immediate neighbor of V is sitting second to my from the statement of T, we don‟t get any
left.‟ doesn‟t satisfy. arrangement further.
So, we can conclude that „S is a False Person‟.
(iv) Considering the Statement of V as True:
(ii) Considering the Statement of U as True:
Also, as W says lies, from his statement-„V is a true
V: Q is sitting opposite to me.
person and he faces U‟.
Q: W is sitting third to my left.
We can conclude that U doesn‟t face V. So we get
W: V is a true person andhe faces U. (Doesn‟t
the final arrangement as,
satisfy)
Since, V is a true person; Q is also a true person. As
per statement of Q we find that W as a false person.
But the statement of W- „V is a true personand faces
U.‟ doesn‟t satisfy.
So, we can conclude that „V and W as a False
Person‟.
From the above assumptions we could end up
finding the list of false and true persons.
False Persons – S, U, V and W
True Persons – P, Q, R and T
From the statements of true persons we get,
P: R, who is an immediate neighbour of V is sitting
second to my left. Set-5:
R: T is sitting second to my left. Directions (19-20):
T: S is a false person. 19) Answer: D
Q: W is sitting third to my left. If a city is an intermediate point along the route, then
Joseph has to enter and exit an equal number of
times and hence there have to be an even number of
roads leading to/from that city. IF any city has an
odd number of roads leading to/from it, then the city
has to be either the starting point or the ending point
(but not the both).
So, from the network of cities given, we could start
and end at either Y or T.
Directions (21-24):

(T – U – V – T – S – Q – R – S – X – V – W – X – Y
– Q – P – Y) 21) Answer: C
The route can be vice versa from Y to T. 22) Answer: E
23) Answer: C
20) Answer: B 24) Answer: C
Other than city U all other cities in the given option Explanation:
has four roads connecting the city. (i) In the outermost wheel, the blue part is at Position
2.
(ii) In the innermost wheel, the orange part is at
Position 4.
(iv) In the middle wheel, the Red part is at Position
6.
(v) The white part in the innermost wheel is to the
immediate right of the Blue part in the outermost
wheel.
From the statements above, we get;
(vii) The Red part in the innermost wheel, the pink (iii) The Pink part in the middle wheel is to the
part in the middle wheel and the yellow part in the immediate Right of Green part in the outermost
outermost wheel were at the same position. wheel.
(viii) Any coloured part in one wheel is neither to the (ix) The Green part in one of the wheels is at
immediate right or left of the same coloured part in Position 1.
another wheel.
(x) The white part in the middle wheel is to the
immediate right of Yellow part in the outermost
wheel.
From the above statements we can fix the position of
Red part in the inner wheel at either Position 8 or
Position 3.
But, as the white part in the middle wheel is to the
immediate right of Yellow part in the outermost
wheel, we cannot fix the Position of Red part at
Position 3.
So, we get

Note: For example, In outermost wheel if white part


is to the immediate right of Pink part, then the
position of white part is to the immediate right of
pink part in the innermost and middle wheel.
Since, in each wheel the colours were in the same
circular order. We can fix the position of all other
parts in the wheel. Thus the final arrangement is,

From this we get only Option (d) as true.

27) Answer: D
After swapping their positions we get,

Set-6:
Directions (25-28):
25) Answer: D
After swapping their positions we get,
Robert is the Son of Q.

28) Answer: E
None is true with respect to the arrangement and
blood relation.
Answer:

So, S sits before T.

26) Answer: D
After swapping their positions we get,

Common Explanation:
Since, the seats are arranged in grid form with three From the above statement, we can conclude that W
rows and three columns, the seats were numbered as is the sister of Robert as already we have concluded
follows: S as his wife.

(i) Since, Robert drives the van, he occupies the seat


numbered 3 having R and Y in his column.
(iii) E, the son of S occupies the seats in the same
row as T and Y, who is his only sister.

(ii) Robert and his father-in-law are not seated in the


same row.
(vi) E and his sister are seated with his maternal
grandmother.
From the above statements, we can conclude that T
is the mother-in-law of Robert and R is the father-in-
law of Robert.

(iv) Q occupies seat numbered 5 and is in same row


as his husband A and is not seated immediately in
front of Robert‟s son E.
(v) Q, the mother of Robert and his daughter are not
seated in the same column.
(ii) 8th and 1st digits are an even number.
From this statement, we can eliminate Cs-1, Cs-
2,Cs-3,Cs-4,Cs-5,Cs-8 and Cs-9.

Set 7:
Directions (29-32):

29) Answer: C
30) Answer: D
31) Answer: B
32) Answer: C
Explanation:
(iii) The Number formed by 7rd and 8th digits is a
product of two consecutive natural numbers.
(vi)TheNumbers formed by 7th and 8th digits is six
less than the number formed by 1st and 2nd digit.
Products of two consecutive natural numbers are
02, 06, 12, 20, 30, 42, 56, 72 and 90.
(iv) Sum of the two-digit numbers formed by 7rd and
From the above two statements we get the following
8th digits, and 1st and 2nd digits is a product of two
possibilities,
consecutive natural numbers.
From this statement, Cs-6i and Cs-6iii gets
eliminated as the 4th and 7th digit is 64 which is a
square number.

Products of two consecutive natural numbers are


02, 06, 12, 20, 30, 42, 56, 72, 90, 110, 132 and 156.
Here we get,
48+42 = 90, this is a product of consecutive number.
62+56 = 118, this is not a product of consecutive
number.
So, we can conclude the 1st, 2nd, 7th and 8th digit of
the roll number.

(vii) In the number, three digits have been used two


times while other two digits have been used only
once.
(i) The Two digit number formed by 3rd and 4th
From the above statements,
digits respectively is square of the number formed by
Only in Cs-6ii we have three digits appeared twice in
5th and 6th digits.
the number. So, all other case gets eliminated.
The square of a number cannot be more than 81
since the square value should be only two digits.
Also, zero doesn‟t come more than once in the
number we can directly omit the squares of 1 to 3.
We get the following possibilities,

So, the Roll number of Mr. Dan Brown is,

Set 08 :
Directions (33-36):

(iv) The Two digit number formed by 4th and 7th


digits respectively is not a perfect square.
(vii) Airline from Country G goes to Country F
33)Answer: C (ix) Airline from Country B goes to Country G
34) Answer: B (ii) Country H receives airline from Country F
35)Answer: D From the above statements we get,
36) Answer: C
Explanation:
(i) Airline-B goes to Country E
(iii) Airline-H belongs to Country A
(v) Airline-D belongs to Country H
(vi) Airline-I belongs to Country C
(viii) Airline-F goes to Country I
(xi) Airline-E goes to Country A
From the above statements we get,
Country C. So we can conclude that Airline-B
belongs to Country D.
(3) Since, only Country E is left we can conclude
that Airline-F belongs to Country E.
(4) We could also find the Country where Airline-D
goes, which is D. Similarly we could find the
Country of the Airlines H and I.

From the given information,

(iv) Only one Airline whose name and the alphabet


of the country it belongs are same.
We can conclude that,
(x) Only one Airline whose name and the alphabet of
(1) Since, Airline-E goes to Country A, it should
thecountry it goes are same.
belong to eitherCountry C or I or B. Already we has
(xii) Airline-A belongs to a country which has
concluded the Airlines belonging to Country B and
Country A as his neighbour.
Country C. So we can conclude that Airline-E
From the above statements we get,
belongs to Country I.
(1) Since, only one Airline whose name and the
(2) Since, Airline-B goes to Country E, it should
country it belongs to are same it could be G.
belong to either Country D or I or C. Already we has
concluded the Airlines belonging to Country I and
(2) From, the given information we can conclude S – T – V – P24 km
that Country B is the neighbour of Country A.So, the S – T – V – X – P25 km
Airline-A belongs to Country B. S – T – V – W – X – P  31 km
So, the final arrangement is, S – T – V – Q – P  31 km
S – R – T – V – P  32 km
S – R – T – V – X – P 33 km
S – R – T – V – W – X – P  39 km
S – R – T – V – Q – P  39 km
Charge:
Fixed Charge
Service Charge
Transport Charge- Rs.5 x 33 km – Rs.165
Total

Set 09 :
Directions (39-40):
39) Answer: D
Explanation:
From the given statements, we can infer that the
person should fit the profile to be CFO. Also, we
could assume that Nilanjan Roymight be a proper
replacement for JayeshSanghrajka. From the
paragraph we can conclude that Infosys Ltd needs a
CFO.
So, Option (D) is the correct choice.

40) Answer: C
Directions (37-38):
Explanation:
37) Answer: C
From the given statements, we can assume that the
The shortest route to travel from P to R is
cost of living in Mumbai is high when compared
P – V – U – R and the distance between them is
with National Capital; as the fuel prices are high it
(6+6+4)= 16 km
affects the local commodity market in large scale
Charge:
which in turn increases the cost of living. Also, we
Fixed Charge
can infer that it leads to increase in price of certain
Service Charge
commodities.
Transport Charge- Rs.5 x 16km – Rs.8
Among the given statements, none of the statement
Total
could be the conclusion of the above paragraph.
So, Option (C) is the correct choice.
38) Answer: B
The Possible routes to travel from S to P is Directions (41-44):
41) Answer: B
Explanation:
From the above paragraph, only option (b) can be
inferred. All other statements were opposite to the
context.

42) Answer: B
Explanation:
From the above paragraph we can conclude that use
of Imidacloprid brings negative effect in the lifecycle
of Bumblebees. So, Option (b) is the conclusion.

43) Answer: B
46) Answer: D
Explanation:
Explanation:
The statement says that it is really fascinating that
the use ofImidacloprid helps us to study the
mechanism behind the patterns of Bumblebee
lifecycle. Only option (b) is relevant and can be
concluded from the paragraph with respect to the
given statement.

44) Answer: C
Explanation:
From the above Paragraph, we can conclude that
using of Imidacloprid alters the lifecycle of
Bumblebee which in fact reduces the growth and We have:
reproduction of Bumblebee. So, only option (c),
could be the course of action that should be taken. Clearly, required sum of digit = (2 + 0 + 8 + 6) = 16.

Set 10 : 47) Answer: C


Directions (45-48): Explanation:
45) Answer: B
Explanation:
Clearly, option B represents „208‟ in given code
language as follow:
53) Answer: A

48) Answer: D We know that,


Explanation: The length of Line 1 is 462
Distance between 2nd and 1st person is multiple of 11
and consecutive number from left to right.
The total person is 8-1=7, which is the number of
difference distance in a line.
The centre of the two person distance is 462/7=66
Or another way,
X+X+11+X+22+X+33+X+44+X+55+X+66=462
7X+231=462
X=33
(i) The distance between K and M is not even
numbered value in line 1 but both the persons are
sitting immediate neighbours.
(ii) N is not an immediate neighbour K. Consecutive
alphabet persons not sit together.
(iii) All the person position in line 1 is same as in
line 2.
(iv) In line 1, the sum of the distance between N and
I and the distance between O and J is same as K and
M.
(v) N sits third to the right of O.

Set-11:
Direction (49-53):
49) Answer: D
50) Answer: E
51) Answer: B
52) Answer: E
Case (ii) and (iv): does not follow the condition
(viii).
Case (iii): does not follow condition (vii) and (vi),
because the length of line 1 is smaller than line 2.
1. And PI (line 1)=JO+NI (line 2).
2. The numbers of persons sit between J and N is 3.
3. So, we get, OJ=X, JL=X+13, LN=X+26,
NI=X+39.
4. Distance between OJ=X; and NI=X+39
5. So, we get, 143=X+X+39
X=52.

JO+NI=KM
So, KM distance value is 99m, there is no another
way.
(vi) The length of the line 1 is smaller than line 2. The distance of P and I is 143,
(vii) The distance between P and I in line 1 is same X+X+13+X+26+X+39+X+52+X+65+X+78
as the sum of the distance between J and O and the 52+65+78+91+104+117+130=637m
distance between N and I in line 2.
(viii) Two persons sit between N and M.
Set-12:
Direction (54-58):
54) Answer: A
55) Answer: E
56) Answer: C
57) Answer: C
58) Answer: E

(i) The one who is studying in ECE department sits (v) And the train turn left from the station C to reach
third to the left extreme end of the line. the fifth station B.
(ii) The train to reach the second station D at (vi) Again turn left from the station B to reach the
7:12a.m., where, Ranjith get into the train but not sits sixth station G at 8:16:30a.m.
immediate left of the one who is studying in IT (vii) Rajesh studying in the Chemical department
department. and waiting in the station B and sits third to the right
(iii) And the train turn right from the station D to of Rahul who is not studying in ECE department.
reach the third station H at 7:36a.m., where the one (viii) The distance between fifth and sixth station is
who is studying in EEE department get into the train. 12km.
(iv) And the train turn right from the station H to (ix) Preethi and the one who studying in EEE
reach the fourth station C at 7:48a.m., where Priya department are sitting together but both are not sits
get into the train but not sits any extreme ends of the immediate neighbour of either Rahul or Rajesh.
line. (x) The one who is studying in Mechanical
department sits immediate left of Rahul.
The train starts at 7a.m. from the station A. So, that we could the distance between each stations
The train takes 3min to cover 2km. as,

From the station To reach the station Time taken Covered the
distance
A at 7a.m. D in 7.12a.m. 12min 8km
D at 7.12a.m. H in 7.36a.m. 24min 16km
H at 7.36a.m. C in 7.48a.m. 12min 8km
C at 7.48a.m. B in 7:58:30a.m. 10min 30sec 7km
B at 7:58:30a.m. G in 8:16:30a.m. 18min 12km

(xi) The train turn left from the station G to reach the seventh station E at 8:46:30a.m. while Preethi get into the
train and sit second to the left of Priya.
Case III and IV: does not follow condition (IX) (xiv) The one who is studying in Civil department
(xii) Three persons sit between the one who waits in waits in the station F. The train turns left from E.
the station E and the one who waits in the station D. (xv) Prisha and Ram not wait in either station G or F.
(xiii) The one who is studying in Automobile (xvi) Prisha is not waits in the starting point.
department sits third to the left of the one who is (xvii) The train reach the last station at 8.57a.m.
studying in IT department.

Case ii: Does not follow the condition (xii)


From the station To reach the station Time taken Covered the distance
A at 7a.m. D in 7.12a.m. 12min 8km
D at 7.12a.m. H in 7.36a.m. 24min 16km
H at 7.36a.m. C in 7.48a.m. 12min 8km
C at 7.48a.m. B in 7:58:30a.m. 10min 30sec 7km
B at 7:58:30a.m. G in 8:16:30a.m. 18min 12km
G at 8:16:30a.m. E in 8:46:30a.m. 30min 20km
E at 8:46:30a.m. F in 8.57a.m. 10min 30sec 7km

Set 13 :
(Directions 59–62):
We have:

Step I: In this step following logic is applied: Clearly, in step II result can be determined by
difference of square of digits.
Step III: In this step following logic is applied:

Clearly, result in step I can be determined by


resultant of above results.
Clearly, result in step III can be determined by
Result = (113 – 24) = 89
difference of results.
Step II: In this step following logic is applied:
Result = (42 – 5) = 37
Step IV: In this step following logic is applied:
61) Answer: C
Explanation:
Clearly, required sum = (2 + 7 + 8 + 9 + 5 + 9 + 5 +
1) = 46
Hence, option C is correct choice.

Clearly, result in step IV can be determined by 62) Answer: E


difference of results. Explanation:
Thus, final output is, Clearly, desired output in step IV = 07
Result = (58– 53) = 05 Hence, option E is correct choice.

From above logical steps we get following results for


given input:
Set 14 :
(Directions 63–66):
We have:
Player  G%P
Above statements are coded as follow:
First letter is the code equivalent to difference of
place value of 2nd letter from left and 2nd letter from
right.
For E.g.:- L = 12; E = 5;
G = (12 – 5) = 7
Symbol is code equivalent to number of letters in a
word.
For E.g.:- @ = 4; # = 5; % = 6 and $ = 7.
Last letter of the code is coded as:-
If number of letters in the given word are even, then
(Last letter – 2).
If number of letters in the given word are odd, then
59) Answer: C (Last letter + 2).
Explanation: Similarly, we have:
Clearly, difference of highest to lowest number = (56 World  C#F
– 17) = 39
Hence, option C is correct choice. 63) Answer: C
60) Answer: A Explanation:
Explanation: We have:
Clearly, lowest number in step III = 14 Smart  E#V
Thus, required difference = (16 – 1) = 15 Project  O$V
Hence, option A is correct choice. Hence, option C is correct choice.
64) Answer: E
Explanation:
We have:
A#V  Count/Mount
I#P  Token
I$A  Victory
Hence, option E is correct choice.

65) Answer: B
Explanation:
Clearly, A is north-west of H.
Genius  P%Q
Hence, option D is correct choice.
Mind E@B
Power  J#T
68) Answer: C
Hence, option B is correct choice.
We have:
66) Answer: A
Explanation:
We have:
Monthly  C$A
Banker  D%P
Hence, option A is correct choice.

(Directions 67–68):
67) Answer: D Clearly, I is South-East of B.
We have: Hence, option C is correct choice.
From above given statements we have:

Set 15 :
Directions (69-73):
 Two persons sit between E and the one whose age is 18. Person those sit at corner are only in odd
number age. C sits second to the left of the one who sits opposite to the one whose age is 18.

 D sits second to the left of F. Neither F nor D age is 18.

 Case 2 will be dropped because D sits second to the  H+E=10+3(A)


left of F.  Among the given age, only one case is possible.
 A sits second to the right of the one who is eldest.  36+19=10+3(15)
Sum of H and E is 10 more than thrice the age of A.

 Case 1A will be dropped because person sit at corner age is only odd number age.
 Case 1C will be dropped because Age of A=18 is not possible.
 Sum of B and F is two less than D.
Step IV: 35 tubsu 14 cbdl squats 67 93 dwodbsdc 80
oqdrdms
Step V: trvbut 35 tubsu 14 cbdl 93 dwodbsdc 80
oqdrdms 66

74) Answer: b)
75) Answer: c)
76) Answer: d)
77) Answer: a)
78) Answer: d)

69) Answer: c)
Set: 17
70) Answer: b)
We have:
71) Answer: d)
 ¥ row contains numbers which are consecutive
72) Answer: d)
multiple of „9‟ starting from „36‟.
73) Answer: d)
 £ row contains numbers which are consecutive
multiple of „13‟.
Set: 16  & row contains numbers which are consecutive
Directions (74-78): multiple of „8‟ starting from „32‟.
Lowest number of letter in a word is arranged in left  % row contains numbers which are consecutive
end in step I and is replaced with its next letters and multiple of „7‟.
highest number is arranged in right end in step I with
 @ row contains numbers which are consecutive
number-1.
multiple of „11‟.
Lowest number is arranged in left end with
number+1 in step II and highest number of letter in a
From above given statements we have:
word is arranged in right end in step II and is
replaced with its previous letter.
Second lowest number of letter in a word is arranged
in left end in step III and is replaced with its next
letter and second highest number is arranged in right
end in step III with number-1.
Above same logic is applied for other steps.
Input: 94 squats 81 expected 13 back 34 present 67
start
Step I:cbdl squats 81 expected 13 34 present 67 start
93
Step II: 14 cbdl squats 81 34 present 67 start 93 79) Answer: C
dwodbsdc Explanation:
Step III: tubsu 14 cbdl squats 34 present 67 93 We have:
dwodbsdc 80 Y = £B &E £A @D
After placing value of strings from given matrix, we
have:
Y = 26 64 13 44
Clearly, a prime number is preceded by a perfect
square then, outcome of the string is product of
digits at tenth place.
Thus, Y = (2 x 6 x 1 x 4) = 48
Clearly, condition I follows. Thus, bulb „Q‟ blinks.
Hence, option C is correct choice.

We have:
80) Answer: A
 The box having Rose is kept second to left of box L.
Explanation:
We have:  The box having Ring is kept just below box L, that
X = ¥C @D £B @B means we have four possible place for box L, in case
After placing value of strings from matrix, we have: (1) box L is kept at 1&, in case (2) L is kept at 1#, in
X = 54 44 26 22 case (3) L is kept at 2& and in case (4) L is kept at
Clearly, all the numbers of string are even numbers. 2#.
Then, outcome is the product of digits at one‟s place.  Box J is kept besides the box having Ring.
Thus, X = (4 x 4 x 6 x 2) = 192  The box J has five neighboring boxes, that means
Clearly, condition IV follows, thus bulb R blinks. case (2) is not valid, in case (1) J is kept at 2#, in
Hence, option A is correct choice. case (3) J is kept at 3%, in case (4) J is kept at 3&.
 Box J having Pen is kept exactly between box having
Ball and box C, that means we have four possible
(Directions 81–83): place for box C, in case (1a) box C is kept at 3#, in
case (1b) box C is kept at 1#, in case (3) box C is
kept at 3&, in case (4) box C is kept at 3#.
 The box having Mango is kept exactly between the
box C and the box B. Box B neither contains Ball
nor Rose, that means in case (1a) box B is kept at
3%, in case (1b) 1&, in case (3) box B is kept at 1&,
in case (4) box B is kept at 1#.
 Based on above given information we have:
Case (2) is not valid as The box J has five the box having banana is kept at 3@, in case (3) the
neighboring boxes. box having banana is kept at 1#.
Again, we have:  The box having Banana is kept exactly below box D,
 The box having LED is kept exactly between box that means case (3) is not valid and in case (1a) box
having Toy and box B. D is kept at 2@.
 Box having Banana is kept adjacent to box B, that  Box D is kept second to left of box E, that means
means case (1b) & case (4) are not valid, in case (1a) box E is kept at 2&.
Based on above given information we have:

Case (1b) & case (4) as box having Banana is kept Again, we have:
adjacent to box B and case (3) is not valid as the 
box having Banana is kept exactly below box D. hree boxes are kept neighbor of box I, which neither
contains Ball nor Rose, that means box I contains neither contains Mango nor kept adjacent to box
Banana. having Ball.
  B
ox I is kept at a gap of one from box G, that means ox H is kept adjacent to box A.
we have two possible place for box G, in case (1a) 
box G is kept at 1@, in case (1c) box G is kept at ox K is kept exactly between the box having Book
3&. and box A, that means in case (1a) box D contains
 Bat and box H contains LED, thus case (1a) is notT
he box having Watch is kept at a gap of one from valid, in case (1c) box D contains Bat and box H
box G, that means in case (1a) the box having Watch contains Rose.
is kept at 1&, in case (1c) the box having Watch is Based on above given information we have final
kept at 3&. arrangement as follow:
 T
he box having Bat is kept adjacent to box H, which

Case (1a) is not valid as Box K is kept exactly between the box having Book and box A.

81) Answer: C Clearly, only statement I and III are true.


Explanation: Hence, option E is correct choice.
Clearly, Box A having Rose is placed at „1@‟.
Hence, option C is correct choice. Set-18:
84) Answer: B
82) Answer: E Explanation:
Explanation: We have:
Clearly, box K contains LED. 13
Hence, option E is correct choice. 17 21 64
For 2nd row:
83) Answer: E  An odd number (prime number) is followed by odd
Explanation: number (non-prime number) so, 17 x 21 = 357.
 Now, the row is 357 64, an odd number (non-prime 
number) followed by a cube number so, 357 + 64 = hus, resultant of row 2 = 1512.
421. 
 As, resultant of difference of both rows = 362 hus, required difference = (1641 – 1512) = 129.
 Thus, resultant of row 1 = (421 – 362) = 59. 
 For 1st row: ence, option E is correct choice.
 An odd number (prime number) is followed by
another odd number (non-prime number) so, 64 – 13 86) Answer: B
= 51. Explanation:
 Now, the row is 51 X, odd number (non-prime We have:
number) followed by X to get resultant 59. 28 9 53
 Thus, 51 + 8 = 59, this follows condition 5. 21 36 X
 Thus, required value of X = 8. For 1st row:
Hence, option B is correct choice.  An even number followed by perfect square so, 28 x
9 = 252.
85) Answer: E  Now, the row is 252 53, an even number followed
Explanation: by an odd number (prime number) so, 252 + 53 =
We have: 305.
32 49 73  Thus, resultant of 1st row = 305.
15 27 36  For 2nd row:
For 1st row:  Clearly, value of X = 305/5 = 61.
  An odd number (non-prime number) followed by anA
n even number followed by perfect square, so 32 x even number, so 21 x 36 = 756.
49 = 1568.  An even number followed by an odd number (prime
 number) so, 756 + 61 = 817. N
ow, the row is 1568 73, an even number followed  nd
Thus, resultant of 2 row = 817.
by odd number (prime number) so, 1568 + 73 =  Hence, option B is correct choice.
1641.
 87) Answer: A T
hus, resultant of row 1 = 1641. Explanation:
 We have: F
or 2nd row: CREW RISK MART COLD FARM
 After rearranging all the letters within words, we get:A
n odd number (non-prime number) followed by a WREC SRKI TRMA OLDC RMFA
cube number so, 15 + 27 = 42. Thus, “SRKI” appears third in dictionary.
 Hence, option A is correct choice. N
ow, the row is 42 36, an even number is followed
by a perfect square so, 42 x 36 = 1512.
88) Answer: A
Explanation: Seventh letter from left end  F
We have: Sixth letter from right end  R
CREW RISK MART COLD FARM Thus, number of letters between „F‟ and „R‟ in an
After rearranging all the letters in alphabetical order alphabetical series = 11.
we get: Hence, option B is correct choice.
AACCDEFIKLMMORRRRSTW

Set-19:
Direction (89-93):
89) Answer: d)
90) Answer: c)
91) Answer: a)
92) Answer: b)
93) Answer: c)

Rack Box Colour Balls


8 H Red 53
7 C Indigo 36
6 E Orange 19
5 A Blue 29
4 F Violet 25
3 B Yellow 13
2 D White 49
1 G Green 47

 Two boxes are kept between box E and the Yellow coloured box, which is kept at one of the odd numbered rack
but not in bottom most rack. Yellow coloured box is kept below the box E. There are 20 balls difference between
the boxes was kept at the rack 5 and rack 2, which has square number of balls.

Case-1 Case-2
Rack Box Colour Balls Box Colour Balls
8 E
7
6 E
5 29 Yellow 29
4
3 Yellow
2 49 49
1

 Three boxes are kept between the box F and the Red  Two boxes are kept between Red coloured box and
coloured box, which is kept immediate above the box A. Box A is kept immediately above the box
box which has the balls in multiple of 12. The box F which contains square number of balls in odd
is kept in one of the even numbered rack. number.

Case-1(a) Case-1(b) Case-2(a) Case-2(b)


Rack Box Colour Balls Box Colour Balls Box Colour Balls Box Colour Balls
8 F Red E E Red
7 A 36 36
6 E 25 E F
5 29 A 29 A Yellow 29 A Yellow 29
4 Red F 25 25 F 25
3 Yellow 36 Yellow
2 49 49 Red 49 49
1 36

 The White coloured box is kept immediately below  Then, the White coloured box must has second
the box B. As many boxes are kept below the White highest no. of balls and the maximum no. of ball in
coloured box is same as above the box C. The White the any box is 53.
coloured box has only four balls less than the box  So, Case-1(a) and Case-2(a) will be dropped.
which has maximum number of balls.
 From the above statements, (Max. num of balls –
Balls in White box)=4.

Case-1(a) Case-1(b) Case-2(a) Case-2(b)


Rack Box Colour Balls Box Colour Balls Box Colour Balls Box Colour Balls
8 F Red E E Red
7 A C 36 C 36
6 E 25 E F
5 29 A 29 A Yellow 29 A Yellow 29
4 Red F 25 25 F 25
3 Yellow 36 B Yellow B
2 49 White 49 Red 49 White 49
1 36

 H has the maximum number of balls. The Orange  As many boxes kept between box H and the Orange
coloured box has 10 balls less than box A and kept coloured box is same as between box B and the
one of the even numbered racks. The Blue coloured Green coloured box.
box is immediately above the Violet coloured box.  So, Case-2(b) will be dropped.

Case-1(b) Case-2(b)
Rack Box Colour Balls Box Colour Balls
8 H Red 53 E Red
7 C 36 C 36
6 E Orange 19 Orange
5 A Blue 29 A Yellow 29
4 F Violet 25 F Blue 25
3 B Yellow B Violet
2 White 49 White 49
1 Green H 53

 The difference between the balls in the Yellow coloured box and box D is equal to the number of balls in the box
C. Box D is kept above the box which contains 47 balls.

Case-1(b)
Rack Box Colour Balls
8 H Red 53
7 C Indigo 36
6 E Orange 19
5 A Blue 29
4 F Violet 25
3 B Yellow 13
2 D White 49
1 G Green 47

Set: 20 Direction (96-97):


Direction (94-96):

96) Answer: e)
97) Answer: b)

Set -21:
Direction (99-103):
94) Answer: d)
99) Answer: c)
95) Answer: b)
100) Answer: c)
96) Answer: a)
101) Answer: a)
102) Answer: d)
103) Answer: b)
Only two persons are sitting between A and H (Either from left or right side), who sits in the even numbered
corner but not 4. F sits second to the left of G and both of them are not an immediate neighbour of A.

As many persons sitting between F and D is same as between C and D. B sits second to the right of D.
So, Case-1(b) and Case-2(b) will be dropped.
 E is not an immediate neighbour of G.
 So, Case-2(a) will be dropped.
 The person who was sitting in middle of the side-34 sits third to the right of the person who was sitting at the
corner-1. Only one person sits between the persons who are sitting in corner-1 and corner-2.
 Hence, G sits third to the right of E. Only one person sits between E and H.

 The person who was sitting at the corner-3 sits second to the left of the person who was sitting at corner-4, who is
an immediate neighbour of the one who was sitting at corner-2.
 Hence, D sits second to the left of B. B is an immediate neighbour of H.

 The one who was sitting at middle of the side-31 sits  Hence, F sits third to the left of A.
third to the left of the person who was sitting at  So, Case-2 will be dropped.
opposite to the person who sitting at middle of the
side-31.
Set: 22
Direction (104-108):

104) Answer: d)
105) Answer: c)
106) Answer: e)
107) Answer: b)
108) Answer: a)
Set: 23
Direction (109-113):

 E sits opposite to the person who sits second to the  G sits third to the right of the one who likes Green in
left of the one who likes Green, who sits one of the the linear row. G sits opposite to the person who sits
extreme ends of the row. immediate right of H in the circle and both of them
 H faces the person who likes Red in the inner circle faces opposite directions.
but none of them sits opposite to E, who does not sit
in the same circle of H sits.

 A faces the one who likes Violet in the circle and likes Red in the circle. The person who likes Blue
both of them are does not sit in the same circle. sitsimmediate right of G in the linear row.
 The one who sits opposite to A in the same circle sits  In the linear row, only one person sits between G and
third to the left of the one who likes Blue. B likes the one who sits immediate right of G in the circle.
Yellow and does not sit opposite to the one who
 The one who likes Red sits fourth to the right of E, who is an immediate neighbour of G in the linear row.
 The one who likes White sits to the immediate right of F in the circle. The one who likes Orange sits third to the
left of F in the linear circle.

 Only three persons are sitting between H and C in  The persons sitting in the extreme ends in the linear
the linear row. A sits to the left of F in the linear row row facing the same direction as A in the circle.
but not immediately.  The one who likes Yellow does not faces the same
 So, Case-2 will be dropped in the linear row. direction as D.
109) Answer: d)
110) Answer: a)
111) Answer: c)
112) Answer: a)
113) Answer: e)
120) Answer: b)
Set: 24 121) Answer: d)
114) Answer: c) 122) Answer: c)
123) Answer: b)

Home -*$3* (Condition ii applies)


Ministry - 3#4#8975 (No conditions apply)

115) Answer: e)
Escalate - %83@2@98 (Condition iii applies)
Module - *$4&2* (Condition ii applies)

116) Answer: a)
Indices - 844#3%# (Condition i applies)  The one whose age is multiple of 11 sits second to
Can - 3@4 (No conditions applied) the right of T and either one of them sits at the
Wide - *#4* (Condition ii applies) extreme ends of the row.
 Here, 44 years is the only choice for multiple of 11
117) Answer: b)
and the one who has 44 years not possible to sit in
Only „after year‟ will represented by the code
the middle of the row. If we make the person who
„5%@7 759%@‟.
sits at the middle of the Row-1 has 44 years, then the
person sits at the extreme right end must be has 52
118) Answer: b)
years.
Distributed - 4#897#2&9%4 (No conditions applied)
 The immediate neighbour of T faces the person who
has the age in multiple of 5.
Set: 25
Direction (119-123):
119) Answer: e)
 Only one person sits between B and C, neither of them sits at the extreme ends of the row. The one whose age is
multiple of 10 sits second to the left of P and faces the immediate neighbour of D.

 The one whose age is thrice the age of S sits third to the right of S. The one whose age is multiple of 9 sits to the
immediate right of D.
 So, Case-2(a) and Case-2(b) will be dropped.
 The difference between the ages of T and E is 3 multiple of 4. S does not face the immediate
years. R is elder than D. A‟s age is not an odd neighbour of B.
number. D‟s age is an even number but not in  So, Case-1(a) will be dropped.

Set: 26 125) Answer: d)


Direction (124-128): 9.40 am+35 minutes=10.15 am=>&^
124) Answer: a)
Train departs at a station at **->6.30 am. It takes 126) Answer: c)
190 minutes to reach destination which means 3 495/48=10 hours 15 minutes->&^
hours 10 minutes. 6.30+3.10->9.40->@+
127) Answer: e)
9.15am+20+10=9.15am+30=>9.54am=@@
128) Answer: e) He has to reach the station 10 minutes earlier which
Departure time of train is 10.50 am. means that he should be in the station at 7.10 am

Set-27:
Directions (129-133):
129) Answer: C
130) Answer: B
131) Answer: D
132) Answer: B
133) Answer: B

Explanation:
From the given information, we can create the following table;

In week 1, Since, Book 3 cannot be read without reading Book


2; R reads Book 4.
In week 2, 135) Answer: E
T and Q continue to read the same book; as Book 2 Explanation:
and 6 takes two weeks to complete it. Also P cannot We have:
read book 3 as one must read Book 2 before reading 34265, 37596, 48367, 65284, 43827
Book 3. After subtracted „2‟ is subtracted from 1st, 3rd& 5th
In week 3-6, digits and „1‟ is subtracted from 2nd& 4th digits, we
Each exchanges their books as per numerical order. have:
In week 7, 13053, 16384, 27155, 44072, 22615
R and T needs Book 2, but P took the book already. Thus, only one such number are possible = 16384.
Without reading Book 2 he cannot read Book 3 and Hence, option E is correct choice.
also he read all other series of books. So, R and T
have no books to read. 136) Answer: A
In week 8, Explanation:
S needs Book 6 but it is with Q, so he has no books We have:
to read. P still has Book 2 so R and T also have no 34265, 37596, 48367, 65284, 43827
books to read. After interchanging 1st and 3rd& 2nd and 5th digits we
In week 9, have:
Still Q reads Book 6, so S has no books to read. Also 25364, 56397, 37468, 24685, 87423
R takes Book 2 so there is no book for T to read. 2nd lowest number after rearrangement = 25364
In week 10, 2nd highest number after rearrangement = 56397
P and Q have read all the books. So, there are no Required difference = (56397 – 25364) = 31033
books left for them to read. Also R takes Book 2 so Hence, option A is correct choice.
there is no book for T to read.
137) Answer: A
Set-28: Explanation:
Directions (134-138): We have:
134) Answer: B 34265, 37596, 48367, 65284, 43827
Explanation: After dividing “34265” by „6‟ we get  5
We have: After dividing “37596” by „9‟ we get  3
34265, 37596, 48367, 65284, 43827 After diving “48367” by „8‟ we get  7
After rearranging digits within 5-digt number we After dividing “65284” by „8‟ we get  4
have: After dividing “43827” by „8‟ we get  3
23456, 35679, 34678, 24568, 23478 Thus, only one such number is possible  65284
2nd lowest number after rearrangement = 23478 Hence, option A is correct choice.
Sum of digits of “23478” = 24
2nd highest number after rearrangement = 34678 138) Answer: C
Sum of digits of “34678” = 28 Explanation:
Required difference = (28 – 24) = 4 We have:
Hence, option B is correct choice. 34265, 37596, 48367, 65284, 43827
20,
30,  Three people sits between Anuj and the one whose28,
weight is 48 kg, who neither sits adjacent to Payal
nor sits adjacent to the one whose weight is 84 kg.
Set-29:  One person sits between the one whose weight is 48
Directions (139-143): kg and the one whose weight is 68 kg.
 The one whose weight is 68 kg and Kavita sits
together at longer side, that means we have six
possibility for Kavita, in case (1a) Kavita sits
immediate right of Anuj, in case (1b) Kavita sits
immediate right of the one whose weight of 48 kg, in
case (1c)& case (2c) weight of Kavita is 84 kg, in
case (2a) Kavita sits third to left of Anuj, in case (2b)
Kavita sits immediate left of Anuj.
 Two person sits between Kavita and Dev whose
We have: weight is 56 kg, that means in case (1a) & case (1b)
 The one whose weight is 84 kg sits at longer side at a Dev sits immediate left of the one whose weight is
gap of two place from Anuj, that means we have four 84 kg, in case (2a) & case (2b) Dev sits immediate
possibility for Anuj, in case (1a) & case (2b) Anuj right of the one whose weight is 84 kg and case (1c)
sits at corner, in case (1b) & case (2a) Anuj sits at & case (2c) are not valid.
smaller side.  Based on above given information we have:

Case (1c) & case (2c) are not valid as two person  The one whose weight is 52 kg sits second to left of
sits between Kavita and Dev whose weight is Rinku.
56kg.  As Payal doesn‟t sit adjacent to the one whose
Again, we have: weight is 48 kg, that means in case (1a) & case (1b)
 The one whose weight is 52 kg sits third to left of Payal sits immediate right of the one whose weight is
Payal, whose weight is 42 Kg. 84 kg and case (2a) & case (2b) are not valid.
 Two people sit between Bittu and Hari, who sits  Either Bittu or Hari sits at corner of the table, that
adjacent to the one whose weight is 64 kg. means Bittu sits at corner and Hari sits immediate
left of the one whose weight is 64 kg.
 Based on above given information we have:

Case (2a) & case (2b) are not valid as the one  One person sits between Ishu and the one whose
whose weight is 52 kg sits second to left of Rinku. weight is 72kg, that means case (1a) is not valid and
Again, we have: in case (1b) Ishu‟s weight is 52 kg.
 The one whose weight is 76kg sits facing Ishu, who  Based on above given information we have:
neither sits adjacent to Sam nor the one whose
weight is 82kg.

Case (1a) is not valid as the one whose weight is 76kg sits facing Ishu and one person sits between Ishu and
the one whose weight is 72kg.
Again after applying given condition of drawn cards we have final arrangement as follow:
Set-30:
Directions (144-148):

139) Answer: C We have:


Explanation:  A, B, C and D are the corner points of the
Clearly, Bittu sits immediate left of Sam in the final rectangular field in such a way that A is west of
arrangement. point B.
Hence, option C is correct choice.  C is south of point A, that means D is east of C.
 The length of largest side of field ABCD is 80m.
140) Answer: A  Side CD is the largest side of the field, that means
Explanation: side AB = CD = 80m.
Clearly, only Payal remains unchanged after final  E, F, G and H are the corner points of another
arrangement. rectangular field having area of 2304 m2 in such a
Hence, option A is correct choice. way that Line EF intersects Line AB at M and Line
CD at N respectively.
141) Answer: B  C is 12m west of point N.
Explanation:  Line GH intersects the rectangular field at L and P.
Clearly, only two person sits between the one whose  H is 32m east of point E.
age is 64 years and the one whose age is 52 years  Area of EFGH = 2304m2.
when counted right of the one whose age is 64 years.  EH = 32m
Hence, option B is correct choice.  Thus, EF = 2304/32 = 72m.
 Based on above given information we have:
142) Answer: C
143) Answer: D

Again, we have:
 D, I, J and K form another rectangular field in such a Explanation:
way that point J is 8m east of point P, that means PJ Clearly, K is north-east of G.
= 8m. GK = √(102 + 82) = 2√41m.
 Thus, DJ = CD – (CN + PN + PJ) Hence, option A is correct choice.
 DJ = 80m – (12m + 32m + 8m) = 28m.
 Area of the rectangular field DIJK is 504m2. 145) Answer: E
 DI = 504/28 = 18m. Explanation:
 Point I is 54m south of point B, that means BI = Total distance covered in “FGPDB” = 132m.
54m. Total distance covered in “FNCMB” = 144m.
Total distance covered in “FGLB” = 132m.
 Thus, BD = (54 – 18) = 36m.
Total distance covered in “FEHLB” = 148m.
 Point H is 8m from point L, that means HL = 8m.
Hence, option E is correct choice.
 PG = HG – (HL + LP)
 PG = 72 – (8 + 36) = 28m.
146) Answer: E
 Based on above given information we have final
Explanation:
arrangement as follow:
Area of MLPN = (36 x 32) = 1152m2.
Area of EHLM = (32 x 8) = 256m2.
Thus, required difference = (1152 – 256) = 896m2.
Hence, option E is correct choice.

147) Answer: C
Explanation:
MH = √(322 + 82) = 4√68m.
Hence, option C is correct choice.

148) Answer: B
Explanation:
Clearly, B is north-east of N.

144) Answer: A

Set-31:
Directions (149-153):
149) Answer: C
150) Answer: B
151) Answer: D
152) Answer: E
153) Answer: C

Explanation:
From the above table we can conclude the following list of matches and winners.

 From the above statement we get two possibilities as


the match against DD and MI might be either Day
match or Day-night match on Monday.
 KKR did not play on Thursday.
 DD won the Tuesday Day match
 MI lost its Day-night match on Wednesday.
 CSK did not play on Monday.
 Since, DD has won only two matches, we can
conclude that Day match was held between DD vs
RR on Tuesday.
 Also from the above table, we get that MI has lost
match with only two teams. So, we can conclude that
MI has Day-night match with CSK on Wednesday.

 DD won match against MI on Monday.

 CSK played two Day matches and two Day-night night match on Tuesday and Day match on Thursday
matches. and Friday.
 Since, CSK played two Day match and two Day- Also, KKR has played four matches other than
night match; we can conclude that CSK has Day- Thursday. So, we can conclude the following.
 So, we can conclude that RR vs. KKR was held on
 RR lost its Day-night match against MI. Monday.
 So, we can conclude that RR vs. MI is held on Since, Case (1) doesn‟t follow the above statement.
Thursday. Case (1) gets eliminated.
 The least winner in the series had a Day match on
Monday.

Also, we can conclude that CSK vs DD is held on


From the table above, Thursday. Since, DD cannot play two matches on the
We can conclude that KKR vs DD is held on same day.
Wednesday. Since, MI cannot play two matches on So, the final arrangement is,
the same day.
Set-32: Explanation:
Directions (154-158):  E lives below D but above B.
 There are two floors between A and B.
 There is exactly one floor between C and E where
none of the five lives.
 C lives above A but below D.

From the above statements we can conclude that D


lives in the floor which is above the four others.
 The floor numbers pressed by B are two perfect
square numbers.
 B pressed the floor number of D.
154) Answer: C
 B does not live in floor numbered 1.
155) Answer: D
 From the above statements we can conclude the floor
156) Answer: B
number of D as 9; also we get four possibilities-
157) Answer: E
Case (1), Case (2), Case (3), Case (4) and Case (5)
158) Answer: C
based on the statements.
 B pressed the buttons of the floors which are above the floor in which he lives.
 From this statement, Case (2), case (4) and Case (5) gets eliminated.
 So, we can conclude that C pressed 6 and E pressed
 C pressed the floor number of E. 2 and 10 in Case (1) and C pressed 8 and E pressed 2
 E pressed the floor number of B. and 8 in Case (3). No two persons press the same
 The difference between the floor numbers pressed by floor number. So, Case (3) gets eliminated.
E is the floor number of C.  A pressed the floor number of C.
 A pressed consecutive floor numbers.
 So, we can conclude that A pressed 8 and 7.

Set-33:
 The difference between the floor numbers pressed by (Directions 159-163):
D is a perfect square.
 The remaining floor numbers were 3, 5 and 1. From
the above statement the only possibility pressed by D
to get a difference of perfect square is 5 and 1.
 So, the final arrangement is,

We have:
 One who likes Horse lives on even numbered floor at
a gap of three floor from one who likes Peacock.
 S, who doesn‟t like Eagle but likes Rat, lives on
floor just below one who likes Peacock, who neither
lives on floor marked as 6 nor on floor marked as 8,
that means we have two different place for one who case (1) one who likes Parrot lives on floor marked
likes Peacock, in case (1) one who likes Peacock 6, in case (2a) one who likes Parrot lives on floor
lives on floor marked 4, in case (2) one who likes marked 8, in case (2b) one who likes Parrot lives on
Peacock lives on floor marked 2. floor marked 4.
 Only one person lives between one who likes Parrot  Only three person lives between one who likes
and one who likes Horse, that means we have three Parrot and W, who likes Cow.
different possible places for one who likes Parrot, in  Based on above given information we have:

Again, we have: Pigeon lives on floor marked 6 and in case (2b) one
 R who likes Owl neither lives on adjacent floor of who likes Pigeon lives on floor marked 8.
one who likes Horse nor one who likes Cow, that  One who likes Lion lives on odd numbered floor just
means in case (1) R lives on floor marked 5, in case below one who likes Sparrow, that means in case
(2a) R lives on top floor, in case (2b) R lives on floor (1a) one who likes Lion lives on floor marked 7, in
marked 3. case (1b) one who likes Lion lives on floor marked
 Only two person lives between one who likes Owl 1, in case (2a) one who likes Lion lives on floor
and one who likes Pigeon, that means we have four marked 3 and in case (2b) one who likes Lion lives
possible place for one who likes Pigeon, in case (1a) on floor marked 7.
W likes Pigeon, in case (1b) one who likes Pigeon Based on above given information we have:
lives on floor marked 8, in case (2a) one who likes
Again, we have: Cat doesn‟t lives on adjacent floor, that means in
 T, who doesn‟t lives on floor adjacent to one who case (1b) and (2a) T lives on floor marked 7, in case
likes Sparrow, lives on odd numbered floor at a gap (2b) T lives on floor marked 5.
of one floor from one who likes Elephant, that means  U who likes Rabbit lives on any floor below one
case (1a) is not valid. who likes Eagle, that means in case (1b) U lives on
 Only three person lives between one who likes Eagle floor marked 4, in case (2a) and case (2b) U lives on
and one who likes Elephant, who lives on adjacent floor marked 2.
floor of one who likes Cat. One who likes Rat and  Based on above given information we have:

Case (1a) is not valid as T lives on odd numbered who likes Horse, that means case (1b) and (2b) is not
floor at a gap of one floor from one who likes valid and we have two possible place for V, in case
Elephant. (2a) V lives on floor marked 8 and in case (2c) V
Again, we have: lives on floor marked 6.
 V, who doesn‟t lives adjacent floor of W, lives on  Q, who neither likes Dove nor Horse, lives at a gap
adjacent floor of one who likes Crow. of one floor from one who likes Dog, that means Q
 At least four person lives between one who likes lives on floor marked 5.
Duck and Crow, who lives on any floor above one  Based on above given information we have:
Case (1b) and case (2b) is not valid as one who  Only two person lives between P and one who likes
likes Crow lives on any floor above one who likes Monkey, that means case (2a) is not valid and in case
Horse. (2c) P lives on floor marked 8.
Again, we have:  Based on above given information we have final
arrangement as follow:

Case (2a) is not valid as one who likes Monkey and P lives at a gap of two floors.

159) Answer: C Explanation:


Explanation: Clearly, X likes dove.
Clearly, combination in option C is not true. Hence, option E is correct choice.

160) Answer: E 161) Answer: A


Explanation: Set-34:
Clearly, only five person lives above one who likes
Sparrow.
Hence, option A is correct choice.

162) Answer: E
Explanation:
Clearly, all the given statements are true.
Hence, option E is correct choice.

163) Answer: E
Explanation:

We have:

nly three boxes are kept between box C and one that
contains Ring, which is kept at top.

nly one box is kept between box that contains Watch
and box C.

ox Q, which contains Watch, is kept at a gap of three
boxes from one which is wrapped with Blue paper,
that means we have two possible place for box Q, in
Clearly, after rearrangement position of none of the case (1) Q is kept third from top and in case (2) box
person remains unchanged. Q is kept third from bottom.
Hence, option E is correct choice. 
he box which contains Book and box C, which
doesn‟t contain Pen, are kept together, that means we
have four possible place for box that contains Book.
Based on above given information we have:
Again, we have:  Only four boxes are kept between one that contains
 Box S, which is neither kept adjacent to box Q nor Pen and one that is wrapped with Red paper, that
one that contains Book, is kept at a gap of three means case (1b) and (2a) is not valid.
boxes from one that contains Pen.  Based on above given information we have:
 Box that contains Pen and Calculator are kept
together.

Case (1b) is not valid as box that contains Pen  The box wrapped with Green paper is kept just
and Calculator are kept together and case (2b) is above box D which contains Laptop, that means in
not valid as box that contains Pen and one case (1a) A is kept second from top, in case (2b) box
wrapped with Red paper are kept at a gap of four A is kept fourth from top, in case (2c) box A is kept
box. fourth from bottom.
Again, we have:  Box which contains Toy and one wrapped with
 Box A is wrapped with Yellow paper is kept at a gap Brown paper is kept together. Box that wrapped with
of three box from one wrapped with Green paper. brown paper is not kept at bottom.
 Box which contains Toy is neither kept adjacent to case (2b) box S is wrapped with Brown paper, in
one contains Laptop nor kept at bottom, that means case (2c) box that contains Toy is kept second from
in case (1a) box wrapped with Brown is kept at top, bottom.
in case (1c) box Q is wrapped with Brown paper, in Based on above given information we have:

Again, we have: place above box R, which is kept at a gap of one box
 Only two boxes are kept between one that contains from box B.
Ball and box which is wrapped with Black paper.  Box which wrapped with Pink paper is kept at any
 Box that contains Cup kept just below one wrapped place above box wrapped with Orange paper. Box P
with White paper. neither wrapped with Brown paper nor contains
 Number of boxes between E and R is same as Book, that means in case (1a) box P is kept at
number of boxes between box P and one which bottom, case (1c), (2b) and (2c) are not valid.
wrapped with Orange paper. Box E, which is neither  Based on above given information we have final
wrapped with Pink nor white paper, is kept at any arrangement as follow:

Case (1c), (2b) & (2c) is not true as number of boxes between box P and one which wrapped
boxes between E and R is same as number of with Orange paper is not valid.
168) Answer: E
Explanation:
Clearly, no box is kept between one that contains
Ball and one wrapped with Blue paper.
Hence, option E is correct choice.

Set-35:
(Directions 169–170):
We have:
 H is either 2m or 3m south of G.
 F is north-west of E.
 B either 2m or 3m west of C.
 A is west of H.
164) Answer: B  F is either 2m or 3m east of G.
Explanation:  E is either 4m or 5m east of D.
Clearly, Box which is wrapped with pink paper is  D is either 4m or 5m north of C.
kept just above one contains Pen.  G is north of C.
Hence, option B is correct choice.  A is north of B.
 Based on above given information we have:
165) Answer: E
Explanation:
Clearly, only six boxes are kept above one that
contains Laptop.
Hence, option E is correct choice.

166) Answer: C
Explanation:
Clearly, the one wrapped with Orange paper contains
Pen.
Hence, option C is correct choice.
Again, we have:
167) Answer: B  Distance between EF is more than CD, that means
Explanation: EF = 5m and CD = 4m.
Clearly, Box R is kept at a gap of one box above box  Distance between BC is more than FG, that means
S, in same way box Q is kept at a gap of one box BC = 3m and FG = 2m.
above box C.  Distance between (AH + GH) = ED, that means AH
Thus, box A is related to box B. = 3m.
Hence, option B is correct choice.
 Distance between CG is equal to 8m, as EF is
hypotenuse for triangle formed by a perpendicular
drawn from F on DE.
 Since, EF = 5m, then perpendicular must be 4 meter.
 Thus, to satisfies all the given of GC = 8m.
 DC = 4m.
 Thus, possible combination of length of GD must be
(2m, 2m) Final result can be determined by difference = (73 –
 Based on above given information we have final 41) = 32
arrangement as follow: Step II: In this step following logic is applied:

Final result can be determined by difference = (65 –


13) = 52
Step III: In this step following logic is applied:

169) Answer: D
Explanation:
Clearly, A is 6m north of B.
Hence, option D is correct choice. Final result can be determined by interchanging
digits of result.
170) Answer: B Step IV: In this step following logic is applied:
Explanation:
Clearly, F is north-east of A.
Hence, option B is correct choice.

(Directions 171-173):
We have given input:

From above logical steps we get following results for Difference = (16 – 4) = 12.
given input: Final result can be determined by interchanging
Step I: In this step following logic is applied: digits of result obtained.
From above logical steps we get following results for
given input:

We have:
 A 5x7 matrix in which rows are labeled 1 to 5 from
top to bottom and columns are labeled as @, %, &,
#, ¥, ^ and £ from right to left.
 All consecutive odd numbers are written in each cell
from left to right in each row such that „1£‟ has
smallest odd number and „5@‟ has the largest
possible odd number.
 All the letters of the word “JOURNEY” are written
in alphabetical order to each cell having multiple of
171) Answer: C
5.
Explanation:
Required sum of digits = (5 + 4) = 9  All the letters of the word “PLASTIC” are written in
Hence, option C is correct choice. reverse alphabetical order to each remaining cell
having number multiple of 3.
172) Answer: E  Based on above given information we have:
Explanation:
Clearly, 19 is second lowest in step I.
Hence, option E is correct choice.

173) Answer: A
Explanation:
Clearly, required difference = (272 – 172) = 440
Hence, option A is correct choice.
Again, we have:
Set-36:  „%‟ is written to each remaining cell which are
Directions (174-178): preceded by a vowel.
 One element is written between „K‟ & „Y‟, that
means we have two possible position for „K‟, in case
(1) K is written in cell „E≦‟, in case (2) K is written
in cell „E@‟.
 „K‟ is immediately preceded by „&‟. possibility for @, in case (1a)& case (2a)„@‟ is
 Two elements are written „@‟ and „#‟ in the same written in „B^‟, in case (1b)& case (2b)„@‟ is written
row. in „D£‟, in case (1c)& case (2c) „@‟ is written in
 „@‟ is written left of „#‟, which is not written in „E£‟.
same column having „U‟, that means we have six Based on above given information we have:

 „G‟ is written after „M‟.


Again, we have:  „M‟ is followed by „≦‟, that means in case (1b), case
 „G‟ is preceded by „£‟. (1c), case (2b) & case (2c) G written in the cell „C^‟,
 „*‟ and „G‟ are written in the same row. and case (1a) & case (2a) are not valid.
 „G‟ and „M‟ are written in the same column next to Based on above given information we have:
each other.
Case (2a) is not valid as „M‟ is written after „G‟.  „V‟ and „W‟ are written in same column.
Again, we have:  „D‟ and „V‟ are written in same row.
 „Q‟ is written before „A‟ but not in any cell having  „β‟ is preceded by „D‟, that means case (1b), case
prime number. Q and A are not written in the same (1c) & case (2c) are not valid and in case (2b) V is
row,that means „Q‟ is written at „A£‟. written „E≦‟.
 At least four and at most eight elements are written  „%‟ and „F‟ are written in same column which means
between „B‟ and „Z‟. B is written before Z. in case (2b) F is written in the cell „E #‟.
 „B‟ is preceded by „$‟.  Based on above given information we have final
 „Z‟ and „Q‟ are not written in same column. Z and Y arrangement as follow:
are not written in the same row.

Case (1b), case (1c) & case (2c) are not valid as „V‟ and „W‟ are written in the same column.

174) Answer: E Hence, option A is the correct choice.


Explanation:
Clearly, correct cell position of „V‟ is „E≦‟. 177) Answer: D
Hence, option E is the correct choice. Explanation:
Clearly, 67& is at „E%‟
175) Answer: C Hence, option D is the correct choice.
Explanation:
Clearly, in each combination 2nd element is second to 178) Answer: C
right of 1stelement and 3rd element is immediate left Explanation:
of 1st element. Clearly, only option C is true with respect to the
Thus, “Vβ&” doesn‟t belong to same group. matrix.
Hence, option C is the correct choice. Hence, option C is the correct choice.

176) Answer: A
Explanation:
Clearly, „#‟ is immediate right of W.
Set-37:
(Direction 179-183):

179) Answer: C a, Case b) based on the position of Benny and


180) Answer: D Martin.
181) Answer: D Mike sits third to the right of the one who likes
182) Answer: A Milkybar.
183) Answer: C Mike faces Hugh and both are immediate neighbours
of one who likes Dairy milk.
From the given conditions we get to know that From the above statement we can conclude the
children sitting in position 1 like Milkybar. position and the favourite chocolate of Children
Lee and Jack belong to same class and there is one in Class III.
children sitting between them. Ben sits between Jake and the one who likes Dairy
The one who likes Kitkat sits immediate right of milk.
Jack. The one who is sitting between Dave and Danny
Dave is junior to Lee and both likes Milkybar. likes Kitkat.
Both Martin and Chris like same Chocolate. From the statement, Martin likes Kitkat in Case a
Martin and Benny exactly face each other and and Benny likes kitkat in Case b.
neither of them sits with Jack nor Jake. Both Tom and Danny were sitting immediate right of
Jake likes munch and is senior to Jack. one who likes Dairy milk.
From the statements, we get to know that Dave As Chris likes Dairy milk in Class V, Martin in
belongs to Class I and Lee, Jack belong to Class Class I should also like Dairy Milk; Case a fails
III and Jake to belong to Class V. the condition. So it gets eliminated.
As Martin and Benny exactly face each other and
neither of them sits with Jack or Jake; they both
belongs to Class I. We get two possibilities (Case
Case a:

Case b:

Set-38:
(Direction 184-188):
184) Answer: D
185) Answer: B
186) Answer: C
187) Answer: A
188) Answer: C
From the above statement we conclude that A
belongs to the either 2nd or 3rd generation.
• C is the grandson of the one who works in North
direction from Meeting point X.
Bangalore is in north direction from meeting point
X, The person who works in Bangalore belongs to
first generation and C belongs to third generation.
• Both the Audi Car was in same colour.
• The person from Mumbai and the one who works
exactly in the difference of 900 from him owns same
Car.
He we get two possibilities either the person working
in Kochi or Hyderabad owns same car as the person
working in Mumbai.
• B is the son in law of one who keeps Audi Car.
So, we can conclude that the person from Bangalore
keeps Audi Car.
• E is the niece of C‟s mother and earns 60k.
So we get C and E in 3rd generation and A and B in
2nd generation. Also we conclude that E earns 60k.
• C earns 80k which is as same as the person who
works in west direction to him.
So, C could work in Chennai, Bangalore or
Hyderabad.
• The one who works in Hyderabad earns less salary
than his son and owns Black coloured Car.
• People with Black Car don‟t earn same salaries.
So, the people who owns Black Car earns salary in
60k, 70k and 80k
• The eldest of the family earns 70k.
So, we can conclude that the person from Bangalore
who keeps Audi earns 70k.
• A earns as same as the salary of his wife‟s Nephew
who works neither in North nor North-east to him.
C is the only possibility, as C earns 80k A also earns
80k.
• G‟s brother-in-law owns a white Car.
Explanation: As, B has black Car, we conclude A has white Car
•The difference between the salary of A and his and G is the wife of A.
Father-in-law is 10k.
•The person who is south-west direction from 27, 29, 31, 37, 41, 43, 47, 53, 59, 61 and 64. Where
meeting point X owns neither Benz nor Audi Car. 27 and 64 are cube numbers and rest were prime
Therefore, A owns Jaguar Car and is the husband of numbers.
D.  Age of Tessa is a cube number and she is elder to
The person in Hyderabad and Mumbai owns Benz. Victoria.
 As the age of Tessa is a cube number it may be
Set-39: either 27 or 64.
(Direction 189-193)  The age of Gwen is as same as her seat number and
189) Answer: D doesn‟t get down at Surat.
190) Answer: B  So the age of Gwen may be 27/29/31
191) Answer: C  The age difference between Amyrah and Nylah is a
192 Answer: A square of a number less than 4 and Amyrah is
193) Answer: D younger to Nylah.
 So the possible cases were
 If Amyrah is 37/27/43 then,
 Nylah is 41/31/47. Only square of 2 differences is
possible between them from the above prime and
cube numbers.
 The person belonging to same destination doesn‟t
faces or sits adjacent to each other.
 Tessa and the person whose age is 20 years less than
Victoria are sitting in same berth of the same side.
 So the age of Victoria may be 61 or 47 from the
above ages.
 Tessa is an immediate neighbour of Victoria who
goes to Delhi.
 We get two possibilities case –a and case – b.
 Only one person gets down at Kota and her age is
one less than the twice the age of Sasha.
 Cynthia goes to Surat in upper berth and her seat
number is the age of the one who goes to Delhi.
 The age of Cynthia is 20 more than her seat number.
 So the only possibility is 47, no other possibility is
there from the above ages. As the age of Cynthia is
determined we can eliminate the possibility of 47
years from Victoria. So the age of Victoria is 61 and
the person sitting in same berth as Tessa is 41 years.
As the age of people was prime number and cube
numbers ranging from 20-65. The numbers were, 23,
Case – a:

Case – b:
 Also the age of Amyrah and Nylah is concluded as
37 years and 41 years based on Sasha‟s age.
 Based on the above statement Sasha sits either in
seat number 31 or 32 in both cases.
 Amyrah is seated in Upper berth and the person who
gets down with Amyrah sits diagonal to her.
 In case-b as there is no possibility for placing Gwen.
So, it gets eliminated and Gwen is placed in seat
numbered 29 in Case-a. And the place of Amyrah is
concluded as seat number 30.
 Zylah and Nylah both sits at Lower berth of different
sides.
 We can conclude Zylah sits at Seat number 28.
 The person who goes to Vadodara and Sasha faces  Neither Amyrah nor Tessa goes to Vadodara.
each other and the difference between their seat  The difference between the seat numbers of Victoria
numbers is 1. and the one who goes to Kota is 2.
 As the person who goes to Kota has one less than the  So, Zylah goes to Kota and Gwen goes to Vadodara
double the age of Sasha, and as Sasha has two as it doesn‟t goes to Surat. Since not more than 2
possibilities either 27= 54-1= 53 or 31= 62-1=61; people goes to same destination, we can conclude
Since 61 years is the age of Victoria we can Amyrah and Tessa who is diagonal to her, goes to
conclude that Sasha‟s age is 27 years and the person Mathura.
who goes to Kota as 53 years.

Case - a:
Case-b:

Explanation:
Set-40:  Daniel sits in seat number 6.
Directions (194-198):  Eric sits third to the right of Daniel.
194) Answer: C  Now we clearly understand from the given data that
195) Answer: E the person who are working for same satellite
196) Answer: C doesn‟t sit together but they are facing same
197) Answer: D direction and the person who are working for
198) Answer: D different satellite are always sit together but they
are facing opposite direction. Because of these
conditions immediate neighbours of Daniel are  Peggy is the most experienced person. So Peggy has
facing same direction and Daniel and his immediate 40 years of experience.
neighbours are working for different satellite. Hence  Serena sits in an even numbered seat and sits second
we can easily find out the directions of all the to the left of the one whose experience is square of a
persons. If Daniel facing centre definitely his number.
immediate neighbours are facing opposite direction  Acaba and Serena are facing same direction and
of Daniel and also find all the persons directions. Acaba experience is twice the amount of time of
Now there are two possible cases. Daniel faces Serena experience.
centre or opposite to the centre.  Already we know that Peggy experience is 40 years.
So, Acaba experience should be 32 years and Serena
experience is 16 years.
 The one who has experience of 32 years sits second
to the left of the one who has experience of 20 years.
 Since, Acaba and Serena facing same direction and
both are sitting in an even numbered seat and Acaba
sits second to the left of the one who has experience
of 20 years, so this person also sits in even numbered
seat .Now the possible cases are

 Joseph experience is four less than that of average of
experience of Robert and Peggy.
 we know that Peggy‟s experience is 40 years and
unknown experience for case 1 is 25, 30 , 27 and 18
and for case 2-25, 30, 27 and 18 finally for case
1(A)- 20 , 30,27 and 18.
 From this we can get experience of that person
which is four less than that of average of Peggy and
Robert by 40 and 18.i.e..((40+18)/2)-4 = 25.
 So we conclude that Robert experience is 18 years
and Joseph experience is 25 years. Case 1(A) and
Case 2(A) are eliminated because Daniel experience
Boe who doesn‟t work for falcon 9 satellite have
is 25 years. Now the case 1 and case 2 becomes
more experience than Eric and never sits in seat
number 3.Boe is 3 years more experience than Eric.
Therefore, only two uncertain experience years we Position 2 is opposite to position 6; and Position 4 is
have 27 and 30. From that statement we definitely opposite to Position 8 and so on.) and change his
say that Boe is 30 years of experience and Eric is 27 direction to opposite direction. (if he faces outside
years of experience. Boe doesn‟t work for falcon 9; the centre in circle A now he face the centre in the
therefore he must be work for Falcon heavy satellite. circle B and vice versa.)
If we know the one person‟s works then we can 3.If the experience of Astronauts is multiple of 4 but
easily find out the satellite which all the persons not a multiple of 5, then he moves to the same
engage their work. (The people who are working for numbered position in circle B and change his
same satellite are not sitting together but they are direction to opposite direction.(if he faces outside the
facing same direction and the people who are centre in circle A now he face the centre in the circle
working for different satellite are always sitting B and vice versa.)
together but they are facing opposite direction.)and 4.If the experience of Astronauts is either a Square of
also case 2 is also eliminated. Because Boe never sits an odd number or a multiple of both 3 and 5, then he
seat number 3. moves a position that is opposite to his current
Robert doesn‟t sits opposite to Boe. Hence we can numbered position in circle B (i.e. Position 2 is
conclude that Robert seat number is 7 and Peggy seat opposite to position 6; and Position 4 is opposite to
number is 5. Also we know the directions of all Position 8 and so on.) and doesn‟t change his
persons. direction.
Now the arrangement becomes 5.Now the final arrangement becomes:

CONDITIONS:
Set-41:
1.If the experience of Astronauts is multiple of 3 but
(Directions 199–203):
not a multiple of 5, then, he moves to the same
We have:
numbered position in circle B and doesn‟t change his
direction.
2.If the experience of Astronauts is multiple of 4 and
also 5, then he moves to a position that is opposite to
his current numbered position in circle B (i.e.
Since M + M = G; also there is no carry in addition
(mentioned in green), we can conclude that G must
be an even number.
So, the possibilities were,
If, (G=2/4/6/8)
M should be (1+1/2+2/3+3/4+4)
Since, G x D = G; the even digit (G) multiplied by a
digit (D) should give the same digit (G).
So the possibilities were,
2 x 6 = 12  Case (1)
4 x 6 = 24  Case (2)
From the above multiplication process; we can 8 x 6 = 48  Case (3)
conclude the following: From all the cases we can conclude that D=6.

G x D = G, G + Q = D and M + M = G;
 Since there is no carry in the last digit (marked in red); we can conclude that G+Q=D (marked in green).

So, In Case (2): 4 + (1/0) = 6 (there can be carry digit


In Case (1): 2 + (4/3) = 6 (there can be carry digit added); 2 is omitted as the value of M =2.
added); 1 is omitted as the value of M =1. In Case (3): There is no such possibility so it can
be eliminated.

In Case (1): Since the value of (2 x E = 1) is not


possible it gets eliminated (marked in green).
In Case (2): The value of (4 x E = 2); we get two
possibilities.

(4 x 3 = 12)  Case (2a)


(4 x 8 = 32)  Case (2b)
In Case (2a), we get (B54 x 3 = 462). So we can
conclude the value of B as 1.
So, the process becomes,

In Case (2a): 3 x 4 = 12; then (K x 3)+1 = 6. There


is no possibility other than substituting 5 as the value
of K. So, we can conclude the value of K as 5.
In Case (2b): 8 x 4 = 32; then (K x 8)+3 = 6. There So, by multiplying 154 x 236; we can find the value
is no possibility of getting odd digit in multiple of of P, Q and Z.
8. So, it gets eliminated. The final solution is,
Thus, “285032” is coded as “MZKQEM”.
Hence, option B is correct choice.

202) Answer: C
Explanation:
We have:
P = 9, G = 4 & B = 1.
Thus, value of “4P + 3G – 3B” = (9x4 + 3x4 -3x1) =
45.
Hence, option C is correct choice.

203) Answer: B
Explanation:
We have:
199) Answer: E B = 1, P = 9, Z = 8 and Q = 0.
Explanation: Thus, “PZ” = 98 & “BQ” = 10
We have: Thus, after multiplication of “98” with “10” we get:
Clearly, P = 8 & M = 2. (98 x 10) = 980
Thus, value of “P2 + 2M” = (9x9 + 2x2) = 85. Thus, “980” is coded as “PZQ”.
Hence, option E is correct choice. Hence, option B is correct choice.

200) Answer: D
Explanation:
Clearly, D = 6, Q = 0 and Z = 8.
Thus, value of “D3 + 4Q – 2Z” =(63 + 4x0 – 2x8) =
496.
Hence, option D is correct choice.

201) Answer: B
Explanation:
We have:
B = 1, K = 5, G = 4, D =6, E = 3, M = 2, P = 9, Z =
8, Q = 0.
Thus, “GKB” = 451 & “DEM” = 632
Thus, after multiplication of “451” with “632” we
get:
(451 x 632) = 285032
Set-42:
Directions (204-208):

204) Answer: D
205) Answer: B
206) Answer: C
207) Answer: B (All the pair has same year of experience)
208) Answer: D

Explanation:
From the given details in the table we can conclude the following;

 A Team cannot have all three members having a  The total “Web development” experience in Team
common domain of experience. Blue is 8 years.
 Team Red includes W and Y.  So, it must be (T + R/Z) = 8;
 If P is in a team, then U cannot be in the same team.
 The total experience of one team is 27, which is  The possible combination to get 27 years of
highest among all teams and Team Orange has total experience is 11+9+7(this is not possible as S and U
experience of 17 years, which is least among all should be together), 8+8+11 and 10+8+9.
teams. The other two teams have different total years  The possible combination to get 17 years of
of experience. experience is 5+9+3 and 7+7+3.
 S and U have to be together.  We get the following cases. Also in Case (2) T, Z
 None of the members in Team Green have any and V has a common domain experience this case
experience in Data Science. gets eliminated.

 Since, the possible combination to get 17 years of  In Case (1a), we cannot fix the employee Z (9) at
experience is 5+9+3 and 7+7+3 also S and U have to Team Orange as the remaining P and V cannot be at
be together. Team Green.
 So we get two possibilities Case (1a) and Case (1b),
 Case (1b) gets eliminated as either P or V should be So, this case gets eliminated.
in Team Green which doesn‟t satisfy the statements.

Hence, the final arrangement is,


Set-43: We have:
(Directions 209–213):  V sits 12m north of the one who is 9m east of Q,
let‟s say that „@‟ sits 9m east of Q.
 P is to the north of Q, that means we have three
possible place for P, in case (1) P sits somewhere
south-west of V, in case (2) P sits west of point V, in
case (3) P sits somewhere north-east of V.
 P is 10m west of the one who is 5m north-west of T,
that means we can say that P sits 10m west of „%‟.
 Based on above given information we have:

 W and N sits at a gap of 6m.


Again, we have:  W sits at any place south of T.
 N sits north of W and is east of Q, that means N is Based on above given information we have:
east of Q.

 O sits at a gap of 4m west of the one who is south of


Again, we have: T, that means O sits 4m west of W.
 W sits east of O and R and O sits in same line.  R sits somewhere in-front of V but not necessarily
north of V, as R and O sits in same line, that means
R is coded as „%‟ and case (1) & case (2) is not  Based on above given information we have:
valid.

Case (1) & case (2) is not valid as R sits


somewhere in-front of V.
Again, we have:
 U sits 20m south of V.
As, UV = 20m & @V = 12m.
Thus, @U = (20 – 12) = 8m.
 S is 13m south-west of T, as S is only remaining
person.
We have, @V = 12m & @N = 5.
Thus, @T = √(@V2 + @N2)
@T = √(122 + 52) = 13m.
Thus, S must be coded as „@‟ and T must be east of 209) Answer: C
V. Explanation:
 Again, RT = 5m OW = 4m. Clearly, U sits south-west of O.
Thus, PQ = (12m + 3m) = 15m. Hence, option C is correct choice.
Based on above given information we have final
arrangement: 210) Answer: E
Explanation:
Clearly, all the above statements are false.
Hence, option E is correct choice.

211) Answer: B
Explanation:
Clearly, TW = (NW + NT)
TW = (6 + 12) = 18m
OW = 4m
Thus, OT = √(OW2 + TW2)
OT = √(182 + 42) = 2√85m.
Tong Coop Sloop Whoop
Hence, option B is correct choice.
Yong Whoop Droop Hoop
212) Answer: E
Explanation: Pong Knoop Roop Stroup
From (I): QS + OW > TV + PR.
We have: To form a sentence the following conditions should
QS = 9m, OW = 4m, TV = 5 & PR = 10m. be considered:
Thus, (9 + 4) > (10 + 5), is not true. 2 Tong – 1 Yong – 2 Pong
From II: PQ > (US + RT) ≥ (WN + QS)  Coop – Whoop (v.v)
We have:  Knoop – Hoop
PQ = 15m, US = 8m, RT = 5m, WN = 6m & QS =  Sloop x Droop
9m.
Thus, 15 > (8 + 5) ≥ (6 + 9), is not true. Set-45:
From III: NS + OW + TR < NW + VT Directions (219-223):
NS = 5m, OW = 4m, TR = 5m, NW = 6m & VT = 219) Answer: C
5m. Explanation:
Thus, (5 + 4 + 5) > (6 + 5), is not true. We have:
From IV: VS + SQ > PQ + TR 13 15 X
VS = 12m, SQ = 9m, PQ = 15m & TR = 5m 21 64 5
Thus, (12 + 9) > (15 + 5), is not true. 36 81 23
Clearly, none of the statements are true. For 2nd row:
Hence, option E is correct choice.  An odd number followed by perfect square so, (64 –
21) = 43.
213) Answer: D  Now, the row is 43 5, an odd number (prime) is
Explanation: followed by another odd number so, 43 x 5 = 215.
Clearly, P sits north-west of W.  Thus, resultant of 2nd row = 215.
Hence, option D is correct choice.
 For 3rd row:
 An even number followed by perfect square so, (81 –
Set-44: 36) = 45.
Directions (214-218):  Now, the row is 45 23, an odd number (non-prime)
214) Answer: D is followed by another odd number (prime), so 45 +
215) Answer: C 23 = 68.
216) Answer: D  Thus, resultant of 3rd row = 68.
217) Answer: C  For 1st row:
218) Answer: C  Resultant of 1st row = 462 – (215 + 68) = 179.
 An odd number (prime) followed by another odd
Explanation:
number so, 13 x 15 = 195.
From the given information we could form the below
table:
 Now, the row is 195 X, an odd number followed by For 1st row:
X to get resultant of 179.  An odd number followed by perfect square so, (36 –
 Thus, 195 – X = 179 21) = 15.
 X = (195 – 179) = 16, which follows Condition (3).  Now, the row is 15 53, an odd number followed by
 Hence, option C is correct choice. another odd number (prime) so, (53 + 15) = 68.
 Thus, resultant of 1st row = 68.
220) Answer: A  For 2nd row:
Explanation:  An even number followed by perfect square so, (48 –
We have: 9) = 39.
24 31 9  Now, the row is 39 49, an odd number followed by
23 27 49 perfect square number. so, (39 - 49) = 10.
32 17 13  Thus, resultant of 2nd row = 10.
For 1st row:  For 3rd row:
 An even number followed by an odd number (prime)  Resultant of 3rd row = 1149 – (10 + 86) = 1071.
so, 24 + 31 = 55.  An even number followed Y thus, after applying
 Now, the row is 55 9, an odd number followed by a Condition (4), we have:
perfect square so, (55 – 9) = 46.  (64 x 18)
 Thus, resultant of 1st row = 46.  Now, the row is (64 x 18)Y, after applying Condition
 For 2nd row: (1) we have:
 An odd number (prime) is followed by another odd  1152 – Y = 1071
number so, (23 x 27) = 621.  1152 – 1071 = 81
 Now, the row is 621 49, an odd number followed  Root of 81 is 9.
by perfect square so, (621 – 49) = 572.  Hence, option B is correct choice.
 Thus, resultant of 2nd row = 572.
 For 3rd row: 222) Answer: D
 An even number followed by odd number (prime) For 1st row:
so, (32 + 17) = 49.  An odd number followed by perfect square so, (15 –
 Now, the row is 49 13, an odd number followed by 9) = 6.
another odd number (prime) = 62.  Now, the row is 6 12, even number is followed by
 Thus, resultant of 3rd row = 62. another even number then the resultant will be the
 Clearly, required product = (6 x 2 x 2) = 24. product of the numbers so, (6 x 12) = 72.
 Hence, option A is correct choice.  Thus, resultant of 1st row = 72.
 For 2nd row:
221) Answer: B  An even number followed by perfect square so, (36 –
Explanation: 20) = 16.
We have:  Now, the row is 16 17, an even number followed by
21 36 53 odd number. So, (16+17) = 33.
48 9 49  Thus, resultant of 2nd row = 33.
64 Y 81  For 3rd row:
 An odd number followed by another odd number so,
(15 +23) = 38.
 Now, the row is 3813, even number is followed by
an odd (prime) number so, (38+13) = 51.
 Thus, resultant of 3rd row = 51.
 Clearly, 2+3+1=6.
 Hence option D is the correct answer.

223) Answer: A
For 1st row:
 An even number followed by an odd number (prime)
so, 24 + 31 = 55.
 Now, the row is 55 X
 Run scored by Jack is a square of the position of
 For 2nd row:
Tom in the scorecard. The run scored by the 4th least
 An even number followed by perfect square so, (48 –
scorer (excluding the players yet to bat) is a cube
9) = 39.
number.
 Now, the row is 39 49, an odd number followed by
 So we can fix that the runs scored by the player in 6th
perfect square number. So, (39 - 49) = 10.
position is a cube number.
 Thus, resultant of 2nd row = 10.
 Paul owns Puma bat and he is yet to bat. Neither of
 For 2nd row:
the players who were yet to bat owns Spartan CG
 An even number followed by perfect square so, (36 –
nor Reebok branded bats.
20) = 16.
 The runs scored by using Puma branded bat is the
 Now, the row is 16 17, an even number followed by
second highest.
odd number. So, (16+17) = 33.
 So we can conclude that the player in 2nd position
Thus, resultant of 2nd row = 33.
owns Puma brand bat.
So, 89-10-33 = 46 (resultant of row1)
 Tom got 8th position in scorecard without scoring
55 - X = 46
any runs. Only Peter scored runs in three digits.
So, the value of X is 9.
 Peter scored 7 runs less than the double the score of
Hence, option A is correct choice.
the only one who uses Sunridges brand bat.
Set-46:  As Peter scored in three digits he must be the highest
(Directions 224-228): scorer and the only possible score of Sunridges is 64
224) Answer: C which can be concluded as Jack‟s score.
225) Answer: B  Mike, who is yet to bat, is in one of the positions
226) Answer: A below Mark in the scorecard which is as same as the
227) Answer: D position of Max above Mark in the scorecard.
228) Answer: C  Already we found that Paul was yet to bat and now
Mike gets added. So, we can conclude the position of
Explanation:
both based on the alphabetical order i.e. Paul in 11th  Lee scored runs as same as the position of Ben in
Position and Mike in 10th position. scorecard and Ben scored as same as the square of
 We get two possibilities Case-a and Case-b based on position of Lee in the scorecard.
the position of Mark.  The players who are in one of the position above 5
and double the same position uses Reebok brand bat.
 Two people owns Spartan CG bat and sum of their
Case-a: score is 27 runs.
 Lee got one of the positions in top 3 least scorers.
 We can fix the position of Ben as 4 not as 5 or 6,
because Position of Lee cannot be 5 and 6 which are
neither a cube nor square number. So Case-a gets
eliminated.
As the position of Ben is fixed, we can conclude the
position of Lee as 7. And the score of Ben is 49 runs
and Lee is 4 runs. Now the remaining runs in 354 is
35 runs in which one must be a cube number. We get
only one possibility 27 and 8, so we can enter the run
of 5th position as 27 and Mark as 8 runs.
As Spartan CG bat users scored 27 runs it must be 27
+ 0, so we can conclude Spartan CG bat user in 5th
Case-b: position and either in 8th or 9th position.
Case-a:

 There are two players between Ben and the one who
scored runs in a square number; both from above and
below of Ben.
230) Answer: C
Case-b: 231) Answer: E
232) Answer: B
233) Answer: C

Explanation:

 Sums of runs scored by using GM Icon brand bat is


the least among other branded bat.
 Mark who is below John and the one who scored  Shortest route between Jaipur and Rajasthan is 200
runs in square number uses SG Cobra branded bat. km
 We can conclude that Ben and Tom has Reebok  Here we get two possibilities- Case (1) and Case (2)
brand bat. We can conclude the position of John and
Peter‟s bat brand from the above statements.
Case b:

 Rajasthan is equidistant from Haryana and Goa


 Shortest route between Gurugram and Goa is 400 km
 Point E is not Rajasthan
 Shortest route between Lucknow and Haryana is 800
km

Set-47:
Directions (229-233):
229) Answer: D
 Shortest route between Lucknow and Kolkata is 600
km
 Case (1) gets eliminated as the distance between C
and F is 800 km.

So, the final arrangement is,

Set-48:
Directions (234-238):

234) Answer: C
235) Answer: B
236) Answer: A
237) Answer: C
238) Answer: D
Explanation:
 The first box is a large Cyan box and the last box is a small Magenta box.

 Every medium sized box has one small box and one  No two boxes of the same size and same colour are
large box adjacent to it on its either side. kept together.
 A small box is never kept to the immediate right side
of the large box.

 Also Peach box is always placed at immediate right


 Peach boxes are not at prime places and also not at of Grey box.
the place that is cube of a natural number.  The Prime and Cube numbers were – 1, 2, 3, 5, 7, 8,
 There are five boxes of each size. Each size has all 11 and 13.
the five coloured boxes.

So, we can conclude the position of medium size Peach box is at 14. Since, Peach and Grey boxes are always
together. We can conclude the position of Grey box is at 13.

On fixing the position of large size Peach box we get two possibilities- Case (1) and Case (2).
 A box is not placed adjacent to same coloured box.
 So, we can fix the position of Small sized Peach box in both the cases.

 The sum of position of all three Magenta coloured boxes is an odd number.
 Here we get two possibilities in each case.
 All three Brown coloured boxes are placed at even numbered positions.
 From the above statement, Case (1b) and Case (2b) gets eliminated.

Since, no two boxes of the same size and same colour are kept together. Case (1a) gets eliminated.
So, the final arrangement is,
Set 49:
Direction (239-243):

239) Answer: B  Height of the container which is headed


240) Answer: D towards Mumbai port is 141ft and number of Cars in
241) Answer: C it is a prime number.
242) Answer: D  Number of Cars and bikes in C2 is a multiple
243) Answer: C of 11.
 No height of a container is above 150ft and
Explanation: below 100ft.
 Number of cars in a container which is
 Number of Cars to be unloaded in Kochi is as headed towards Tuticorin is as same as number of
same as number of cars in C2. bikes in C2. The container which is 106ft tall has 29
 Container C4 is 44ft taller than one of the cars in it.
containers placed below it.
 There are two containers between the one eliminated as Number of cars in a container which is
which is headed towards Tuticorin and Mumbai port. headed towards Tuticorin is as same as number of
 Number of bikes in C1 is even and the height bikes in C2, so C2 container is not headed towards
of the same container is an odd number. Tuticorin.
From the above statements we get, two possibilities We can conclude C5 as Kochi, as C3 cannot be
(Case a, Case b). Kochi because the number of Cars in C3 is 29 which
As the place of the container headed towards is not a divisible of 11.
Mumbai and Tuticorin is unknown. But Case b gets

 Difference between number of bikes in the  There is 88 Cars in one of the containers and
container which is headed towards Kandla and the container which is headed towards Kochi is 17ft
Mumbai port is as same as the difference between shorter than Kolkata port.
bikes in the container which is headed towards As the number of containers headed towards Kochi
Mumbai and Kochi port. is 17ft shorter than the container which is headed
towards Kolkata port. We can conclude the container If we take 132ft as height of C2, we get height of C5
with 106ft is headed towards Kandla and C2 is as 115ft. Hence on adding all the height of the
headed towards Kolkata. container we get 644ft as the total height which is
Let us assume the Height of the C2 as 121ft, 132ft or incorrect.
143ft which is divisible by 11,
Possibility 1: Possibility 3:
If we take 121ft as height of C2, we get height of C5 If we take 143ft as height of C2, we get height of C5
as 104ft. Hence on adding all the height of the as 126ft. Hence on adding all the height of the
container we get 622ft as the total height which is container we get 666ft as the total height which is
incorrect. correct.
Possibility 2:
From the statement, container which is headed towards Mumbai and
Difference between number of bikes in the container Kochi port. Case b gets eliminated.
which is headed towards Kandla and Mumbai port is
as same as the difference between bikes in the

Set-50:
Directions (244-248):

244) Answer: C Explanation:


245) Answer: A  Immediate neighbours of V are facing towards
246) Answer: D the centre in the table. The Kenya team captain sits
247) Answer: E at one of the position right to T in the press meet.
248) Answer: B The difference between ODI ranking of U and T‟s
team is a prime number.
 There are 3 team captains sitting between T and table. Kenya got the highest ODI rank and the ODI
Afghanistan team captain, who sits at extreme end. rank of India is an even number but not 4.
Only the team which got ODI ranking in perfect So, from the possibilities of U-T = prime number;
square faces outside the table. The product of ODI we get the ODI rank of U as 3 as T is an even
ranks of U and V‟s team is a perfect cube number. number.
We get two possibilities (case-a, case-b) in linear 2–4=2 3–5=2
arrangement as the Afghanistan captain could at any 2–5=6 3–6=3
of the ends. 2–7=5 3–8=5
As V faces outside we can conclude that ODI rank of 2–9=5 3 – 10 = 7
V is a perfect square number. Since we get U as 3,
U x V = Cube number (1, 4, 9 are perfect square The case U x V = Cube number (1, 4, 9 are perfect
number) square number) becomes,
So the possibilities were, 1x8=8
8x1=8 2x4=8
2x4=8 3 x 9 = 27
3 x 9 = 27 So the ODI rank of U is 3 and ODI rank of V is 9.
U-T = Prime number (2, 3, 5, 7) The position of Kenya team Captain and T and V
Now U must be either 2 or 3 so, the possible cases gets concluded. The Rank of Kenya is 1.
were,  Zimbabwe got ODI rank of 5 and is an
2–4=2 3–5=2 immediate neighbour of Kenya in the table. There is
2–5=6 3–6=3 only one cube number in ODI rankings, which is the
2–7=5 3–8=5 rank of P. Zimbabwe team captain W sits third to the
2–9=5 3 – 10 = 7 right of R, who represents Pakistan in the table.
 P attends press meet at one of the position right As we concluded that P as Kenya‟s team captain and
to W; the difference between their ODI ranks is the W as Zimbabwe captain and their ODI rankings
ODI rank of R. R sits at extreme end and he is not were known. We can conclude the following too,
the captain of Afghanistan team. The ODI rank of P – W = ODI rank of R
Afghanistan is 2 positions below W. 1-5 = 4
P – W = ODI rank of R W + 2 = ODI rank of Afghanistan team
W + 2 = ODI rank of Afghanistan team 5+2=7
 Bangladesh got ODI ranking of 3 and is an  England team captain sits second from the left
immediate neighbour of Zimbabwe and India in end and got ODI ranking of 2.In press meet,
Press meet. Only one person sits between Australia team captain sits immediate right of T and
Afghanistan and Zimbabwe‟s team captain in both there is three people sitting between S and Australia
press meet and table. team captain.
 T faces towards centre and Kenya‟s team captain Case-b gets eliminated as Australia team captain sits
sits right to the T in the table. There are 2 team immediate right of T.
captains sitting between T and V in either side of the
Set-51: b) Number of consonants in all the words without
Direction (249-253): repeating any letters are written as second letter in
249) Answer: A first box.
250) Answer: C E.g.
251) Answer: B Time, variety, Tone, Lend
252) Answer: E Number of vowels without repeating->I, E, O, A->4
253) Answer: A letters->equivalent alphabet is D.
Number of consonants->T, M, V, R, Y, N, L, D->8-
Explanation: > equivalent alphabet is H.
Step I: Step II: Consider the two letter value in English
a) Number of vowels in all the words without alphabet as a whole number. Now take the difference
repeating any letters are written as first letter in first between the first and second box. Similarly for
box. second and third. Put the equivalent letter value of
first letter in first box in step II. Then for second
letter, (second letter of first box and second letter of Step IV: Difference between the first letter of both
second box)^2. the boxes is written in first box and similarly
E.g. difference between the second letter of both the
(DH-DF)=(48-46)=2=B boxes is written in second box.
(H-F)^2=4=D Step V: Number of consonants in between those two
Step III: For Both the box, first and second letter letters obtained in step IV.
will be +2 and +3 respectively.
Set-52:  P who works in SBI was born in one of the month
Directions (254-258): having 30 days and was born at a gap of two month
from his wife.
 Q is the only daughter of the one who likes Orange
and is married to the one who works in SBI.
 P‟s wife likes Banana but was not born on adjacent
month of the one who works in RBI, that means in
case (1) P was born in April and Q was born in July,
in case (2) P was born in June and Q was born in
September, in case (3) P was born in September and
Q was born in June, in case (4) P was born in
September and Q was born in December, in case (5)
P was born in November and Q was born in August.
 The one who works in BOB was born in adjacent
month of the one who works in RBI.
 H was born at a gap of two month from the one who
works in BOB, that means in case (1) the one who
works in BOB was born in March and H was born in
June, in case (2) the one who works in BOB was
born in January and H was born in April, in case (3)
the one who works in BOB was born in January and
H was born in April, in case (4a) the one who works
in BOB was born in January and H was born in
April, in case (4b) the one who works in BOB was
born in March and H was born in June, in case (5a)
the one who works in BOB was born in January and
H was born in April, in case (5b) the one who works
in BOB was born in March and H was born in June.
 The one who works in HDFC was born in May.
 Based on above given information we have:
We have:
 The one works in RBI was born in February and is
sister of the one who works in PNB.
 The one who likes Guava is sister of the one who
likes Apple, that means H is the sister of the one who
likes Apple.
 M is father of the one who likes Cherry, that means
H must be married to M.
 U‟s father works in ICICI and was born in
November, that means case (5a) & case (5b) are not
valid.
Again, we have:
 Based on above given information we have:
 The one who works in CBI also likes Cherry.
 The one who likes Cherry is only son of H, who
likes Guava.
U‟s father was born in November, that means L must
Case (5a) & case (5b) are not valid as U‟s father be grandson of the one who works in ICICI.
works in ICICI and was born in November.  One person was born between the one who likes
Cherry and M, who works in ICICI, as there is not
more than one month gap between birth of two
person, that means in case (1a) the one who likes
Cherry was born in September, in case (1b) the one
who likes Cherry was born in August and no person
was born in October, in case (1c) the one who likes
Cherry was born in August and no person was born
in September, in case (2), case (3), case (4a) & case
Again, we have:
(4b) the one who likes Cherry was born in August
 L is only child of the one who likes Litchi and is
and no person was born in October.
grandson of the one who was born in November, as
 K is sister-in-law of the one who likes Litchi, that  The one who likes Papaya is married to H and is
means K must be married to brother the one who brother-in-law of the one who likes Apple, as H is
likes Litchi. married to M, that means M must likes Papaya.
 Based on above given information we have:

Again, we have:  D is unmarried member of the family and is sister-in-


 The one who works in HDFC is mother of the one law of the one who likes Papaya, that means D likes
who works in BOM. Apple.
 As only nine member are there in the family and works in BOM was born in December and case (1a),
each person was born in different month, that means case (1b) & case (1c) are not valid.
in case (1a) no person was born in January, August  No person was born between the one who works in
and December, in case (1b) no person was born in BOI and D.
January, October and December, in case (1c) no  Two person were born between K and the one who
person was born in January, September and works in BOI, who doesn‟t like Guava.
December, in case (2) no person was born in March,  One person was born between the one who works in
July and October, in case (3) & case (4a) no person PNB and U.
was born in March, July and October, in case (4c) no  L was neither born after October nor works in BOB,
person was born in March, June and October, in case since D likes Apple, thus in case (4a) the one who
(4b) no person was born in two of three months works in BOI was born in June and L was born in
January, April & July. January, in case (4b) L was born in January and
 Two person were born between E and the one who works in BOI and case (2), case (3) & case (4c) are
works in BOM, who was born in adjacent month of not valid.
the one who likes Papaya, that means in case (2), Based on above given information we have:
case (3), case (4a), case (4b) & case (4c) the one who
Case (1a), case (1b), case (1c) & case (4c) are not person was born between the one who works in
valid as two person were born between E and the BOI and D, case (3) is not valid as L was neither
one who works in BOM, case (2) is not valid as No born after October nor works in BOB.

Again, after comparing above given figure we can conclude following results:
Again, we have:  One person was born between the one who likes
 U‟s mother works in PNB, since E is only son of M Grapes and his mother, since D is unmarried member
and H works in PNB in remaining valid cases, that of the family, thus case (4a) is not valid.
means U must be daughter of H and L must be son  K is married to P‟s father-in-law and neither likes
of U. Grapes nor works in PNB, since only nine members
 Three person were born between L‟s mother and the are there in family, that means L must likes Grapes
one who likes Mango, since U is mother of L, thus in and K must married to E.
case (4a) L likes Mango and in case (4b) P likes  As only three married couples are there in family,
Mango. that means the one who likes Orange must be
 The one who works in RBI neither likes Litchi nor married to E.
Orange, that means in case (4a) K likes Grapes.  After comparing above given information we have
final arrangement as follow:
256) Answer: E
Explanation:
Clearly, statement A , B and C are not true.
Hence, option E is correct choice.

257) Answer: C
Explanation:
Clearly, the one who likes Guava was born in June.
Hence, option C is correct choice.

258) Answer: E
Explanation:
Clearly, all the above given combinations are not
true.
Hence, option E is correct choice.

Case (4a) is not valid as one person was born Set-53:


between the one who likes Grapes and his mother. (Directions 259–263):
We have:

Step I: In this step following logic is applied:

254) Answer: E
Explanation:
Clearly, result in step I, can be determined by
Clearly, the one who works in RBI is maternal aunty
difference of square of results:
of E.
Result = (12)2 – (9)2= 63
Hence, option E is correct choice.
Step II: In this step following logic is applied:
255) Answer: B
Explanation:
Clearly, only three person was born between the one
who likes Grapes and the one who works in PNB.
Hence, option B is correct choice.
Case I: if result in step I is odd then, required result
can be determined as follow:
 Clearly, final result can be determined as average ofD
ifference of square of digits in step I. difference of square of digits in step IV.
(6)2 – (3)2 = 27 Result = ((8)2 - (6)2)/2 = 14
 A
fter difference interchange the digits of obtained
result.
Thus, final result = 72.
Case II: if result in step I is even then, required result
can be determined as follow:
 S
um of square of digits in step I.
(5)2 + (6)2 = 61
 A
fter difference interchange the digits of obtained
result.
Thus, final result = 16.
Step III: In this step following logic is applied:

Clearly, result in step III can be determined as sum


Thus, from above given steps we final following
of digits in step II:
result for given input:
Result = (9 + 7) = 16.
259) Answer: A
Step IV: In this step following logic is applied:
We have:
Sum digits of highest number in step II = (9 + 2) =
11
Sum of digits of lowest number in step II = (2 + 5) =
7
Clearly, results in step IV can be determined by Required difference = (11 – 7) = 4
difference of square of digits in step III. Hence, option A is correct choice.
After the difference final result can be obtained by
sum of digits of obtained number. 260) Answer: C
Result = (3 + 5) = 8. Clearly, final output in step V = 36.
Step V: In this step following logic is applied. Hence, option C is correct choice.
261) Answer: E
Clearly, second highest number in step I = 63
Second lowest number in step I = 52
Required difference = (63 – 52) = 11
Hence, option E is correct choice.  Science book having Black cover is kept at any place
below Math book.
262) Answer: B  Geography book having Pink cover is kept at any
Clearly, lowest number in step IV = 3. place below Hindi book, that means Geography book
Hence, option B is correct choice. is kept at bottom.
Based on above given information we have:
263) Answer: D
Explanation:
From I:
We have:
 Geography is kept at bottom having pink cover.
 Math is at second position from top.
 Book having Red cover is kept at the top.
 History book is just placed below math book.
 There is a gap of one place between English and
Hindi, who has cover of Green color.
 English having Blue cover is above Hindi, that
means English book is kept just below History book.
 Based on above given information we have: Clearly, exact position Yellow covered book is not
known.
Hence, statement II is not sufficient alone.
From I and II:
After combining statement I and II we have:

Clearly, no exact position for science book is known.


Hence, statement I is not sufficient alone.
From II:
We have:
 Science is kept third from bottom just below English Clearly, exact position of Yellow covered book is
having Blue cover. not known.
 GK book is kept on top at a gap of one place from Hence, statement I and II together not sufficient.
History book. Hence, option D is correct choice.
Set-54: %8F@5#Ω1≧4A>≤WHD63&U^0M£
Directions (264-268): QB2LI$R¥9E₹7
We have: Thus, from above given information we have final
8%DF4@H#1≧A>6&U^5M£3ΩQB2 arrangements as follow:
≤LI$0R≦9E₹W7 %8F@5#Ω1≧4A>≤WHD63&U^0M£
From step I: Such special character that is QB2LI$R¥9E₹7
immediately preceded by a number and immediately
followed by a consonant. 264) Answer: C
8%DF4@H#1µA>6&U^5M£3 ΩQB2 Explanation:
≤LI$0R¥9E₹W7 We have:
Thus, only four such special characters are possible %8F@5#Ω1≧4A>≤WHD63&U^0M£
 %, @, Ω and ≤. QB2LI$R¥9E₹7
After placing all such special characters to the Thus, only two such special characters are possible
immediate left of 1st, 5th, 9th and 13th element, we  @ and ¥.
get: Hence, option C is correct choice.
%8DF@4H#Ω1≧A>≤6&U^5M£3QB
2LI$0R¥9E₹W7 265) Answer: A
From step II: Such digits immediately preceded by a Explanation:
special symbol and immediately followed by a We have:
consonant. %8F@5#Ω1≧4A>≤WHD63&U^0M£
%8DF@4H#Ω1≧A>≤6&U^5M£3QB QB2LI$R¥9E₹7
2LI$0R¥9E₹W7 After drooping elements preceded by prime digits,
Thus, only five such digits are possible  8, 4, 5, 3 we have:
and 0. %8F@5Ω1≧4A>≤WHD63U^0M£QB
After placing all such digits to the immediate right of 2I$R¥9E₹7
3rd, 7th, 11th, 15th and 19th element in descending Element 8th to left of 13th element from right end =
order, we have: (8 + 13) = 21st element from right end.
%D8F@H5#Ω1≧4A>≤63&U^0M£Q Thus, 21st element from right end  W.
B2LI$R¥9E₹W7 Hence, option A is correct choice.
From step III: Such alphabets immediately preceded
by a special character and immediately followed by a 266) Answer: C
number. Explanation:
%D8F@H5#Ω1≧4A>≤63&U^0M£Q We have:
B2LI$R¥9E₹W7 %8F@5#Ω1≧4A>≤WHD63&U^0M£
Thus, only three such alphabets are possible  D, H QB2LI$R¥9E₹7
and W. Clearly, number of elements between 1st to 2nd
After placing all such alphabets immediate right of element is one less than number of elements between
„≤‟ in reverse alphabetical order, we get: 2nd to 3rd element either going forward or backward.
Thus, “6MI” doesn‟t belong to that group.
Hence, option C is correct choice.
Clearly, we don‟t know exact position of the one
whose hobby is Cricket.
267) Answer: A Hence, statement I is not sufficient alone.
Explanation: From II:
We have: We have:
%8F@5#Ω1≧4A>≤WHD63&U^0M£  Q sits third to left of S, who sits second to right of
QB2LI$R¥9E₹7 the one who likes Singing.
Thus, only two such alphabets are possible  W &  P sits third to left Q, who like Boxing.
Q.  The one who likes Football sit third to left of P.
Hence, option A is correct choice.  U sits second to left of R, who sits second to left of
the one who likes Boxing.
268) Answer: E  The one likes Boxing sits immediate right of W.
Explanation:  T likes Singing and sits third to left of the one who
From I: likes Cricket, that means U likes Cricket.
We have: Based on above given information we have:
 U sits third to right of T, who likes Singing.
 S sits immediate left of U.
 The one who likes Football sit in-front of the one
who likes Badminton, who sits third to left of T, that
means the one who likes Football sits immediate left
of S.
 Q having Boxing as hobby sit second to right of R,
who sit third to right of S, that means R likes
Badminton.
 P sit third to left of Q.
 V sit third to right of one likes Dancing, who sit
second to right of P, that means V likes Football. Clearly, we don‟t know hobby of the R, who sits
 The one who likes singing sits second to left of the second to right of the one who likes Cricket.
one like Drawing, that means S likes Drawing. Hence, statement II is not sufficient alone.
Based on above given information we have: From I and II:
After combining both statements we have:
Clearly, R (Badminton) sits second to right of U  Distances are considered only on the
(Cricket). perimeter of the square table.
Hence, statement I and II together sufficient. Perimeter = 4a =320m (given)
Hence, option E is correct choice. There are 8 persons sitting in a square table. So
320/8 = 40m, which is the distance between two
Set-55: successive person in the group.
Directions (269-273):
269) Answer: a
270) Answer: c
271) Answer: c
272) Answer: e
273) Answer: d

 All the persons are sitting along the perimeter


of the square table at an equal distance between
them.  Berner sits 80m away from Jane who is not
 The perimeter of the square is 320m and the sits at any corner of the table.
perimeter of the square is 4a, where a is the side of
the square.

 Three person sits between Jane and Ashoke.


 Noam sits at corner of the table and Alan sits 40m to the right of Noam.

 Blackburn and Shinya sits opposite to each other.


 Alan is not an immediate neighbour of Shinya.

 Only one person left in seating, that person is James.


 Jane doesn‟t sit immediate left of James. From this statement, case 2 is eliminated and case 1 becomes
final arrangement.

Set-56: 274) Answer: C


Directions (274-278): 275) Answer: B
276) Answer: A
277) Answer: E
278) Answer: B

Explanation:
Their feet sizes are 7.5, 8, 8.5, 9 and 9.5.
Received shoe sizes were 7.5, 8.5, 9, 9.5 and 9.5.
From this we can create the below table,
The shoe size 7.5 is only wore by the one with foot Jaakko didn‟t order the shoes size received by
size 7.5 (none can wear the shoes smaller than their Jammy. So, we can conclude that Jaakko ordered
foot size) shoe size – 9.
Also, the one with foot size 9.5 can wore only the
shoe size of 9.5 (none can wear the shoes smaller
than their foot size)
So, the one with 7.5 and 9.5 have Narrow foots. The
foot size of Jasbir cannot be 9.5 as none of them
received a pair that was neither the one that they
ordered nor one that was recommended for their feet
size.
So the table becomes,

Set-57:
Directions (279-283):

So the remaining feet sizes were – 8, 8.5 and 9.


Remaining shoe sizes were 8.5, 9 and 9.5.
Also, the foot size of Jammy can‟t be 8.5 or 9 (since
it satisfies the recommendation of shoe size he
received). So we can conclude that the feet size of
Jammy is 8.
279) Answer: B  In round 2, V is sitting opposite R.
280) Answer: E  So we can fix the positions of R and T in Round 2
281) Answer: D and Position of W in Round 1.
282) Answer: B  From the above statements we get,
283) Answer: C

Explanation:
From the given information we can conclude the
positions and direction of Both Rounds,

 In round 1, Q is sitting opposite R.


 Here we get two possibilities based on the position
of Q and R.- Case (1) and Case (2).

 In round 1, S is sitting 2 places to the left of U.


 Case (2) gets eliminated as there is no possibility of
 In round 2, W is sitting 3 places to the right of S. placing W in Round 2. Also we can‟t fix the position
of T and P in Round 1; U and Q in Round 2.
So, the final arrangement is,
Set-58:
Directions (284-288):

284) Answer: B Hence, we can conclude that there were total 10


285) Answer: D matches played. Also it is said that each win gains 2
286) Answer: C points. So, we can conclude that there is a total of
287) Answer: C 10x2=20 points in the table.
288) Answer: B
 IIT-B didn‟t win any match.
Explanation:  From this we can conclude that IIT-B stands last in
As each team played exactly one match against the points table.
every other team, we can list the matches as below.  There was only one match which didn‟t produce any
result. Every team scored at least one point.
 IIT-D won two matches and lost one in the whole
series.
 From the above points we can conclude that, IIT-B
vs IIT-D match has No-Result. Since, every team has
scored one point and the information given about
IIT-D clearly says that one of its matches has No-
Result.
IIT-G beat only IIT-B.
From this we can conclude the following,

 The performance chart (number of won-Lost-N/R  Since the total points were 20; we can conclude that
matches) for only IIT-M and IIT-K was identical. 20-(1+5+2) = 12; thus IIT-M and IIT-K has got 6
 IIT-M topped the list because of its fabulous victory points each with three victories. Also it is given that
against IIT-K in the last match at higher net run rate. IIT-M topped the list.
So, the final table becomes,

Set-59:  One who belongs to 8th grade, who doesn‟t sit


(Directions 289–293): adjacent to one who belongs to 9th grade, sits second
to left of U, that means in case (1) U sits facing south
and in case (2) U sits facing north.
 W, who belongs to 10th grade, sits immediate right of
U.
 Only two person sits between W and X, who neither
We have:
sits adjacent to R nor adjacent to one who belongs to
 One who belongs to 9th grade sits third from either
4th grade, that means we have four possible place for
end of the row, that means in case (1) one belongs to
X, in case (1a) X sits second from left end, in case
9th grade sits third from left end and in case (2) one
(1b) X belongs to 8th grade, in case (2a) X belongs to
belongs to 9th grade sits third from right end.
8th grade, in case (2b) X belongs to second from right
 Only two person sits between U and one who
end.
belongs to 9th grade.
Based on above given information we have:

Again, we have: end, in case (2a)& case (2b)one belongs to 7th grade
 R, who neither sits adjacent to U nor belongs to 12th sits at right end.
grade, sits third to right of one belongs to 7th grade  S, who neither sits at end nor belongs to 9th grade,
who neither sit adjacent to U nor sits adjacent to W, sits fourth to left of one who belongs to 5th grade,
as R doesn‟t sit adjacent to X, that means in case that means in case (1a) & case (2b) S belongs to 8th
(1a)& case (1b)one belongs to 7th grade sits at left grade and case (1b) & case (2a) is not valid.
 Based on above given information we have:

Case (1b) & (2a) are not valid as S sits fourth to  Only three person sits between one who belongs to
left of one who belongs to 5th grade. 6th grade and one who belongs to 11th grade.
Again, we have:  One who belongs to 6th grade sits third to right of Q,
 P, who doesn‟t belong to 9th grade, sits second to who sits facing north, that means in case (1a) X
right of one who belongs to 4th grade. belongs to 11th grade and Q belongs to 9th grade, in
 Person sitting at end of the row sits facing in case (2b) Q belongs to 4th grade and U belongs to 6th
opposite direction, that means in case (1a) P sits grade.
third from right end and in case (2b) P sits third from  T, who doesn‟t sit adjacent to one who belongs to 5th
left end. grade, sits second to left of V, that means case (1a) is
not valid and in case (2b) V belongs to 9th grade.
Based on above given information we have: Clearly, W sits third to left of one who belongs to
11th grade.
Hence, option B is correct choice.

290) Answer: C
Explanation:
Clearly, direction of facing of S is not known.
Hence, option C is correct option.

291) Answer: D
Explanation:
After rearranging students in ascending order of
grade from left to right we get:
Case (1a) is not valid as T sits second to left of V.
Again, we have:
 Person adjacent to V sits facing in same direction but Clearly, only two person remains unchanged.
opposite in direction to V, that means X sits facing Hence, option D is correct choice.
north.
 X and W sits facing in opposite same direction, that 292) Answer: E
means W also sits facing south. Explanation:
 At least four person sits facing south. Clearly, direction of facing of P is not known.
 Based on above given information we have final Hence, option E is correct choice.
arrangement as follow:
293) Answer: B
Explanation:
Since, Q sits second to left of P, that means P must
facing north.
As, at least four person sits facing south that means S
must facing south.
289) Answer: B
Thus, option B is correct choice.
Explanation:

Set-60:
(Directions 294–298):
294) Answer: B
295) Answer: C
296) Answer: E
297) Answer: A
298) Answer: C

Explanation:
We have:
 S sits third to left of T, who sits third to left
of PO. Neither PO nor S sits at end of the row. W
sits third to left of V, who is a Teacher and Teacher
sits immediate neighbor of T and the one who is a
writer sits immediate left of W, that means we have
three possible case for V. In case (1) S sits second
from left end and V sits immediate right of T, in case
(2a) S sits third from left end and V sits immediate Again, we have:
 The one who is a HR sits third to left of Q,
left of T, in case (2b) S sits third from left end and V
sits immediate right of T. who does not sit at extreme ends of the line. Q is not
 U sits immediate neighbor of one who is PO a PO. Only three persons sits between HR and Poet,
and sits third to right of Teacher. Distance between S who sits immediate neighbor of R, that means case
and W is 5 meter. (1) is not valid, in case (2a) HR sits immediate left of
Based on above given information, we get following V and U is Poet, similarly case (2b) HR sits
result: immediate left of T and Poet sits at left end of the
row.
 Distance between V and T is the highest,
since no person sits between V and T in both case
(2a) and (2b) thus, possible highest value is 10m.
Based on above given information, we get following
result:
Again, we have: 16m and only one person sits between Q and R, thus
 P sits second to left of Doctor, who neither only possible combination available is (9 + 7) or (7
sits immediate neighbor of HR nor immediate +9). As QU is common in both cases thus, only
neighbor of U, that means in case (2a) P is HR and T possible distance between Q and U is 7m. Thus,
must be a Doctor, in case (2b) no such place distance between TQ is 6m and that of UR is 9m.
available for P, thus case (2b) is not valid.  Distance between HR and Poet is 25m, as
 Distance between Poet and Doctor is 13m distance between V and U, who is poet is 23m thus,
and distance between Q and R is 16m, since only one distance between P, who is HR and V is (25 – 23) =
person sits between T, who is Doctor and U, who is 2m.
poet thus, only possible combination of distance is (6 Based on above given information, we have
+ 7) or (7 + 6), similarly distance between Q and R is following result:
distance between Clerk and lawyer is either (7 + 9 +
Again, we have: 3) = 19m or (7 + 9 + 8) = 24m thus, none of
 Number of person sits between Z and P is available distance is true, if W is Lawyer then
same as that of Clerk and Y. Distance between Z and distance between Clerk and lawyer is either (5 + 2 +
MR is 9m, that means Z must sits at left end of the 10 + 6 + 3) = 26m thus, distance between S and P is
row, possible distance Z and W is 4m, thus Q must 3m.
be clerk and Y sits at extreme right end.  Thus, Y must be Engineer and distance
 Distance between Clerk and Lawyer is 26m, between R and Y be 8m.
since only two possible distance is available 3m and Based on above given information, final arrangement
8m. As only two possible position available for is as follow:
Lawyer, if Lawyer sits at extreme right end, then
Set-61:
(Directions 299–303):

299) Answer: D Explanation:


300) Answer: C The distance between immediate neighbours of any
301) Answer: B two people should not be greater than 144m and less
302) Answer: A than 48m. A is 312m away from F who is third
303) Answer: B eldest person in a family. Four possible cases.
H faces north and sits third to the right of her son in south and sits second to the right of B. Try the
law. Only one person sits to the left of H‟s son in arrangement for these statements separately and then
law. Number of persons sits left of H is two less compare this arrangement with above four possible
than that of number of person sits right of B. A faces cases. All the cases are eliminated except Case 2.
B who is not married person sits 288m away from Immediate neighbour of D belongs to same gender.
eldest person. E born in 2005 and sits at one of the Therefore B is male. And finally we
extreme ends. E‟s age is 16 and sits at right extreme knowgenerations of H, C and B. As H should be
end. elder and C should be elder than B but younger than
H‟s daughter in law is an immediate neighbour of F H and B is youngest.
and B. G and C are immediate neighbours. So H‟s G – 1953 – 64 Years
daughter in law is D. H – 1957 – 60 Years
A‟s father who is not C sits second to the right of E. F – 1982 – 35 Years
So G is father of A and E faces south. By the C – 1983 – 34 Years
statement persons at the extreme ends faces same D – 1985 – 32 Years
directions F also faces south direction. Not more A – 1987 – 30 Years
than two people facing same direction are sitting E – 2005 -12 Years
together. By this we know all the persons directions. B – 2001 – 16 Years
A is sister of F. E is daughter of C. D is two year The family tree and final arrangement is
elder than her sister-in-law. A is not born in
1985.D‟s age is 32and A‟s age is 30.
PUZZLE-1
Directions (1-5): Study the following information carefully and answer the given
questions.
Eight friends Venkat, Abdulla, Rakshan, Vikram, Prakash, Suman, John and
Manoj live on eight different floors of a building but not necessarily in the same
order. The lowermost floor of the building is numbered 1 and the topmost floor of the
building is numbered 8. Each of them likes different teams of Champions trophy
cricket viz, England Team, India Team, Australia Team, Bangladesh Team, New
Zealand Team, Pakistan Team, South Africa Team and Sri Lanka Team but not
necessarily in the same order.
The one who likes India Team lives on an even-numbered floor but not on the
topmost floor. Only one person lives between Suman and the one who likes New
Zealand Team. Only two persons live between Suman and the one who likes India
Team. Neither Prakash nor Rakshan lives on the first floor. Only one person lives
between Rakshan and the one who likes Australia Team. Venkat lives just above
Suman. Only two persons live between Prakash and Venkat. The one who likes
New Zealand Team does not live on floor number one, Abdulla lives on an even-
numbered floor and just above Rakshan. The one who likes Bangladesh Team lives
on an even numbered floor and lives just above the person who likes Sri Lanka
Team. Rakshan does not like New Zealand Team or Sri Lanka Team. Only two
persons live between the one who likes Pakistan Team and the one who likes
England Team. Vikram does not like South Africa Team. The one who likes Pakistan
Team does not live on an odd-numbered floor. Manoj lives just below the one who
likes Sri Lanka Team.

1). Who lives on fifth floor?


a) Abdulla
b) Prakash
c) The one who likes Bangladesh Team
d) The one who likes New Zealand Team
e) Both option b and d

2). John likes which of the following team?


a) South Africa Team
b) Australia Team
c) Bangladesh Team
d) Sri Lanka Team
e) England Team

3). How many persons are between the one who likes Bangladesh Team and the
one who stays on 4th floor?
a) One
b) Two
c) Three
d) Four
e) Five

4). Four of the following five are alike in a certain way and hence they form a group.
Which one of the following does not belong to that group?
a) Venkat
b) Rakshan
c) Manoj
d) Abdulla
e) John

5). Which of the following combinations is true?


a) 1 – Vikram - South Africa Team
b) 4 – Prakash - India Team
c) 3 – Suman - Sri Lanka Team
d) 6 – Manoj - Pakistan Team
e) 7 – Rakshan - England Team
PUZZLE-2
Directions (6-10): Study the following information carefully and answer the given
questions.
Varun, Rajesh, Sunil, Jeeva, Vasanth, Parthiban and Avinash are seven
persons from different States, viz Gujarat, Bihar, Maharashtra, Haryana, Punjab,
Assam and Rajasthan. They go to top 4 cities in India, viz. Mumbai, Hyderabad,
Bangalore and Chennai only on Friday but not necessarily in the same order. At
least one Person goes to one city, but no city is visited by more than two Persons.
Parthiban, who is from Maharashtra, goes alone to Bangalore. The one who is from
Haryana does not go to Mumbai. Also, he never goes either with Vasanth or with
Avinash. Jeeva goes to Chennai with the person who is the Person from Rajasthan.
Sunil goes to Hyderabad. Avinash is not a Person from Rajasthan. The one who is
from Haryana goes to the city with the person who is the Person of Gujarat. The one
who is from Punjab goes to Chennai. Varun is a Person from neither Bihar nor
Rajasthan. The one who is from Assam goes to Mumbai neither with Vasanth nor
with Jeeva. The person who is from Bihar goes to the city with Varun.

6). Who among the following persons visit Hyderabad?


a) Jeeva and Rajesh
b) Vasanth and Sunil
c) Rajesh and Sunil
d) Parthiban and Rajesh
e) Cannot be determined

7). Rajesh is the person from which of the following state?


a) Haryana
b) Gujarat
c) Bihar
d) Rajasthan
e) Either Haryana/Gujarat

8). Which of the following city visited by Avinash?


a) Bangalore
b) Mumbai
c) Chennai
d) Hyderabad
e) None of these

9). Which of the following statements is wrong?


a) Jeeva – Hyderabad - Punjab
b) Vasanth – Chennai - Rajasthan
c) Parthiban – Bangalore - Maharashtra
d) Varun – Mumbai - Assam
e) All statements are true

10). Which of the following combinations is true?


a) Jeeva – Chennai - Bihar
b) Varun – Hyderabad - Assam
c) Vasanth – Chennai - Punjab
d) Avinash – Mumbai - Bihar
e) None of these

Explanation With Answer Key:


Directions (Q. 1-5):
1). Answer: e)
2). Answer: a)
3). Answer: c)
4). Answer: b)
5). Answer: d)

Directions (Q. 6-10):

6). Answer: c)
7). Answer: e)
8). Answer: b)
9). Answer: a)
10). Answer: d)

PUZZLE-3
Directions (Q. 6-11): Study the following information carefully and answer the
questions given below:
Seven employees H, I, J, K, L, M and N working in different companies reach
their offices every day by a train which stops at five stations A, B, C, D and E
respectively after leaving station Z.
 Three of them board the train at station Z.
 K deboards at the station next to the station at which M deboards.
 I does not deboard either with H or with L.
 N alone boards at station C and deboards with J after passing one station.
 None of them boards at station B and none of them deboards the train at station A.
 J boards with M but does not board with either I or with K.
 L boards with two others and deboards alone after K.
 I and K deboard together at station C.
 H travels between two consecutive stations and deboard at the station E.
 M does not board after L.

6). At which of the following stations does L deboard?


a) B
b) D
c) C
d) Can't be determined
e) None of these

7). At which of the following stations do J and M board the train?


a) A
b) C
c) D
d) E
e) None of these

8). After how many stations does L deboard?.


a) One
b) Two
c) Three
d) Four
e) None of these

9). How many persons deboard at station E?


a) None
b) One
c) Two
d) Three
e) None of these

10). H deboards the train how many stations after I deboards?


a) One
b) Two
c) Three
d) None
e) None of these

11). Who among the following board at station A?


a) I,K
b) M, L
c) K,J
d) L, M, J
e) None of these
Answers:
6)b 7)e 8)d 9)d 10)b 11)a

Explanation:
Directions (Q. 6-10):

PUZZLE-4
Directions (1-5): Study the following information carefully and answer the given
questions.
In HDFC Bank the staffs Gopinath, Sanjay, Anwar, Latif, Jemima, Shalini,
Monica, Mishraya are working and they come to the Bank by different express such
as Shiv Ganga Express, Malwa Express, Suryanagri Express, Magadh Express,
Pandian Express, Nellai Express, Archana Express and Vaishali Express not in the
respective order. And they are selected for special seminar. They planned to take
seminar individually with other staffs in different States such as Bihar, Haryana,
Rajasthan, Kerala, Karnataka, Maharashtra, Assam and Gujarat in a week starting
from Sunday to Saturday. The maximum number of seminar taken by each person is
one.
Gopinath going to take seminar before Sanjay and after Mishraya who did not
come to the bank by Suryanagri Express. The staff who is going to take seminar in
Rajasthan, come to Bank by Nellai Express. Anwar come to Bank by Malwa Express
and he did not take seminar on week’s starting day. The staff who take seminar in
Gujarat, attend seminar in Maharashtra and Karnataka in the previous day.
Mishraya likes to take seminar in Bihar and she did not come by Archana Express.
Only two persons giving their seminar on same day due to other commitments. The
person who is coming to Bank by the way of Pandian Express likes to take seminar
on Thursday. Sanjay likes to take seminar in Assam and the day in between the
persons who take seminar in Kerala and Gujarat. Monica comes to Bank by Archana
Express and she did not take seminar on Saturday. The persons who come to the
bank by Magadh Express and Malwa Express likes to take seminar on same day.
The person who takes seminar in Assam come to the bank by Shiv Ganga Express
and attend Jemima’s seminar who come to Bank by Pandian Express in the next
day. Gopinath who did not come to the bank by Vaishali Express taking seminar on
the next day of Latif’s seminar. Latif who making arrangement with another staff for
taking seminar on same day and he did not take seminar in Maharashtra.

1). Latif comes to the Bank by which express?


a) Magadh Express
b) Shiv Ganga Express
c) Pandian Express
d) Nellai Express
e) Malwa Express

2). Who takes the seminar on Friday?


a) The one who takes seminar in Haryana
b) Monica
c) Both a and b
d) The one who takes seminar in Assam
e) The one who come to the Bank by Shiv Ganga Express
3). Who come to the Bank by Nellai Express?
a) Mishraya
b) Latif
c) Monica
d) Shalini
e) Jemima

4). Two seminars held on which of the following day?


a) Thursday
b) Tuesday
c) Saturday
d) Friday
e) Monday

5). Which of the following combinations is true?


a) Mishraya - Archana Express – Bihar - Thursday
b) Sanjay - Shiv Ganga Express – Assam - Wednesday
c) Gopinath - Suryanagri Express – Kerala - Wednesday
d) Shalini - Vaishali Express – Rajasthan - Saturday
e) None of these

PUZZLE-5
Directions (6-10): Study the following information carefully and answer the given
questions.
Eight persons Akram, Manish, Vinay, Mishra, Harshan, Irfan, Ramkumar and
Santosh are seated around a circular table such that four faces the centre and four
are facing opposite to the centre. They like different IPL Cricket teams: Kolkata
Knight Riders, Rising Pune Giants, Gujarat Lions, Mumbai Indians, Sunrisers
Hyderabad, Delhi Daredevils, Kings XI Punjab and Royal Challengers Bangalore not
necessarily in the same order. The arrangement is based on the following rules:
One who likes Royal Challengers Bangalore is seated opposite to Akram. Mishra is
3rd to the right of the one who likes Delhi Daredevils who is seated between Manish
and the one who likes Gujarat Lions. Ramkumar is 3rd to the left of Vinay who likes
Kings XI Punjab. Santosh is to the immediate left of the one who likes Sunrisers
Hyderabad. One who likes Royal Challengers Bangalore is a neighbour of Mishra.
Akram does not like Delhi Daredevils and Ramkumar does not like Royal
Challengers Bangalore. Akram is 2nd to the left of Harshan who is not a neighbour
of Mishra. Either Akram or Ramkumar likes Sunrisers Hyderabad. Mishra and
Manish are seated facing the same direction.. The one who likes Gujarat Lions is
2nd to the right of the one who likes Rising Pune Giants. Mishra and Santosh do not
like Rising Pune Giants, Ramkumar faces outside the centre. One who likes Mumbai
Indians is facing the opposite direction faced by Vinay. Not more than two persons
who sits on consecutive seats face the same direction.

6). Who among the following is not facing centre?


a) Vinay
b) Harshan
c) Akram
d) Santosh
e) Irfan

7). Who among the following likes Delhi Daredevils?


a) Akram
b) Santosh
c) Irfan
d) Harshan
e) None of these
8). Who is sitting third to the right of the person who like Kings XI Punjab team?
a) The one who likes Gujarat Lions team
b) The one who likes Rising Pune Giants team
c) The one who likes Royal Challengers Bangalore team
d) The one who likes Delhi Daredevils team
e) None of these

9). The only person who is sitting between Santosh and Ramkumar likes which of
the following teams?
a) Gujarat Lions
b) Mumbai Indians
c) Sunrisers Hyderabad
d) Kings XI Punjab
e) Kolkata Knight Riders

10). Which of the following is correctly matched?


a) Mishra – Facing Centre – Kolkata Knight Riders
b) Santosh - Facing Outside – Gujarat Lions
c) Irfan – Facing Outside – Royal Challengers Bangalore
d) Vinay – Facing Centre – Mumbai Indians
e) Manish – Facing Outside – Rising Pune Giants

Explanation With Answer Key:


Directions (Q. 1-5):
1). Answer: a)
2). Answer: c)
3). Answer: d)
4). Answer: e)
5). Answer: b)

Directions (Q. 6-10):


6). Answer: a)
7). Answer: d)
8). Answer: b)
9). Answer: c)
10). Answer: e)

PUZZLE-6
Directions (1-5): Study the following information carefully and answer the given
questions.
Nine people Sandeep, Harish, Lenin, Ravi, Abbas, Dinesh, Nisar, Venkat and
Sumanth stay in a building. The building has nine floors and only one person stays
on one floor. Each of them likes different colour uniforms namely- Red, Blue, Green,
Yellow, White, Orange, Brown, Grey and Violet. Each person belongs to different
companies, i.e. Samsung, Onida, Panasonic, Videocon, LG, Whirlpool, Toshiba,
Sharp and Sony but not necessarily in the same order. The ground floor is
numbered 1, the floor above it is numbered 2, and so on, and the topmost floor is
numbered 9.
The one who belongs to Sharp Company stays on the 4th floor. There are 4 floors
between the floors on which Ravi & Nisar stay with Ravi living above Nisar. Lenin
stays on the floor immediately above Venkat's floor. The one who likes Red colour
uniform stays on an even numbered floor and is a neighbour of the one who likes
Blue colour uniform. Sumanth does not belong to Onida company. The one who
belongs to Samsung company stays on the topmost floor. The one who likes Brown
colour uniform stays immediately below the one who likes White colour uniform.
Abbas likes Grey colour uniform. Dinesh belongs to LG company and stays on 8th
floor. There are 3 persons between the one who likes Yellow colour uniform and the
one who like Grey colour uniform. There are two floors between the floors on which
people from Sony company and Sharp company stay. Abbas stays immediately
below the floor on which the person from Videocon company lives. The person who
likes Orange colour uniform belongs to Onida company and stays on an odd
numbered floor. Sumanth stays on the bottommost floor. The person who likes
Green colour uniform is from Videocon company and stays on 6th floor. The person
from Toshiba company stays on 2nd floor. The persons from Whirlpool company
and Onida company are not the neighbours of the person from Videocon company.
Ravi likes Blue colour uniform and Nisar does not like Red colour uniform. Harish
does not live below the floor of Sandeep.

1). Which of the following persons sits exactly between Sumanth and the one who
likes Violet colour uniform?
a) Venkat and Nisar
b) Lenin and Abbas
c) Nisar and Ravi
d) Sandeep and Venkat
e) Harish and Abbas

2). Who lives in the Third floor?


a) Abbas
b) The one who likes Violet colour uniform
c) The one who is from Onida company
d) Dinesh
e) None of these

3). Green Colour uniform like by which of the following person?


a) Ravi
b) Sandeep
c) Nisar
d) Sumanth
e) Lenin
4). How many persons sit between Nisar and the one who likes Yellow colour
uniform?
a) Five
b) Three
c) Four
d) Two
e) Six

5). Which of the following combinations is true?


a) 4 - Sharp - Lenin - Yellow colour uniform
b) 3 –Onida – Nisar - White colour uniform
c) 8 - Samsung – Harish - Green colour uniform
d) 7 - Sony - Ravi - Blue colour uniform
e) None of these

PUZZLE-7
Directions (6-10): Study the following information carefully and answer the given
questions.
Nine persons Subash, Kumaran, Tharoon, Ajay, Yadav, Praveen, Achuthan, Niraj
and Ashok are sitting in a row facing in either north or south direction but not
necessarily in the same order. They work in different countries they are Australia,
India, Russia, USA, Japan, Kuwait, Canada, Thailand and UK but not necessarily in
this order. The following information is known about them.
Praveen and Ashok are immediate neighbours of Tharoon but face in opposite
directions to each other. Achuthan and Niraj face in the same direction. Subash who
works in Japan sits in the middle and faces towards south. The person working in
Kuwait sits to the immediate right of a person facing in north direction. The person
sitting at first and last position from left face in north direction and they work in
Canada and Russia respectively. The person working in Thailand sits to the
immediate right of Kumaran. Praveen does not sit with Subash. Yadav sits second
to the left of Ajay but does not work in Russia. Niraj sits two places away from
Subash. Ajay sits exactly between Achuthan and Niraj. Also, there is only one
person sitting between Achuthan and Niraj. Kumaran and Tharoon face in the same
direction. The person who works in India sits fourth to the left of the person working
in Kuwait but neither of them is Niraj. The persons working in UK and Australia are
sitting adjacent to each other. The person working in Australia sits fourth to the right
of Praveen.

6). Who works in Australia Country?


a) Kumaran
b) Praveen
c) Niraj
d) Ajay
e) Yadav

7). In which of the following country Achuthan works?


a) USA
b) Kuwait
c) Thailand
d) UK
e) Russia

8). Which of the following persons sits at the extreme ends of row?
a) Ajay and Tharoon
b) Kumaran and Ashok
c) Niraj and Achuthan
d) Ashok and Yadav
e) Yadav and Subash

9). The one who works in Japan sits third to the left of ______________
a) Achuthan
b) Praveen
c) The one who works in UK
d) The one who works in India
e) Both option c and d

10). Which of the following combinations is true?


a) Achuthan - Kuwait – Facing South
b) Tharoon – India – Facing South
c) Yadav – Australia – Facing North
d) Ajay – Canada – Facing South
e) None of these

Explanation With Answer Key:


Directions (Q. 1-5):

1). Answer: a)
2). Answer: c)
3). Answer: b)
4). Answer: e)
5). Answer: d)
Directions (Q. 6-10):

6). Answer: d)
7). Answer: b)
8). Answer: d)
9). Answer: e)
10). Answer: a)

PUZZLE-8
Directions (1-5): Study the following information carefully and answer the given
questions.
Twelve friends are sitting in two parallel rows of chairs containing six people
each, in such a way that there is equal distance between adjacent persons. In row 1:
Dinesh, Vignesh, Rakesh, Charan, Eshwar and Karthik are seated and all of them
are facing south. In row 2: Hafiz, Jaypal, Naren, Praveen, Murugan and Arjun are
seated and all of them are facing north. Each of them likes different car such as,
Maruti Suzuki, Hyundai, Honda, Toyota, Tata, Ford, Chevrolet, Volkswagen,
Mahindra, Nissan, Skoda and Fiat but not necessarily in the same order. Each of
them like different players: Roger Federer, Rafael Nadal, Novak Djokovic, Andy
Murray, Stan Wawrinka, Milos Raonic, Kei Nishikori, David Ferrer, Fernando
Verdasco, Jack Sock, Dominic Thiem and Pete Sampras. In the given seating
arrangement, each member seated in a row faces another member of the other row.
Eshwar, whose favourite player is Milos Raonic sits third to the left of the person
who likes Hyundai and not sitting at the extreme ends, Vignesh, who likes Toyota,
does not sit at an extreme end of the row. If Eshwar sits at an extreme end then
Vignesh doesn’t sit to the immediate right of the one who likes Hyundai. Naren and
Hafiz face the persons whose favourite players are Milos Raonic and Stan Wawrinka
respectively. There are two persons sitting on the right side of Jaypal, who sits
second to the right of the person, whose favourite player is Jack Sock. Jaypal, who
likes Tata, faces the immediate neighbour of Dinesh. Dinesh likes Honda. Hafiz sits
with the persons who like Tata and Fiat. Karthik faces the person who likes Skoda
car and Pete Sampras player but Karthik does not like Hyundai. The one who likes
Fiat car doesn’t sit with the one who likes Skoda car. The person, who likes Nissan
car, sits second to the right of the person who likes Ford. Praveen does not like
Skoda and faces the person, who likes Maruti Suzuki. The person who likes
Fernando Verdasco is an immediate neighbour of the person who likes Volkswagen.
Rakesh’s favourite player is Andy Murray and he doesn’t sit with Dinesh. Naren
does not like Rafael Nadal player. The person, whose favourite player is Roger
Federer, faces the person, who sits second to the right of the person, whose
favourite player is Stan Wawrinka. The person whose favourite player is David
Ferrer does not sit in row 2 and faces the person who likes Dominic Thiem. Murugan
faces the person who likes Kei Nishikori. Rakesh doesn’t face the person who likes
Ford car. The one who likes Chevrolet sits in Row 2.

1). Who likes the Jack Sock player?


a) Murugan
b) Hafiz
c) Arjun
d) Naren
e) Praveen
2). The one who likes Kei Nishikori sits third to the right of ______________
a) Dinesh
b) Vignesh
c) Eshwar
d) Karthik
e) Rakesh

3). Which of the following car likes by Hafiz?


a) Skoda
b) Hyundai
c) Honda
d) Ford
e) Volkswagen

4). Which of the following persons sits at the extreme ends of one row?
a) Praveen and Hafiz
b) Murugan and Naren
c) Charan and Karthik
d) Dinesh and Eshwar
e) Praveen and Arjun

5). Which of the following combinations is true?


a) Karthik – Maruti Suzuki - Novak Djokovic
b) Eshwar – Ford – Milos Raonic
c) Naren – Nissan – Stan Wawrinka
d) Vignesh – Toyota - David Ferrer
e) None of these
PUZZLE-9
Directions (6-10): Study the following information carefully and answer the given
questions.
The seven foreign banks Deutsche Bank, Doha bank, Barclays Bank, Citi Bank,
Standard Chartered Bank, Royal Bank of Canada, HSBC Bank are having their
headquarters in different places namely USA, Canada, China, Japan, Russia, UK
and Qatar not in the respective order. And the banks making contract with insurance
companies such as New India Assurance, The Oriental Insurance Company, United
India Insurance Company, National Insurance Company, Export Credit Guarantee
Corporation Of India, General Insurance Corporation and Life Insurance
Corporation. The insurance companies are located in different places such as
Chennai, Hyderabad, Bangalore, Delhi, Mumbai, Kolkata and Patna not in
the respective order.
 Life Insurance Corporation and National Insurance Company are not contact with
Canada and UK based banks respectively. Either Deutsche Bank or HSBC Bank
likes to contract with United India Insurance Company which is not located in
Kolkata and Hyderabad
 The insurance company which located in Mumbai contract with the bank that is
located in Japan.
 Royal Bank of Canada contract with Hyderabad based insurance company and is
not located in Russia and Canada.
 General Insurance Corporation is Bangalore based insurance company. And
Kolkata based insurance company not deals with HSBC Bank and Deutsche Bank.
 USA and China based banks making contract with Delhi and Patna based insurance
companies respectively.
 Deutsche Bank is not contracting with the Delhi and Mumbai based insurance
companies. Barclays Bank is having headquarters in Qatar.
 Citi Bank do not contract with insurance companies located in Chennai, Mumbai and
also General Insurance Corporation Company
 The Mumbai based The Oriental Insurance Company does not contract with
Standard Chartered Bank and Doha bank. HSBC Bank which has headquarters in
Japan doesn’t contract with Chennai based New India Assurance company. General
Insurance Corporation will contract with Qatar based bank.
 Chennai based insurance company making contract with Standard Chartered Bank
which not have headquarters in Canada and UK
 Export Credit Guarantee Corporation of India is not located in Patna and Kolkata
and makes deal with Doha bank Company

6). Which of the following bank headquarters located in Russia?


a) HSBC Bank
b) Barclays Bank
c) Royal Bank of Canada
d) Doha bank
e) Standard Chartered Bank

7). Citi Bank Contract with which of the following insurance company?
a) Export Credit Guarantee Corporation of India
b) National Insurance Company
c) United India Insurance Company
d) Life Insurance Corporation
e) New India Assurance

8). The Oriental Insurance Company contract with which of the following banks?
a) Barclays Bank
b) Royal Bank of Canada
c) Standard Chartered Bank
d) HSBC Bank
e) Doha bank
9). If ‘Deutsche Bank’ is related to ‘Kolkata’, ‘Citi Bank’ is related to ‘Hyderabad’,
then which following is ‘Royal Bank of Canada’ related to?
a) Mumbai
b) Chennai
c) Delhi
d) Bangalore
e) Patna

10). Which of the following combinations is true?


a) Doha Bank – Russia – Export Credit Guarantee Corporation Of India - Chennai
b) Deutsche Bank – Japan – The Oriental Insurance Company - Mumbai
c) Citi Bank - Canada – National Insurance Company - Kolkata
d) Standard Chartered Bank – USA – New India Assurance - Delhi
e) None of these

Explanation With Answer Key:


Directions (Q. 1-5):

1). Answer: a)
2). Answer: c)
3). Answer: d)
4). Answer: e)
5). Answer: d)

Directions (Q. 6-10):

6). Answer: e)
7). Answer: b)
8). Answer: d)
9). Answer: a)
10). Answer: c)

PUZZLE-10
Directions (Q. 1-5): Study the following information carefully to answer the given questions:
There are six persons- Janani, Vimala, Ramya, Bharati, Hasina and Narmada living in a
triple floor building with six flats. The floors are First, Second and Third, each having two
flats. Those who have four or more degree occupy the Third floor and using different
Devices. The one who work in Yes Bank uses Mobile and she is not Narmada while the one
who work in Federal Bank uses Tablet. Both of them occupied same floor. Vimala who work
in HDFC Bank has three degree and not stayed in the Third floor. Ramya, an unmarried
woman, does not have any degree, occupies the Second floor. Hasina and Narmada have
Luxury watches while the rest have Sports watches. Bharati, a Second floor occupant, uses
Computer. One of the two persons having two degree uses Laptop and work in Axis Bank
also occupied in first floor. Two persons, of whom one work in ICICI Bank, do not using any
Devices.

1). What is the least number of degrees had by all the persons in the entire building?
a) 14
b) 16
c) 15
d) 17
e) Cannot be determined

2). Who among the following occupies the Third floor?


a) Janani
b) Hasina
c) Ramya
d) Bharati
e) None of these

3). In which Bank does Bharati work in?


a) ICICI Bank
b) Yes Bank
c) Axis Bank
d) Federal Bank
e) Cannot be determined

4). How many degrees does Janani have?


a) None
b) One
c) Two
d) Three
e) None of these
5). Which of the following device Ramya Using?
a) Tablet
b) Mobile
c) Tablet or Mobile
d) No Device

e) Computer

PUZZLE-11
Directions (Q. 6-10): Study the following information carefully and answer the given
questions
Eight friends- Vishnu, Suresh, Aravind, Selvam, Prem, Johnson, Deepak and Mahesh visit
different countries viz. Japan, Singapore, Russia and Switzerland are sitting around a
circular table facing the centre of the table. Each country is being visited by two people only,
but not necessarily in the same order. All these friends have different mobiles i.e. -
Samsung, Microsoft, Vivo, Oppo, Motorola, Redmi, HTC and Lenovo. No two people visiting
the same country are sitting adjacent to each other except those visiting Singapore. The
person who has Oppo mobile is sitting on the immediate left of the person who has Lenovo
mobile. Prem neither has Lenovo nor Vivo mobile. Mahesh has Redmi mobile and visits
Russia and is sitting to the immediate left of Suresh, who visits Singapore. Suresh does not
have Vivo mobile. Johnson has Motorola mobile and visits Singapore, who is sitting
opposite to Deepak. Only Vishnu, who has Microsoft mobile, is sitting between Deepak,
who has Oppo mobile and the person who has HTC mobile. Persons who visit Japan are
sitting opposite to each other. Each of the persons who visit Switzerland is sitting adjacent
to a person who visits Japan. Selvam does not have Lenovo mobile.

6). Which mobile is owned by Aravind?


a) Lenovo
b) Vivo
c) Samsung
d) Microsoft
e) None of these
7). Who has a Samsung mobile?
a) Suresh
b) Selvam
c) Aravind
d) Prem
e) None of these

8). Which of the following pair is sitting adjacent to the person who has Motorola mobile?
a) Suresh and Mahesh
b) Prem and Selvam
c) Selvam and Mahesh
d) Mahesh and Aravind
e) None of these

9). Who among the following visit Japan?


a) Aravind and Mahesh
b) Prem and Vishnu
c) Selvam and Aravind
d) Vishnu and Johnson
e) None of these

10). Who is sitting opposite to the person who has Redmi mobile?
a) The person who has Oppo mobile
b) The person who has HTC mobile
c) The person who has Microsoft mobile
d) The person who has Vivo mobile
e) None of these
ANSWERS - a,b,e,c,d,a,a,e,c,b
PUZZLE-12
Directions (Q. 6-11): Study the following information carefully and answer the
questions below:
A, B, C, D, E, F, G and H are sitting around a circular table facing the centre.
Each of them has a mobile of a different company viz, Samsung, Nokia, LG, HTC,
Lenovo, Moto C, Apple and Intex, but not necessarily in the same order.
 Only two persons sit between G and the one who has Apple.
 Only one person sits between E and the one who has Nokia.
 B sits second to the left of the one who has Nokia.
 H is an immediate neighbour of D. The one who has LG sits second to the right of
H. D is not an immediate neighbour of B or E and he does not have Apple.
 E sits second to the left of the one who has Apple.
 Only three people sit between the one who has LG and the one who has HTC. F
sits second to the left of the one who has HTC.
 H does not have Lenovo. The one who has Lenovo is not an immediate neighbour
of G or H.
 C sits on the immediate left of the one who has Lenovo.
 The one who has Intex sits on the immediate right of the one who has Samsung.
6). Who among the following has Moto G?
a) A
b) B
c) H
d) C
e) None of these

7). Who among the following sits exactly between B and the one who is on the
immediate right of A?
a) The one who has Lenovo
b) The one who is opposite D
c) The one who has Intex
d) The one who has Moto G
e) None of these

8). How many persons sit between the one who has Samsung and the one who has
Lenovo?
a) One
b) Two
c) Three
d) Four
e) None of these

9). Which of the following statements is/are true?


a) Only one person sits between C and G.
b) The one who has HTC sits second to the left of C.
c) G has Intex.
d) The one who has Samsung sits second to the left of A.
e) None of these

10). C has which of the following mobiles?


a) Apple
b) Lenovo
c) Moto G
d) Nokia
e) None of these

11). What is the position of G with respect to the one who has Intex?
a) Second to the left
b) Third to the right
c) Third to the left
d) Fifth to the left
e) None of these

Explanation With Answer Key:

Directions (Q. 6-10):

6). B)
7). A)
8). C)
9). D)
10). A)
11). C)

PUZZLE-13
Directions (Q. 1-5): Study the following information carefully to answer the given
questions:
There are seven people Naveen, Kailash, Laxman, Lokesh, Santhosh,
Abinash and Udayan. They like different Sims, Wonders and were born in different
countries and months. They all were born in the same year. One of the Sim is JIO.
One of the Countries is Poland. One of the Wonders is Machu Picchu. One of the
persons was born in the month of August. The following information is known about
them. Abinash was born in India or in Malaysia. The one who was born in the month
of April likes Great Pyramid of Giza and Vodafone sim and is either Laxman or
Udayan. Lokesh is not the one who likes Petra. The one who likes Colosseum likes
Idea sim. The one who was born in Malaysia is the youngest one and is either
Kailash or Udayan. The one who likes Christ the Redeemer was born in Singapore
but is not Kailash. Santhosh was born in the month of January. Lokesh was born in
Bangladesh in the month of June and likes Docomo sim. The one who likes Taj
Mahal was born in October but is not the youngest person. Kailash likes Aircel sim.
The one who likes Colosseum was born in China but that person is not Laxman who
likes BSNL sim. The one who was born in Russia likes Airtel sim but is not
Santhosh. Abinash is the second youngest person. The one who likes Great Wall of
China is one month younger than Santhosh.

1). Who likes JIO sim?


a) Abinash
b) Udayan
c) Naveen
d) Kailash
e) Laxman

2). Which of the following Wonders does Lokesh like?


a) Taj Mahal
b) Machu Picchu
c) Great Pyramid of Giza
d) Great Wall of China
e) Christ the Redeemer

3). Which month Laxman was born?


a) February
b) April
c) August
d) October
e) Cannot be determined

4). Who was born in Poland?


a) Santhosh
b) Naveen
c) Kailash
d) Udayan
e) Laxman

5). Which of the following statements is correct?


a) Santhosh likes Machu Picchu Bank
b) Naveen likes Vodafone sim
c) Udayan was born in the month of August
d) Kailash was born in India
e) None of these

PUZZLE-14
Directions (Q. 6-10): Study the following information carefully and answer the given
questions.
E, F, G, H, I, J, K and L are eight family members sitting around a circular
table in a restaurant. All of them are facing the centre. Three of the family members
are engineering graduates and others are arts graduates. All the eight family
members are bankers. They are working in different banks Andra Bank, Dena Bank,
Syndicate Bank, Bank of India, Vijaya Bank, UCO Bank, Canara Bank and
Allahabad Bank, but not necessarily in the same order. There are two married
couples, three brothers, two daughters, one granddaughter and one grandson in the
group. Two of the female members in the group E and L get same degree. They
studied different degrees in colleges, they are B.com, BCA, M.com, BE, MBA, ME
and B.sc.
K, works in Andra Bank, is the head of the family, and is immediate neighbour
of L and H. The granddaughter J is sitting third to the left of K. The grandmother of J
is sitting on the immediate left of her. J is arts graduate. I is sitting second to the
right of his father K. He works in Vijaya Bank, and studied B.sc. Only K’s daughter is
sitting exactly between E and I. She studied MBA and she works in UCO Bank. L
works in Syndicate Bank and F works in Bank of India. The son of L is sitting on the
immediate left of his father. His father works in Dena Bank. G works in UCO Bank
and is sitting on the immediate left of her mother. H, the husband of the one who
works in Syndicate Bank, studied ME and his mother studied BE. The one who
works in Bank of India sits second to the right of E, who is an immediate neighbour
of the one who works in UCO Bank and the Canara Bank. The one who works in
Canara Bank does not studied B.com and the one who works in Andra Bank does
not studied M.com or BCA.

6). The only person who is sitting between J and G is working on which of the
following bank?
a) Dena Bank
b) Bank of India
c) Andra Bank
d) Syndicate Bank
e) Allahabad Bank

7). Who among the following working in Canara Bank?


a) J
b) H
c) G
d) E
e) None of these

8). Who among the following is studied B.com degree?


a) J
b) K
c) E
d) F
e) None of these

9). The person who studied BE is working in which of the following banks?
a) Allahabad Bank
b) Bank of India
c) Syndicate Bank
d) Both Allahabad Bank and Syndicate Bank
e) Canara Bank

10). Which of the following is correctly matched?


a) F – Bank of India – M.com
b) I – Allahabad Bank – B.sc
c) H – Dena Bank - ME
d) G – UCO Bank – ME
e) None of these

Explanation With Answer Key:


Directions (Q. 1-5):
1). Answer: a)
2). Answer: b)
3). Answer: c)
4). Answer: d)
5). Answer: e)

Directions (Q. 6-10):

Family Tree:
Male (+); Female (-)
6). Answer: e)
7). Answer: a)
8). Answer: b)
9). Answer: d)
10). Answer: c)

PUZZLE-15
Directions (Q. 1-6): Study the following information carefully and answer the
questions given below:
Eight people P, Q, R, S, T, U, V and W are sitting around a circular table
facing the centre, but not necessarily in the same order. Each of them belongs to a
different state, viz Delhi, Jharkhand, Punjab, MP, UP, Kerala, Bihar and TN, but not
necessarily in the same order.
The one who belongs to Jharkhand is on the immediate right of T. T is third to
the left of V. T is not from Delhi. Neither Q nor W is an immediate neighbour of T. U
belongs to MP and is third to the right of the one who belongs to Jharkhand. The
one who belongs to Bihar sits second to the left of T. The person who belongs to UP
sits second to the right of the one who belongs to Punjab. R sits third to the left of U.
The one who belongs to Kerala sits second to the left of the one who belongs to MP.
P sits exactly between W and U and belongs to Punjab. Q sits fourth to the right of
W.
1). Who among the following comes from TN?
a) S
b) T
c) Q
d) P
e) None of these

2). Which of the following statements is true about Q?


a) Q sits on the immediate left of W.
b) Q sits second to the right of U.
c) Q belongs to Kerala.
d) All are true
e) None of these

3). Who sits second to the right of the one who belongs to TN?
a) The one who belongs to Kerala
b) The one who is an immediate neighbour of V and R
c) The one who sits third to the left of the one who belongs to Punjab.
d) The one who sits third to the right of the one who belongs to UP
e) All of these

4). Who among the following comes from Delhi?


a) R
b) W
c) V
d) Can't be determined
e) None of these

5). Which of the following combinations is true?


a) U - Kerala
b) T – Delhi
c) R - MP
d) S - UP
e) Q - Jharkhand

6). The one who is exactly between S and R belongs to which of the following
states?
a) Bihar
b) Jharkhand
c) UP
d) MP
e) None of these
Explanation With Answer Key:
Directions (Q. 1-6):

1). Answer: b)
2). Answer: c)
3). Answer: e)
4). Answer: c)
5). Answer: d)
6). Answer: e)
PUZZLE-16
Directions (Q. 1-5): Study the following information carefully and answer the given
questions.
Sanjay, Vasu, Gandhi, Jairaj, Prathap, Kumar, Suresh and Ramnath are eight
friends sitting around a circular table. Four of them are facing towards the centre and
four of them are facing away from the centre. All of them like different Currencies,
viz. Dollar, Pound, Dinar, Peso, Euro, Rupee, Yen and Riyal, but not necessarily in
the same order. Prathap faces towards the centre and likes Dollar. Both the
immediate neighbours of Prathap face away from the centre and like Dinar or Peso.
Jairaj faces away from the centre. Both the immediate neighbours of Jairaj do not
face away from the centre. Prathap sits third to the right of Kumar, who likes Euro
and faces away from the centre. Gandhi sits third to the left of Kumar. The one who
likes Dinar sits opposits of Kumar. The one who likes Pound is not the immediate
neighbour of Kumar and faces away from the centre. Sanjay sits second to the left of
Gandhi and does not like Yen or Riyal. The one who likes Yen sits between
Ramnath and Kumar. Vasu faces away from the centre and does not like Pound.

1). Who among the following sits second to the right of Vasu?
a) Jairaj
b) Ramnath
c) Gandhi
d) Kumar
e) None of these

2). The only person who is sitting between Prathap and Suresh, likes which of the
following currency?
a) Dinar
b) Euro
c) Rupee
d) Dollar
e) Peso

3). Who like the Rupee?


a) Prathap
b) Vasu
c) Sanjay
d) Ramnath
e) Gandhi

4). How many persons sit between Jairaj and Prathap, count from the right of Jairaj?
a) Five
b) Four
c) Three
d) Two
e) None of these

5). Which of the following is correctly matched?


a) Gandhi - Facing Centre - Euro
b) Suresh - Facing Outside - Peso
c) Kumar - Facing Centre - Dollar
d) Sanjay – Facing Centre - Rupee
e) None of these

PUZZLE-17
Directions (6-10): Study the following information carefully and answer the given
questions.
There are nine people Balaji, Darshan, Kalam, Sethu, Bilal, Ramnath, Lalit, Yuvan
and Nakul who sits around a circular table. Some are facing towards and some are
facing away from the centre. They like different pens among Parker, Cello, Camlin,
Hero and Montex. Not more than 2 people like the same pens. The following
information is known about them. Kalam and Lalit form the only pair that is sitting
together and like the same pens. They are facing in opposite directions to each
other. The one who likes Camlin sits third to the left of Darshan but is neither Kalam
nor Ramnath. Balaji sits second to the left of Ramnath, who likes Cello. The one
who likes Parker is an immediate neighbour of Yuvan and Bilal but none of them sits
adjacent to Kalam or Lalit. Balaji, Nakul and Yuvan face in same direction as that of
Bilal. Nakul sits third to the right of Sethu, who is facing away from centre and likes
Hero. Bilal sits second to the left of Kalam and likes Cello. The people who like Cello
face towards the centre. Balaji neither likes Camlin or Hero.

6). Who sits second to the right of Darshan?


a) Yuvan
b) Lalit
c) Kalam
d) Nakul
e) Bilal

7). Which of the following person likes Parker?


a) Lalit and Balaji
b) Yuvan and Darshan
c) Sethu and Nakul
d) Bilal and Kalam
e) Balaji and Nakul

8). Which of the following pen like by Yuvan?


a) Camlin
b) Parker
c) Cello
d) Montex
e) None of these

9). How many persons facing outside the circle?


a) Two
b) Five
c) Three
d) Six
e) Four

10). Which of the following combinations is true?


a) Yuvan– Facing outside –Camlin
b) Bilal– Facing centre– Parker
c) Balaji– Facing centre–Cello
d) Kalam – Facing outside – Montex
e) None of these

Explanation With Answer Key:


Directions (Q. 1-5):

1). Answer: a)
2). Answer: e)
3). Answer: c)
4). Answer: b)
5). Answer: d)

Directions (Q. 6-10):

6). Answer: b)
7). Answer: e)
8). Answer: a)
9). Answer: c)
10). Answer: d)

PUZZLE-18
Directions (Q. 1-5): Study the following information carefully and answer the
questions given below:
There are eight persons D, E, G, H, R, S, T and U sitting around a circular
table. Some of the persons are not facing the centre. They all like four different types
of drinks, viz sprite, Slice, Limca and Pepsi, and each drink is liked by two persons,
Note: Same direction means that if one person is facing the centre then the
other person also faces the centre and vice versa. Opposite direction means if one
person is facing the centre then the other person faces outside and vice versa.
T sits on the immediate right of E, who likes Sprite. G sits third to the left of U,
who likes Pepsi and both are facing the same direction. G and E are not facing the
same direction but G is an immediate neighbour of R. R sits fourth to the left of T
and both are facing opposite directions but like the same soft drink. The persons
who like Sprite sit adjacent to each other, but face opposite directions. The persons
who like Slice sit opposite each other. The immediate neighbours of R are facing the
centre. The person who likes Pepsi is an immediate neighbour of the persons who
like Limca. H and S are immediate neighbours of U. H is not facing the centre and
likes Limca. The one who is on the immediate left of S is not facing the centre. S sits
second to the right of G.

1). Who among the following like Pepsi?


a) R, T
b) G, U
c) S, D
d) H, U
e) None of these

2). How many persons are not facing the centre?


a) One
b) Two
c) Three
d) Four
e) None of these

3). Which of the following statements is true about D?


a) D is an immediate neighbour of G and the person who likes Slice.
b) The immediate neighbours of D are facing the centre.
c) D is third to the right of the person who likes Limca.
d) None of the given statements is true
e) D sits opposite U, who likes Pepsi.

4). Who among the following sits third to the right of U?


a) T, who likes Pepsi
b) B, who likes Sprite
c) R, who likes Slice
d) Can't be determined
e) None of these

5). Four of the following five are alike in a certain way and hence form a group.
Which is the one that does not belong to that group?
a) S,U
b) R, H
c) G,D
d) E, H
e) U,E

PUZZLE-19
Directions (Q. 6-10): Study the following information carefully and answer the
questions given below:
There are four counters I, II, III and IV at the Railway station for Tatkal,
Platform, Reservation and General tickets, but not necessarily in the same order.
Four persons W, X, Y and Z are standing on these counters to buy a ticket, but not
necessarily in the same order.
W does not buy Tatkal or Reserved Ticket and he buys neither from counter
no. II nor from counter no. III. Z goes to the station to receive a guest. Y does not
buy ticket from counter no. II. X does not buy Tatkal ticket. Z buys ticket from
counter no. IV.

6). Which of the following tickets can be bought from counter no. IV?
a) Tatkal
b) Platform
c) Reservation
d) Can't be determined
e) None of these

7). W stands at which of the following counters?


a) I
b) II
c) IV
d) Either I or IV.
e) Can't be determined

8). Which of the following counters is for Tatkal tickets?


a) I
b) II
c) III
d) W
e) Can't be determined

9). Which of the following statements is not true about Y?


a) Y does not buy ticket from counter no. I.
b) Y does not buy Platform ticket.
c) Y buys Reservation ticket from counter no. III.
d) All statements are true
e) None of these

10). Which of the following statements is/are not true about counter no. I?
a) Counter no. I is for General ticket.
b) X does not buy ticket from counter no. I.
c) W stands at counter no. I.
d) Only a) and c) are not true
e) None of these

Answer:
1)b 2)c 3)e 4)b 5)e 6)b 7)a 8)c 9)c 10)e

Explanation:
Directions (Q. 1-5):

Directions (Q. 6-10):


PUZZLE-20
Directions (1-5): Study the following information carefully and answer the given
questions.
C, D, G, H, J, K and L lives in a seven storey building, The lower-most floor is
numbered 1; floor above it is numbered 2 and so on. There are four males and three
females among them.
Miss L lives in floor number 2. No female lives above the floor in which Miss D lives.
H lives in an odd numbered floor and he does not live either on the top-most or
lower-most floor. J has to use most number of stairs to reach to his flat. More than
anyone else in the building, None of the female lives at the lower-most or top-most
floor. K lives just a floor below his sister G's floor. No two male or female live
immediately above or below each other.

1). Who among the following exactly sitting between L and H?


a) G and K
b) J and D
c) G and D
d) J and K
e) C and D

2). Who among the following lives on floor number 5?


a) K
b) J
c) D
d) L
e) H

3). Which of the following floor number G Lives?


a) Two
b) Six
c) Five
d) Four
e) Three

4). Which of the following combinations is true?


a) 7 - Female - G
b) 3 - Male - C
c) 6 - Female - D
d) 5 - Male - J
e) None of these

5). If ‘J’ is related to ‘Floor number 4’, ‘G’ is related to ‘Floor number 1’, then which
following is ‘H’ related to?
a) Floor number 7
b) Floor number 2
c) Floor number 5
d) Floor number 4
e) Floor number 6

PUZZLE-21
Directions (6-10): Study the following information carefully and answer the given
questions.
Eight persons from eight floors and different states such as Maharashtra, Andra
Pradesh, Tamil Nadu, Kerala, Rajasthan, Haryana, Karnataka and Bihar eating 8
different foods such as Biryani, Dosa, Idly, Poori, Chapati, Parotta ,Noodles and
Upma . And each of them having a mobile in the brand names of Samsung,
Microsoft, Sony and Motorola
 The person who is from Andra Pradesh eating Upma as a food
 The person who is eating Noodles having Motorola mobile
 The third floor stayed person eating Dosa and having a mobile in the brand name of
Samsung.
 Idly is eaten by the person who is having Microsoft mobile.
 Minimum of three persons stayed below of Bihar state man. And the person who
belongs to Kerala not stayed in the odd numbered floor.
 The mobile’s brand name Microsoft and Motorola having same number of state-men
 Tamil Nadu state man stayed two floor below from the person who is belongs to
Andra Pradesh.
 The top floor is not occupied by the person from Karnataka.
 The Sony owned person eating Biryani.
 The persons from Kerala and Andra Pradesh having same mobile brand and both of
them not having Microsoft mobile and also three persons stayed in between them.
 The persons who are eating Poori food and Dosa are having same mobile brand
with the one who is eating Chapati. The Sony owned person stayed in fourth floor.
 There are four persons stayed in between of Karnataka and Kerala. The lowest floor
is occupied by the Rajasthan state man.
 Only three persons having one kind of mobile brand name and the person from
Karnataka who is eating Chapati is one of them.
 The state man from Haryana and the person who eating Chapati are not stayed in
the even numbered floor. Only one person having Sony mobile
 The person from Maharashtra who likes to eat Parotta stayed in even numbered
floor and is having same mobile brand name with the person, who is eating Idly also
stayed in first floor.
 Motorola mobile is owned by the person who is from Kerala.
 The person who eating Upma stayed above the floor of the person who is eating
Noodles
 The Haryana who is eating Dosa having same mobile brand with the state man from
Bihar
6). How many persons have Samsung mobiles?
a) One
b) Two
c) Three
d) Four
e) Five

7). Who lives in the sixth floor?


a) The one who is from Andra Pradesh
b) The one who is from Kerala
c) The one who is from Karnataka
d) The one who is from Maharashtra
e) The one who is from Bihar

8). Chapati food eats by which of the following state person?


a) Tamil Nadu
b) Andra Pradesh
c) Kerala
d) Maharashtra
e) Karnataka

9). Which of the following combinations is true?


a) 2 – Idly – Motorola - Rajasthan
b) 4 - Biryani - Sony - Tamil Nadu
c) 8 – Biryani – Microsoft - Rajasthan
d) 7 - Dosa – Samsung - Maharashtra
e) 3 – Poori - Sony - Haryana

10). Which of the following person sits exactly between who are eating Biryani and
Upma?
a) The one who eats Dosa food
b) The one who eats Noodles food
c) The one who eats Parotta food
d) The one who eats Poori food
e) The one who eats Idly food

Explanation With Answer Key:

Directions (Q. 1-5):

1). Answer: a)
2). Answer: e)
3). Answer: d)
4). Answer: c)
5). Answer: b)

Directions (Q. 6-10):


6). Answer: c)
7). Answer: a)
8). Answer: e)
9). Answer: b)
10). Answer: d)

PUZZLE-22
Directions (Q. 1-5): Study the following information to answer the given questions.
Seven stores viz. A, B, C, D, E, F and G sold different number of TV's of a
particular brand viz. 4, 6, 9, 12, 15, 18 and 27 on seven different days of the same
week starting from Monday and ending on Sunday, but not necessarily in the same
order.
 A sold the TV's on one of the days after Friday. Only three stores sold TV's between
A and the store which sold 12 TV's. The store which sold TV's on the day
immediately after the store which sold 12 TV's sold more than 12 TV's but not odd
number of TV's.
 Only two stores sold TV's between F and the store which sold 4 TV's. The store
which sold 4 TV's sold these before F. 4 TV's were not sold on Monday.
 Only three stores sold TV's between F and D. The difference between the number
of TVs sold on Wednesday and Saturday is more than 10.
 B sold the TV's on a day immediately before the day on which 9 TV's were sold. 9
TV's were not sold on Friday.
 Only two stores sold TV's between B and G.
 Difference between the number of TV's sold by G and D is less than 5. C did not sell
the least number' of TV's.

1). In which of the following pairs did both the stores sell odd, number of TV's?
a) D, E
b) A, B
c) E, F
d) F, G
e) C, D

2). Which of the following stores sold 4 TV’s?


a) F
b) A
c) C
d) G
e) E

3). Based on the given arrangement, which of the following is true?


a) D sold the TVs on Wednesday.
b) Only one store sod TV's between C and F.
c) A sold 12 TV's on Sunday.
d) C sold 18 TV's on Thursday.
e) None of these

4). How many stores sold less number of TV's than B?


a) Five
b) Three
c) Six
d) Two
e) One

5). What is the difference between the number or TV’s sold on Monday and Friday?
a) 9
b) 12
c) 18
d) 3
e) 11

PUZZLE-23
Directions (Q. 6-10): Study the following information carefully and answer the
questions given below:
J, K, L, M, N and O are six members of a group in which there are three
female members. All the three females work in three different departments, viz HR,
Accounts and Publishing. All the persons working in the same department are not on
the same floor. All the females sit on three different floors, viz 1st, 2nd and 3rd. Two
persons work on each floor and in each department.
K and N work in the same department but not in Publishing. N and J are on
the 1st and 3rd floors respectively. M works in the Accounts department. J works in
the same department in which O works. L, a male, works on the first floor. M is a
female but does not sit on the 2nd floor. J is not a female.

6). Which of the following pairs of employees works on the second floor?
a) N, K
b) J, O
c) K, O
d) K, I
e) O, N

7). O works in which of the following departments?


a) Accounts
b) Publishing
c) HR
d) Either a) or b)
e) None of these

8). Which of the following groups of employees are females?


a) M, N, O
b) J,M, O
c) M, O, K
d) L, K, N
e) None of these

9). Which of the following pairs of persons works in Accounts department?


a) L, M
b) O, M
c) I, M
d) L, O
e) Data inadequate

10). If O is transferred to HR and N is transferred to Accounts, who is to be


transferred to Publishing to maintain the original distribution of departments on each
floor?
a) K
b) L
c) M
d) Data inadequate
e) None of these
Answer:
1)b 2)e 3)d 4)c 5)a 6)c 7)b 8)a 9)a 10)d

Explanation:

Directions (Q. 1-5):

Directions (Q. 6-10):

Male (+), Female (-)


PUZZLE-24
Directions (Q. 1-5): Study the following information and answer the questions.
There are nine persons Prem, Vinoth, Harish, Ramesh, Satish, Atreya,
Mohan, Gowtham and Yadav stay on a nine floors building, but not necessarily in
the same order. Only one person stays on each floor. All of them clear either SBI PO
or IBPS PO examination, Each of them presently working as a clerk in different
banks i.e. Andra Bank, Bank of India, Canara Bank, Indian Bank, Syndicate Bank,
Punjab National Bank, UCO Bank, Vijaya Bank and IDBI Bank, but not necessarily
in the same order. Only five of them clear IBPS PO exam. The ground floor is
numbered 1 and the topmost floor is numbered 9. There are two floors between the
persons who working in a Punjab National Bank cleared SBI PO exam and who
working in Vijaya Bank cleared IBPS PO exam. Harish working in Canara Bank
cleared SBI PO exam. Atreya does not working in IDBI Bank but he cleared SBI PO
exam. Prem stays on even numbered floor below the even numbered floor on which
Gowtham stays. The one who working in Syndicate Bank cleared IBPS PO exam
stays on the fourth floor. Ramesh stays on the second floor and working in Indian
Bank cleared IBPS PO exam. The one who working in Bank of India cleared IBPS
PO exam stays on the topmost floor. Prem does not working in UCO Bank cleared
IBPS PO exam. Atreya working in Punjab National Bank cleared SBI PO exam and
stays on an odd-numbered floor. There is only one floor between the floors on which
Atreya and Mohan stay. Satish stays immediate above the floor on which Yadav
stay. Mohan does not stay on the ground floor. Vinoth stays immediate above the
floor on which Gowtham stay. There are three floors between the floors on which
Harish and Atreya stay. The one who working in Vijaya Bank cleared IBPS PO exam
stays immediate above Mohan .Satish stays on the fourth floor. The one who
working in IDBI Bank clearedSBI PO exam stays on the third floor.

1). Who lives on the 7th floor?


a) Mohan
b) Vinoth
c) The one who working in IDBI Bank cleared IBPS PO exam
d) Atreya
e) The one who working in Vijaya Bank cleared IBPS PO exam

2). The person who stays on the 4th floor working in which of the following bank?
a) Canara Bank
b) UCO Bank
c) Indian Bank
d) Syndicate Bank
e) Andra Bank

3). How many persons are between the one who working in Bank of India and the
one who stays on 4th floor?
a) Five
b) Three
c) Four
d) Seven
e) None of these

4). Which of the following bank Atreya working?


a) Andra Bank
b) Canara Bank
c) Syndicate Bank
d) IDBI Bank
e) Punjab National Bank

5). Four of the following five are alike in a certain way and hence they form a group.
Which one of the following does not belong to that group?
a) Canara Bank
b) Vijaya Bank
c) Bank of India
d) UCO Bank
e) IDBI Bank

PUZZLE-25
Directions (6-10): Study the following information carefully and answer the given
questions.
Seven people namely Lakshitha, Brinda, Krithika, Charvi, Neeja, Umika and
Mohini buy different company laptops viz. Dell, HP, Asus, Apple, Lenovo and Acer
and different company computers viz. Microsoft, Samsung, Sony and LG from
different sites say Amazon, Shopclues, Flipkart and Snapdeal. Not more than two
people buy from same site. It is known that each person buys both the items from
the same site. The people who buy HP company laptop get from the same site and
they don’t buy the same company computers and is neither Lakshitha nor Mohini.
Umika buys from Snapdeal. Charvi and Umika buy same company computers which
are bought by no one apart from them and they get from different sites. The only
person who orders LG company computer from Flipkart and is neither Brinda nor
Krithika. Charvi doesn’t buy on Amazon. Acer company laptop is get from Amazon
but not by Lakshitha. Krithika buy Asus company laptop. One who buys Microsoft
company computer buys from Amazon and is not Brinda and also buys either Dell or
Lenovo company laptop. The persons who buy Samsung company computer get on
different sites but not on Shopclues. Only one person buys Microsoft company
computer. Krithika doesn’t get Sony company computer. Umika neither orders Dell
or Apple company laptop.

6). Who among the following person buying in Shopclues?


a) Lakshitha
b) Krithika
c) Charvi
d) Brinda
e) Mohini
7). Who among the following persons bought the Samsung company computer?
a) Mohini
b) Krithika
c) Brinda
d) All of the above
e) None of these

8). Which of the following company laptop and computer bought by Charvi?
a) Dell and Samsung
b) Apple and Sony
c) Acer and LG
d) Apple and Microsoft
e) Lenovo and Sony

9). Which of the following company laptop bought by two persons?


a) HP
b) Acer
c) Asus
d) Dell
e) Lenovo

10). Which of the following combinations is true?


a) Neeja – HP – LG – Amazon
b) Krithika – Asus – Microsoft - Snapdeal
c) Lakshitha – Dell – Samsung - Amazon
d) Mohini - Lenovo - Samsung - Amazon
a) e) Brinda - HP - Samsung - Flipkart

Explanation With Answer Key:


Directions (Q.1-5):
1). Answer: a)
2). Answer: d)
3). Answer: c)
4). Answer: e)
5). Answer: b)
Directions (Q. 6-10):

6). Answer: c)
7). Answer: d)
8). Answer: b)
9). Answer: a)
10). Answer: e)

PUZZLE-26
Directions (1-5): Study the following information carefully and answer the given
questions.
Naresh, Fayaz, Suvetha, Milan, Lavanya, Sahana, Archana, Renuka, Sarojini,
Vidya, Geeta and Jemima are 12 persons sitting in two rows, among them Naresh,
Suvetha, Milan, Sahana, Sarojini and Geeta are facing north while the remaining are
facing south. Each person faces exactly one person in the other row. Each one of
them likes a different city from among London, Paris, Dubai, Tokyo, Rome, Vienna,
Mumbai, New York, Bangkok, Chennai, Amsterdam and Bangalore (not necessarily
in the same order) and belongs to one country each from among Australia, China,
South Africa, Japan, Canada, France, India, Netherland, Poland, Qatar, Thailand
and Iceland (again not necessarily in the same order).
The one who likes Rome is from France and is facing Sahana. None of Fayaz,
Jemima and Lavanya likes Rome or London. The persons from South Africa and
Iceland are the immediate neighbours of Vidya.
Fayaz is second to the right of Archana, who is from India. Fayaz like Mumbai city
and faces the person from Qatar, who is third to the right of Milan. Neither Naresh
nor Milan likes Paris city or from China.
Geeta, like the Dubai city, sits second to the right of the person who likes Chennai
city, who faces the person from Iceland, None of them sits at an extreme end.
The Paris city like by the person from Thailand, sits as far as possible from Milan.
Jemima, likes the Vienna city, sits opposite the person who likes the Amsterdam
city, who is an immediate neighbour of the person sitting at an extreme end.
The Mumbai and Tokyo city liker’s are from South Africa and Australia respectively.
The person who likes Vienna city and Tokyo city face the same direction.
Sahana, like the New York city and from Japan, sits equidistant in the same row
from the Bangkok and the city from China.
The city from Poland is opposite the city from Australia and second to the left of the
city from Japan. Naresh, who is not like the Amsterdam city, is third to the left of
Sarojini; and neither of them like Paris city.
Renuka likes the Bangalore city and is third to the right of person who likes Vienna
city from Canada.

1). Who sits second to the left of Vidya?


a) Lavanya
b) Fayaz
c) Renuka
d) Archana
e) Jemima

2). Which of the following person belongs to Qatar?


a) Geeta
b) Sarojini
c) Vidya
d) Naresh
e) Jemima

3). Which of the following persons sits at the extreme ends of one row?
a) Lavanya and Fayaz
b) Naresh and Suvetha
c) Milan and Suvetha
d) Archana and Renuka
e) Sahana and Milan

4). The one who belongs to Canada sits immediate of ______________


a) The one who belongs to South Africa
b) The one who belongs to France
c) The one who likes London city
d) The one who likes Mumbai city
e) The one who likes Tokyo city

5). Which of the following combinations is true?


a) Sahana - New York – South Africa
b) Renuka - Bangalore - Iceland
c) Sarojini - Chennai - China
d) Jemima – Mumbai - Canada
e) None of these

PUZZLE-27
Directions (Q. 6-10): Study the following information carefully and answer the given
questions.
Ten persons Harshika, Kokila, Uma, Meena, Chitra, Dhiya, Nisha, Prema, Thamarai
and Aruna are sitting in two rows with five persons in each row. The persons in row
one are facing south and the persons in row two are facing north. Each person in
row one faces a person from the other row. All of them have a book of different
authors, viz Chetan Bhagat, Kiran Desai, Arundhati Roy, Anita Desai, Salman
Rushdie, Vikram Seth, Amitav Ghosh, Jhumpa Lahiri, Anita Nair and Ruskin Bond,
but not necessarily in the same order.
The persons who like Salman Rushdie and Vikram Seth sit opposite each other.
Dhiya sits opposite to Harshika, who likes Chetan Bhagat. The one who likes Kiran
Desai sits opposite the one who likes Jhumpa Lahiri. Aruna is not facing north but
sits third to the left of Nisha, who likes Kiran Desai. There is only one person
between Kokila and Uma. Chitra sits at one of the ends of the row and likes Vikram
Seth. The one who likes Jhumpa Lahiri is on the immediate right of Meena, who
does not like Ruskin Bond. The persons who like Arundhati Roy and Anita Desai
respectively are not facing north. Uma likes Amitav Ghosh. The one who likes Anita
Desai sits opposite the one who is second to the right of Kokila. Thamarai does not
like Ruskin Bond. Chitra sits opposite the one who sits second to the left of the one
who likes Arundhati Roy.

6). Who have Anita Nair book?


a) Chitra
b) Kokila
c) Meena
d) Dhiya
e) Nisha

7). Who sits opposite to Kokila?


a) The one who have Chetan Bhagat book
b) The one who have Arundhati Roy book
c) Prema
d) The one who have Salman Rushdie book
e) Nisha

8). Who sits opposite to the one who sits second to the left of Chitra?
a) The one who have Arundhati Roy book
b) The one who have Anita Desai book
c) The one who have Anita Nair book
d) The one who have Kiran Desai book
e) None of these

9). Who among the following is the immediate neighbour of the one who have
Chetan Bhagat book?
a) The one who have Anita Nair book
b) Uma
c) Thamarai
d) The one who have Kiran Desai book
e) None of these
10). Who sits at the right end in Row II?
a) Prema
b) Aruna
c) Nisha
d) Chitra
e) Meena

Solutions:

1). Answer: e)
2). Answer: a)
3). Answer: c)
4). Answer: d)
5). Answer: b)

Facing Lavanya Renuka Vidya Fayaz Jemima Archana


South
(Tokyo) (Bangalore) (Rome) (Mumbai) (Vienna) (London)

(Australia) (Iceland) (France) (South (Canada) (India)


Africa)

Facing Milan Naresh Sahana Geeta Sarojini Suvetha


North
(Bangkok) (Chennai) (New (Dubai) (Amsterdam) (Paris)
York)
(Poland) (Netherland) (Qatar) (China) (Thailand)
(Japan)

6). Answer: c)
7). Answer: e)
8). Answer: a)
9). Answer: b)
10). Answer: d)

Facing South Prema Nisha (Kiran Dhiya Thamarai Aruna


(Ruskin Desai) (Arundhati (Anita Desai) (Salman
Bond) Roy) Rushdie)

Facing North Meena (Anita Kokila Harshika Uma (Amitav Chitra


Nair) (Jhumpa (Chetan Ghosh) (Vikram Seth)
Lahiri) Bhagat)

PUZZLE-28
Directions (Q. 1-5): Study the following information carefully and answer the
questions given below:
A, B, C, D, E, F, G and H are friends having different occupations, viz – Clerk,
Manager, PO, Professor, Teacher, Director, Supervisor and Editor. All are sitting
around a circular table facing the centre, but not necessarily in the same order.
C sits third to the right of the Teacher. Only two persons sit between the Teacher
and H. The Manager and the Clerk are immediate neighbours. Neither C nor H is a
manager or Clerk. The Manager is not an immediate neighbor of the Teacher. The
PO sits third to the right of D, who is not a Manager. F sits on the immediate right of
the Editor. The Director sits second to the left of A, who is not an immediate
neighbor of H. The Director is an immediate neighbor of both the professor and the
PO. C is not an Editor. B is not an immediate neighbour of C. E is not an immediate
neighbour of the Director. H is not a Professor and sits second to the left of the
Editor. E. is an immediate neighbour of D and C.

1). Who among the following is the Editor?


a) A
b) B
c) E
d) G
e) None of these

2). Which of the following statements is/are true?


a) A is an Editor and sits on the immediate right of F.
b) The Supervisor and the Professor are immediate neighbours of G.
c) F is a Teacher and sits second to the left of E.
d) C is a PO and sits second to the left of
e) None is true

3). How many persons sit between the Editor and the Supervisor?
a) None
b) One
c) Two
d) Three
e) None of these

4). What is the occupation of D?


a) Editor
b) Professor
c) Clerk
d) Director
e) None of these

5). Who among the following sits second to the left of the Supervisor?
a) Clerk
b) PO
c) Teacher
d) Director
e) None of these

Answers:
1)a 2)c 3)d 4)c 5)a
Explanation:
Directions (Q. 1-5):

PUZZLE-29
Directions (Q. 1-5): Study the following information carefully and answer the given
questions.
There are seven candidates – Akshar, Naren, Ganesh, Baskar, Vimal, Mishra
and Karthik – who attends different examinations which started on 25th December
and ended on 31st December. Each of them wrote different Exams, viz SBI PO,
IBPS PO, SYNDICATE PO, INDIAN PO, DENA PO, BOB PO and BOM PO, but not
necessarily in same order. They like different currencies, viz Rupee, Dinar, Dollar,
Pound, Riyal, Euro and Yen, but not necessarily in same order.
Karthik, who likes Euro, attended exam on fourth day of week but attended
neither SBI PO nor IBPS PO Exam. Two exams were held between the exams of
Karthik and Mishra, neither of whom attended on 1st day of week. There was one
exam between exam of Baskar and Ganesh. But Ganesh’s exam did not happen
either on 1st or on 3rd day of week. Ganesh likes Dinar currency and attended SBI
PO Exam. The one who attended SYNDICATE PO Exam on the last day of week,
likes Dollar currency. Naren did not attend exam on day either immediately before or
immediately after performance of Baskar, who does not like either Riyal or Rupee or
Pound currency. Vimal attended immediately after Ganesh exam and he likes
Rupee currency. Akshar does not like Riyal currency and attended INDIAN PO
Exam. Baskar did not attend either BOM PO or BOB PO Exam. One who attended
in DENA PO was scheduled immediately after SBI PO exam.

1). Which of the following exam held on 28 th December?


a) IBPS PO
b) BOB PO
c) BOM PO
d) Either BOB PO or BOM PO
e) Either IBPS PO or INDIAN PO

2). Which of the following currency Mishra likes?


a) Dollar
b) Yen
c) Rupee
d) Euro
e) Dinar

3). Which of the following day SBI PO exam was organized?


a) 30th December
b) 25th December
c) 29th December
d) 31st December
e) 26th December

4). Which of the following candidate like Pound currency?


a) Vimal
b) Akshar
c) Ganesh
d) Baskar
e) Mishra

5). Which of the following statement is true?


a) 29th December – Ganesh - SBI PO - Riyal
b) 30th December – Vimal - BOB PO - Rupee
c) 26th December - Mishra - INDIAN PO - Pound
d) 31st December – Akshar - SYNDICATE PO - Dollar
e) 27th December – Baskar - IBPS PO - Yen

PUZZLE-30
Directions (Q. 6-10): Study the following information carefully to answer the given
questions:
Eight friends Sameer, Taranga, Devnath, Parvesh, Harendra, Jaswant,
Gurdeep and Waseem live on eight different floors of a building but not necessarily
in the same order. The lowermost floor of the building is numbered 1 and the
topmost floor of the building is numbered 8. Each of them likes different players of
grand prix viz, Valtteri Bottas, Kimi Räikkönen, Max Verstappen, Nico Rosberg,
Daniel Ricciardo, Fernando Alonso, Lewis Hamilton and Sebastian Vettel but not
necessarily in the same order.
The one who likes Kimi Räikkönen lives on an even-numbered floor but not
on the topmost floor. Only one person lives between Jaswant and the one who likes
Daniel Ricciardo. Only two persons live between Jaswant and the one who likes
Kimi Räikkönen. Neither Harendra nor Devnath lives on the first floor. Only one
person lives between Devnath and the one who likes Max Verstappen. Sameer lives
just above Jaswant. Only two persons live between Harendra and Sameer. The one
who likes Daniel Ricciardo does not live on floor number one, Taranga lives on an
even-numbered floor and just above Devnath. The one who likes Nico Rosberglives
on an even numbered floor and lives just above the person who likes Sebastian
Vettel. Devnath does not like Daniel Ricciardo or Sebastian Vettel. Only two persons
live between the one who likes Fernando Alonsoand the one who likes Valtteri
Bottas. Parvesh does not like Lewis Hamilton. The one who likes Fernando Alonso
does not live on an odd-numbered floor. Waseem lives just below the one who likes
Sebastian Vettel.

6). Who lives on Fourth floor?


a) Parvesh
b) Taranga
c) Gurdeep
d) Sameer
e) Waseem

7). Jaswant lives on which of the following floor?


a) Seventh
b) Sixth
c) Third
d) Fifth
e) Fourth

8). Which of the following player Harendra likes?


a) Nico Rosberg
b) Kimi Räikkönen
c) Fernando Alonso
d) Max Verstappen
e) Daniel Ricciardo

9). Which of the following statement is true?


a) 6 – Jaswant - Sebastian Vettel
b) 4 – Taranga – Lewis Hamilton
c) 3 – Devnath - Valtteri Bottas
d) 2 – Gurdeep – Nico Rosberg
e) 2 – Parvesh - Max Verstappen

10). Four of the following players form a group, which of the following player does
not belong to that group?
a) Lewis Hamilton
b) Nico Rosberg
c) Fernando Alonso
d) Valtteri Bottas
e) Kimi Räikkönen

Explanation With Answer Key:


Directions (Q. 1-5):

1). Answer: d)
2). Answer: a)
3). Answer: c)
4). Answer: b)
5). Answer: e)

Directions (Q. 6-10):


6). Answer: b)
7). Answer: a)
8). Answer: e)
9). Answer: c)
10). Answer: d)

PUZZLE-31
Directions (Q. 1-6): Study the following Information carefully and answer the
questions given below:
Nine people A, B, C, D, E, F, G, II and J are living in a nine-storey building.
The ground floor is numbered 1 and the topmost floor is numbered 9. All of them like
a different colour, viz Red, Pink, Green, Yellow, Orange, Violet, Black, Blue and
White, but not necessarily in the same order.
B likes Yellow and lives on an even-numbered floor. J lives on even-
numbered floor below the floor on which B lives. The one who likes Blue lives on the
fourth floor. E lives on the second floor and likes Green. The one who lives on the
third floor likes White. J does not like Blue. There are two floors between the floors
on which the persons who like Orange and Yellow respectively, live. There are three
floors between the floors on which G and C live respectively. D does not like White.
There is one floor between the floor on which D and C live respectively. F lives on a
floor immediately above A's floor. The one who lives on the topmost floor likes Red.
D does not live on the ground floor. G likes Pink. C does not live on topmost floor.
There is only one floor between the ones who like Black and Yellow respectively.

1). How many floors are there between the floor on which A lives and the floor on
which J lives?
a) One
b) Two
c) Three
d) None
e) None of these

2). On which of the following floors does C live?


a) First
b) Third
c) Fifth
d) Second
e) None of these

3). Which of the following colours does the person like who lives on the first floor?
a) Red
b) Black
c) Pink
d) Violet
e) None of these

4). J likes which of the following colours?


a) Blue
b) Violet
c) Yellow
d) Orange
e) None of these

5). Who lives on the floor which is exactly between the floor on which B lives and the
floor on which J lives?
a) A
b) G
c) E
d) D
e) None of these

6). Who among the following lives on the topmost floor?


a) E
b) A
c) C
d) H
e) None of these

Answers:
1)b 2)c 3)c 4)e 5)d 6)d
Explanation:
Directions (Q.1-6):
PUZZLE-32
Directions (Q. 1-5): Study the following information carefully and answer the given
questions.
Seven flights - Emirates, Singapore Airlines, Qatar Airways, Etihad Airways,
Lufthansa, Qantas Airways and Turkish Airlines - depart from Delhi Airport on
Friday. Each flight departure at different times 4 AM, 7 AM, 8 AM, 10 AM, 1 PM, 3
PM and 6 PM not necessarily in the same order.
Qatar Airways is the earliest to depart. Its departure time is not after 7 a.m. and its
travel time is two hours less than the travel time of Qantas Airways. Since Emirates
has to cover the least distance, it leaves last, but before 7 p.m. and still completes
its journey at 9 p.m. Etihad Airways leaves Delhi Airport at Friday 10 a.m. and takes
19 hours to reach its destination. There is a four hour gap between the departure
times of Lufthansa, leaving at 8 a.m. and Qatar Airways. Lufthansa takes one hour
more than the travel time of Qatar Airways to reach its destination. Emirates leaves
five hours after Singapore Airlines departs but takes only three hours to reach its
destination. Qantas Airways and Turkish Airlines take 10 hours and 8 hours to reach
their respective destinations. Qantas Airways leaves at 3 p.m. and there is gap of 8
hours between the departure times of Qantas Airways and Turkish Airlines. One
flight reaches its destination at 6 p.m.

1). Which of the following flights reach the destination on Saturday?


a) Etihad Airways
b) Qantas Airways
c) Both Etihad Airways and Qantas Airways
d) Turkish Airlines
e) Singapore Airlines

2). How many flights departure between Singapore Airlines and Qantas Airways?
a) None
b) One
c) Two
d) Three
e) Four

3). Which of the following flight departure last on Friday?


a) Qantas Airways
b) Emirates
c) Etihad Airways
d) Turkish Airlines
e) Lufthansa

4). What is the difference between the departure timing of Etihad Airways and
Emirates flights?
a) 2 hours
b) 4 hours
c) 6 hours
d) 8 hours
e) 10 hours

5). Which of the following statement is true as per the given information?
a) Singapore Airlines - 3:00 PM - 6:00 PM
b) Emirates - 6:00 PM - 8:00 PM
c) Turkish Airlines - 9:00 AM - 1:00 PM
d) Lufthansa - 10:00 AM - 7:00 PM
e) Qatar Airways - 4:00 AM - 12:00 noon

PUZZLE-33
Directions (Q. 6-10): Study the following information carefully and answer the given
questions.
In an Interview seven candidates such as Kevin, Monica, Akshara, Bala, Lokesh,
Varun and Ramesh are attending interview in three different panels at three different
sessions in a day. Only three candidates going to attend in a panel at different
session of the day. Four CEO’s Usha Ananthasubramanian, Subramaniakumar,
Rajkiran Rai and Sankara Narayanan are interviewed each panel, One of the three
panels has two CEO’s. Two panels will have only one CEO to interview the people.
One of the three panels has two CEO’s for recruiting the people in afternoon
session. All the CEO will be present at their allocated interview panels and the
session will start as per scheduled. Every candidates should be interviewed in their
respective allocated interview panels and only two panels going to interview the
same number of people.
 Kevin and Monica not going to attend the interview at evening. The candidate Bala
interviewed by the CEO Rajkiran Rai at 9.00 A.M
 Ramesh going to attend interview after 2.00 P.M
 Lokesh and Kevin going to get interviewed at the same panels
 Usha Ananthasubramanian is the CEO who is not going to interview the people at
panel III.
 Rajkiran Rai is the CEO going to interview Varun and one other candidate in
different sessions.
 In the Evening session Akshara and Varun going to interviewed and both of them
not interviewed at panel I.
 Ramesh going to be interviewed at the same session with Lokesh and not in the
panel III
 Monica’s interview time is 9.00 A.M. and He is not interviewed by Subramaniakumar
and Sankara Narayanan.

 Akshara Interviewed at Panel II.


6). How many persons attend the interview on Afternoon session?
a) Two
b) Three
c) One
d) Either two or three
e) Either One or Two

7). Rajkiran Rai is the CEO going to interview on __________ session


a) Morning
b) Afternoon
c) Evening
d) Both Morning and Afternoon
e) Both Morning and Evening

8). Which of the following candidates attends the interview on afternoon session?
a) Kevin
b) Lokesh
c) Ramesh
d) All of the above
e) Only Kevin and Lokesh

9). If “Monica” related to “Akshara” and “Bala” related to “Varun” on a certain basis,
on the same basis “Kevin” related to?
a) Both Lokesh and Ramesh
b) Only Lokesh
c) Only Ramesh
d) Bala
e) Monica

10). Which of the following statement is true as per the given information?
a) More than two peoples attend interview at Morning session
b) Bala and Varun going to interview at panel I
c) Usha Ananthasubramanian is the CEO who is going to interview the people at panel
II
d) Monica attend the interview on Evening session
e) None of the options are true
Explanation With Answer Key:

Directions (Q. 1-5):

1). Answer: c)
2). Answer: a)
3). Answer: b)
4). Answer: d)
5). Answer: e)

Directions (Q. 6-10):

6). Answer: b)
7). Answer: e)
8). Answer: d)
9). Answer: a)
10). Answer: c)

PUZZLE-34
Directions (Q. 1-5): Study the following information carefully and answer the given
questions.
Six competitive exams – IBPS PO, IBPS Clerk, RRB PO, RRB Clerk, SBI PO and
SBI Clerk were written by three males – Ahmad, Gagan and Kaushik and three
females – Sandhya, Roshni and Charita. Three of these competitive exams were
held in Mumbai and the rest in Delhi. Each of these exams was organized by
different IT Companies– Infosys, TCS, HCL, Wipro, Oracle and Tech Mahindra (may
not be in the same order). Each exam was written by a male and a female.
None of the students wrote in a city twice and no two competitive exams were
written by the same pair of male and female. The females who written IBPS PO and
IBPS Clerk did not write the exam organized by Oracle and Tech Mahindra. The
male student was same for the exams – IBPS PO and IBPS Clerk. Gagan written
RRB PO but did not write the exam organized by Oracle or Infosys. Roshni did not
write any exam with Ahmad, but written RRB Clerk with Kaushik. Sandhya wrote an
exam with Kaushik. Tech Mahindra organized the exam in Mumbai and Wipro
organized the exam in Delhi which was written by Ahmad and HCL organized IBPS
Clerk. SBI PO was organized by Infosys. Oracle and TCS organized the exams in
the same city. IBPS Clerk and RRB PO exam were written by the same female.

1). Who among the following persons wrote IBPS PO exam?


a) Gagan and Roshni
b) Kaushik and Roshni
c) Ahmad and Sandhya
d) Gagan and Charita
e) Kaushik and Sandhya
2). Who among the following persons wrote the exam with Roshini in Delhi?
a) Kaushik
b) Gagan
c) Ahmad
d) Either Kaushik or Gagan
e) None of these

3). Which of the following IT company conduct SBI Clerk exam?


a) HCL
b) Tech Mahindra
c) TCS
d) Infosys
e) Oracle

4). Which of the following exam wrote by Ahmad and Charita?


a) IBPS PO
b) RRB PO
c) SBI PO
d) SBI Clerk
e) IBPS Clerk

5). Which of the following combinations is true?


a) IBPS Clerk – Ahmad – Charita – Delhi - HCL
b) RRB Clerk – Gagan – Roshni - Mumbai - Tech Mahindra
c) SBI PO – Kaushik – Sandhya - Mumbai - Oracle
d) RRB PO – Gagan – Charita - Delhi - TCS
e) All the statements are wrong
PUZZLE-35
Directions (Q. 6-10): Study the following information carefully and answer the given
questions.
Eight friends Ricardo, James, Shane, Roberto, Alberto, Lawrence, George and
Fernando live on eight different floors of a building but not necessarily in the same
order. The lowermost floor of the building is numbered 1 and the topmost floor of the
building is numbered 8. Each of them likes different brands of City viz, Mumbai,
Chennai, Hyderabad, Bangalore, Kolkata, Delhi, Kochi and Patna but not
necessarily in the same order.
The one who likes Chennai lives on an even-numbered floor but not on the topmost
floor. Only one person lives between Lawrence and the one who likes Kolkata. Only
two persons live between Lawrence and the one who likes Chennai. Neither Alberto
nor Shane lives on the first floor. Only one person lives between Shane and the one
who likes Hyderabad. Ricardo lives just above Lawrence. Only two persons live
between Alberto and Ricardo. The one who likes Kolkata does not live on floor
number one, James lives on an even-numbered floor and just above Shane. The
one who likes Bangalore lives on an even numbered floor and lives just above the
person who likes Patna. Shane does not like Kolkata or Patna. Only two persons live
between the one who likes Delhi and the one who likes Mumbai. Roberto does not
like Kochi. The one who likes Delhi does not live on an odd-numbered floor.
Fernando lives just below the one who likes Patna.
6). Alberto lives on which of the following floor?
a) 7
b) 5
c) 3
d) 4
e) 2

7). Which of the following person like Kochi city?


a) George
b) Shane
c) Fernando
d) Lawrence
e) Roberto

8). How many persons are between the one who likes Patna and George?
a) One
b) Two
c) Three
d) Four
e) Five

9). Which of the following statement is true?


a) 6 – Fernando - Mumbai
b) 1 –George- Hyderabad
c) 4 – James - Chennai
d) 2 – Shane - Delhi
e) 8 – Ricardo - Kolkata

10). Four of the following five are alike in a certain way and hence they form a
group. Which one of the following does not belong to that group?
a) Patna
b) Hyderabad
c) Mumbai
d) Kolkata
e) Bangalore

Explanation With Answer Key:


Directions (Q. 1-5):
1). Answer: c)
2). Answer: a)
3). Answer: b)
4). Answer: e)
5). Answer: d)

Directions (Q. 6-10):

6). Answer: b)
7). Answer: a)
8). Answer: d)
9). Answer: c)
10). Answer: e)
PUZZLE-36
Directions (1-5): Study the following information carefully and answer the given
questions.
Ten students namely viz Varun, Kishore, Prakash, Sundar, Bhavesh, Jeevika, Usha,
Harsha, Durga and Subha called for document verification but not necessarily in the
same order have document verification on five different days starting from Monday
to Friday of the same week. Students have document verification at two different
time slots, i.e 10.00 A.M and 02.00 P.M.
Durga have document verification on Tuesday at 10.00 A.M. The number of people
who have document verification between Usha and Sundar is same as the number
of people who have document verification between Prakash and Harsha. Kishore
have document verification immediately before Durga. Durga does not have
document verification on any of the days before Usha. The one who have document
verification at 10.00 A.M immediately before Subha. Harsha does not have
document verification at 02.00 P.M. Sundar have document verification immediately
after the day of one who have document verification on Monday. Jeevika does not
have document verification at 02.00 P.M. Sundar does not have document
verification on any one of the days after Bhavesh. Only three people have document
verification between Usha and Bhavesh. Neither Bhavesh nor Usha does not have
document verification on Friday. Only two people have document verification
between Jeevika and Subha. Jeevika does not have document verification on any of
the days after Harsha.

1). Which of the following students attends document verification on Wednesday?


a) Durga and Prakash
b) Kishore and Bhavesh
c) Bhavesh and Prakash
d) Usha and Harsha
e) Subha and Sundar
2). Jeevika and Varun have document verification on which of the following day?
a) Friday
b) Thursday
c) Wednesday
d) Monday
e) Tuesday

3). How many persons have document verification after Sundar?


a) Two
b) Three
c) Four
d) Five
e) Six

4). Four of the following students form a group, which of the following student does
not belong to that group?
a) Harsha
b) Varun
c) Kishore
d) Subha
e) Prakash

5). Which of the following statement is true as per the given information?
a) Varun attend document verification after six students
b) Durga and Prakash attends document verification on same day
c) Two students attend document verification between Sundar and Varun
d) Five students have document verification after Bhavesh
e) None of these
PUZZLE-37
Directions (6-10): Study the following information carefully and answer the given
questions.
Bikas, Jalal, Nanda, Tarun, Manas, Francis, Ikshan and Larshan are eight
employees of Indian Bank who attends training in the four months (May, September,
November and December) of the same year. The trainings are held in each month
on either 17th or 28th. Only two trainings are held in a month. It is further known
that:
Bikas attends training in a month with 30 days. Only two people attend trainings
after Nanda. Bikas and Tarun attend their training on consecutive turns. Tarun and
Manas attend trainings on same date. Manas attends training in a month with 31
days. Larshan attends training before Francis and Ikshan. Jalal or Larshan are not
the first to go for training. Neither Ikshan nor Jalal has training on 28th. Ikshan and
Francis do not have trainings in the same month.
6). Which of the following persons exactly attends training between Bikas and
Nanda?
a) Jalal
b) Larshan
c) Nanda
d) Francis
e) Ikshan

7). Ikshan attends training on which of the following month?


a) November
b) September
c) May
d) December
e) None of these

8). Which of the following persons attends the training on December month?
a) Ikshan and Francis
b) Jalal and Francis
c) Manas and Bikas
d) Jalal and Nanda
e) Tarun and Jalal

9). Four of the following form a group, which of the following does not belong to that
group?
a) Manas
b) Tarun
c) Ikshan
d) Larshan
e) Jalal

10). Which of the following statement is true as per the given information?
a) Larshan attends the training on December
b) Ikshan attends the training on 28th November
c) Tarun and Bikas attend the training on September
d) Nanda and Francis attends the training on 17th
e) None of these

Answer Key:
1).c 2).b 3).e 4).a 5).d 6).e 7).a 8).b 9).d 10).c

Explanation With Answer Key:


Directions (Q. 1-5):
Directions (Q. 6-10):

PUZZLE-38
Directions (Q. 1-5): Study the following information carefully and answer the
questions given below.
Eight persons P, Q, R, S, T, U, V and W stay in an apartment in eight different floors
but not necessarily in the same order. The lowermost floor of an apartment is
numbered 1 and topmost floor is numbered 8. They are born in different month i.e.
January, February, March, April, May, June, July and August.
S lives on the odd numbered floor above the floor on which P lives. Q has born in
the month of January and lives in an odd numbered floor, below the floor on which P
lives but not immediate below. The number of floors between W & U is double the
number of floors between P & Q. There is no month gap between month of V and W
was born. V lives neither immediate above nor immediate below on which W live.
One who has born on May month lives on even numbered floor but not on the eighth
floor. R is adjacent only with one who born in the month of minimum number of days
amongst all. P lives on the fourth floor and does not born on the month which has
maximum number of days. One who has born on March lives on the topmost floor.
One who has born in June month lives below the floor on one who born in July
month. U lives one of the floors above W.

1). Who among the following person had born on May month?
a) S
b) T
c) U
d) V
e) W

2). Who lives on floor number 3?


a) R
b) S
c) T
d) U
e) V

3). Which of the following combinations of month, person and floor is true?
a) June - P - Four
b) May - T - Six
c) Feb – U -Seven
d) Feb - W - Seven
e) None is true
4). Which of the following statement is/are definitely true?
a) W is an immediate neighbour of Q
b) Two floors in between P and T
c) W is not lives immediate below the floor on which R lives
d) One who born in June and August are adjacent to each other
e) Both a) and c)

5). Four of the following five are alive in a certain way based on the given
information and so form a group. Which is the one that does not belong to that
group?
a) Q
b) W
c) R
d) V
e) S

PUZZLE-39
Directions (Q. 6-10): Study the following information carefully and answer the
questions given below.
There are eight persons namely A, B, C, D, E, F, G and H born in the same month of
different years 1935, 1946, 1961, 1969, 1974, 1992, 1996, 2004. Their ages are
calculated as on the same month of 2016. A is born before independence. E is born
in even number year, which is completely divisible by four but, not completely
divisible by three. H is 8 years elder to B, who is 5 years older than D. G is 46 year
younger than A. C is not youngest among all.

6). Who among the following was born in 1974?


a) G
b) D
c) E
d) F
e) H

7). Who among the following is the youngest?


a) B
b) E
c) F
d) D
e) G

8). Which of the following year B is born?


a) 1946
b) 1961
c) 1974
d) 1996
e) none of these

9). How many people is/are born before H?


a) One
b) Two
c) Three
d) Four
e) No one

10). What is the age of G?


a) 46 years younger than A
b) 50 years younger than A
c) 56 years younger than B
d) Both a) and c)
e) None of these
Answers:
1). d) 2). c) 3). c) 4). e) 5). b) 6). b) 7). c) 8). e) 9). b) 10). a)

Explanation:
Directions (Q. 1-5):

Directions (Q. 6-10):


PUZZLE-40
Directions (1-5): Study the following information carefully and answer the given
questions.
Each of the five students - Sukumar, Gokul, Arvind, Sharma and Vikram - studies 5
subjects -Current Affairs, Reasoning, Aptitude, Computer and English - in a week
starting on Monday and ending on Friday.
Exactly five subjects were studied on each of the five days, no two of which were
same.
 No student studied more than one subject on any of the five days.
 Arvind studied Aptitude on Tuesday.
 Sharma studied English on Wednesday. He didn't study Current Affairs on
Thursday.
 Vikram studied Computer on Monday.
 Sukumar studied the same subject on Monday which Sharma studied on Friday.
The subject was not Aptitude.
 Gokul studied Current Affairs, Reasoning and Computer on Monday, Wednesday
and Friday respectively.
 Arvind studied the same subject on Monday which Sukumar studied on Friday.

1). Which of the following subjects learned by Sharma on Wednesday?


a) Reasoning
b) Aptitude
c) Current Affairs
d) Computer
e) English

2). Gokul learned how many subjects between Current Affairs and Computer?
a) None
b) One
c) Two
d) Three
e) Four

3). Sukumar studied Aptitude on which of the following day?


a) Thursday
b) Wednesday
c) Friday
d) Monday
e) Tuesday

4). If Arvind and Vikram interchange her subjects on all days, then Vikram studies
reasoning on which of the following day?
a) Thursday
b) Friday
c) Monday
d) Wednesday
e) Tuesday

5). Which of the following statement is true as per the given information?
a) Monday – Reasoning - Current Affairs – English – Computer - Aptitude
b) Tuesday – Computer – Aptitude – English - Current Affairs - Reasoning
c) Wednesday – Aptitude – Reasoning – Computer – English - Current Affairs
d) Thursday – Reasoning – Aptitude – Current Affairs – Computer - English
e) Friday – English – Computer - Current Affairs – Aptitude - Reasoning

PUZZLE-41
Directions (6-10): Study the following information carefully and answer the given
questions.
Gowri, Jadhav, Lakshita, Dulal, Pakshi, Wamika and Ramani are seven
people live on seven different floors of a building but not necessarily in the same
order. The lower most floor of the building is numbered 1, the one above that is
numbered 2 and so on till the topmost floor is numbered 7. Each one of them born in
different years. i.e. 1989, 1988, 1987, 1990, 1991, 1992 and 1993 (But not
necessarily in the same order). The one who born on 1989 lives immediately above
the one who born on 1993. Only one person lives between Jadhav and Pakshi.
Jadhav lives on one of the floors above Pakshi. Neither Lakshita nor Gowri born on
1990. Pakshi does not born on 1987. Gowri lives on an odd numbered floor but not
on the floor numbered three. The one who born on 1991 lives immediately above
Gowri. Only two people live between Gowri and the one who born on 1987. The one
who born on 1988 lives on one of the odd numbered floors above Dulal. Only three
people live between Lakshita and the one who born on 1988. The one who born on
1987 lives immediately above Lakshita. Wamika born on 1987.

6). Which of the following persons exactly sit between Ramani and Gowri?
a) The one who lives in sixth floor
b) Jadhav
c) The one who born on 1991
d) All of the above
e) None of these

7). Dulal born on which of the following year?


a) 1991
b) 1992
c) 1993
d) 1990
e) 1989

8). How many persons stay between the one who born in 1991 and Wamika?
a) Three
b) Four
c) Five
d) None
e) One

9). Four of the following persons form a group, which of the following person does
not belong to that group?
a) Jadhav
b) Lakshita
c) Wamika
d) Dulal
e) Pakshi

10). Which of the following statement is true as per the given information?
a) 5 – Pakshi - 1989
b) 7 – Ramani - 1992
c) 2 – Wamika - 1988
d) 1 – Lakshita - 1990
e) 6 – Jadhav - 1991
Answer Key:
Direction (01-05):

Students Monday Tuesday Wednesday Thursday Friday


Current
Sukumar Reasoning Computer Aptitude English
Affairs
Current
Gokul English Reasoning Aptitude Computer
Affairs
Current
Arvind English Aptitude Computer Reasoning
Affairs
Current
Sharma Aptitude English Computer Reasoning
Affairs
Current
Vikram Computer Reasoning English Aptitude
Affairs
1. e) 2. d) 3. b) 4. a) 5. c)

Direction (06-10):

Floor Persons Years


7 Ramani 1990
6 Jadhav 1991
5 Gowri 1988
4 Pakshi 1989
3 Dulal 1993
2 Wamika 1987
1 Lakshita 1992

6. d) 7. c) 8. a) 9. b) 10. e)
PUZZLE-42
Directions (1-5): Study the following information carefully to answer these
questions.
A, B, C, D, E, F and G are seven students of the Engineering college. All of them
belongs to different branch of Engineering: Mechanical, Chemical, Electrical, Computer
Science, IT, Textile and Electronics but not necessarily in the same order. Each one of
them likes different sports Snooker, Golf, Tennis, Badminton, Cricket, Football and
Volleyball not necessarily in the same order. All of them are of different height. D is
taller than C and F. E is taller than G.B studies in Mechanical and likes Football. E
studies in Computer Science but not likes Tennis or Cricket. The one who studies in IT
likes Snooker. F likes Golf but does not studies in Chemical or Electronics. C is taller
than E. B is taller than only A.A person who studies in Electrical likes Badminton. The
one who studies in Electronics does not like Cricket. G studies in IT and C likes Tennis.
The one who is tallest likes Badminton.

Q1.Who plays the Cricket?


(a) D
(b)A
(c)E
(d)D or E
(e) None of these

Q2.E plays which game?


(a) Badminton
(b) Golf
(c) Cricket
(d) Volleyball
(e) None of these

Q3.Which of the following combinations of branch-person-sports is definitely


correct?
(a) Electrical – B –Badminton
(b) Chemical – E – Volleyball
(c) Electronics – D – Cricket
(d) Chemical – D – Cricket
(e) None of these

Q4.Who studies Textile?


(a) C
(b) D
(c) G
(d) F
(e) None of these

Q5.Who is the Golfer?


(a) C
(b) F
(c) D
(d) Cannot be determined
(e) None of these

Solutions(1-5):

D>C>E>G>B>A
From given information, we know about F that he is surely less than D and can come
anywhere from D to B.

S1. Ans.(b)
Sol.

S2. Ans.(d)
Sol.

S3. Ans.(e)
Sol.

S4. Ans.(d)
Sol.

S5. Ans.(b)
Sol.

PUZZLE-43
Directions (6-10): Study the following information carefully to answer these
questions.
Seven family members G, H, J, K, L, M and N are going to different countries viz. UK,
US, Germany, France, Italy, Russia, China not necessarily in the same order. Each one
of them has a different profession from amongst CA, Admin, Accountant, HR, Manager,
Engineer and Professor not necessarily in the same order.
G is the father of K's only brother L. The one who is wife of L is anEngineer and she is
going to France. The one who is brother of L is anAdmin and he does not go UK.H is
the sister-in-law of K. The person who is CA goes to US. M goes to Italy. The one who
is Professor goes to China. K is uncle of N.M and N are brothers. L is a Manager and he
goes to Germany. G is an HR.The one who is sister of N does not go to US.J is sister of
M.

Q6.Who is the Accountant?


(a) J
(b) N
(c) K
(d) M
(e) None of these

Q7.Which of the following combinations of person, profession and country is


definitely correct?
(a) G – HR – Germany
(b) K – Professor - China
(c) L – Manager - US
(d) G – HR – UK
(e) None of these

Q8.Who is going to US?


(a) J
(b) K
(c) M
(d) N
(e) None of these

Q9. Granddaughter of G goes to which country?


(a) Russia
(b) China
(c) US
(d)Chandigarh
(e) None of these

Q10.Who is the Professor?


(a) N
(b) J
(c) M
(d) L
(e) None of these

Solution(6-10):
S6. Ans.(d)
Sol.

S7. Ans.(d)
Sol.

S8. Ans.(d)
Sol.

S9. Ans.(b)
Sol.

S10. Ans.(b)
Sol.

PUZZLE-44
Directions (11-15): Read the following information carefully and answer the
questions given below.
Eight friends P,Q,R,S,T,U, V and W are seated in a straight line at an equal distance
between each other, but not necessarily in the same order. Some of them are facing
north and some are facing south. T is an immediate neighbour of one who is sitting at
an extreme end of the line. Only three people sit between T and V. S sits second to the
right of V. S does not sit at an extreme end of the line. W sits on the immediate left of P.
W is not an immediate neighbour of V. The immediate neighbour of P faces opposite
directions.(i.e. If one neighbour faces north then other faces south and vice versa.) The
persons sitting at the extreme ends faces opposite directions.(i.e. If one person faces
north then other faces south and vice versa.)Q sits second to the left of U. U faces
north. U is not an immediate neighbour of T. The immediate neighbours of U faces
same directions.(i.e. If one neighbour faces north then other also faces north and if one
neighbour faces south then other also faces south).Both T and Q face a direction
opposite to that of S.(i.e. If S faces north then T and Q faces south and vice-versa.)

Q11. As per the following arrangements, which of the following statements is not
true with respect to P . ?
(a) P faces south.
(b) P is fourth to the right of Q.
(c) P is 2nd to left of T.
(d) P is between V and W.
(e) None of these

Q12. How many person sit on the left of Q ?


(a) One
(b) Two
(c) Three
(d) Four
(e) None of these

Q13. What is the position of S with respect to P ?


(a) Immediate left
(b) Third to left
(c) Third to right
(d) Fourth to left
(e) None of these

Q14. Which of the following is immediate neighbour of Q?


(a) T,S
(b) T,U
(c) V,U
(d) R,S
(e) None of these

Q15 Four of the given five are alike in a certain way based on the given
arrangement and hence form a group. Which of them does not belong to that
group .?
(a) R
(b) S
(c) P
(d) U
(e) V

Solutions(11-15):
S11. Ans.(c)
S12. Ans.(b)
S13. Ans.(c)
S14. Ans.(a)
S15. Ans.(d)

PUZZLE-45
Directions (6-10): Read the following information carefully and answer the
questions that follow.
Ten persons are sitting in two parallel rows containing five persons in each. In row 1, P,
Q R, S, and T are sitting and all of them are facing south. In row 2, A, B, C, D, and E are
sitting and all of them are facing north. In the given seating arrangement, each member
seated in a row faces another member of the other row but not necessarily in the same
order. Each of them is going to watch different movie viz. Bahubali, Alien, Half
Girlfriend, Radha, Posto, Dino Planet, Lahoriye, Sarkar, Rosetta, and Smurfs but not
necessarily in the same order.
There are two persons sitting between the person who is going to watch Bahubali who
sits at an extreme end, and T. A, who sits middle of the row, and not an
immediate neighbour of B, who is not going to watch Smurfs. E is sitting at an extreme
end of the row. T is going to watch Posto , sits on the immediate right of the person who
is going to watch Alien and faces the immediate neighhour of B. R does not sit at the
extreme left end of the row. A does not watch Rosetta. There is only one person
between Q and S, who is going to watch Bahubali. D is going to watch Dino Planet, is
an immediate neighbour of the person is going to watch Smurfs and does not face the
person who is going to watch Half Girlfriend. C, who is going to watch Radha, is an
immediate neighbour of the person who is going to watch Rosetta, who in turn faces the
immediate neighbour of the person who is going to watch Posto. There are two persons
between the person who is going to watch Lahoriye and the person who is going to
watch Half Girlfriend. R does not watch Lahoriye. E does not watch Sarkar.

Q6. Who among following sit at the extreme end of the line?
(a) R,C
(b) S,A
(c) Q,B
(d) E,T
(e) B,S

Q7. The immediate neighbor of A faces whom among the following?


(a) The person who who is going to watch Bahubali
(b) The person who who is going to watch Half Girlfriend
(c) The person who is going to watch Lahoriye.
(d) The person who who is going to watch Rosetta.
(e) Can't be determined

Q8.Who among the following sits diagonally opposite to R?


(a) E-Smurfs
(b) D-Dino Planet
(c) A-Radha
(d) E-Rosetta
(e) None of these

Q9. Four of the following five are alike in a certain way based on their seating
positions and so form a group. Which of the following is different from the
group?
(a) T
(b) Q
(c) A
(d) P
(e) D

Q10. Which of the following statements is/are definitely false?


(a) P-Lahoriye
(b) E-Smurfs
(c) T-Posto
(d) C-Radha
(e) All are true

Solution(6-10):

S6. Ans.(e)
S7. Ans.(c)
S8. Ans.(a)
S9. Ans.(a)
S10. Ans.(e)
PUZZLE-46
Directions (6-10): Study the information and answer the given questions:

Eight friends A, B, C, D, E, F, G, and H are sitting in a straight line but not necessarily in
the same order. Some of them are facing north while some are not. All of them like
different colour viz. violet, red, blue, black, white, yellow, pink and orange but not
necessarily in the same order. The following information is known about them.
A sits fourth to right of one who likes orange colour. H likes pink colour and sits second
to right of A. There are two friends sit between H and G, who likes yellow colour. One of
immediate neighbor of G sits extreme end. There are four friends sit between A and F,
who likes white colour. Immediate neighbor of G faces opposite direction(opposite
direction means if one neighbor of G faces north then other neighbor of G faces south
and vice-versa). E likes black colour and does not sit adjacent to H. The person who
likes red colour sits second to right of G. A does not like violet colour. The friends who
sit on the extreme end of line faces opposite direction(Faces opposite direction means if
one friend faces north then other friend faces south and vice-versa). B sits immediate
left of C. The one who likes red colour sits immediate right of H, who faces south
direction. C and D face same direction.

Q6. A likes which of the following colour?


(a) black
(b) yellow
(c) red
(d) blue
(e) pink

Q7. How many persons faces north direction?


(a) two
(b) four
(c) three
(d) five
(e) None of these

Q8. Who among following sits fifth to right of G?


(a) D
(b) H
(c) C
(d) B
(e) F

Q9. Who among following likes violet colour?


(a) D
(b) H
(c) C
(d) B
(e) F
Q10. If F is related to violet and A is related to black, in the same way H is related
to?
(a) Blue
(b)Black
(c) White
(d) Red
(e) Orange
Solution (6-10):

S6. Ans.(d)
S7. Ans.(c)
S8. Ans.(d)
S9. Ans.(c)
S10. Ans.(e)

PUZZLE-47
Directions (11-15): Study the information and answer the given questions:
Seven people P, Q, R, S, T, U and V live on separate floors of a 7-floor building.
Ground floor is numbered 1, first floor is numbered 2 and so on until the topmost floor
is numbered 7. Each one of them belongs to different cities, viz Noida, Delhi, Agra,
Meerut, Kanpur, Pune and Puri but not necessarily in the same order. Only three
people live above the floor on which P lives. Only one person lives between P and the
one belongs to Pune. U lives immediately below the one who belongs to Delhi. The
one belongs to Delhi lives on an even-numbered floor. Only three people live between
the ones belongs to Pune and Agra. T lives immediately above R. T does not belong to
Agra. Only two people live between Q and the one who belongs to Kanpur. The one
who belongs to Kanpur live below the floor on which Q lives. The one who belongs to
Noida does not live immediately above or immediately below Q. S does not live
immediately above or immediately below P. V does not belong to Meerut.

Q11. Which of the following is true with respect to V as per the given information?
(a) The one who lives immediately below V is belong to Delhi.
(b) V lives on floor no. 7.
(c) V lives immediately below T.
(d) V lives on the lowermost floor.
(e) V belongs to Pune.

Q12. Who among the following lives on floor no. 3?


(a) The one who belongs to Meerut
(b) The one who belongs to Kanpur
(c) R
(d) V
(e) T

Q13. Who lives on the floor immediately above T?


(a) P
(b) Q
(c) S
(d) V
(e) U

Q14. S belongs to which of the following city?


(a) Delhi
(b) Pune
(c) Agra
(d) Kanpur
(e) Meerut

Q15. How many people live between the floors on which S and the one who
belongs to Delhi?
(a) None
(b) Two
(c) One
(d) More than three
(e) Three
Solution(11-15):
S11. Ans.(a)
S12. Ans.(b)
S13. Ans.(e)
S14. Ans.(e)
S15. Ans.(b)

PUZZLE-48
Directions (1-5): Study the following information carefully and answer the
questions given below:
There are eight friends A, B, C, D, E, F, G and H are sitting around a square table in
such a way that four of them sit at the four corners of the square table while four sit in
the middle of each of four sides. The one who sits at the four corners face the center
and those who sit in the middle of the sides face outward. Each of them likes
different colour, viz Red, Green, Violet, Black, White, Pink, Blue and Orange.
F sits third to left of the one who likes Green. The one who likes Green faces outward.
Only two friends sit between F and A. The one who likes Red sits on the immediate right
of A. The one who likes White sits second to right of B, who is not immediate neighbor
of A or F.B does not like Green. Only one friend sits between H and one who likes
White. E sits on the immediate left of the one who likes Black. B does not like Black. D
likes Violet but he is not immediate neighbor of H. The one who likes Orange is an
immediate neighbor of D. The one who likes Pink is an immediate neighbor of C. C is an
immediate neighbor of both E and the one who likes Orange.

Q1. Who among following likes Pink colour?


(a) E
(b) A
(c) H
(d) G
(e) F

Q2. Who among following sits opposite to D?


(a) G
(b) F
(c) C
(d) B
(e) A

Q3. Who among following sits second to right of D?


(a) A
(b) E
(c) C
(d) B
(e) G
Q4. Four of the following five are alike in a certain way and hence they form a
group. Which one of the following does not belong to that group?
(a) A
(b) H
(c) B
(d) E
(e) F

Q5. Who among following sits diagonally opposite to H ?


(a) E
(b) B
(c) F
(d) A
(e) None of these

Solution(1-5):

S1. Ans.(a)
S2. Ans.(e)
S3. Ans.(c)
S4. Ans.(a)
S5. Ans.(b)

PUZZLE-49

Directions (1-5): Study the following information carefully and answer the
questions given below:

Farmer A of Nirman Nagar goes to trade fair with their family after completing their
daily work. There are nine members in the family after including farmer, i.e. A, B, C,
D, E, F, G, H and I. Each member eats different food items viz. Chocolate, Pizza,
Burger, Rasmalai, Samosa, Jalebi, Khulfi, Dosa and Idli. The relation of farmer with
the family members is defined as Father, Mother, Brother, Sister, Wife, Daughter,
Son, and Brother-in-law but not in the same order. All the family members are
enjoying circular ride, but not necessarily in the same order. Some of them are facing
towards centre while others are facing outward.
I is brother of A’s daughter. A’s brother is seated on the immediate right of the
brother of A’s sister and both face the same direction. I’s father eats Pizza. H has a
sister. The mother of A’s son is E and sits second to the left of the married son of A’s
father. F’s daughter-in-law eats Idli. A faces outward. F is a male. A’s father sits third
to the right A’s daughter. Either I or H sits third to the left of the mother of C, but
both I and H face the same direction. A’s brother and sister eats Burger and Chocolate
respectively. G is not father of H. In two generations, the members of the same
generation are sitting immediate next to each other and face the same direction. A’s
father does not eat Dosa and Khulfi. F’s grandson eats Rasmalai. B is a female and of
the same generation as of A. G sits on the immediate left of the granddaughter of A’s
father. The one, who is the wife of F likes Kulfi . A’s brother-in-law neither eats Dosa
nor Samosa. A’s father doesn’t sit immediate next to his children. G is facing
the centre and is second to the right of A. B sits second to the left of F. No three
members seated together can face the same direction.
Q1. How is B’s husband related to A’s father?
(a) Brother
(b) Father
(c) Son
(d) Grandson
(e) Son-in-law

Q2. Who sits third to the right of A’s mother?


(a) B
(b) H’s brother-in-law
(c) A’s brother-in-law
(d) E’s husband
(e) Both (b) and (c)

Q3. Who among the following sits exactly between A and his wife?
(a) F
(b) A’s sister
(c) A’s daughter
(d) D
(e) None of these

Q4. Four of the following five are alike in a certain way and hence they form a
group. Which one of the following does not belong to that group?
(a) C
(b) F
(c) H
(d) I
(e) A

Q5. Which of the following persons faces inside?


(a) B, G, F and I.
(b) A, G, D and I.
(c) G, D, A’s father and G’s wife.
(d) D, G, B and C.
(e) None of these

Solution(1-5):

S1. Ans.(e)
S2. Ans.(e)
S3. Ans.(b)
S4. Ans.(b)
S5. Ans.(c)

PUZZLE-50
Directions(6-10): Answer the questions on the basis of the information given
below.
Six kids– Z, X, C, V, B and N- watch in different Cartoon serials on their TV viz.
Ducktails, Doremon, Shinchain, Baloo, Chota beem and Hanuman – and they are lives
on six different floors of a building, numbered 1 to 6. They like six different Cartoon
Characters viz. Dholu, Sizuka, Munni, Lui, Jiyan and Pumba, in no particular order.
Each Cartoon serial is cast on a different day from Monday to Saturday. X
likes Jiyan and Cartoon serial likes by him is Doremon and lives on the 6th floor of the
building. B and Z lives on even numbered floors who’s cartoon cast on Saturday and
Monday. The one who watch Ducktails is lives on the lowest floor and it is cast on
Tuesday. Baloo Cartoon serial watch by that kid who lives on the floor next to X and
it is cast on Thursday. The kid who watches Hanuman likes Pumba and his cartoon is
cast on Monday. N likes Lui and he lives on first floor. One of the kid who watches
Cartoon serials which cast on friday lives on even numbered floors. V lives two floors
below C and he likes Munni. B, who does not live immediately above or below of C,
likes Dholu and his serial cast on Saturday. The kid who likes Chota beem is lives on
an even numbered floor.
Q6. Who is watching Ducktails cartoon serial?
(a) Z
(b) N
(c) V
(d) X
(e) C

Q7. Which Character does B like?


(a) Pumba
(b) Jiyan
(c) Dholu
(d) Lui
(e) Sizuka

Q8. Who among the following watch that cartoon which is cast on Thursday?
(a) V
(b) N
(c) Z
(d) X
(e) C

Q9. If V is related to C and B is related to Z, in the same way N is related to?


(a) X
(b) N
(c) Z
(d) V
(e) C

Q10. Which of the following combination is correct?


(a) V – Shinchain – Sizuka
(b) C – Baloo – Munni
(c) Z – Chota beem – Pumba
(d) X – Doremon – Dholu
(e) N – Ducktails – Lui

Solution(6-10):
S6. Ans.(b)
S7. Ans.(c)
S8. Ans.(e)
S9. Ans.(d)
S10. Ans.(e)

PUZZLE-51
PUZZLE-52

PUZZLE-53
PUZZLE-54
PUZZLE-55
PUZZLE-56
PUZZLE-57
PUZZLE-58
PUZZLE-59
PUZZLE-60
PUZZLE-61
PUZZLE-62
PUZZLE-63
PUZZLE-64
PUZZLE-65
PUZZLE-66
PUZZLE-67
PUZZLE-68
PUZZLE-69
PUZZLE-70
PUZZLE-71
PUZZLE-72
PUZZLE-73
PUZZLE-74
PUZZLE-75

PUZZLE-76
Directions (1-5): Study the following information carefully and answer the given
questions.
There are seven boys Sudhan, Hassan, Nakul, Rushil, Vishnu, Ishat and Amir who
participated in a game competition which started on Monday and ended on Sunday.
In the first round of the competition, each of them played game under different
teams Red, Blue, Green, Yellow, Pink, Orange and White, but not necessarily in the
same order. They like different States, Maharashtra, Kerala, Assam, Bihar, Tamil
Nadu, Gujarat and Haryana, but not necessarily in the same order. Hassan did not
play on the day either immediately before or immediately after the game of Rushil,
who does not like either Tamil Nadu or Maharashtra or Bihar State. Two games
were held between the game of Amir and Ishat, neither of them played on Monday.
There was one game between the games of Rushil and Nakul. But Nakul’s game did
not happen either on Monday or on Wednesday. Nakul likes Kerala State and
played Red. The one, who played under Green team on the last day of competition,
likes Assam State. Vishnu played immediately after Nakul and he likes Maharashtra
State. Sudhan does not like Tamil Nadu State and played under Yellow team. Rushil
did not play for either White or Orange ream. The one who played for Pink team was
scheduled immediately after the game of Red team. Amir, who likes Gujarat, played
on the fourth day of the competition but played neither Red team nor Blue team.

1). Who among the following like Tamil Nadu state?


a) Hassan
b) The one who played for Yellow team
c) The one who played for Pink team
d) The one who played for Red team
e) Vishnu

2). The one who played on Friday likes which of the following state?
a) Tamil Nadu
b) Gujarat
c) Haryana
d) Kerala
e) Maharashtra

3). Who among the following like Bihar State?


a) Amir
b) Hassan
c) Sudhan
d) Vishnu
e) Ishat

4). Amir plays for which of the following team?


a) Orange
b) White
c) Red
d) Green
e) Either Orange or White

5). Which of the following statement is true?


a) Thursday – Amir – White - Haryana
b) Tuesday – Sudhan – Yellow - Bihar
c) Monday – Rushil - Blue - Haryana
d) Sunday – Ishat – Pink - Assam
e) Saturday – Vishnu - Orange - Maharashtra

PUZZLE-77
Directions (6-10): Study the following information carefully and answer the given
questions.
Seven friends namely Subha, Praveen, Zaheer, Ganesh, Bharathi, Kaviya and Nisar
attend different Exams coaching namely NIACL Clerk, SBI PO, BOB PO, NICL AO,
DENA PO, BOI PO and LVB PO, not necessarily in the same order, from Monday to
Sunday (of the same week). Subha attends a coaching on Friday. Only two people
attend coaching between Subha and the one who is attending NICL AO class.
Kaviya attends a coaching immediately before Ganesh, Neither Kaviya nor Ganesh
is attending NICL AO class. Only one person attends a coaching between Kaviya
and the one who is attending BOB PO. The one who is attending BOB PO class
does not have a coaching on Monday. Praveen attends a coaching immediately
before the one who is attending NIACL Clerk class. Subha is not attending NIACL
Clerk class. Only one person has a coaching between Kaviya and Bharathi. Zaheer
is attending BOI PO class. Only two people have coaching between Zaheer and the
one who is attending DENA PO class. Bharathi is not attending LVB PO class.

6). BOI PO class held on which of the following day?


a) Friday
b) Saturday
c) Sunday
d) Monday
e) Tuesday

7). How many persons attend the class between Bharathi and Nisar?
a) Four
b) Three
c) Five
d) Two
e) One

8). Praveen attends which of the following coaching class?


a) LVB PO
b) NICL AO
c) SBI PO
d) BOB PO
e) NIACL Clerk

9). Who among the following attend the class on Wednesday?


a) Bharathi
b) Ganesh
c) Nisar
d) Subha
e) Kaviya

10). Which of the following statement is true?


a) Sunday – Zaheer - BOB PO
b) Friday – Kaviya - BOI PO
c) Thursday – Ganesh - DENA PO
d) Tuesday – Bharathi - SBI PO
e) Wednesday – Praveen - NICL AO

Explanation with answers key:


Directions (Q. 1-5):

1). Answer: a)
2). Answer: d)
3). Answer: c)
4). Answer: e)
5). Answer: b)

Directions (Q. 6-10):


6). Answer: c)
7). Answer: a)
8). Answer: b)
9). Answer: e)
10). Answer: c)

PUZZLE-78
Directions (1-5): Study the following information carefully and answer the given
questions.
There are nine persons Ratan, Samar, Darshan, Gokul, Yoonus, Vasu, Manish,
Faraz and Bibek stay on a nine floors building, but not necessarily in the same
order. Only one person stays on each floor. All of them have either Computer or
Laptop of different companies i.e. Microsoft, Acer, Samsung, Lenovo, Apple, HP,
Asus, Dell and Sony, but not necessarily in the same order. Only five of them have
Laptop. The ground floor is numbered 1 and the topmost floor is numbered 9. There
are two floors between the floors on which the persons having a HP Computer and
Dell Laptop stay. Darshan owns a Samsung Computer. Vasu does not own a Sony
Computer. Ratan stays on even numbered floor below the even numbered floor on
which Faraz stays. The one who owns Apple Laptop stays on the fourth floor. Gokul
stays on the second floor and owns the Lenovo Laptop. The one who owns the Acer
Laptop stays on the topmost floor. Ratan does not own Asus Laptop. Vasu owns a
HP Computer and stays on an odd-numbered floor. There is only one floor between
the floors on which Vasu and Manish stay. Yoonus stays immediate above the floor
on which Bibek stay. Manish does not stay on the ground floor. Samar stays
immediate above the floor on which Faraz stay. There are three floors between the
floors on which Darshan and Vasu stay. The one who owns the Dell Laptop stays
immediate above Manish. Yoonus stays on the fourth floor. The one who owns the
Sony Computer stays on the third floor.

1). Vasu lives on which of the following floor?


a) 1st floor
b) 3rd floor
c) 5th floor
d) 7th floor
e) 9th floor

2). Which company laptop used by the person who lives in 4th floor?
a) Apple
b) Lenovo
c) Asus
d) Acer
e) Dell

3). Four of the following five are alike in a certain way and hence they form a group.
Which one of the following does not belong to that group?
a) Ratan
b) Darshan
c) Vasu
d) Bibek
e) Manish
4). How many persons are between the one who uses Lenovo laptop and the one
who uses Microsoft computer?
a) Two
b) Three
c) Four
d) Five
e) One

5). Which of the following person uses Sony computer?


a) Gokul
b) Darshan
c) Faraz
d) Bibek
e) Samar

PUZZLE-79
Directions (6-10): Study the following information carefully and answer the given
questions.
Eight friends Deepan, Jalil, Nagaraj, Sharma, Hanuman, Tilak, Sukumar and
Prasanna live on eight different floors of a building but not necessarily in the same
order. The lowermost floor of the building is numbered 1 and the topmost floor of the
building is numbered 8. Each of them likes different players of grand prix viz, Kimi
Räikkönen, Daniel Ricciardo, Max Verstappen, Lewis Hamilton, Fernando Alonso,
Sebastian Vettel, Jenson Button and Nico Rosberg but not necessarily in the same
order.
The one who likes Daniel Ricciardo lives on an even-numbered floor but not on the
topmost floor. Only one person lives between Tilak and the one who likes Fernando
Alonso. Only two persons live between Tilak and the one who likes Daniel Ricciardo.
Neither Hanuman nor Nagaraj lives on the first floor. Only one person lives between
Nagaraj and the one who likes Max Verstappen. Deepan lives just above Tilak. Only
two persons live between Hanuman and Deepan. The one who likes Fernando
Alonso does not live on floor number one, Jalil lives on an even-numbered floor and
just above Nagaraj. The one who likes Lewis Hamilton lives on an even numbered
floor and lives just above the person who likes Nico Rosberg. Nagaraj does not like
Fernando Alonso or Nico Rosberg. Only two persons live between the one who likes
Sebastian Vettel and the one who likes Kimi Räikkönen . Sharma does not like
Jenson Button. The one who likes Sebastian Vettel does not live on an odd-
numbered floor. Prasanna lives just below the one who likes Nico Rosberg.

6). Sukumar likes which of the following player?


a) Jenson Button
b) Max Verstappen
c) Lewis Hamilton
d) Nico Rosberg
e) Kimi Räikkönen

7). Four of the following five are alike in a certain way and hence they form a group.
Which one of the following does not belong to that group?
a) Deepan
b) Nagaraj
c) Prasanna
d) Jalil
e) Sukumar

8). How many persons are between the one who likes Lewis Hamilton and the one
who stays on 4th floor?
a) One
b) Two
c) Three
d) Four
e) Five

9). Who lives on fifth floor?


a) Jalil
b) Hanuman
c) The one who likes Lewis Hamilton
d) The one who likes Fernando Alonso
e) Both option b and d

10). Which of the following combinations is true?


a) 1 – Sharma - Jenson Button
b) 4 – Hanuman - Daniel Ricciardo
c) 3 – Tilak - Nico Rosberg
d) 6 – Prasanna - Sebastian Vettel
e) 7 – Nagaraj - Kimi Räikkönen

Explanation With Answers Key:


Directions (Q. 1-5):

1). Answer: c)
2). Answer: a)
3). Answer: e)
4). Answer: b)
5). Answer: d)

Directions (Q. 6-10):

6). Answer: a)
7). Answer: b)
8). Answer: c)
9). Answer: b)
10). Answer: d)

PUZZLE-80
Directions (1-5): Study the following information carefully and answer the given
questions.
Eight Bank MD and CEOs – Shika Sharma, Rakesh Sharma, Kishor Kharat,
Koteeswaran, Arundhati, Srinivas, Chanda Kochhar and Usha - have come together
to discuss a strategy for the development of the banks. They are sitting around a
round table such that exactly half of them are facing the centre. Each MD and CEO
heads a different bank viz, Indian Bank, Axis Bank, ICICI Bank, UCO Bank, Dena
Bank, State Bank of India, Canara Bank and Bank of India. Of these, Uco Bank and
Bank of India fall in the same zone. Kishor Kharat, Rakesh Sharma, Koteeswaran
and Srinivas are the male CEOs.
Rakesh Sharma, from Indian Bank and facing the centre, sits opposite to the CEO
from Dena Bank. Usha, an immediate neighbour of Rakesh Sharma, is fourth to the
right of the CEO from State Bank of India, who is facing the centre. Kishor Kharat,
from Canara Bank, sits second to the right of Srinivas and both are facing outside.
The CEO from ICICI Bank also the person whose bank name starts with the Second
alphabet letter is facing the centre. He is also to the immediate right of Usha. To the
immediate left of Koteeswaran is Arundhati, who is facing outside and is opposite to
Chanda Kochhar, who is the CEO from Bank of India and faces the centre. The
CEOs from the same zone are sitting opposite each other. Only two males face the
centre. No two people of the same gender sit next to each other.

1). Who sits second to the left of the person the one who represents for Canara
Bank?
a) The one who represents for Dena Bank
b) Shikha Sharma
c) Chanda Kochhar
d) The one who represents for Indian Bank
e) None of these

2). Usha represents for which of the following bank?


a) State Bank of India
b) Bank of India
c) ICICI Bank
d) Indian Bank
e) Axis Bank
3). How many persons sit between the person who represent for ICICI Bank and the
person who represent for Indian Bank? (Count from the left of the person who
represent for ICICI Bank)
a) Four
b) Two
c) Five
d) One
e) Three

4). Who represents for UCO Bank?


a) Usha
b) Arundhati
c) Shikha Sharma
d) Srinivas
e) None of these

5). Which of the following combinations is true?


a) Srinivas – Facing Centre – Dena Bank
b) Kishore Kharat – Facing Outside – UCO Bank
c) Koteeswaran – Facing Centre – State bank of India
d) Usha – Facing Outside – Axis Bank
e) None of these

PUZZLE-81
Directions (6-10): Study the following information carefully and answer the given
questions.
Eight persons Suresh, Prakash, Arun, Kavi, Mohan, Jaffar, Naveen and
Dinesh are seated around a circular table such that four faces the centre and four
are facing opposite to the centre. They like different Pro Kabaddi League teams:
Bengaluru Bulls, U Mumba, Puneri Paltan, Bengal Warriors, Jaipur Pink Panthers,
Patna Pirates, Telugu Titans and Dabang Delhi not necessarily in the same order.
The arrangement is based on the following rules:
 One who likes Dabang Delhi is seated opposite to Suresh
 Kavi is 3rd to the right of the one who likes Patna Pirates who is seated between
Prakash and the one who likes Puneri Paltan
 Naveen is 3rd to the left of Arun who likes Telugu Titans
 Dinesh is to the immediate left of the one who likes Jaipur Pink Panthers
 One who likes Dabang Delhi is a neighbour of Kavi
 Suresh does not like Patna Pirates and Naveen does not like Dabang Delhi
 Suresh is 2nd to the left of Mohan who is not a neighbour of Kavi
 Either Suresh or Naveen likes Jaipur Pink Panthers
 Kavi and Prakash are seated facing the same direction.
 The one who likes Puneri Paltan is 2nd to the right of the one who likes U Mumba
 Kavi and Dinesh do not like U Mumba, Naveen faces outside the centre
 One who likes Bengal Warriors is facing the opposite direction faced by Arun.
 Not more than two persons who sits on consecutive seats face the same direction.

6). Who among the following likes Patna Pirates?


a) Suresh
b) Dinesh
c) Jaffar
d) Mohan
e) None of these

7). The only person who is sitting between Dinesh and Naveen likes which of the
following teams?
a) Puneri Paltan
b) Bengal Warriors
c) Jaipur Pink Panthers
d) Telugu Titans
e) Bengaluru Bulls

8). Who among the following is not facing centre?


a) Arun
b) Mohan
c) Suresh
d) Dinesh
e) Jaffar

9). Who is sitting third to the right of the person who like Telugu Titans team?
a) The one who likes Puneri Paltan team
b) The one who likes U Mumba team
c) The one who likes Dabang Delhi team
d) The one who likes Patna Pirates team
e) None of these

10). Which of the following is correctly matched?


a) Kavi – Facing Centre – Bengaluru Bulls
b) Dinesh - Facing Outside – Puneri Paltan
c) Jaffar – Facing Outside – Dabang Delhi
d) Arun – Facing Centre – Bengal Warriors
e) Prakash – Facing Outside – U Mumba

Answer Key:
1. a) 2. e) 3. c) 4. b) 5. d)

6. d) 7. c) 8. a) 9. b) 10. e)

PUZZLE-82
Directions (1-5): Study the given information carefully to answer the given questions.
Seven people - P, Q, R, S, T, U and V live on seven different floors of a
building but not necessarily in the same order. The lower most floor of the building is
numbered 1, the one above that is numbered 2 and so on till the top most floor is
numbered 8. Each one of them likes a different watch— Casio, Citizen, Fossil,
Seiko, Tissot and Fastrack, but not necessarily in the same order.
U lives on floor numbered 4. Only two people live between U and the one who
likes Citizen. Only three people live between the one who likes Citizen and the one
who likes Fastrack. S lives on one of the even numbered floors above the one who
likes Fastrack. Only one person lives between S and the one who likes Seiko. V
lives one of the odd numbered floors below the one who likes Seiko. Only one
person lives between V and the one who likes Fossil. P lives with immediately above
or immediately below the one who likes Fossil. T lives immediately above P. Only
one person lives between Q and the one who likes .Casio. The one who likes Tissot
lives on one of the floors above the one who likes Omega.

1). Who lives immediately below S ?


a) T
b) Q
c) U
d) The one who likes Omega
e) The one who likes Tissot

2). How many people live above the one who likes Seiko ?
a) More than three
b) Two
c) None
d) Three
e) One

3). Which of the following watches does T like ?


a) Omega
b) Fastrack
c) Fossil
d) Tissot
e) Casio
4). On which of the following floor numbers does Q live ?
a) 7
b) 5
c) 3
d) 2
e) 6

5). How many people live between the one who likes Fastrack and the one who likes
Casio ?
a) None
b) One
c) More than three
d) Two
e) Three

Answer Key:
1).b 2).d 3).c 4).b 5).b
Explanation:
Directions (Q. 1-5):
PUZZLE-83
Directions (1-5): Study the following information carefully and answer the given
questions.
Nine persons Lalith, Devan, Mahendran, Ragu, Indran, Samuel, Vadivel,
Mani, Sathish are sitting in a row facing in either north or south direction but not
necessarily in the same order. They work in different companies they are Sony,
Nestle, Microsoft, IBM, Wipro, Bajaj, Apple, Samsung and TCS but not necessarily
in this order. The following information is known about them.
Samuel and Sathish are immediate neighbours of Mahendran but face in opposite
directions to each other. Vadivel and Mani face in the same direction. Lalith who
works in Wipro sits in the middle and faces towards south. The person working in
Bajaj sits to the immediate right of a person facing in north direction. The person
sitting at first and last position from left face in north direction and they work in Apple
and Microsoft respectively. The person working in Samsung sits to the immediate
right of Devan. Samuel does not sit with Lalith. Indran sits second to the left of Ragu
but does not work in Microsoft. Mani sits two places away from Lalith. Ragu sits
exactly between Vadivel and Mani. Also, there is only one person sitting between
Vadivel and Mani. Devan and Mahendran face in the same direction. The person
who works in Nestle sits fourth to the left of the person working in Bajaj but neither of
them is Mani. The persons working in TCS and Sony are sitting adjacent to each
other. The person working in Sony sits fourth to the right of Samuel.

1). In which of the following company Vadivel works?


a) IBM
b) Bajaj
c) Samsung
d) TCS
e) Microsoft

2). The one who works in Wipro sits third to the left of ______________
a) Vadivel
b) Samuel
c) The one who works in TCS
d) The one who works in Nestle
e) Both option c and d

3). Who works in Sony Company?


a) Devan
b) Samuel
c) Mani
d) Ragu
e) Indran

4). Which of the following persons sits at the extreme ends of row?
a) Ragu and Mahendran
b) Devan and Sathish
c) Mani and Vadivel
d) Sathish and Indran
e) Indran and Lalith

5). Which of the following combinations is true?


a) Vadivel - Bajaj – Facing South
b) Mahendran – Nestle – Facing South
c) Indran – Sony – Facing North
d) Ragu – Apple – Facing South
e) None of these
PUZZLE-84
Directions (6-10): Study the following information carefully and answer the given
questions.
Virat Kohli, Saina Nehwal, Abhinav Bindra, Vijender Singh, Anwar Ali and
Sardar Singh are six sports players, who belong to six different states viz–Haryana,
Punjab, Maharashtra, Assam, Bihar and Rajasthan but not necessarily in the same
order, each one has won the trophy in different countries, viz Australia, Malaysia,
Singapore, China, Argentina and Brazil but not necessarily in the same order. They
are sitting around a circular table facing the centre. The persons who have won the
trophy in China, Singapore and Argentina are neither from Rajasthan nor from
Haryana. The persons who are from Assam and Punjab have won the trophies
neither in China nor in Brazil. Virat Kohli is neither from Rajasthan nor sits on the
immediate left of the person who won the trophy in Malaysia. The only person who is
between Anwar Ali and Sardar Singh has won the trophy in Australia. The person
who is on the left side of the person from Rajasthan has not won the trophy in Brazil.
Vijender Singh has won the trophy in Malaysia and is from Punjab. He is facing the
person who has won the trophy for Australia. One who has won the trophy for China
is sitting opposite the person from Bihar, while the person from Punjab is on the left
of the person who has won the trophy in Argentina. One who has won the trophy in
Brazil is on the immediate right of the person from Rajasthan but on the left of the
person who has won the trophy in Singapore. Abhinav Bindra has not won the
trophy in Singapore and Sardar Singh has not won the trophy in China.

6). Which of the following player belongs to Rajasthan?


a) Abhinav Bindra
b) Virat Kohli
c) Saina Nehwal
d) Anwar Ali
e) Sardar Singh
7). Saina Nehwal belongs to which of the following state?
a) Punjab
b) Rajasthan
c) Haryana
d) Maharashtra
e) Bihar

8). Who sits second to the left of the player the one who won the trophy in Malaysia?
a) Virat Kohli
b) The player who won the trophy in China
c) The player who belongs to Rajasthan
d) The player who won the trophy in Brazil
e) The player who belongs to Maharashtra

9). Which of the following player won the trophy in China?


a) Abhinav Bindra
b) Anwar Ali
c) Saina Nehwal
d) Vijendar Singh
e) Virat Kohli

10). Which of the following combinations is true?


a) Abhinav Bindra – Haryana – Australia
b) Anwar Ali – Punjab - China
c) Virat Kohli – Assam - Argentina
d) Sardar Singh – Bihar - Singapore
e) None of these

Answer Key:
1). b) 2). e) 3). d) 4). d) 5). a)

6). a) 7). e) 8). d) 9). b) 10). c)

PUZZLE-85

Directions (1-5): Study the following information carefully and answer the given

questions.

Seven people namely Sahith, Umesh, Maaran, Duruvan, Wasim, Aadhav and
Gopi buy different colour shirts viz. Red, Blue, Green, Yellow, White and Orange
and different colour pants viz. Black, Grey, Sandal and Brown from different shops
say Kumaran Silks, Saravana Stores, Chennai silks and Pothys. Not more than two
people buy from same shop. It is known that each person buys both the items from
the same shop. The people who buy Blue colour shirt get from the same shop and
they don’t buy the same colour pants and is neither Sahith nor Gopi. Aadhav buys
from Pothys. Duruvan and Aadhav buy same colour pants which are bought by no
one apart from them and they get from different shops. The only person who orders
Brown colour pant from Chennai silks and is neither Umesh nor Maaran. Duruvan
doesn’t shop on Kumaran Silks. Orange colour shirt is get from Kumaran Silks but
not by Sahith. Maaran buy Green colour shirt. One who buys Black colour pant buys
from Kumaran Silks and is not Umesh and also buys either Red or White colour
shirt. The persons who buy Grey colour pant get on different shops but not on
Saravana Stores. Only one person buys Black colour pant. Maaran doesn’t get
Sandal colour pant. Aadhav neither orders Red or Yellow colour shirt.

1). Who among the following person shopping in Saravana Stores?


a) Sahith
b) Maaran
c) Duruvan
d) Umesh
e) Gopi

2). Who among the following persons bought the Grey colour pant?
a) Gopi
b) Maaran
c) Umesh
d) All of the above
e) None of these

3). Which of the following colour shirt and pant bought by Duruvan?
a) Red and Grey
b) Yellow and Sandal
c) Orange and Brown
d) Yellow and Black
e) White and Sandal
4). Which of the following colour shirt bought by two persons?
a) Blue
b) Orange
c) Green
d) Red
e) White

5). Which of the following combinations is true?


a) Wasim – Blue – Brown – Kumaran silks
b) Maaran – Green – Black - Pothys
c) Sahith – Red – Grey - Kumaran Silks
d) Gopi - White - Grey - Kumaran Silks
e) e) Umesh - Blue - Grey - Chennai silks

PUZZLE-86
Directions (6-10): Study the following information carefully and answer the given
questions.
There are eight people namely A, B, C, D, E, F, G and H who sit around a
circular table some of them facing towards centre and some of them facing away
from the centre. They like to buy different mobiles. They are Samsung Galaxy C7
Pro, Samsung Galaxy J2 Prime, Samsung Galaxy J5, Samsung Galaxy On5,
Samsung Galaxy J2 Ace, Samsung Galaxy J1, Samsung Galaxy A7 and Samsung
Galaxy C9 Pro. The following information is known about them.
H’s neighbours face in same direction but opposite to that of H. G sits third to the left
of E who faces towards the centre. D’s spouse who likes to buy Samsung Galaxy J5
sits third to the left of D. C sits third to the right of a male and they face in the same
direction. B’s immediate neighbours face in same direction but opposite to that of B.
The female who likes to buy Samsung Galaxy J2 Prime sits opposite to the male
who buys Samsung Galaxy C7 Pro. A who wants Samsung Galaxy A7 sits second
to the left of D and they face in the same direction. The immediate neighbours of C
like to have Samsung Galaxy J2 Prime and Samsung Galaxy C9 Pro. B and F sit
together and none of them sits with A and D. G’s spouse who likes to buy Samsung
Galaxy On5 sits second to the left of G. The immediate neighbours of H’s only sister
who are neither C nor E are A and D. B who buys Samsung Galaxy J1 is the only
one whose both the immediate neighbours are of opposite gender to B.

6). Who likes to buy Samsung Galaxy J2 Prime?


a) B
b) E
c) D
d) H
e) A

7). How many persons facing centre?


a) Five
b) Six
c) Four
d) Three
e) Two

8). Who is sitting second to the left of person who likes to buy Samsung Galaxy J1?
a) The one who uses Samsung Galaxy J2 Ace
b) The one who uses Samsung Galaxy C9 Pro
c) The one who uses Samsung Galaxy J1
d) The one who uses Samsung Galaxy J2 Prime
e) The one who uses Samsung Galaxy J5

9). How many persons sit between who likes to buy Samsung Galaxy J5 and who
likes to buy Samsung Galaxy J2 Ace, Count from the right of the person who likes to
buy Samsung Galaxy J5?
a) 1
b) 5
c) 3
d) 2
e) 4

10). Which of the following is correctly matched?


a) G – Male – Samsung Galaxy J2 Ace
b) H – Female – Samsung Galaxy A7
c) A – Female – Samsung Galaxy J1
d) C – Male – Samsung Galaxy On 5
e) None of these

Answer Key:

Direction (01 to 05):


Person Shirt Colour Pant Colour Shops
Sahith Red Black Kumaran Silks
Umesh Blue Grey Chennai silks
Maaran Green Grey Pothys
Duruvan Yellow Sandal Saravana Stores
Wasim Blue Brown Chennai silks
Aadhav White Sandal Pothys
Gopi Orange Grey Kumaran Silks

1. c) 2. d) 3. b) 4. a) 5. e)

Direction (06 to 10):


6. c) 7. a) 8. e) 9. b) 10. d)

PUZZLE-87
Directions (1-5): Study the following information carefully and answer the given
questions.
Ishanth, Thilak, Yaswanth, Gulam, Charles, Kavin, Rashid, Bhasker, Jeyaraj and
Eeswar have some Bikes and cars. Both kind of vehicles are 1-10 in number. No 2
persons have the same number of same vehicles. It is also known that:
 Thilak has 6 bikes more than Rashid.
 Thilak has 10 vehicles in total.
 Eeswar has 3 cars.
 The total number of cars owned by Yaswanth and Jeyaraj is equal to the number of
cars that Bhasker has. Yaswanth has less number of cars than Jeyaraj.
 Only Bhasker has 18 vehicles in total such that the number of cars he has is more
than his bikes.
 Only Yaswanth has equal number of bike and cars.
 Only Rashid has cars twice than the bikes.
 Ishanth has 9 vehicles in total. He has more cars than bikes.
 The number of bikes owned by Gulam is equal to the number of cars owned by
Charles.
 Charles has 8 vehicles in total.
 Charles and Kavin have 16 cars in total.
 Jeyaraj and Eeswar have 16 bikes in total.

1). Who among the following have 3 bikes?


a) Bhasker
b) Eeswar
c) Rashid
d) Kavin
e) Thilak

2). What is the sum of total number of vehicles owned by Gulam and Jeyaraj
together?
a) 25
b) 20
c) 17
d) 22
e) 28

3). Who among the following have 10 cars?


a) Yaswanth
b) Bhasker
c) Charles
d) Jeyaraj
e) Gulam

4). Which of the following combinations is true?


a) Rashid – 6 bikes – 3 cars
b) Eeswar - 10 bikes – 8 cars
c) Thilak – 1 bikes – 9 cars
d) Gulam – 7 bikes – 4 cars
e) None of these

5). If ‘Ishanth’ is related to ‘Gulam’, ‘Charles’ is related to ‘Thilak’, then which


following person is ‘Rashid’ related to?
a) Yaswanth
b) Gulam
c) Kavin
d) Ishanth
e) Eeswar

PUZZLE-88
Directions (6-10): Study the following information carefully and answer the given
questions:
Six cricketers Amla, Maxwell, Watson, Gayle, Narine and Lynn are going to
play for IPL Teams namely Punjab, Kolkata and Bangalore. They are coming in
three different Cars such as, Hyundai, Ford and Toyota. The Cricketers are making
presence in advertisements such as, Samsung, Oppo, Vivo, Motorola, Micromax
and Gionee not necessary in the same order. And they like different bats they are
Adidas, GM Icon, Puma, SS Ton, Reebok not necessary in the same order.
 One of the players from Bangalore and Punjab teams coming through same cars.
Amla who belongs to the Kolkata team does not come with Maxwell by Toyota.
 The one who presenting ad in Oppo and likes GM Icon bat going to play with the
cricketer who likes SS Ton bat. One of the player from Punjab presenting ad in
Motorola.
 Each cars carries same number of cricketers and one of the cricketers comes by
Toyota, playing for Punjab with his team mate Watson. Amla not making presents in
Motorola and Gionee ads.
 Lynn and Gayle not comes by same Cars but plays in same team. And one of them,
makes presence in Vivo ad and also likes Puma bat.
 Maxwell and Narine did not play for the same team but they comes by same cars.
The two players who comes by Ford like SS Ton bat and both of them not play for
Kolkata.
 Lynn does not plays for Punjab And Maxwell do not like Adidas bat and SS Ton bat.
Amla and Gayle do not come by Ford.

6). Which of the following player presenting in the Gionee advertisement?


a) Watson
b) Narine
c) Amla
d) Maxwell
e) Lynn

7). Which of the following players come by the Toyota car?


a) Amla and Gayle
b) Gayle and Lynn
c) Watson and Maxwell
d) Narine and Maxwell
e) Gayle and Lynn

8). Which of the following players like the SS Ton bats?


a) Watson and Lynn
b) Amla and Lynn
c) Narine and Amla
d) Gayle and Maxwell
e) None of these

9). The person who presenting in Samsung advertisement like which of the following
bats?
a) Puma
b) Adidas
c) Reebok
d) SS Ton
e) GM Icon

10). Which of the following combinations is true?


a) Narine - Kolkata – Micromax – Adidas - Toyota
b) Lynn - Bangalore – Gionee - SS Ton - Hyundai
c) Watson - Punjab - Motorola - SS Ton - Ford
d) Maxwell – Punjab – Samsung - GM Icon - Toyota
e) None of these

Answer Key:
Direction (01 to 05):
Name Bikes Cars
Rashid 3 6
Bhasker 8 10
Yaswanth 2 2
Gulam 7 4
Ishanth 4 5
Thilak 9 1
Charles 1 7
Kavin 5 9
Jeyaraj 6 8
Eeswar 10 3
1. c) 2. a) 3. b) 4. d) 5. e)
Direction (06 to 10):

6. e) 7. d) 8. a) 9. b) 10. c)

PUZZLE-89
Directions (1-5): Study the following information carefully and answer the given
questions.
Ten persons from different banks viz. Allahabad Bank, Bank of India, Canara
Bank, Dena Bank, Syndicate Bank, UCO Bank, Indian Bank, Andra Bank, Vijaya
Bank and Bank of Baroda are sitting in two parallel rows containing five people
each, in such a way that there is an equal distance between adjacent persons. In
row-1 Arun, Barvesh, Jagadish, Naren and Prathap are seated and all of them are
facing south. In row-2 Gowshiik, Vikaas, Chandru, Raagavan and Krishnan are
seated and all of them are facing north. Therefore in the given seating arrangement,
each member seated in a row faces another member of the other row. (All the
information given above does not necessarily represent the order of seating in the
final arrangement.)
Gowshiik faces one of the immediate neighbours of the person from Bank of India.
Gowshiik does not face Arun. The person from Allahabad Bank sits second to the
right of the person from Andra Bank. Naren faces one of the immediate neighbours
of the person from Canara Bank. Raagavan is not from Canara Bank. Naren is not
from Syndicate Bank. Chandru sits second to the left of the persons from Bank of
Baroda. Arun sits third to the right of person from UCO Bank. Only one person sits
between the person from Vijaya Bank and Vikaas. The person from Dena Bank sits
to the immediate right of Vikaas. Jagadish sits to the immediate left of the person
who faces Vikaas. Only two people sit between Barvesh and Prathap. The person
from Syndicate Bank sits second to the right of the one who faces Raagavan.
Raagavan does not sit at an extreme end of the line. One of the immediate
neighbours of the person from Syndicate Bank faces Vijaya Bank.
1). Which of the following Bank likes by Chandru?
a) UCO Bank
b) Andra Bank
c) Dena Bank
d) Allahabad Bank
e) Indian Bank

2). Which of the following persons sits at the extreme ends of one row?
a) Arun and Barvesh
b) Chandru and Gowshiik
c) Naren and Jagadish
d) Vikaas and Raagavan
e) Prathap and Arun

3). Who sits second to the left of the person the one who likes Canara Bank?
a) Krishnan
b) The one who likes Syndicate Bank
c) The one who likes Dena Bank
d) Chandru
e) None of these

4). Who likes the Vijaya Bank?


a) Raagavan
b) Arun
c) Krishnan
d) Jagadish
e) Naren

5). The one who likes Bank of Baroda sits opposite to the _____________
a) The one who likes Andra Bank
b) The one who likes Allahabad Bank
c) Naren
d) Both a and c
e) None of these

PUZZLE-90
Directions (6-10): Study the following information carefully and answer the given
questions.
Nine people Daniel, Akash, Giri, Kiran, Sadiq, Yogendra, Firoz, Hemant and
Balaji stay in a building. The building has nine floors and only one person stays on
one floor. Each of them likes different company shoes namely- Reebok, Bata, Nike,
Adidas, Liberty, Woodland, Puma, Paragon and Action. Each person belongs to
different states, i.e. Bihar, Haryana, Karnataka, Kerala, Maharashtra, Rajasthan,
Punjab, Gujarat and Sikkim but not necessarily in the same order. The ground floor
is numbered 1, the floor above it is numbered 2, and so on, and the topmost floor is
numbered 9.
The one who belongs to Gujarat stays on the 4th floor. There are 4 floors between
the floors on which Kiran & Firoz stay with Kiran living above Firoz. Giri stays on the
floor immediately above Hemant's floor. The one who likes Reebok stays on an even
numbered floor and is a neighbour of the one who likes Bata. Balaji does not belong
to Haryana. The one who belongs to Bihar stays on the topmost floor. The one who
likes Puma stays immediately below the one who likes Liberty. Sadiq likes Paragon.
Yogendra belongs to Maharashtra and stays on 8th floor. There are 3 persons
between the one who likes Adidas and the one who like Paragon. There are two
floors between the floors on which people from Sikkim and Gujarat stay. Sadiq stays
immediately below the floor on which the person from Kerala lives. The person who
likes Woodland belongs to Haryana and stays on an odd numbered floor. Balaji
stays on the bottommost floor. The person who likes Nike is from Kerala and stays
on 6th floor. The person from Punjab stays on 2nd floor. The persons from
Rajasthan and Haryana are not the neighbours of the person from Kerala. Kiran
likes Bata and Firoz does not like Reebok. Akash does not live below the floor of
Daniel.

6). Who lives in the Third floor?


a) Sadiq
b) The one who likes Action shoes
c) The one who is from Haryana
d) Yogendra
e) None of these

7). Nike Shoes like by which of the following person?


a) Kiran
b) Daniel
c) Firoz
d) Balaji
e) Giri

8). Which of the following persons sits exactly between Balaji and the one who likes
Action shoes?
a) Hemant and Firoz
b) Giri and Sadiq
c) Firoz and Kiran
d) Daniel and Hemant
e) Akash and Sadiq
9). How many persons sit between Firoz and the one who likes Adidas?
a) Five
b) Three
c) Four
d) Two
e) Six

10). Which of the following combinations is true?


a) 4 - Gujarat - Giri - Adidas
b) 3 –Haryana – Firoz - Liberty
c) 8 - Bihar – Akash - Adidas
d) 7 - Sikkim - Kiran - Bata
e) None of these

Answer Key:

1. e) 2. b) 3. a) 4. c) 5. d)
Floor States Names Shoes
9 Bihar Akash Adidas
8 Maharashtra Yogendra Reebok
7 Sikkim Kiran Bata
6 Kerala Daniel Nike
5 Karnataka Sadiq Paragon
4 Gujarat Giri Action
3 Haryana Hemant Woodland
2 Punjab Firoz Liberty
1 Rajasthan Balaji Puma
6. c) 7. b) 8. a) 9. e) 10. d)

PUZZLE-91
Directions (1-5): Study the following information carefully and answer the given
questions.
8 people Loknath, Rohan, Gupta, Prabhas, Habib, Nitin, Tanvir and Wahid sit
around a circular table facing the center. They belong to Austria, Kuwait, China,
Japan, USA, UK, Malaysia and Singapore. They belong to different ages youngest
being 17 and oldest being 64 years old.
One from Singapore is as much older than Gupta, who is from Japan, as the age of
Loknath. Difference of ages of Prabhas and Rohan is 41. One who belongs to UK
sits 2nd right to one from Austria. One from Japan sits 2nd left to one from
Singapore. Age of Habib is 36 years. Loknath is 3rd youngest among all the people.
Two people sit between Nitin and the one whose age is 35. One who belongs to
Malaysia and one who belongs to Singapore sit adjacent to each other and Prabhas
is not from Malaysia. Wahid is the youngest of them and Prabhas is not the oldest.
One who belongs to China is opposite to one whose age is 47. One person sits
between Rohan, who is from USA, and Habib. Nitin sits 3rd left of the from UK who
is the oldest person them. Loknath is sitting 3rd left from the one who is from China.
Sum of ages of Rohan and Wahid is equal to the age of Gupta. One whose age is
35 sits 3rd right of the one whose age is 18. The age of Nitin is average of ages of
Gupta and Prabhas. One from USA and UK are sitting adjacent to each other. One
person sits between Nitin and one who is from Malaysia.

1). Who sits third to the left of the person the one who is from Japan?
a) Loknath
b) Rohan
c) Tanvir
d) Habib
e) Nitin

2). What is the age of Nitin?


a) 18
b) 36
c) 47
d) 24
e) 31

3). Which of the following persons belongs to China?


a) Habib
b) Tanvir
c) Rohan
d) Gupta
e) Nitin

4). Wahid belongs to which of the following country?


a) Kuwait
b) UK
c) Japan
d) Austria
e) Malaysia

5). Which of the following combinations is true?


a) Prabhas – Malaysia - 59
b) Gupta – Japan - 36
c) Loknath – Kuwait - 24
d) Tanvir – UK - 64
e) None of these

PUZZLE-92
Directions (6-10): Study the following information carefully and answer the given
questions.
Six friends Aditi, Geeta, Mayuri, Bhuvana, Vasanti and Leela stay on a 6 floor
building with the ground floor numbered as floor number 1 and so on. All the friends
have dresses on different colours such as; Red, Blue, Green, Yellow, Orange and
Pink not necessarily in the same order. They also like different days via; Sunday,
Monday, Tuesday, Wednesday, Thursday and Friday not necessarily in the same
order. The arrangement is based on the following rules:
 The person on the top floor having dress on colours either Pink or Green
 The one who likes Monday lives 2 floors below the one who has dress as Green
 Vasanti likes Friday and lives on an even numbered floor
 One who have dress on Green colour likes either Wednesday or Tuesday
 Mayuri lives 4 floors below Geeta
 One who lives on 2nd floor has the dress of Yellow colour and likes neither Friday
nor Monday
 Bhuvana’s dress colour is Orange and she lives 2 floors above Mayuri
 Mayuri does not like Yellow
 One who has the dress of Red colour likes Thursday
 One who likes Wednesday lives on an odd numbered floor
 Aditi likes Tuesday and stays just above the person who likes Wednesday
 One who has Orange colour dress does not like Wednesday.

6) Which of the following person like Monday?


a) Geeta
b) Leela
c) Mayuri
d) Aditi
e) Bhuvana

7) The one who likes Tuesday have which of the following colour dress?
a) Pink
b) Green
c) Orange
d) Blue
e) Red

8) Which of the following colour dress and day like by Mayuri?


a) Blue - Tuesday
b) Red - Thursday
c) Green - Friday
d) Pink - Monday
e) Blue - Wednesday

9) Who lives in the second floor?


a) The one who likes Sunday
b) Leela
c) The one who have Yellow colour dress
d) All of the above
e) None of these

10) Which of the following combinations is true?


a) 3 – Bhuvana – Orange - Friday
b) 1 - Mayuri - Yellow- Thursday
c) 5 – Geeta – Green - Wednesday
d) 6 – Aditi –Orange- Tuesday
e) None of these

Explanation With Answer Key:

Directions (Q. 1-5):

1. Answer: b)
2. Answer: c)
3. Answer: a)
4. Answer: e)
5. Answer: d)
Directions (Q. 6-10):

6. Answer: e)
7. Answer: a)
8. Answer: b)
9. Answer: d)
10. Answer: c)

PUZZLE-93
Directions (1-6) : Read the following information and answer the given
question.Seven people, namely O, P, Q, R, S, T and U have to attend a practical but
not necessarily in the same order, on seven different months (of the same year)
namely February, March, April, June, August, September and November. Each of
them also likes a different subject namely Chemistry, Biology, Psychology, Hindi,
English, Geography and Account but not necessarily in the same order.
The one who likes Hindi will attend a practical in a month which has 31 days. Only
two people will attend a practical between the one who likes Hindi and T. Only three
people will attend a practical between T and Q. Only one person will attend a
practical between Q and the one who likes Accounts. The one who likes Chemistry
will attend a practical in one of the months before the one who likes Accounts. The
one who likes Chemistry will attend a practical in the month which has less than 30
days. Only two people will attend a practical between the one who likes Chemistry
and S.
Only one person will attend a practical between S and the one who likes
Psychology. The one who likes Psychology will attend a practical on one of the
months after S. Only two people will attend a practical between the one who likes
Psychology and P. The one who likes Geography will attend a practical immediately
before P. Only three people will attend a practical between P and the one who likes
English. Only one people will attend a practical between O and R. O will attend a
practical in a month which has 31 days.
1). Who amongst the following likes Biology ?
a) P
b) T
c) U
d) R
e) O

2). How many people will attend a practical after P ?


a) Three
b) One
c) Two
d) None
e) More than three

3). Which of the following represents the people who will attend a practical
immediately before and immediately after O ?
a) T, P
b) P, Q
c) S, Q
d) S, P
e) U, P
4). As per the given arrangement February is related to T and March is related to P
following a certain pattern, which of the following is August related to following the
same pattern ?
a) U
b) Q
c) R
d) S
e) O

5). Which of the following represents the month in which U will attend a practical?
a) November
b) February
c) April
d) June
e) Cannot be determined
6). Who amongst the following likes English ?
a) S
b) T
c) U
d) R
e) Other than those given as options

PUZZLE-94
Directopms (7-12) : Study the following information and answer the given questions.
Seven people namely D, E, F, G, H, I and J like seven different monuments namely,
Taj Mahal, Red Fort, Charminar, India Gate, Qutub Minar, Victoria Memorial and
Sanchi Stupa. Each of them work in either of the three fields viz. Economics,
Management and Pharmacy with atleast two of them in a field. (Note.: None of the
information given is necessarily in the same order.) The one who likes Qutub Minar
works in field of Management only with G. The one who likes Charminar works with
the one who likes Sanchi Stupa. J works with the one who likes India Gate. I neither
works with G nor in the field of Pharmacy. J does not like Charminar. D likes Red
Fort. D does not work with J. F works with only one person. F does not like Qutub
Minar. H works with I. I does not like Charminar. Neither G nor F likes Taj Mahal.

7).Which of the following combinations represents the combination of people who


work in the field of Economics ?
a) The ones who like Victoria Memorial, Taj Mahal and India Gate.
b) The ones who like Charminar and Sanchi Stupa.
c) The ones who like Char Minar, Sanchi Stupa and Victoria Memorial.
d) The ones who like Taj Mahal and India Gate.
e) The Ones who like Sanchi Stupa, Red Fort and Char Minar.

8). Which of the following statements is true ?


a) H likes Sanchi Stupa
b) E works only with J.
c) All of the given statements are true.
d) Both E and G work in the, same field.
e) Only two people work in ti field of economics.

9). Which of the following combinations represents th`e field in which H works and
the monument he likes ?
a) Economics-Taj Mahal
b) Pharmacy-India Gate
c) Management- India Gate
d) Economics-Charminar
e) Pahrmacy-Charminar
10). Four of the following five are alike in a certain way as per the given
arrangement and hence form a , group. Which of the following does not belong to
that group ?
a) HI
b) JF
c) GE
d) EJ
e) ID

11). Which of the following monuments does E like ?


a) Taj Mahal
b) Qutub Minar
c) Sanchi Stupa
d) Char Minar
e) India Gate

12). Who amongst the following likes Victoria Memorial ?


a) H b) J c) I d) G e) F
Answer Key:

Month People Subject


February U Chemistry
March T Geography
April P Biography
June S Accountant
August O Hindi
September Q Psychology
November R English

1). A 2). E) 3). C 4). B 5). B 6).D


Management E Qutub Minar
G Victoria Memorial
Economics I Sanchi Stupa
H Charminar
D Red Fort
Pharmacy F India Gate
J Taj Mahal

7). E 8). D 9). D 10). D 11).B 12). D

PUZZLE-95
Directions (1-5): Study the following information carefully and answer the given
questions.
The seven mobile manufacturing companies Samsung, Nokia, Lenovo, HTC,
Oppo, Motorola, Vivo are having their plant in places namely Maharashtra, Andra
Pradesh, Karnataka, Haryana, Tamil Nadu, West Bengal and Bihar not in the
respective order. And the companies making contract with Sim dealers such as
AIRTEL, VODAFONE, IDEA, BSNL, AIRCEL, DOCOMO and JIO. The Sim dealers
are located in different places such as Chennai, Hyderabad, Bangalore, Delhi,
Mumbai, Kolkata and Patna not in the respective order.
 JIO and BSNL are not contact with Andra Pradesh and West Bengal based
companies respectively. Either Samsung or Vivo likes to contract with IDEA which is
not located in Kolkata and Hyderabad
 The dealer which located in Mumbai contract with the company that is located in
Haryana.
 Motorola contract with Hyderabad based dealer and is not located in Tamil Nadu
and Andra Pradesh.
 DOCOMO is Bangalore based dealer. And Kolkata based dealer not deals with Vivo
and Samsung.
 Maharashtra and Karnataka based companies making contract with Delhi and Patna
based Sim dealers respectively.
 Samsung is not serving to the Delhi and Mumbai based dealer. Lenovo is having
plant in Bihar.
 HTC do not dealing with dealers of Chennai, Mumbai and also DOCOMO dealer
 The Mumbai based VODAFONE does not contract with Oppo and Nokia. Vivo which
is located in Haryana doesn’t deal with Tamil Nadu based AIRTEL dealer.
DOCOMO will contract with Bihar based company.
 Chennai based dealer making deal with Oppo which is not located in Andra Pradesh
and West Bengal
 AIRCEL is not from Patna and Kolkata and makes deal with Nokia Company

1). HTC Contract with which of the following dealer?


a) AIRCEL
b) BSNL
c) IDEA
d) JIO
e) AIRTEL

2). VODAFONE contract with which of the following companies?


a) Lenovo
b) Motorola
c) Oppo
d) Vivo
e) Nokia

3). Which of the following company plant located in Tamil Nadu?


a) Vivo
b) Lenovo
c) Motorola
d) Nokia
e) Oppo

4). Which of the following combinations is true?


a) Nokia – Tamil Nadu – AIRCEL - Chennai
b) Samsung – Haryana – VODAFONE - Mumbai
c) HTC - Andra Pradesh – BSNL - Kolkata
d) Oppo – Maharashtra – AIRTEL - Delhi
e) None of these

5). If ‘Samsung’ is related to ‘Kolkata’, ‘HTC’ is related to ‘Hyderabad’, then which


following is ‘Motorola’ related to?
a) Mumbai
b) Chennai
c) Delhi
d) Bangalore
e) Patna

PUZZLE-96
Directions (6-10): Study the following information carefully and answer the given
questions.
Eight persons from different countries such as Austria, Belgium, Canada,
Denmark, Finland, Germany, Iceland and Mauritius eating 8 different fruits such as
Apple, Orange, Banana, Strawberry, Pears, Grapes ,Pineapple and Papaya . And
each of them having a watch in the brand names of Fastrack, Citizen, Titan and
Timex
The person who is from Belgium eating Papaya as a fruit. The person who is eating
Pineapple having Timex watch. The third floor stayed person eating Orange and
having a watch in the brand name of Fastrack. Banana is eaten by the person who
is having Citizen watch. Minimum of three persons stayed below of Mauritius country
man. And the person who belongs to Denmark not stayed in the odd numbered
floor. The watch’s brand name Citizen and Timex having same number of country-
men. Canada country man stayed two floor below from the person who is belongs to
Belgium. The top floor is not occupied by the person from Iceland. The Titan owned
person eating Apple. The persons from Denmark and Belgium having same watch
brand and both of them not having Citizen watch and also three persons stayed in
between them. The persons who are eating Strawberry fruit and Orange are having
same watch brand with the one who is eating Pears. The Titan owned person stayed
in fourth floor. There are four persons stayed in between of Iceland and Denmark.
The lowest floor is occupied by the Finland country man. Only three persons having
one kind of watch brand name and the person from Iceland who is eating Pears is
one of them. The country man from Germany and the person who eating Pears are
not stayed in the even numbered floor. Only one person having Titan watch. The
Austrian who likes to eat Grapes stayed in even numbered floor and is having same
watch brand name with the person , who is eating Banana also stayed in first floor.
Timex watch is owned by the person who is from Denmark. The person who eating
Papaya stayed above the floor of the person who is eating Pineapple. The Germany
who is eating Orange having same watch brand with the country man from Mauritius

6). Pears fruit eat by which of the following country person?


a) Canada
b) Belgium
c) Denmark
d) Austria
e) Iceland

7). Which of the following person sits exactly between who are eating apple and
papaya?
a) The one who eats Orange fruit
b) The one who eats Pineapple fruit
c) The one who eats Grapes fruit
d) The one who eats Strawberry fruit
e) The one who eats Banana fruit

8). Who lives in the sixth floor?


a) The one who is from Belgium
b) The one who is from Denmark
c) The one who is from Iceland
d) The one who is from Austria
e) The one who is from Mauritius

9). How many persons have Fastrack watches?


a) One
b) Two
c) Three
d) Four
e) Five

10). Which of the following combinations is true?


a) 2 – Banana – Timex - Finland
b) 4 - Apple - Titan - Canada
c) 8 – Apple – Citizen - Finland
d) 7 - Orange – Fastrack - Austria
e) None of these
Answer Key:
Direction (01 to 05):

1. b) 2. d) 3. e) 4. c) 5. a)

Direction (06 to 10):

6. e) 7. d) 8. a) 9. c) 10. b)
PUZZLE-97
Directions (1-5): Study the following information carefully and answer the given
questions.
Twelve friends are sitting in two parallel rows of chairs containing six people each, in
such a way that there is equal distance between adjacent persons. In row 1: Dravid,
Laxman, Sachin, Ganguly, Sehwag and Kumble are seated and all of them are
facing south. In row 2: Rohit, Yuvraj, Raina, Dhoni, Kohli and Dhawan are seated
and all of them are facing north. Each of them likes different month such as,
January, February, March, April, May, June, July, August, September, October,
November and December but not necessarily in the same order. Each of them like
different countries: USA, China, Australia, New Zealand, South Africa, West Indies,
England, Bangladesh, India, Sri Lanka, Pakistan and Zimbabwe. In the given
seating arrangement, each member seated in a row faces another member of the
other row.
Sehwag, whose favourite country is West Indies sits third to the left of the person
who likes February and not sitting at the extreme end. Laxman, who likes April, does
not sit at an extreme end of the row. If Sehwag sits at an extreme end then Laxman
doesn’t sit to the immediate right of the one who likes February. Raina and Rohit
face the persons whose favourite countries are West Indies and South Africa
respectively. There are two persons sitting on the right side of Yuvraj, who sits
second to the right of the person, whose favourite country is Sri Lanka. Yuvraj, who
likes May, faces the immediate neighbour of Dravid. Dravid likes March. Rohit sits
with the persons who like May and December. Kumble faces the person who likes
November month and Zimbabwe country but Kumble does not like February. The
one who likes December month doesn’t sit with the one who likes November month.
The person, who likes October month, sits second to the right of the person who
likes June. Dhoni does not like November and faces the person, who likes January.
The person who likes India is an immediate neighbour of the person who likes
August. Sachin’s favourite country is New Zealand and he doesn’t sit with Dravid.
Raina does not like China country. The person, whose favourite country is USA,
faces the person, who sits second to the right of the person, whose favourite country
is South Africa. The person whose favourite country is Bangladesh does not sit in
row 2 and faces the person who likes Pakistan. Kohli faces the person who likes
England. Sachin doesn’t face the person who likes June month. The one who likes
July sits in Row 2.
1) The one who likes England sits third to the right of ______________
a) Dravid
b) Laxman
c) Sehwag
d) Kumble
e) Sachin

2) Which of the following month likes by Rohit?


a) November
b) February
c) March
d) June
e) August

3) Which of the following combinations is true?


a) Kumble – January - Australia
b) Sehwag – June – West Indies
c) Raina – October – South Africa
d) Laxman – April - Bangladesh
e) None of these

4) Who likes the Sri Lanka country?


a) Kohli
b) Rohit
c) Dhawan
d) Raina
e) Dhoni

5) Which of the following persons sits at the extreme ends of one row?
a) Dhoni and Rohit
b) Kohli and Raina
c) Ganguly and Kumble
d) Dravid and Sehwag
e) Dhoni and Dhawan

PUZZLE-98
Directions (6-10): Study the following information carefully and answer the given
questions.
In Infosys company the staffs Vijay Kumar, Mithran, Sathya, Akilan, Sahana,
Gayathri, Hafiza, Varsha are working and they come to the company by different
way of rivers such as Ganges River, Yamuna River, Brahmaputra River, Mahanadi
River, Godavari River, Narmada River, Krishna River, Kaveri River not in the
respective order. And they are selected for special seminar. They planned to take
seminar individually with other staffs in different Cities such as Mumbai, Bangalore,
Kolkata, Chennai, Hyderabad, Delhi, Ahmadabad, Kochi in a week starting from
Sunday to Saturday.
 Vijay Kumar going to take seminar before Mithran and after Varsha who did not come
by the way of Brahmaputra River
 The staff who is going to take seminar in Kolkata, come to company by the way of
Narmada River.
 Sathya come to company by the way of Yamuna River and he did not take seminar
on week’s starting day.
 The staff who take seminar in Kochi, attend seminar in Delhi and Hyderabad in the
previous day.
 Varsha likes to take seminar in Mumbai and she did not come by Krishna River.
 Only two persons giving their seminar on same day due to other commitments
 The person who is coming to company by the way of Godavari River likes to take
seminar on Thursday.
 Mithran likes to take seminar in Ahmadabad and the day in between the persons who
take seminar in Chennai and Kochi.
 Hafiza come to company by the way of Krishna River and she did not take seminar
on Saturday.
 The persons who come by the way of Mahanadi River and Yamuna River likes to
take seminar on same day.
 The person who takes seminar in Ahmadabad come by the way of Ganges River and
attend Sahana’s seminar who come to company by Godavari River in the next day.
 Vijay Kumar who did not come by the way of Kaveri River taking seminar on the next
day of Akilan’s seminar. Akilan who making arrangement with another staff for taking
seminar on same day and he did not take seminar in Delhi.
6) Who takes the seminar on Friday?
a) The one who takes seminar in Bangalore
b) Hafiza
c) Both a and b
d) The one who takes seminar in Ahmadabad
e) The one who come to the company by the way of Ganges River

7) Who come to the company by the way of Narmada River?


a) Varsha
b) Akilan
c) Hafiza
d) Gayathri
e) Sahana

8) Two seminars held on which of the following day?


a) Thursday
b) Tuesday
c) Saturday
d) Friday
e) Monday

9) Akilan comes to the company by the way of which river?


a) Mahanadi River
b) Ganges River
c) Godavari River
d) Narmada River
e) Yamuna River

10) Which of the following combinations is true?


a) Varsha - Krishna River – Mumbai - Thursday
b) Mithran - Ganges River – Ahmadabad - Wednesday
c) Vijay Kumar - Brahmaputra River – Chennai - Wednesday
d) Gayathri - Kaveri River – Kolkata - Saturday
e) None of these

Answer Key:
Direction (01-05):
1) c 2) d 3) d 4) a 5) e

Direction (06-10):

6) c 7) d 8) e 9) a 10) b

PUZZLE-99
Directions (1-5): Study the following information carefully and answer the given
questions.
There are nine people Udit, Emaan, Jagan, Mandeep, Sankar, Murali, Akbar,
Nirmal and Rasul who sits around a circular table. Some are facing towards and
some are facing away from the centre. They like different airports among Kurnool
Airport, Rajkot Airport, Shimla Airport, Dhanbad Airport and Gwalior airport. Not
more than 2 people like the same airports. The following information is known about
them. Jagan and Akbar form the only pair that is sitting together and like the same
airports. They are facing in opposite directions to each other. The one who likes
Shimla Airport sits third to the left of Emaan but is neither Jagan nor Murali. Udit sits
second to the left of Murali, who likes Rajkot Airport. The one who likes Kurnool
Airport is an immediate neighbour of Nirmal and Sankar but none of them sits
adjacent to Jagan or Akbar. Udit, Rasul and Nirmal face in same direction as that of
Sankar. Rasul sits third to the right of Mandeep, who is facing away from centre and
likes Dhanbad Airport. Sankar sits second to the left of Jagan and likes Rajkot
Airport. The people who like Rajkot Airport face towards the centre. Udit neither likes
Shimla Airport or Dhanbad Airport.

1. Which of the following person likes Kurnool Airport?


a) Akbar and Udit
b) Nirmal and Emaan
c) Mandeep and Rasul
d) Sankar and Jagan
e) Udit and Rasul

2. How many persons facing outside the circle?


a) Two
b) Five
c) Three
d) Six
e) Four

3. Which of the following airport like by Nirmal?


a) Shimla Airport
b) Kurnool Airport
c) Rajkot Airport
d) Gwalior Airport
e) None of these

4. Who sits second to the right of Emaan?


a) Nirmal
b) Akbar
c) Jagan
d) Rasul
e) Sankar

5. Which of the following combinations is true?


a) Nirmal– Facing outside –Shimla Airport
b) Sankar– Facing centre– Kurnool Airport
c) Udit– Facing centre–Rajkot Airport
d) Jagan – Facing outside – Gwalior Airport
e) None of these

PUZZLE-100
Directions (6-10): Study the following information carefully and answer the given
questions.
Twelve countries have sent one representative Raheem, Bibek, Jackson, Dylan,
Milton, Abdul, Mohamed, Patel, Irfan, Sundar, Salman and Abbas each for an OPEC
Summit. They meet in a rectangular summit hall where four representatives occupy
the four corners and two representatives each occupy places on each of the four
sides. All of them face the center of the hall.
The following information is also given:
 Abdul, from Nigeria, sits second to the left of Mohamed, from Kuwait.
 Abbas, the representative from Saudi Arabia is diagonally opposite to Abdul, who
sits second to the right of Dylan.
 Dylan, the representative from Iran occupies the third place to the left of Bibek, who
is the representative from Angola.
 Raheem, from Algeria, faces Mohamed while Irfan sits between Patel and Dylan.
 The representatives from Ecuador and Qatar are diagonally opposite to each other
and Jackson from Ecuador is fourth to the left of Raheem.
 Sundar, from Venezuela, is sixth to the right of Salman, who represents UAE.
 Milton sits third to the right of Bibek. There are representatives from countries in Iraq
and Libya as well.

6. Patel likes which of the following countries?


a) Libya
b) Iraq
c) Either Iraq or Libya
d) Nigeria
e) Iran

7. How many persons it between Irfan and Bibek? (Counted from the left of Irfan)
a) Seven
b) Four
c) Six
d) Five
e) Three

8. Who sits third to the right of Mohamed?


a) Sundar
b) Dylan
c) Salman
d) Irfan
e) Either Sundar or Salman

9. Irfan is a representative of which of the following country?


a) Algeria
b) Qatar
c) UAE
d) Venezuela
e) Kuwait
10. Who sits third to the right of Raheem?
a) Abdul
b) Milton
c) Irfan
d) Dylan
e) None of these

Answer Key:
Direction (01-05):

1. e) 2. c) 3. a) 4. b) 5. d)

Direction (06-10):
6. c) 7. a) 8. e) 9. b) 10. d)

PUZZLE-101

Directions (1-5): Study the following information carefully and answer the given
questions.

Eight friends- Darshana, Hasina, Tanika, Bhavani, Vidya, Iniya, Akhila and
Usha visit different countries viz. Australia, Japan, Germany and Brazil are sitting
around a circular table facing the centre of the table. Each country is being visited by
two people only, but not necessarily in the same order. All these friends like different
Dams i.e. -Mitti dam, Salal Dam, Konar Dam, Bhadra Dam, Mettur Dam, Vaigai
Dam, Tehri Dam and Bargi Dam. No two people visiting the same country are sitting
adjacent to each other except those visiting Japan. The person who likes Bhadra
Dam is sitting on the immediate left of the person who like Bargi Dam. Vidya neither
like Bargi Dam nor Konar Dam. Usha likes Vaigai Dam and visits Germany and is
sitting to the immediate left of Hasina, who visits Japan. Hasina does not like Konar
Dam. Iniya likes Mettur Dam and visits Japan, who is sitting opposite to Akhila. Only
Darshana, who like Salal Dam, is sitting between who like Bhadra Dam and the
person who like Tehri Dam. Persons who visit Australia are sitting opposite to each
other. Each of the persons who visit Brazil is sitting adjacent to a person who visits
Australia. Bhavani does not like Bargi Dam.

1). Who sits second to the left of the person the one who likes Mitti Dam?
a) The one who likes Bhadra Dam
b) The one who likes Bargi Dam
c) Usha
d) Bhavani
e) The one who likes Mettur Dam

2). Who of the following person likes Tehri Dam?


a) Hasina
b) Akhila
c) Darshana
d) Bhavani
e) Vidya

3). Which of the following persons visiting Germany?


a) Darshana and Usha
b) Usha and Hasina
c) Akhila and Iniya
d) Bhavani and Vidya
e) Darshana and Tanika

4). Which of the following dam like by Tanika?


a) Tehri Dam
b) Mettur Dam
c) Bargi Dam
d) Vaigai Dam
e) Salal Dam

5). Which of the following combinations is true?


a) Usha – Germany – Bargi Dam
b) Bhavani – Brazil – Konar Dam
c) Akhila – Australia – Bhadra Dam
d) Hasina – Japan – Mitti Dam
e) None of these

PUZZLE-102
Directions (6-10): Study the following information carefully and answer the given
questions.
Hariram, Nikhil, Vikas, Gaurav, Lalit, Ramkumar, Chandrika, Jamuna, Mala, Pallavi,
Kavita and Dhanya are 12 persons sitting in two rows, among them Hariram, Vikas,
Gaurav, Ramkumar, Mala and Kavita are facing north while the remaining are facing
south. Each person faces exactly one person in the other row. Each one of them
likes a different festival from among Dussehra, Diwali, Mahashivaratri, Holi, Ugadi,
Onam, Lohri, Pongal, Bonalu, Navaratri, Brahmotsavam and Bohag Bihu (not
necessarily in the same order) and belongs to one state each from among
Maharashtra, Kerala, Tamil Nadu, Assam, Manipur, Bihar, Gujarat, Karnataka,
Rajasthan, Sikkim, Nagaland and Haryana (again not necessarily in the same
order).
 The one who likes Ugadi is from Bihar and is facing Ramkumar. None of Nikhil,
Dhanya and Lalit likes Ugadi or Dussehra. The persons from Tamil Nadu and
Haryana are the immediate neighbours of Pallavi.
 Nikhil is second to the right of Chandrika, who is from Gujarat. Nikhil like Lohri
festival and faces the person from Sikkim, who is third to the right of Gaurav. Neither
Hariram nor Gaurav likes Diwali festival or from Kerala.
 Kavita, like the Mahashivaratri festival, sits second to the right of the person who
likes Navaratri festival, who faces the person from Haryana, None of them sits at an
extreme end.
 The Diwali festival like by the person from Nagaland, sits as far as possible from
Gaurav.
 Dhanya, likes the Onam festival, sits opposite the person who likes the
Brahmotsavam festival, who is an immediate neighbour of the person sitting at an
extreme end.
 The Lohri and Holi festival liker’s are from Tamil Nadu and Maharashtra
respectively. The person who likes Onam festival and Holi festival face the same
direction.
 Ramkumar, like the Pongal festival and from Assam, sits equidistant in the same
row from the Bonalu and the festival from Kerala.
 The festival from Rajasthan is opposite the festival from Maharashtra and second to
the left of the festival from Assam. Hariram, who is not like the Brahmotsavam
festival, is third to the left of Mala; and neither of them like Diwali festival.
 Jamuna likes the Bohag Bihu festival and is third to the right of person who likes
Onam festival from Manipur.

6). Which of the following person belongs to Sikkim?


a) Kavita
b) Mala
c) Pallavi
d) Hariram
e) Dhanya

7). Who sits second to the left of Pallavi?


a) Lalit
b) Nikhil
c) Jamuna
d) Chandrika
e) Dhanya

8). Which of the following persons sits at the extreme ends of one row?
a) Lalit and Nikhil
b) Hariram and Vikas
c) Gaurav and Vikas
d) Chandrika and Jamuna
e) Ramkumar and Gaurav

9). Which of the following combinations is true?


a) Ramkumar - Pongal – Tamil Nadu
b) Jamuna - Bohag Bihu - Haryana
c) Mala - Navaratri - Kerala
d) Dhanya – Lohri - Manipur
e) None of these

10). The one who belongs to Manipur sits third to the left of ______________
a) The one who belongs to Tamil Nadu
b) The one who belongs to Bihar
c) The one who likes Dussehra festival
d) The one who likes Lohri festival
e) The one who likes Holi festival

Answer Key:
1. b) 2. e) 3. a) 4. c) 5. d)

6. a) 7. e) 8. c) 9. b) 10. d)

PUZZLE-103
Directions (Q. 1-5): Study the following information carefully and answer the questions
given below:
There are five classes which have been scheduled for students in the days of the week
starting from Monday and ending on Friday. The subjects are Maths, Reasonin English,
Computer and GA. There are five teacherswhgo' belong to a different cities, viz Delhi,
Kolkata, Allahabad Bhopal and Patna. Among the five teachers only two are males.
The teacher who teaches Computer belongs to Delhi The one who teaches GA belongs
neither to Kolkata nor to Bhopal. A female belongs to Bhopal.
The one who belongs to Patna is a female and she teaches on Tuesday. The
Reasoning classes are scheduled on Wednesday. The males can take classes
alternately but not on Monday. Neither Maths nor Computer can be scheduled on
Monday. English classes are scheduled immediately after Reasoning. The one who
teaches Reasoning does not belong to Bhopal.
1). The teacher of which subject belongs to Kolkata?
a) Reasoning
b) Maths
c) Computer
d) Can't be determined
e) None of these

2). Male members take the classes on which' of the following days?
a) Wednesday & Monday
b) Thursday & Friday
c) Thursday
d) Wednesday & Friday
e) None of these

3). WhO among the following takes the class on Monday?


a) The one who belongs to Patna
b) The one who belongs to Allahabad
c) The one who teaches Reasoning
d) The one who teaches English
e) None of these

4). Who among the following takes class of Computer?


a) The one who takes class on Friday
b) The one who belongs to Bhopal
c) The one who takes class on Wednesday
d) The one who belongs to Patna
e) None of these

5). Which of the following combinations is true?


a) Female — Wednesday — Maths — Kolkata
b) Female — Thursday — English — Delhi
c) Male Friday Computer — Delhi
d) Male — Thursday — English — Bhopal
e) None of these

PUZZLE-104
Directions (6-11): Study the following information carefully and answer the questions
given below:
Eight friends P, Q, R, S, T, U, V and W are sitting 'around a circular table, but not facing
the centre. Each of them has a different rank in final-semester exam, viz 1 st to 8th, but
not necessarily in the same order.
The one who got 1st rank sits on the immediate left of the one who got 8th rank. V and
T are not immediate neighbours. V, who does not have 5th rank, sits on the immediate
right of W. P is second to the right of the one whose rank is 7th. S's rank is neither 6th
nor 7th. There are two persons sitting between R and the one whose rank is 6th. There
is only one person between those whose ranks are 6th and 8th. T and Q are immediate
neighbours. The one whose rank is 5th sits second to the right of U. W's rank is neither
1st nor 2nd but he is an immediate neighbour of the one whose rank is 5th. The one
whose rank is 1st cannot sit with the person whose rank is 4th. P and R, whose rank is
3rd, can never sit together. There are two persons between the persons whose ranks
are 7th and 5th.
6). What is the position of W with respect to the person whose rank is 6th?
a) Third to the right
b) Immediate left
c) Fourth to the left
d) Third to the left
e) From the right
7). How many persons are there between R and Q?
a) One
b) Two
c) Three
d) None
e) None of these

8). Who among the following sits second to the left of the one who sits opposite the
person whose rank is 2nd?
a) 1)T
b) 2) Q
c) 3) U
d) 4) V
e) 5) S

9). Who among the following sits third to the right of S?


a) The one whose rank is 5th
b) The one whose rank is 6th
c) The one whose rank is 7th
d) The one whose rank is 1st
e) None of these

10). What is the position of P with respect to V?


a) Second to the left
b) Third to the right
c) Fifth to the right
d) Can't be determined
e) Third to the left

11). Which of the following statements is/are true?


a) S's rank is 4th.
b) R is second to the right of the one whose rank is 6th.
c) The one whose rank is 8th sits opposite V.
d) None is true
e) The one whose rank is 7th sits second to the right of P.

Solution(1-5):

SUBJECT DAY CITY GENDER


Maths Tuesday Patna Female
Reasoning Wednesday Kolkata Male
English Thursday Bhopal Female
Computer Friday Delhi Male
GA Monday Allahabad Female

1) a 2) d 3) b 4) a 5) c

Solution ( 6-11):

6) d 7) c 8) a 9) c 10) b 11) c
PUZZLE-105
Directions (1-5): Study the following information carefully and answer the given
questions.
There are seven boys, Sukumar, Gokul, Nagesh, Basha, Vinayak, Manoj and
Rajesh who participated in a game competition which started on Monday and ended
on Sunday. In the first round of the competition, each of them played different
games, Cricket, Football, Hockey, Tennis, Boxing, Badminton and Volleyball, but not
necessarily in the same order. They like different colours, Red, Blue, Green, Yellow,
Orange, White and Pink, but not necessarily in the same order
Gokul did not play on the day either immediately before or immediately after the
game of Basha, who does not like either Orange or Red or Yellow colour. Two
games were held between the game of Rajesh and Manoj, neither of whom played
on Monday. There was one game between the game of Basha and Nagesh. But
Nagesh’s game did not happen either on Monday or on Wednesday. Nagesh likes
Blue colour and played Cricket. The one, who played Hockey on the last day of
competition, likes Green colour. Vinayak played immediately after Nagesh and he
likes Red colour. Sukumar does not like Orange colour and played Tennis. Basha
did not play either Volleyball or Badminton. The one who played Boxing was
scheduled immediately after the game of Cricket. Rajesh, who likes White, played
on the fourth day of the competition but played neither Cricket nor Football.

1). Manoj likes which of the following colour and plays which of the following game?
a) Blue - Cricket
b) Green - Hockey
c) Yellow - Hockey
d) Pink - Tennis
e) White - Boxing

2). Who among the following plays Football game?


a) Gokul
b) Nagesh
c) Sukumar
d) Basha
e) Vinayak

3). Which of the following games held on Tuesday?


a) Cricket
b) Hockey
c) Boxing
d) Basha
e) Tennis

4). The person who plays Hockey like which of the following colour?
a) Green
b) Yellow
c) Blue
d) White
e) Orange

5). Which of the following combinations is true?


a) Tuesday – Sukumar – Orange - Tennis
b) Wednesday - Basha – Pink - Cricket
c) Saturday – Vinayak – Red - Boxing
d) Friday – Nagesh – Blue - Football
e) None of these
PUZZLE-106
Directions (6-10): Study the following information carefully and answer the given
questions.
M, N, O, P, Q, R and S lives in a seven storey building. The lower-most floor
is numbered 1; floor above it is numbered 2 and so on. There are four males and
three females among them.
 Miss S lives in floor no. 2.
 No female lives above the floor in which Miss N lives.
 P lives in an odd numbered floor and he doesn't live either on the top-most or lower-
most floor.
 Q has to use most no. of stairs to reach to his flat.
 More than anyone else in the building, None of the female lives at the lower-most or
top-most floor.
 R lives just a floor below his sister O's floor.
 No two male or female live immediately above or below each other.

6. Who among the following lives on floor number 5?


a) R
b) Q
c) N
d) S
e) P

7. Which of the following floor number O Lives?


a) Two
b) Six
c) Five
d) Four
e) Three
8. If ‘Q’ is related to ‘Floor number 4’, ‘O’ is related to ‘Floor number 1’, then which
following is ‘P’ related to?
a) Floor number 7
b) Floor number 2
c) Floor number 5
d) Floor number 4
e) Floor number 6

9. Who among the following exactly sitting between S and P?


a) O and R
b) Q and N
c) O and N
d) Q and R
e) M and N

10. Which of the following combinations is true?


a) 7 - Female - O
b) 3 - Male - M
c) 6 - Female - N
d) 5 - Male - Q
e) None of these
Answer Key:
Direction (01-05):
Days Person Game Colour
Monday Gokul Orange Badminton/Volleyball
Tuesday Sukumar Yellow Tennis
Wednesday Basha Pink Football
Thursday Rajesh White Volleyball/Badminton
Friday Nagesh Blue Cricket
Saturday Vinayak Red Boxing
Sunday Manoj Green Hockey
1. b) 2. d) 3. e) 4. a) 5. c)
Direction (06-10):
Floor Gender Person
7 Male Q
6 Female N
5 Male P
4 Female O
3 Male R
2 Female S
1 Male M

6. e) 7. d) 8. b) 9. a) 10. c)

PUZZLE-107
Directions (1-5): Study the following information carefully and answer the given
questions.
Eight friends - Ajith, Vijay, Surya, Vikram, Siva, Jeeva, Madhavan and Karthi
likes to work in different banks viz, Syndicate Bank, Allahabad Bank, Bank of India,
Canara Bank, Dena Bank, Indian Bank, Bank of Baroda, and Vijaya Bank, not
necessarily in the same order. Each of them also owns a mobile from among
Samsung, HTC, Microsoft, Oppo, Vivo, Micromax, Lenovo and Apple, not
necessarily in the same order. It is also known that:
Ajith likes Allahabad Bank and does not own the HTC while Vikram does not work in
Canara Bank and owns the Oppo. Siva neither works in Canara Bank nor in Indian
Bank but owns either the Vivo or Microsoft. The HTC is not owned by Surya or
Jeeva while Karthi owns the Apple. The person who works in Indian Bank owns the
Micromax but is not Surya. The person who works in Canara Bank owns neither the
Lenovo nor the Apple. Indian Bank is not the bank of which Madhavan works, who
owns the Lenovo. Surya does not work in Bank of Baroda and does not own the
Samsung. The person who owns the HTC works neither work in Canara Bank nor in
Bank of India and the person who works in Vijaya Bank owns neither the Lenovo nor
the HTC. Vikram works in Dena Bank while Madhavan works neither in Bank of
India nor in Bank of Baroda.

1). Who among the following owns the HTC mobile?


a) Madhavan
b) Siva
c) Surya
d) Vijay
e) None of these

2). Who works in Vijaya Bank?


a) Karthi
b) Jeeva
c) Siva
d) Either Karthi or Siva
e) None of these

3). The person works in Bank of Baroda owns which of the following phones?
a) HTC
b) Oppo
c) Microsoft
d) Apple
e) Lenovo

4). Which of the following combinations is true?


a) Ajith - Syndicate Bank - Samsung
b) Jeeva - Indian Bank - HTC
c) Vikram - Dena Bank - Oppo
d) Madhavan - Allahabad Bank - Lenovo
e) None of these

5). If ‘Vijay’ is related to ‘Oppo’, ‘Vikram’ is related to ‘Micromax’, then which


following is ‘Jeeva’ related to?
a) Samsung
b) Microsoft
c) HTC
d) Lenovo
e) Apple

PUZZLE-108
Directions (6-10): Study the following information carefully and answer the given
questions.
Six friends Priya, Jaya, Richa, Deepa, Suvetha and Harini participated in the
cultural fest held in their college. Each of them participates in different dances:
Bharatanatyam, Kathak, Kathakali, Kuchipudi, Odissi and Mohiniyattam, not in same
order starting from Monday upto Sunday. There is a holiday in between when no
event is scheduled.
Holiday is not on Monday or Saturday. Deepa performs on Sunday and
Bharatanatyam is scheduled on Wednesday. There is two days gap between
Holiday and Suvetha's performance. Priya performs after Suvetha and Jaya
performs before Holiday. There is two days gap between Kuchipudi and Kathakali
competitions and Kathakali is not performed on Monday. Mohiniyattam is scheduled
on the next day of Richa's performance. Deepa is not a performing Kuchipudi. Odissi
is not done by Suvetha But it is scheduled before Kathak Competition. Kuchipudi is
scheduled before holiday.Suvetha and Harini did not perform on Monday.

6). The one who perform on Saturday perform which of the following dance?
a) Odissi
b) Kathak
c) Mohiniyattam
d) Bharatanatyam
e) Kuchipudi

7). Which of the following person performs on Tuesday?


a) Richa
b) Priya
c) Suvetha
d) Jaya
e) Harini

8). Which of the following days Kuchipudi dance performs?


a) Monday
b) Saturday
c) Tuesday
d) Wednesday
e) Thursday

9). Which of the following dance perform by Richa?


a) Odissi
b) Mohiniyattam
c) Kathakali
d) Kuchipudi
e) Bharatanatyam

10). Which of the following combinations is true?


a) Tuesday - Deepa - Mohiniyattam
b) Monday - Harini - Odissi
c) Sunday - Priya - Kathakali
d) Saturday - Harini - Kathak
e) None of these
Answer Key:

Person Banks Mobiles


Ajith Allahabad Bank Samsung
Vijay Bank of Baroda HTC
Surya Canara Bank Microsoft / Vivo
Vikram Dena Bank Oppo
Siva Bank of India / Vijaya Bank Vivo / Microsoft
Jeeva Indian Bank Micromax
Madhavan Syndicate Bank Lenovo
Karthi Vijaya Bank / Bank of India Apple

1. d) 2. d) 3. a) 4. c) 5. e)

Day Person Event


Monday Richa Odissi
Tuesday Suvetha Mohiniyattam
Wednesday Jaya / Priya Bharatanatyam
Thursday Priya / Jaya Kuchipudi
Friday - -
Saturday Harini Kathak
Sunday Deepa Kathakali

6. b) 7. c) 8. e) 9. a) 10. d)
PUZZLE-109
Directions (1-5): Study the following information carefully and answer the given
questions:
Seven Summits ASEAN Summit, BRICS Summit, SAARC Summit, NAM
Summit, BIMSTEC Summit, SKOCH Summit, and OPEC Summit were scheduled to
be held in China, Japan, Russia, Iceland, India, Germany and Canada, on one day
in a week starting from Monday and ending on Sunday.
BIMSTEC Summit was held in Iceland on Friday. Only one summit was held
between BIMSTEC Summit and BRICS. SKOCH Summit was held immediately after
SAARC but immediately before OPEC. SKOCH Summit was not held in India. The
summit held on Monday was held in Russia. Only one summit was held between
OPEC Summit and the summit held in China. The SAARC was not held in China.
Summit in Japan was held immediately before summit in India. NAM Summit was
not held on Monday. Summit in Canada was not held after summit in China.

1). On which of the following days was the SAARC Summit held?
a) Monday
b) Tuesday
c) Wednesday
d) Thursday
e) Friday

2). If Russia is related to Japan and OPEC is related to NAM in a certain way, then
to which of the following would India be related to, following the same pattern?
a) Germany
b) Canada
c) China
d) Russia
e) India
3). Which of the following summits was held on Monday?
a) OPEC
b) SKOCH
c) SAARC
d) ASEAN
e) None of these

4). In which of the following countries was OPEC Summit held?


a) Canada
b) China
c) Russia
d) Germany
e) India

5). How many summits were held between ASEAN Summit and NAM Summit?
a) Four
b) One
c) Two
d) Three
e) Five

PUZZLE-110
Directions (6-10): Study the following information carefully and answer the given
questions:
Eight friends Praveen, Salim, David, Arafath, Yadav, Naresh, Arnold and
Johnson sitting around a rectangular table, four of them facing inside the table and
four of them facing outside.
David is going to Sikkim and sitting third to the right of Yadav. David is facing
outside the table. Arnold who is going to Assam is sitting third to the right of Naresh
and facing outside the table. Salim is facing inside and sitting second right of Arnold.
Person going to Punjab sitting second left of person who is going to Odisha. Person
going to Karnataka sitting second left of person who is going to Manipur. Naresh is
going to Bihar and sitting fourth left of Praveen who is facing opposite direction of
Salim. Yadav is going to Haryana and facing inside the table. Naresh is facing
outside the table. The persons going to Punjab and Manipur are facing each other.
Arafath is not going to either Manipur or Odisha and facing same direction of Yadav.
Johnson is facing opposite direction of Arnold, is sitting to the immediate left of
Yadav.

6). What is the position of Yadav with respect to David?


a) Second to the left
b) Fourth to the left
c) Third to the left
d) Third to the right
e) None of these

7). Who of the following is sitting third to the left of Yadav?


a) Praveen
b) Salim
c) David
d) Arnold
e) Arafath

8). Praveen is facing which direction?


a) Inside the table
b) Outside the table
c) Inside or outside
d) Cannot be determined
e) None of these

9). Which of the following pair is correct?


a) David – Sikkim
b) Arafath – Punjab
c) Praveen – Karnataka
d) Yadav – Haryana
e) All of these

10). Four of the following five are same in a certain way so form a group. Which of
the following does not belong to that group?
a) Yadav, Naresh
b) Arafath, David
c) David, Salim
d) Praveen, Salim
e) Johnson, Arnold

Answer Key:
Direction (01 to 05):

Days Country Subject of summit


Monday Russia ASEAN
Tuesday Canada SAARC
Wednesday Japan SKOCH
Thursday India OPEC
Friday Iceland BIMSTEC
Saturday China NAM
Sunday Germany BRICS
1. b) 2. c) 3. d) 4. e) 5. a)
Direction (06 to 10):
6. d) 7. a) 8. b) 9. e) 10. c)

PUZZLE-111
Directions (Q. 1-5): Study the following information carefully and answer the given
questions.
Shah Rukh Khan, Aamir Khan, Akshay Kumar, Hrithik Roshan, Ranbir Kapoor, Ajay
Devgan, Abhishek Bachchan and Irrfan Khan are eight fighters sitting around a
circular table. Four of them are facing towards the centre and four of them are facing
away from the centre. All of them like a different National parks, viz. Jim Corbett
National Park, Ranthambore National Park, Kaziranga National Park, Periyar
National Park, Bandipur National Park, Manas National Park, Madhav National Park
and Satpura National Park, but not necessarily in the same order. Ranbir Kapoor
faces towards the centre and likes Jim Corbett National Park. Both the immediate
neighbours of Ranbir Kapoor face away from the centre and like Kaziranga National
Park or Periyar National Park. Hrithik Roshan faces away from the centre. Both the
immediate neighbours of Hrithik Roshan do not face away from the centre. Ranbir
Kapoor sits third to the right of Ajay Devgan, who likes Bandipur National Park and
faces away from the centre. Akshay Kumar sits third to the left of Ajay Devgan. The
one who likes Kaziranga National Park sits opposits of Ajay Devgan. The one who
likes Ranthambore National Park is not the immediate neighbour of Ajay Devgan
and faces away from the centre. Shah Rukh Khan sits second to the left of Akshay
Kumar and does not like Madhav National Park or Satpura National Park. The one
who likes Madhav National Park sits between Irrfan Khan and Ajay Devgan. Aamir
Khan faces away from the centre and does not like Ranthambore National Park.

1). Who like the Manas National Park?


a) Ranbir Kapoor
b) Aamir Khan
c) ShahRukh Khan
d) Irrfan Khan
e) Akshay Kumar

2). How many persons sit between Hrithik Roshan and Ranbir Kapoor, count from
the right of Hrithik Roshan?
a) Five
b) Four
c) Three
d) Two
e) None of these

3). The only person who is sitting between Ranbir Kapoor and Abhishek Bachchan,
likes which of the following parks?
a) Kaziranga National Park
b) Bandipur National Park
c) Manas National Park
d) Jim Corbett National Park
e) Periyar National Park

4). Who among the following sits second to the right of Aamir Khan?
a) Hrithik Roshan
b) Irrfan Khan
c) Akshay Kumar
d) Ajay Devgan
e) None of these

5). Which of the following is correctly matched?


a) Akshay Kumar - Facing Centre - Bandipur National Park
b) Abhishek Bachchan - Facing Outside - Periyar National Park
c) Ajay Devgan - Facing Centre - Jim Corbett National Park
d) Shah Rukh Khan – Facing Centre - Manas National Park
e) None of these

PUZZLE-112
Direction (6-10): Study the given information carefully to answer the given questions:
Ten Persons belongs to different states i.e. Assam, Bihar, Gujarat, Haryana,
Maharashtra, Karnataka, Kerala, Nagaland, Sikkim and Manipur are sitting in two
parallel rows containing five persons each, in such a way that there is an equal
distance between adjacent persons. In row-1 Rajesh, Vidya, Surya, Prakash and
Deepika are seated (not necessarily in the same order) and all of them are facing
south. In row-2 Sanjay, Dinesh, Balaram, Janani and Kaushik are seated (not
necessarily in the same order) and all of them are facing north. Therefore in the
given seating arrangement each member seated in row faces another member of
the other row. All of them have relation with each other.
Only one person sits between Surya’s wife and Rajesh’s brother. Only two persons
sit between Surya’s father and Janani’s father. Dinesh’s father sits at one of the end.
Neither Vidya nor Janani’s grandfather faces Janani. Dinesh belong to Haryana and
Vidya is neither belongs to Assam nor Manipur. Janani’s husband, who belongs to
Karnataka, sits third to the right of Janani’s uncle. Sanjay sits second to the left of
Janani’s husband. Rajesh has two sons. The person facing Janani’s brother, who
belongs to Gujarat, sits immediate right of Rajesh’s daughter-in-law. Deepika is the
grandmother of Sanjay and Janani. Prakash is the brother-in-law of Deepika. Surya
is the father of Sanjay and Husband of Vidya. Sanjay’s grandfather, who belongs to
Assam, is not an immediate neighbour of Vidya. Kaushik is the husband of Janani
and brother of the one, who belongs to Maharashtra. Janani belongs to Kerala and
Deepika belongs to Sikkim. Surya neither belongs to Nagaland nor Bihar. Balaram
belongs to Maharashtra. Prakash does not belong to Bihar.

6). Who is second to the left of the person who belongs to Sikkim?
a) The One who belongs to Manipur
b) Rajesh
c) Deepika
d) The One who belongs to Bihar
e) The One who belongs to Nagaland

7). How is Sanjay related to Rajesh?


a) Son
b) Brother
c) Son in law
d) Grand son
e) None of these

8). What is the position of Kaushik with respect of Kaushik’s wife?


a) Third to the right
b) Immediate left
c) Second to the right
d) Immediate right
e) Second to the left

9). Balaram’s Brother belongs to, which of the following state?


a) Gujarat
b) Manipur
c) Assam
d) Bihar
e) Karnataka

10). Which of the following statement is true?


a) Rajesh belongs to Assam and he is the father of Prakash
b) Balaram belongs to Manipur and he is the brother of Kaushik
c) Vidya belongs to Bihar and she is the wife of Surya
d) Sanjay belongs to Kerala and he is the brother of Janani
e) None of these

Explanation:
Directions (Q. 1-5):

1). Answer: c)
2). Answer: b)
3). Answer: e)
4). Answer: a)
5). Answer: d)
Directions (Q. 6-10):

Family tree:

6). Answer: a)
7). Answer: d)
8). Answer: b)
9). Answer: e)
10). Answer: c)

PUZZLE-113
Directions (Q. 1-5): Study the following information carefully and answer the given
questions.
Ten Persons are sitting in two parallel rows of six seats each. One seat is vacant in
each row. Mahima, Sathya, Karthick, Vimal and Priyanka are sitting in row-1 facing
south. Lokesh, Murugan, Lalitha, Meena and Begum are facing north. Each likes a
different brand of Tooth pastes i.e. Colgate, Pepsodent, Sensodyne, Oral- B, Dant
Kanti, Vicco, Amway, Himalaya, Close Up and Meswak. Meena sits third to the right
of Lalitha and likes Oral- B. Only two people sit between Murugan and the vacant
seat. Murugan does not like Dant Kanti or Sensodyne Tooth paste. Priyanka is not
an immediate neighbour of Karthick. Sathya likes Meswak. The one who likes
Sensodyne Tooth paste faces the one who likes Himalaya. The one who likes
Sensodyne sits opposite to the one who sits third right of the person who sits
opposite to Meena. Karthick is not an immediate neighbour of Vimal. Begum, who
likes neither Dant Kanti nor Vicco, does not face the vacant seat. Neither Meena nor
Lalitha sits at any of the extreme ends of the row. Vimal faces Lalitha. Vacant seats
are not opposite to each other. Two seats are there between Karthick and Sathya,
who sits third right of the one who likes Amway. The one who likes Close Up Tooth
paste faces the one who likes Oral- B. The persons who like the Colgate and
Himalaya are adjacent to each other. Vacant seat of row – 1 is not an immediate
neighbour of Vimal. Murugan sits at one of the extreme ends of the row. Lalitha does
not like Colgate and Himalaya. Vacant seat of row-1 does not face Meena who
doesn’t sit at any of the extreme ends of the row.

1). Which of the following brand Begum likes?


a) Meswak
b) Oral- B
c) Amway
d) Pepsodent
e) Close Up

2). Who among the following likes Vicco?


a) Begum
b) Meena
c) Murugan
d) Vimal
e) None of these

3). How many people will sit between Lokesh and Murugan?
a) One
b) Two
c) Three
d) Four
e) More than four

4). Which of the following is correctly matched?


a) Vimal – Meswak
b) Lokesh – Dant Kanti
c) Begum – Amway
d) Murugan – Close Up
e) Lalitha – Oral- B

5). Four among the following form a group in a certain way. Which of the following
does not belong to Group?
a) Priyanka, Meswak
b) Lokesh, Sensodyne
c) Murugan, Dant Kanti
d) Sathya, Close Up
e) Begum, Oral- B

PUZZLE-114
Directions (Q. 6-10): Study the following information carefully and answer the given
questions.
Abhinav plays Pokemon Go. He caught 7 different Pokemons on 7 different days of
the week from Monday to Sunday. One of the Pokemons is Ratata. These Pokemon
had 2 characteristics: Type and Combat Power (CP). One of the Pokemons is
Poison type.
The following information is known:

• The combat powers of the 7 Pokemons are 25, 31, 38, 46, 49, 52 and 60.
• Charmander is not the Ground type Pokemon.
• The Pokemon caught on Monday has CP 46 while Spearow was the last Pokemon
to be caught.
• Squirtle, an Electric Pokemon, was caught on Wednesday.
• There is only one Pokemon whose CP is lower than that of Pidgey.
• The number of Pokemons caught before Pikachu is the same as the number of
Pokemons caught after Charmander. Pikachu was caught before Charmander.
• The Pokemon that has the highest CP was not caught on Saturday.
• A Rock type Pokemon whose CP is 25 was caught on Friday and a Water type
Pokemon whose CP is 52 was caught on Tuesday.
• Bulbasaur is a Fire type Pokemon which was not caught on either Thursday or
Saturday and Spearow is a Fighting type Pokemon.
• The Pokemon caught on Sunday has CP less than 40.

6). What is the CP of Charmander?


a) 38
b) 46
c) 49
d) 60
e) 52

7). Which is the rock type Pokemon?


a) Ratata
b) Pikachu
c) Charmander
d) Bulbasaur
e) Pidgey

8). What is the difference between the CP of Squirtle and that of the fighting type
Pokemon?
a) 14
b) 24
c) 27
d) 22
e) 11

9). Arrange the Pokemons in the order they were caught:


a) Bulbasaur, Pikachu, Ratata, Pidgey, Charmander
b) Bulbasaur, Pikachu, Pidgey, Ratata, Charmander
c) Pikachu, Bulbasaur, Pidgey, Ratata, Charmander
d) Bulbasaur, Pidgey, Pikachu, Ratata, Charmander
e) Pikachu, Bulbasaur, Pidgey, Charmander, Ratata

10). Bulbasaur is related to 'Electric' in some manner and Ratata is related to


'Fighting' in that manner. Which Pokemon is related to 'Rock' in the same manner?
a) Pikachu
b) Pidgey
c) Charmander
d) Spearow
e) Squirtle

Explanation With Answers Key:


Directions (Q. 1-5):
1). Answer: d)
2). Answer: c)
3). Answer: a)
4). Answer: b)
5). Answer: e)

Directions (Q. 6-10):

6). Answer: c)
7). Answer: a)
8). Answer: d)
9). Answer: b)
10). Answer: e)

PUZZLE-115
Directions (Q. 1-5): Study the following information and answer the questions given
below.
In a conference 8 people Rekha, Salma, Ganesh, Mishra, Gibbs, Sahana,
Pooja and Aarya from different countries Australia, Germany, China, Japan, Russia,
Switzerland, Poland and Brazil not necessarily in same order sitting around a
rectangular table. 3 persons are sitting on each longer side and each on the smaller
sides. Mishra is sitting second to the right of the person who is from Brazil. Gibbs is
sitting third to the left of the person who is from Switzerland. Sahana and Pooja are
sitting opposite each other. Ganesh is sitting diagonally opposite the person from
Poland. Salma is sitting opposite the person who is from Australia. The person from
Japan is sitting second to the right of the person from Russia and second to the left
of Rekha, who is not sitting near the person who is from Poland. Pooja is sitting on
the smaller side and to the right of the person who is from Poland. Persons from
Russia and Japan are not on the same side of the table. The person from Japan is
sitting third to the right of the person from Brazil, who is not sitting diagonally
opposite the person from Germany. The person from Japan is third to the left of
Ganesh. Sahana sits second to the left of Aarya. The person from Germany sits
opposite to the person from Russia.
1). Which of the following combinations is correctly matched?
a) Aarya –Brazil
b) Mishra – Russia
c) Pooja – Japan
d) Rekha– China
e) None of these
2). Who is sitting third to the right of the person from Switzerland?
a) Gibbs
b) Person from Russia
c) Rekha
d) Person from Poland
e) None of these
3). From which country does Rekha belong?
a) China
b) Australia
c) Russia
d) Brazil
e) Germany
4). Who is definitely sitting diagonally opposite to Mishra?
a) Rekha
b) Ganesh
c) Person from Germany
d) Person from China
e) None of these
5). According to the sitting arrangement what will come in place of question mark?
Mishra : Japan ::Ganesh : ?
a) Switzerland
b) Russia
c) China
d) Brazil
e) None of these

PUZZLE-116
Directions (Q. 6-10): Study the following information and answer the questions given
below.
Eight friends having different Sims as Airtel, Vodafone, Aircel, Docomo, Idea,
BSNL, JIO and Reliance are sitting around a circular table, not necessarily in the
same order. Some of them are facing outside and some of them are facing towards
the centre. Each of them has birthday in different year.
Only two people sit between the one who uses Idea and the one who uses Aircel.
The friends who use Vodafone and Airtel are not neighbours. The one who uses
Airtel is sitting just right of one who uses Idea. The one who uses JIO sits second to
the right of one who uses Aircel. The person who uses Idea faces outside and he is
the oldest guy having birthdays in 1975. Only one person sits in between friends
who use Reliance and Airtel. The one liking Vodafone is 8 years older than the one
liking BSNL. The one who uses Reliance sits third to the left of one who uses
Vodafone and he is the youngest among all having birthday in 1989. The person
who uses BSNL faces towards the direction opposite to the one who uses JIO, but
same to the one who uses Aircel. Person liking BSNL is not a neighbour of either
who uses Airtel or JIO. The one who uses Airtel was born in 1983 and the one who
uses JIO is 4 years younger than him. Immediate neighbours of one who uses
Docomo face in the same direction. The one who uses Airtel faces towards the
centre. The one who uses Vodafone and the one who uses Aircel face the direction
just opposite to the one who uses Docomo. The person facing the one who uses
Airtel was born in 1979. The immediate neighbours of the one who uses Airtel have
faces opposite to each other and they have the age gap of 6 years.
6). Who uses Docomo sim born on which year?
a) 1983
b) 1981
c) 1975
d) 1979
e) None of these
7). How many persons facing outside?
a) Four
b) Three
c) Two
d) Six
e) None of these
8). Which of the following is correctly matched?
a) Airtel – Facing Centre - 1981
b) Jio – Facing Outside - 1983
c) Aircel – Facing Centre - 1987
d) Idea – Facing Outside - 1975
e) None of these
9). Who is sitting second to the left of person who born in 1975?
a) One who uses Docomo sim
b) One who born in 1989
c) One who uses JIO sim
d) One who born in 1981
e) One who uses Vodafone sim
10). How many persons sit between who uses the Airtel sim and who uses BSNL
sim, count from the right of one who uses Airtel sim?
a) 1
b) 3
c) 5
d) 4
e) 6
Answer Key:
Direction (01-05):

1. c) 2. d) 3. e) 4. b) 5. a)
Direction (06-10):
6. b) 7. a) 8. d) 9. e) 10. d)

PUZZLE-117

Directions (Q. 1-5): Study the following information carefully and answer the given
questions.
There are seven boys Jadeja, Saha, Pujara, Kohli, Vijay, Rahul and Rahane

who participated in a game competition which started on Monday and ended on

Sunday. In the first round of the competition, each of them played different games,
Cricket, Tennis, Football, Hockey, Badminton, Volleyball and Basketball, but not

necessarily in the same order. They like different Social Networks, Facebook,

Twitter, Hike, Whatsapp, Telegram, Instagram and Google, but not necessarily in

the same order. Saha did not play on the day either immediately before or

immediately after the game of Kohli, who does not like either Telegram or Facebook
or Whatsapp Social Network. Two games were held between the game of Rahane

and Rahul, neither of them played on Monday. There was one game between the

games of Kohli and Pujara. But Pujara’s game did not happen either on Monday or
on Wednesday. Pujara likes Twitter Social Network and played Cricket. The one,

who played Football on the last day of competition, likes Hike Social Network. Vijay

played immediately after Pujara and he likes Facebook Social Network. Jadeja does
not like Telegram Social Network and played Hockey. Kohli did not play either

Basketball or Volleyball. The one who played Badminton was scheduled

immediately after the game of Cricket. Rahane, who likes Instagram, played on the

fourth day of the competition but played neither Cricket nor Tennis.

1). Which game organised on Tuesday?

a) Tennis

b) Badminton
c) Football

d) Hockey
e) Cricket

2). Rahul like which of the following social Network?


a) Hike
b) Facebook

c) Twitter
d) Whatsapp

e) None of these

3). Which of the following statement is true?

a) Monday – Saha – Volleyball - Telegram


b) Friday – Pujara - Cricket - Twitter

c) Thursday – Rahane - Basketball - Instagram


d) Wednesday – Jadeja – Hockey - Facebook

e) Sunday – Rahul – Football - Google


4). Which of the following day Cricket game is organized?

a) Sunday

b) Saturday
c) Wednesday

d) Monday

e) Friday

5). Which of the following boy like the Instagram social Network?
a) Vijay

b) Pujara
c) Rahane

d) Saha

e) Kohli

PUZZLE-118

Directions (Q. 6-10): Study the following information carefully and answer the given
questions.
There are nine people A, B, C, D, E, F, G, H and I in a family. It is a three

generation family. They are sitting in a row facing in North and South directions not
necessarily in the same order. The following information is known about them. C

who is A’s father sits fourth to the right of B who is the only daughter of E and they

both face in the opposite directions. D sits third to the right of F who is B’s

grandmother and one of them sits adjacent to I. D is unmarried and faces in a


direction similar to that of G. E who is the sister of D sits at one of the ends with a

female. C’s neighbours are females and face in opposite directions. F’s spouse C
faces in the North direction. I who is the wife of G sits at the middle facing in South

direction. A, who is male, is the only sibling of G sits exactly between D and B’s
cousin who is a male. The people sitting at ends face in North direction. A faces in

same direction as that of F. C sits adjacent to I.

6). Who among the following sits at one of the ends?


a) F’s son

b) H’s mother

c) A’s daughter

d) G’s son
e) None of these

7). How is D related to A?


a) Wife

b) Sister in law

c) Brother in law

d) Husband

e) None of these
8). How many males are there in the family?
a) 2

b) 3
c) 1

d) 5

e) 4

9). C and D face in which direction in the respective order?

a) North, South

b) South, North

c) North, North
d) South, North

e) None of these
10). Who sits third to the right of H’s mother?

a) B

b) A
c) F

d) G

e) H

Answer Key:
Direction (01 to 05):

1. d) 2. a) 3. b) 4. e) 5. c)

Direction (06 - 10):


6. d) 7. b) 8. e) 9. c) 10. b)

PUZZLE-119
Directions (1 -6): Study the following information carefully and answer the questions
given below:
Isha, Swati, Ravi, Mohan, Ruchi, Raj, Sohan and Rani are eight friends travelling by
three different models of cars, viz Alto, Scorpio and WaganR. Each model of car is
used by at least one female. The cars go to three places- Dwarka, Varanasi and
Agra. There are three days of travel – Monday, Tuesday and Wednesday. Each
place is visited by at least on female. On each day at least one female travels. Isha
and Mohan travel to Dwarka on the same day by the same model of car. Raj travels
on Monday in Alto, but not to Agra. A person travelling to Agra uses Scorpio on
Monday. WagonR goes only to Varanasi and only on Tuesday. Sohan travels to
Varanasi in Alto on the day before the day on which Isha travels. Ruchi travels to
Agra but she likes to travel in Alto. No male member travels in WagonR. Isha travels
on the same day on which Swati travels. Ravi travels to Varanasi on Wednesday in
Scorpio. Rani does not travel on Tuesday. No person travels to Dwarka on Monday.
Mohan travels by the same model of car by which Sohan travels. Ruchi and Ravi
travel on the same day.

1). Ruchi travel on which of the following day?


a) Monday
b) Wednesday
c) Either Monday or Tuesday
d) Tuesday
e) None of these
2). Who among the following travel to Varanasi on Monday?
a) Raj and Rani
b) Sohan and Raj
c) Swati and Sohan
d) Rani and Swati
e) None of these
3). How many persons are travelling to Dwarka?
a) One
b) Two
c) Three
d) Can’t be determined
e) None of these
4). Swati travels in which of the following cars?
a) Scorpio
b) Alto
c) WagonR
d) Either Alto or Scorpio
e) None of these
5). Which of the following combinations is / are true?
a) Isha – WagonR – Dwarka – Tuesday
b) Ravi – Scorpio – Varanasi – Wednesday
c) Ruchi – Alto – Dwarka – Monday
d) Rani – Alto – Varanasi – Wednesday
e) None of these
6). Which of the following statements is / are true?
a) Sohan goes to Varanasi on Monday
b) Mohan travels in Scorpio on Tuesday
c) Raj Travels in Alto to Agra
d) Swati goes to Varanasi on Wednesday
e) None of these

PUZZLE-120
Directions (Q. 7-11): Study the following information and answer the following
questions:
A, B, C, D, E, G and I are seven friends who study in three different standards,
namely 5th, 6th and 7th, such that not less than two friends study in the same
standard. Each friend has a different favourite subject, namely History, Civics,
English, Marathi, Hindi, Maths and Economics also but not necessarily in the same
order.
A likes Maths and studies in the 5th standard with only one friend who likes Marathi. I
studies with two other friends. Both the friends who study with I like languages (here
languages include only Hindi, Marathi and English). D studies in the 6 th standard
with only one person and does not like Civics. E studies with only one friend. The
one who likes History does not study in the 5th or 6th standard. E does not like
languages. C does not like English, Hindi or Civics.

7). Which combination represents E’s favourite subject and the standard in which he
studies?
a) Civics and 7th
b) Economics and 5th
c) Civics and 6th
d) History and 7th
e) Economics and 7th
8). Which of the following is I’s favourite subject?
a) History
b) Civics
c) Marathi
d) Either English or Marathi
e) Either English or Hindi

9). Who among the following studies in the 7th standard?


a) G
b) C
c) E
d) D
e) Either D or B

10). Which of the following combinations is definitely correct?


a) I and Hindi
b) G and English
c) C and Marathi
d) B and Hindi
e) E and Economics

11). Which of the following subjects does G like?


a) Either Maths or Marathi
b) Either Hindi or English
c) Either Hindi or Civics
d) Either Hindi or Marathi
e) Either Civics or Economics

Answers:
1). b) 2). b) 3). b) 4). c) 5). b) 6). a) 7). c) 8). a) 9). a) 10). c) 11). b)
Directions(1-6):

Name Car Place Day


Isha Alto Dwarka Tuesday
Swati WagonR Varanasi Tuesday
Ravi Scorpio Varanasi Wednesday
Mohan Alto Dwarka Tuesday
Ruchi Alto Agra Wednesday
Raj Alto Varanasi Monday
Sohan Alto Varanasi Monday
Rani Scorpio Agra Monday

Questions (7-11):

Person Standard Subject

A 5th Maths

C 5th Marathi

D 6th Economics

E 6th Civics

I 7th History

B 7th Hindi/English

G 7th Hindi/English
PUZZLE-121
Directions (Q. 1-5): Study the following information carefully and answer the given
questions.
Six department exams – Computer, Maths, Science, Commerce, Economics
and Engineering were written by three males – Gokul, Sukumar and Magesh and
three females – Deepa, Priya and Divya. Three of these department exams were
held in Chennai and the rest in Bangalore. Each of these exams was organized by
different universities– Manipal, SRM, VIT, Amity, IIT and IIS (may not be in the same
order). Each exam was written by a male and a female.
None of the students wrote in a city twice and no two department exams were
written by the same pair of male and female. The females who written Computer and
Maths did not write the exam organized by IIT and IIS. The male student was same
for the exams – Computer and Maths. Sukumar written Science but did not write the
exam organized by IIT or Manipal. Priya did not write any exam with Gokul, but
written Commerce with Magesh. Deepa wrote an exam with Magesh. IIS organized
the exam in Chennai and Amity organized the exam in Bangalore which was written
by Gokul and VIT organized Maths. Economics was organized by Manipal. IIT and
SRM organized the exams in the same city. Maths and Science exam were written
by the same female. The Engineering exam was not organized by IIT or Amity and it
was not written by Deepa.

1). Who wrote the Maths department exam?


a) Magesh and Divya
b) Sukumar and Priya
c) Gokul and Deepa
d) Gokul and Divya
e) None of these

2). Which of the following exams organized in Chennai?


a) Maths, Economics and Computer
b) Maths, Commerce and Engineering
c) Maths, Economics and Engineering
d) Science, Economics and Engineering
e) Maths, Economics and Science

3). Science exam is conducted by which university?


a) SRM
b) VIT
c) IIT
d) IIS
e) None of these

4). Which of the following exam written by Magesh and Priya?


a) Science
b) Commerce
c) Computer
d) Engineering
e) Maths

5). Which of the following statement is true?


a) Computer – Gokul – Deepa – Amity - Chennai
b) Commerce - Magesh - Priya – VIT - Bangalore
c) Maths - Gokul – Divya – VIT - Bangalore
d) Engineering - Sukumar – Deepa - IIS - Chennai
e) Economics – Magesh – Deepa – Manipal - Chennai
Direction (01-05):

PUZZLE-122
Directions (Q. 1-5): Study the following information carefully and answer the given
questions.

M, N, O, P, Q, R, S and T are eight family members sitting around a circular

table in a restaurant. All of them are facing the centre. Three of the family members
are non- vegetarian and others are vegetarian. All the eight family members are

professionals. They are Banker, Engineer, Accountants, Police, Lawyer, Scientist,

Professor and Teacher, but not necessarily in the same order. There are two
married couples, three brothers, two daughters, one granddaughter and one

grandson in the group. Two of the female members in the group M and T order for

the same food. The food ordered in the restaurant are Khichdi, Kufta, Chapatti,
Butter chicken, Vegetable Biryani, Tandoori Chicken and Puri.

S, a Banker, is the head of the family, and is immediate neighbour of T and P. The

granddaughter R is sitting third to the left of S. The grandmother of R is sitting on the

immediate left of her. R is vegetarian. Q is sitting second to the right of his father S.

He is a Lawyer, and orders for Puri. Only S’s daughter is sitting exactly between M

and Q. She orders for Vegetable Biryani and she is a Scientist. T is an Accountant
and N is a Police. The son of T is sitting on the immediate left of his father. His

father is an Engineer. O is a Scientist and is sitting on the immediate left of her

mother. She orders for Vegetable Biryani. P, the husband of the Accountants,
ordered for Tandoori Chicken and his mother orders for Butter chicken. The one who

is a Police sits second to the right of M, who is an immediate neighbour of the

Scientist and the Professor. The Professor does not like Khichdi and the Banker

does not like Chapatti or Kufta.


1). Who among the following working as Professor?

a) R
b) P

c) O

d) M

e) None of these

2). Who among the following is ordering Khichidi food?


a) R
b) S

c) M
d) N

e) None of these

3). Which of the following is correctly matched?

a) N – Police - Chappatti

b) Q – Teacher - Puri

c) P – Engineer - Tandoori Chicken

d) O – Scientist – Tandoori Chicken


e) None of these
4). The only person who is sitting between R and O is working on which of the

following professions?

a) Engineer
b) Police

c) Banker

d) Accountant

e) Teacher
5). The person who ordered Butter chicken is working as which of the following

professions?
a) Teacher

b) Police

c) Accountant

d) Both Teacher and Accountant

e) Professor

PUZZLE-123

Directions (Q. 6-10): Study the following information carefully and answer the given

questions.
Ten persons Daniel, Rajesh, Michael, Yusuf, Anand, Balaji, Lokesh, Kannan,

Jaffar and Praveen are sitting in two rows with five persons in each row. The

persons in row one are facing south and the persons in row two are facing north.

Each person in row one faces a person from the other row. All of them have a laptop

of different companies, viz Acer, HP, Asus, Dell, LG, Toshiba, Samsung, Lenovo,

Apple and Microsoft, but not necessarily in the same order.

The persons who like LG and Toshiba sit opposite each other. Balaji sits opposite to

Daniel, who likes Acer. The one who likes HP sits opposite the one who likes
Lenovo. Praveen is not facing north but sits third to the left of Lokesh, who likes HP.

There is only one person between Rajesh and Michael. Anand sits at one of the

ends of the row and likes Toshiba. The one who likes Lenovo is on the immediate
right of Yusuf, who does not like Microsoft. The persons who like Asus and Dell

respectively are not facing north. Michael likes Samsung. The one who likes Dell sits

opposite the one who is second to the right of Rajesh. Jaffar does not like Microsoft.

Anand sits opposite the one who sits second to the left of the one who likes Asus.
6. Who among the following is the immediate neighbour of the one who have Acer

laptop?
a) The one who have Apple laptop

b) Michael

c) Jaffar

d) The one who have HP laptop

e) None of these
7. Who have Apple laptop?
a) Anand

b) Rajesh
c) Yusuf

d) Balaji

e) Lokesh

8. Who sits at the right end in North facing Row ?

a) Kannan

b) Praveen

c) Lokesh
d) Anand

e) Yusuf
9). Who sits opposite to Rajesh?

a) The one who have Acer laptop

b) The one who have Asus laptop


c) The one who have Apple laptop

d) The one who have LG laptop

e) The one who have HP laptop

10). Who sits opposite to the one who sits second to the left of Anand?
a) The one who have Asus laptop

b) The one who have Dell laptop


c) The one who have Apple laptop

d) The one who have HP laptop

e) None of these

Answer Key:

Direction (01-05):
1. a) 2. b) 3. c) 4. e) 5. d)

Direction (06-10):

Kannan
Facing South Lokesh (HP) Balaji (Asus) Jaffar (Dell) Praveen (LG)
(Microsoft)

Rajesh Michael Anand


Facing North Yusuf (Apple) Daniel (Acer)
(Lenovo) (Samsung) (Toshiba)

6. b) 7. c) 8. d) 9. e) 10. a)

PUZZLE-124
Directions (Q. 1-5): Study the following information carefully to answer the given
questions:
Eight People – Haroon, Amir, Prem, Devan, Ramesh, Siva, Mohan and Irfan
live in eight different floors of building (but not necessarily in the same order). The
lowermost floor of the building is numbered one, the one above that is numbered
two, and so on till the topmost floor is numbered eight. Each one of them also owns
a different brands of watch, namely Fastrack, Titan, Casio, Timex, Sonata, Citizen,
Giardano and Maxima (but not necessarily in the same order).
Only one person lives between Amir and the one who owns Sonata. Siva lives an
odd numbered floor above the floor numbered four. Only three people live between
Devan and the one who owns Citizen. Prem lives on one of the odd numbered floors
above the one who owns Citizen. The one who owns Giardano lives immediately
above Mohan, Mohan owns neither Citizen nor Fastrack. Ramesh does not own
Giardano. Only three people live between Mohan and Haroon. The one who owns
Maxima lives immediately above the one who owns Titan, but not on the topmost
floor. Only one person lives between Siva and the one who owns Casio. The
number of people living above Siva is same as the number of people living between
Siva and Devan. Only two people live between Prem and the one who owns
Fastrack.

1). Who amongst the following stays on the top–most floor?


a) Who owns Timex Watch
b) Mohan
c) Amir
d) Both a and c
e) None of these

2). How many floors are there between the floor on which Siva stays and the floor
who owns Titan watch?
a) None b) Three c) Two d) One e) More than three
3). Ramesh owns which of the following Watches?
a) Sonata
b) Citizen
c) Fastrack
d) Titan
e) Casio
4). On which of the following floor does Irfan stay?
a) 3rd b) 5th c) 7th d) 2nd e) 1st
5). Which of the following statement is true as per the given information?
a) Irfan stays on a floor immediately below the floor on which Mohan stays
b) Prem stays on a floor immediately above the floor on who owns Citizen Watch
c) Haroon stays on sixth floor and owns a Timex watch
d) There are two persons live between Ramesh and Amir
e) None is true

PUZZLE-125
Directions (Q. 6-10): Study the following information carefully to answer the given
questions:
Eight friends Vikram, Ashok, Varun, Gautam, Charan, Basha, Yashwant and
Edward live on eight different floors of a building but not necessarily in the same
order. The lowermost floor of the building is numbered 1 and the topmost floor of the
building is numbered 8. Each of them likes different flavours of fruit juices viz,
Mango Juice, Orange Juice, Apple Juice, Grapes Juice, Lemon Juice, Banana
Juice, Pomegranate Juice and Papaya Juice but not necessarily in the same order.
The one who likes Orange Juice lives on an even-numbered floor but not on the
topmost floor. Only one person lives between Basha and the one who likes Lemon
Juice. Only two persons live between Basha and the one who likes Orange Juice.
Neither Charan nor Varun lives on the first floor. Only one person live between
Varun and the one who likes Apple Juice. Vikram lives just above Basha. Only two
persons live between Charan and Vikram. The one who likes Lemon Juice does not
live on floor number one, Ashok lives on an even-numbered floor and just above
Varun. The one who likes Grapes Juice lives on an even numbered floor and lives
just above the person who likes Papaya Juice. Varun does not like Lemon Juice or
Papaya Juice. Only two persons live between the one who likes Banana Juice and
the one who likes Mango Juice. Gautam does not like Pomegranate Juice. The one
who likes Banana Juice does not live on an odd-numbered floor. Edward lives just
below the one who likes Papaya Juice.
6). On which of the following floor does Ashok stay?
a) Third
b) Fourth
c) Fifth
d) Second
e) Seventh
7). How many persons are there between who likes apple juice and who likes
grapes juice?
a) Six
b) One
c) Three
d) Four
e) Five
8). Who likes the Papaya Juice?
a) One who lives in floor number 5
b) Basha
c) Gautam
d) One who lives in floor number 3
e) Vikram
9). Which of the following is wrong as per the given information?
a) Charan stays on a floor immediately above the floor on who likes Orange Juice
b) Only one person stay between who likes Mango Juice and Gautam
c) Basha lives on seventh floor and likes Banana Juice
d) More than two persons living between Yashwant and Edward
e) All statements are true
10). Who amongst the following stays on the fifth floor?
a) Edward
b) Varun
c) Ashok
d) Basha
e) None of these

Answer with Explanation:

Direction (01 to 05):


Floor Person Watch
8 Amir Timex
7 Prem Casio
6 Haroon Sonata
5 Siva Citizen
4 Ramesh Fastrack
3 Irfan Giardano
2 Mohan Maxima
1 Devan Titan
1. d) 2. b) 3. c) 4.a) 5. e)

Direction (01 to 06):


Floor Person Mobiles
8 Vikram Grapes Juice
7 Basha Papaya Juice
6 Edward Banana Juice
5 Charan Lemon Juice
4 Ashok Orange Juice
3 Varun Mango Juice
2 Yashwant Pomegranate Juice
1 Gautam Apple Juice
6. b) 7. a) 8. b) 9. c) 10. e)
PUZZLE-126
Directions (Q. 1-5): Study the following information carefully to answer the given
questions:
There are six persons- Aarthi, Shreya, Kavya, Vidya, Diya and Madhu living in a
triple floor building with six flats. The floors are Ground, Middle and Top, each
having two flats. Those who have four or more books occupy the top floor and go by
different Vehicles. The one who is in engineering department in a company goes by
Cycle and she is not Madhu while the one who is in Computer department of a
company goes by Bus, Both of them occupied same floor. Shreya who is in Science
department has three books and not stayed in the Top floor. Kavya, an unmarried
woman, does not have any book, occupies the middle floor. Diya and Madhu have
Round purses while the rest have Square purses. Vidya, a middle floor occupant,
goes by Car. One of the two persons having two books goes by Bike and is in Agri
department in a company also occupied in ground floor. Two persons, of whom one
is in Commerce department, do not go by Vehicles.
1). Who among the following occupies the top floor?
a) Aarthi
b) Diya
c) Kavya
d) Vidya
e) None of these
2). By which Vehicle does Kavya go?
a) Bus
b) Cycle
c) Bus or Cycle
d) No Vehicle
e) None of the above
3). What is the least number of books owned by all the persons in the entire
building?
a) 14
b) 16
c) 15
d) 17
e) Cannot be determined
4). In which department does Vidya work in?
a) Commerce
b) Engineering
c) Agri
d) Computer
e) Cannot be determined
5). How many books does Aarthi have?
a) None
b) One
c) Two
d) Three
e) None of these

PUZZLE-127
Directions (Q. 6-10): Study the following information carefully to answer the given
questions:
There are seven people M, N, O, P, Q, R and S. They like different Mobiles, Banks
and were born in different States and months. They all were born in the same year.
One of the Mobiles is HTC. One of the States is Maharashtra. One of the Banks is
Bank of India. One of the persons was born in the month of September. The
following information is known about them. R was born either in Tamil Nadu or in
Goa. The one who was born in the month of May likes Indian Bank and Apple
mobile and is either O or S. P is not the one who likes Syndicate Bank. The one who
likes Canara Bank likes LeEco mobile. The one who was born in Goa is the
youngest one and is either N or S. The one who likes Dena Bank was born in
Rajasthan but is not N. Q was born in the month of February. P was born in Bihar in
the month of July and likes Micromax mobile. The one who likes Vijaya Bank was
born in November but is not the youngest person. N likes Motorola mobile. The one
who likes Canara Bank was born in Punjab but that person is not O who likes Nokia
mobile. The youngest person likes Syndicate Bank and he is not S.The one who
was born in Kerala likes Samsung mobile but is not Q. R is the second youngest
person. The one who likes UCO Bank is one month younger than Q.
6). Which of the following Banks does P like?
a) Vijaya Bank
b) Bank of India
c) Indian Bank
d) UCO Bank
e) Dena Bank
7). When was O born?
a) March
b) May
c) September
d) November
e) Cannot be determined
8). Who likes HTC mobile?
a) R
b) S
c) M
d) N
e) O
9). Which of the following statements is correct?
a) E likes Bank of India Bank
b) M likes Apple mobile
c) S was born in the month of September
d) N was born in Tamil Nadu
e) None of these
10). Who was born in Maharashtra?
a) Q
b) M
c) N
d) S
e) O

Explanation with Answer key:

Directions (Q. 1-5):

1). Answer: b)
2). Answer: d)
3). Answer: c)
4). Answer: e)
5). Answer: c)
Directions (Q. 6-10):

6). Answer: b)
7). Answer: c)
8). Answer: a)
9). Answer: e)
10). Answer: d)

PUZZLE-128
Directions (Q. 1-5): Study the following information carefully to answer the given
questions:
Udhaya, Mahesh, Arun, Fasal, Raja, Gopi and Johnson are seven people live on
seven different floors of a building but not necessarily in the same order. The lower
most floor of the building is numbered 1, the one above that is numbered 2 and so
on till the topmost floor is numbered 7. Each one of them earn different amount per
month. i.e. 24000, 18000, 12000, 27000, 33000, 39000 and 45000. (But not
necessarily in the same order). The one who earn Rs.24000 lives immediately
above the one who earns Rs.45000. Only one person lives between Mahesh and
Raja. Mahesh lives on one of the floors above Raja. Neither Arun nor Udhaya earns
Rs.27000. Raja does not earn Rs. 12000. Udhaya lives on an odd numbered floor
but not on the floor numbered three. The one who earns Rs.33000 lives immediately
above Udhaya. Only two people live between Udhaya and the one who earns
Rs.12000. The one who earns Rs.18000 lives on one of the odd numbered floors
above Fasal. Only three people live between Arun and the one who earns 18000.
The one who earn Rs.12000 lives immediately above Arun. Gopi earns 12000.

1). Four among the following form a group in a certain way. Which of the following
does not belong to Group?
a) Johnson – 18000
b) Udhaya – 45000
c) Mahesh – 24000
d) Gopi – 45000
e) Raja – 12000

2). Which among the following is the salary of Mahesh?


a) Rs. 24000
b) Rs. 45000
c) Rs. 39000
d) Rs. 33000
e) None of these

3). Who among the following earns Rs. 39000?


a) Udhaya
b) Arun
c) Fasal
d) Gopi
e) None of these.

4). Which of the following combination is true as per the given arrangement?
a) Udhaya – 18000
b) Arun – 33000
c) Mahesh – 39000
d) Gopi – 24000
e) None of these.

5). Who among the following lives in floor no 3?


a) Udhaya
b) Arun
c) Fasal
d) Gopi
e) None of these.

PUZZLE-129
Directions (Q. 6-10):
A group of seven friends Chandru, Chahal, Charan, Chamu, Chandran, Charu and
Chadrasekar has four males and three females. Each of them has wear different
coloured T-shirts, viz Yellow, Orange, Red, White, Green, Blue and Grey, and each
of them belongs to different states –Maharashtra , Odisha, Tamilnadu, Uttar
Pradesh, Madhya Pradesh, Manipur and Karnataka but not necessarily in the same
order. None of the females wear Blue or Yellow. Charan wear Red and she belongs
to neither Uttar Pradesh nor Manipur. Chandru is from Manipur and wear neither
yellow nor blue. Chahal does not wear White. Chandran has Grey and from either
Odisha or Maharashtra . Charu has Yellow or Green and not belong to Maharashtra.
Chadrasekar has either Blue or Yellow T-shirt and from Tamilnadu. The one who
wears White belongs to Karnataka. The one who wears Orange either from Manipur
or Maharashtra . None of the female either from Manipur or Odisha.

6). Which of the following combinations is true?


a) Chandran -Grey-Odisha-Female
b) Chandru -Green-Odisha-Male
c) Chadrasekar -Blue-Maharashtra-Female
d) Charan -Red-Madhya Pradesh-Male
e) None of these
7). Which of the following T-shirt colour does Chamu have?
a) Blue
b) White
c) Orange
d) Red
e) None of these

8). Which of the following statements is true?


a) Chamu is from Maharashtra.
b) Charan is a male and possesses Red T-shirt.
c) Charu possesses Yellow and from Uttar Pradesh .
d) Chadrasekar is a female and possesses Blue.
e) All are true

9). Which of the following groups has only female members?


a) Charu,Chamu,Chahal
b) Chandran,Charan,Chadrasekar
c) Charan, Chamu,Chahal
d) Chamu,Chandru,Chahal
e) None of these

10). The one who has Yellow T-shirt is from which of the following states?
a) Odisha
b) Madhya Pradesh
c) Manipur
d) Uttar Pradesh
e) Cannot be determined
Directions (Q. 1-5):

1). Answer: d)
2). Answer: d)
3). Answer: b)
4). Answer: a)
5). Answer: c)

Directions (Q. 6-10):

6). Answer: e)
7). Answer: b)
8). Answer: c)
9). Answer: c)
10). Answer: d)

PUZZLE-130
Directions (Q.1 -5): Study the following information carefully and answer the given
questions.
Ten students namely viz Rakesh, Naveen, Kumar, John, Suresh, Divya,
Priya, Ramya, Anjali and Deepa from a banking coaching institute but not
necessarily in the same order have exam on five different days starting from Monday
to Friday of the same week. Students have exam at two different time slots, i.e 09.30
A.M and 01.30 P.M. Anjali has a exam on Tuesday at 09.30 A.M. The number of
people who have exam between Priya and John is same as the number of people
who have exam between Kumar and Ramya. Naveen has a exam day immediately
before Anjali. Anjali does not have exam on any of the days before Priya. There is
one student who has exam at 09.30 A.M immediately before Deepa. Ramya does
not has exam at 01.30 P.M. John has an exam immediately after the day of one
who has exam on Monday. Divya does not have exam at 01.30 P.M. John does not
has exam on any one of the days after Suresh. Only three people have exam
between Priya and Suresh. Neither Suresh nor Priya does not have exam on Friday.
Only two people have exam between Divya and Deepa. Divya does not have exam
on any of the days after Ramya.

1). Which of the following is correctly matched?


a) John – Monday
b) Kumar – Tuesday
c) Rakesh – Friday
d) Priya – Tuesday
e) Suresh – Wednesday
2). How many persons have exam at 01.30 P.M between Priya and Kumar?
a) 5
b) 6
c) 2
d) 4
e) 1

3). Who among the following have exam on Friday?


a) Anjali, Suresh
b) Ramya, Deepa
c) Naveen, Rakesh
d) John, Suresh
e) Suresh, Kumar

4). Who among the following person has exam at 01.30 P.M?
a) Kumar
b) Priya
c) Anjali
d) Divya
e) Suresh

5). Four among the following form a group in a certain way. Which of the following
does not belong to Group?
a) Priya – Tuesday
b) Anjali – Wednesday
c) Divya – Friday
d) Priya – Wednesday
e) Suresh – Thursday
Direction (01-05):
Students
Days
Morning shift Noon shift
Monday Priya Naveen
Tuesday Anjali John
Wednesday Suresh Kumar
Thursday Divya Rakesh
Friday Ramya Deepa

PUZZLE-131
Directions (Q.1 -5): Study the following information carefully and answer the given
questions.
9 friends Ajay, Bhisup, Civesh, Dinesh, Evesh, Fervesh, Gukan, Harish and Ismail were
seated around a circular table facing the centre. They were playing the game of cards
and everyone was holding a single card bearing a number from 2-10 not necessarily in
the same order. It was further known that:

 Evesh and Bhisup had cards whose product was equal to the product of Dinesh and
Civesh 's cards.
 Dinesh and Civesh had even numbered cards.
 B had a number that was twice of Civesh 's card number. Bhisup and Civesh sit
together.
 The person with the lowest card number was 2nd to the left of Harish
 Dinesh sat 4th to the left of Civesh and the sum of their card numbers was equal to
the card number of Fervesh
 Gukan got a smaller card number than Evesh
 3 persons were seated between the persons having the card numbers 7 and 5 when
counted in a clockwise manner from 5
 Ismail was seated 3rd to the left of Bhisup. Ismail had a card number greater than
Bhisup
 Harish was to the immediate right of Fervesh.
1). What is the position of Ajay with respect to the one have card number 6?
a) Immediate left
b) Immediate right
c) Second to the right
d) Second to the left
e) Third to the right
2). How many persons had a card number greater than the card number of the one who
to the immediate right of Evesh?
a) None
b) One
c) Two
d) Three
e) More than three
3). What is he difference between Ajay and Dinesh’s card numbers?
a) 1
b) 2
c) 3
d) 4
e) 5
4). In which of the following groups is the 3rd person seated exactly in between the
1st and the 2ndpersons?
a) Bhisup Civesh Ajay
b) Ajay Dinesh Evesh
c) Dinesh Harish Evesh
d) G Civesh Bhisup
e) Bhisup Gukan Farvesh
5). What card number does the person has who is to the immediate left of Ismail?
a) 3
b) 5
c) 9
d) 6
e) 8

PUZZLE-132
Directions (Q.6-10): Study the following information carefully and answer the given
questions.
Twelve people are sitting in two parallel rows. In row 1, Avikshit, Avinash, Ayush,
Atulya, Avanish and Atmaja are sitting facing towards south. In row 2, Aadesh, Aabheer,
Aafreen, Aakash, Aashish and Abhilash are sitting facing towards north. They are sitting
in a manner that each person sitting in row 1 faces exactly one person sitting in row 2.
Each of these twelve persons except two has a different favorite colour.

 Aabheer likes pink and is sitting opposite to the one who is second to the left of
Ayush who likes Grey.
 Neither Aabheer nor Ayush sits at any of the extreme end.
 Aakash and the one who likes Green sit at extreme end of the row, but they do not
face each other.
 Only three people sit between Aafreen and the person who likes Blue.
 Aafreen faces Avanish, who likes Red.
 Atmaja sits second to the right of Atulya, who likes Brown.
 Four people sitting at extreme end like white, green, blue and purple color.
 Aadesh likes black and is sitting third to the left of Aafreen
 Avinash does not face Aabheer while Aashish faces Atulya
 Aakash is a neighbour of Aadesh while Abhilash is not
 There are three people between the two who like green and grey
 The one who likes yellow is sitting immediate left of the person who likes purple
6). Who is sitting second to the right of the one who faces Atulya?
a) Aadesh
b) Aabheer
c) Aafreen
d) Aakash
e) Aashish

7). Find the odd one out?


a) Atmaja
b) Avinash
c) Aakash
d) Abhilash
e) Atulya

8). Who is sitting between Atulya and the person who likes Red color?
a) Avikshit
b) Avinash
c) Ayush Atulya
d) Avanish
e) None

9). What is the favourite color of Aashish?


a) Blue
b) Black
c) White
d) Grey
e) None

10). Which of the following combination between person and color is incorrect?
a) Aadesh-Black
b) Atulya -Brown
c) Ayush -Grey
d) Avanish-Red
e) All are correct

Answer Key:
Solution(1-5):

1) Answer: e)
2) Answer: b)
3) Answer: a)
4) Answer: d)
5) Answer: a)

Solution(6-10):
6) Answer: c)
7) Answer: e)
8) Answer: a)
9) Answer: e)
10) Answer: e)

PUZZLE-133
Directions (1-5): Study the following information carefully and answer the
questions given below:
Farmer A of Nirman Nagar goes to trade fair with their family after completing their
daily work. There are nine members in the family after including farmer, i.e. A, B, C,
D, E, F, G, H and I. Each member eats different food items viz. Chocolate, Pizza,
Burger, Rasmalai, Samosa, Jalebi, Khulfi, Dosa and Idli. The relation of farmer with
the family members is defined as Father, Mother, Brother, Sister, Wife, Daughter,
Son, and Brother-in-law but not in the same order. All the family members are
enjoying circular ride, but not necessarily in the same order. Some of them are facing
towards centre while others are facing outward.
I is brother of A’s daughter. A’s brother is seated on the immediate right of the
brother of A’s sister and both face the same direction. I’s father eats Pizza. H has a
sister. The mother of A’s son is E and sits second to the left of the married son of A’s
father. F’s daughter-in-law eats Idli. A faces outward. F is a male. A’s father sits third
to the right A’s daughter. Either I or H sits third to the left of the mother of C, but
both I and H face the same direction. A’s brother and sister eats Burger and Chocolate
respectively. G is not father of H. In two generations, the members of the same
generation are sitting immediate next to each other and face the same direction. A’s
father does not eat Dosa and Khulfi. F’s grandson eats Rasmalai. B is a female and of
the same generation as of A. G sits on the immediate left of the granddaughter of A’s
father. The one, who is the wife of F likes Kulfi . A’s brother-in-law neither eats Dosa
nor Samosa. A’s father doesn’t sit immediate next to his children. G is facing the
centre and is second to the right of A. B sits second to the left of F. No three
members seated together can face the same direction.
Q1. How is B’s husband related to A’s father?
(a) Brother
(b) Father
(c) Son
(d) Grandson
(e) Son-in-law
Q2. Who sits third to the right of A’s mother?
(a) B
(b) H’s brother-in-law
(c) A’s brother-in-law
(d) E’s husband
(e) Both (b) and (c)

Q3. Who among the following sits exactly between A and his wife?
(a) F
(b) A’s sister
(c) A’s daughter
(d) D
(e) None of these

Q4. Four of the following five are alike in a certain way and hence they form a
group. Which one of the following does not belong to that group?
(a) C
(b) F
(c) H
(d) I
(e) A

Q5. Which of the following persons faces inside?


(a) B, G, F and I.
(b) A, G, D and I.
(c) G, D, A’s father and G’s wife.
(d) D, G, B and C.
(e) None of these
PUZZLE-134
Directions(6-10): Answer the questions on the basis of the information given
below.
Six kids– Z, X, C, V, B and N- watch in different Cartoon serials on their TV viz.
Ducktails, Doremon, Shinchain, Baloo, Chota beem and Hanuman – and they are lives
on six different floors of a building, numbered 1 to 6. They like six different Cartoon
Characters viz. Dholu, Sizuka, Munni, Lui, Jiyan and Pumba, in no particular order.
Each Cartoon serial is cast on a different day from Monday to Saturday. X likes Jiyan
and Cartoon serial likes by him is Doremon and lives on the 6th floor of the building. B
and Z lives on even numbered floors who’s cartoon cast on Saturday and Monday. The
one who watch Ducktails is lives on the lowest floor and it is cast on Tuesday. Baloo
Cartoon serial watch by that kid who lives on the floor next to X and it is cast on
Thursday. The kid who watches Hanuman likes Pumba and his cartoon is cast on
Monday. N likes Lui and he lives on first floor. One of the kid who watches Cartoon
serials which cast on friday lives on even numbered floors. V lives two floors below C
and he likes Munni. B, who does not live immediately above or below of C, likes
Dholu and his serial cast on Saturday. The kid who likes Chota beem is lives on an
even numbered floor.

Q6. Who is watching Ducktails cartoon serial?


(a) Z
(b) N
(c) V
(d) X
(e) C

Q7. Which Character does B like?


(a) Pumba
(b) Jiyan
(c) Dholu
(d) Lui
(e) Sizuka

Q8. Who among the following watch that cartoon which is cast on Thursday?
(a) V
(b) N
(c) Z
(d) X
(e) C
Q9. If V is related to C and B is related to Z, in the same way N is related to?
(a) X
(b) N
(c) Z
(d) V
(e) C

Q10. Which of the following combination is correct?


(a) V – Shinchain – Sizuka
(b) C – Baloo – Munni
(c) Z – Chota beem – Pumba
(d) X – Doremon – Dholu
(e) N – Ducktails – Lui

PUZZLE-135
Directions (11-15): Study the following information carefully and answer the
questions.
There are seven friends – S, P, M, A, D, K and C. All of them are working in Army and
they like different type of machine gun like Anti-aircraft machine gun(AA), machine
gun(MG), light Machine gun(LMG), medium machine gun (MMG), heavy machine
gun(HMG), general-purpose machine gun (GPMG), and squad automatic machine
gun(SAMG). They go to temple one by one in the same week starting from Monday but
not necessarily in the same order.
S goes to temple on the third day of the week and he does not like LMG and HMG.
Only one friend goes to temple between S and P, who likes AA. There are two friends
go to temple between the days on which P and M, who likes MG. The one who likes
SAMG goes to temple on the fourth day of the week. Only one friend goes to temple
between the A and one who likes SAMG. The one who likes GPMG goes to temple on
last day of the week. The one who likes HMG goes to temple neither immediately
above nor below the days on which S goes to temple. There are as many as friend go
to temple between the D and one who likes LMG and as between the D and C. K does
not go to the temple last day of the week.

Q11. Who among following goes to temple on Monday?


(a) P (b) D (c) S (d)M (e) K

Q12. S likes which of the following machine gun?


(a) GPMG
(b) SAMG
(c) LMG
(d)MMG
(e)None of these

Q13. S goes to temple on which of the following day?


(a) Friday
(b) Tuesday
(c) Monday
(d) Wednesday
(e) Sunday

Q14. Who among following likes HMG?


(a) K
(b) M
(c) C
(d) A
(e) P

Q15. K goes to temple on which of the following day?


(a) Monday
(b) Tuesday
(c) Wednesday
(d) Thursday
(e)None of these
PUZZLE-136
Directions (11-15): Study the following information carefully and answer the
questions.
There are seven students – M, N, O, P, Q, R and S – whose favourite subjects are-
Hindi, English, Math, Science, Physics, Chemistry and Biology but not necessarily in
the same order. They go to the library on different days of the week starting from
Monday. Also, they like different colours, viz- Red, Green, Black, White, Yellow,
Violet and Orange but not necessarily in the same order.
N, who likes Violet, goes to library fourth day of the week but his favourite subject is
neither Hindi nor Biology. Two students go to the library between the days on which N
and R go to the library and neither of them goes to library 1st day of the week. There
is one student goes the library between M and O. But O does not go either on 1st or
3rd day of the week. O likes Green colour and his favourite subject is Hindi. The one
whose favourite subject is Math goes on the last day of the week and like the Black
colour. P does not go to the library on the day either immediately before or
immediately after the day on which M goes to the library, who doesn’t like either
Yellow or Red or White colour.
S goes to the library immediately after O and he likes Red colour. Q does not like the
Yellow colour and his favourite subject is Chemistry. M’s favourite subject is neither
Physics nor Science. The one whose favourite subject is English goes to the library
immediately after the day when the student whose favourite subject Hindi goes.

Q11. Which of the following combinations is definitely false?


(a)O – Green – Hindi
(b)M – Orange –Biology
(c)Q – White –Chemistry
(d)R – Green – Hindi
(e)None of these
Q12.Who among the following goes to the library on the 5th day of the week?
(a)The student who likes Orange colour
(b)The student whose favourite subject is Math
(c)The student whose favourite subject is Hindi
(d)The student who likes White colour
(e)None of these

Q13.Whose favourite subject is Science?


(a)The student who likes Violet colour
(b)The student who likes Orange colour
(c)The student who likes Yellow colour
(d)Either the student who likes Yellow colour or the student who likes Violet colour
(e)None of these

Q14.If ‘N’ is related to ‘Green’ and ‘S’ is related to ‘Black’ then which of the
following is ‘M’ related to?
(a)Yellow
(b)Violet
(c)White
(d)Orange
(e)None of these

Q15.Whose favourite subject is Biology?


(a)The student who goes to library fourth day of the week
(b)The student who goes to library before Q
(c)The student who goes to library just after Q
(d)Can’t be determined
(e)None of these
PUZZLE-137
Directions (1-5): Study the following information given below and answer the
given questions.
Eight friends, viz A, B, C, D, E, F, G and H are sitting in a straight line but not
necessarily in the same order and all of them are facing north direction. Each of them
has belong to diierent cities, viz Varanasi, Patna, Agra, Delhi, Noida ,Puri, Puna and
Chandigarh but not necessarily in the same order. G sits third to the right of the
person who belongs to Noida. The person who belongs to Chandigarh sit second to the
right of G. A and E are immediate neighbours of each other. Neither A nor E belong to
Noida or Chandigarh. Neither A nor E is an immediate neighbour of G. H sits third to
the right of the person who belongs to Varanasi. Neither A nor E belongs to Varanasi.
H does not belong to Chandigarh. Only two people sit between E and the person who
belongs to Puna. The person who belongs to Patna sit on the immediate left of D. Only
one person sits between E and B. C sits second to left of one who belongs to Puna. E
does not belong to Agra. The one who belongs to Puri sits at extreme end of the line.

Q1. Who among following sits on the corner?


(a) A,H
(b) D,G
(c) A,G
(d) D,C
(e) None of these

Q2.How many persons sits between E and H?


(a) two
(b) three
(c) two
(d) more than three
(e) None of these

Q3. C belongs to which of the following city ?


(a) Delhi
(b) Noida
(c) Varanasi
(d) Puna
(e) None of these

Q4. Who among following sits fourth to left of H?


(a) E (b) C (c) B (d) F(e) None of these
Q5. If E is related to Varanasi and B is related to Agra, in the same way F is related
to?
(a) Puma
(b) Agra
(c) Patna
(d) Chandigarh
(e) More than three

PUZZLE-139
Directions(11-15): Read the following information carefully and answer the
questions which follow.
There are seven friends named P, Q, R, S, T, U, V from different cities Agra, Noida,
Patna, Delhi, Varanasi, Jaipur and Puri. They are wearing shoes of different brands
like Nike, Lotto, Lancer, Sparx, Puma, Adidas and Reebok. They like different colours
like Orange, Red, Yellow, Blue, Amber, Indigo, and Pink. T is wearing Sparx shoes and
likes Indigo colour. The one who is wearing Lancer shoes does not like Blue and Pink
colour. U does not wear Reebok and Lotto shoes and likes Blue colour. P does not like
pink or yellow colour. S is from Delhi wears Adidas shoes. Q is from Patna and does
not wear Nike shoes. V wears either Puma or Lotto shoes. Q does not wear Lotto
shoes. The one who likes yellow colour does not belong to either Noida or Delhi.
Person from Jaipur is like Indigo colour. The one who belongs to Varanasi likes Pink
colour and he is wearing Reebok shoes. The person wearing Nike shoes is belong to
either Puri or Jaipur. S likes Orange colour . The one who is wearing Lotto shoes likes
Amber colour. U does not belong to Puri.

Q11. U belongs to which of the following city?


(a) Jaipur
(b) Noida
(c) Agra
(d) Either (a) or (b)
(e) Either(b) or (c)
Q12. P likes which of the following colour?
(a) Red
(b) Amber
(c) Indigo
(d) Blue
(e) None of these

Q13. Who among following is wearing Lancer shoes?


(a) U
(b) R
(c) S
(d) Q
(e) None of these

Q14. Who among following belongs to Puri?


(a) P
(b) R
(c) Q
(d) V
(e) None of these

Q15. Which among the following combination is correct?


(a) R- Pink- Varanasi
(b) U- Yellow- Lancer
(c) P- Indigo- Jaipur
(d) V- Delhi- Lotto
(e) None of these
PUZZLE-140
Directions (1-5): Study the following information carefully and answer the given
questions.
Ten persons are sitting in two parallel rows containing five persons in each. In row 1,
A, B, C, D and E are sitting and all of them are facing south. In row 2, P, Q, R, S and T
are sitting and all of them are facing north. In the given seating arrangement, each
member seated in a row faces another member of the other row but not necessarily in
the same order. All of them like different colours viz. Red, Yellow, Violet, Green,
Black, Brown, White, Pink, Blue, and Orange but not necessarily in the same order.
There are two persons sits between Q and P, who likes violet. Immediate neighbour Q
faces D, who likes blue. The one who likes red sits third to the left of E, one of them
sits to the extreme end of the row. B is not an immediate neighbour of E. There is
only one person sits between T and Q, who likes orange. The immediate neighbour of
E faces the person who likes pink colour. One of the immediate neighbour of E likes
yellow colour. A is not an immediate neighbour of D and he likes white colour. The
immediate neighbour of R faces the person who likes brown colour. The immediate
neighbour of E faces T, who likes green.

Q1. S faces which of the following person?


(a) A
(b) B
(c) C
(d) D
(e) E

Q2. Who among following sits to the extreme end of the row ?
(a) A,S
(b) B,P
(c) E,Q
(d) D,R
(e) A,Q

Q3. Who among following sits second to the right of P?


(a) T
(b) The one who likes orange colour
(c) The one who likes violet colour
(d) The one who likes black colour
(e) Q
Q4. Who among following sits diagonally opposite to R?
(a) C
(b) The one who likes red colour.
(c) The one who likes blue colour.
(d) E
(e) D

Q5. Who among following sits third to right of B?


(a) A
(b) E
(c) C
(d) D
(e) None of these

PUZZLE-141
Directions (11-15): Study the following information carefully and answer the given
questions.
There are nine teachers A, B, C, D, E, F, G, H, and I of different subjects like Hindi,
English, Science, Sanskrit, Computer, Math, Art, Commerce and History but not
necessarily in the same order. They all are living on a different floor. The ground floor
is numbered one and so on up to top floor is numbered nine. Only three teachers live
between I and one who teaches History, both are living on odd numbered floor but
none of them is living either top floor or bottom floor. F and G do not like Science.
Only one teacher lives between I and A, who teaches commerce. There is only one
teacher lives between H and C, who teaches Computer. D teaches Hindi and lives on
the even numbered floor. There is only one teacher lives between the floor on which
History’s teacher and English teacher lives. B teaches Art and lives immediately below
on the floor on which A lives. E does not live immediately above or below on the
floor on which C lives. E does not live on first floor. There is as many as teacher lives
between E and one who teaches English and between G and one who teaches Sanskrit.
Only two teachers live between I and H, who teaches Math. The teacher who teaches
science lives on one of the floors above the floor on which teacher who teaches
Sanskrit lives.

Q11. F teaches which of the following subject?


(a) History
(b) Math
(c) Computer
(d) English
(e) None of these

Q12. Who among following teaches Science?


(a) A
(b) D
(c) E
(d) I
(e) B

Q13. Who among following lives on top floor?


(a) B
(b) E
(c) F
(d) G
(e) A

Q14. Who among following lives between the floor on which I and E lives?
(a) A
(b) C
(c) D
(d) G
(e) B

Q15. If A is related to Science and H is related to Computer, in the same way B is


related to ?
(a) Sanskrit
(b) Math
(c) Hindi
(d) History
(e) Computer
PUZZLE-142
Directions (1-5): Study the information given below and answer the given
questions.
Nine people A, B, C, D, E, F, G, H and I stay in a building. The building has nine
floors and only one person stays on one floor. Each of them have a puppy whose
names are- Tiger, Rony, Panther, Rocky, Jumbo, Sheru, Gabru, Bullet and
Cheetah. Each person belongs to different cities, i.e. London, Bangkok, Paris,
Dubai, Istanbul, New York, Singapore, Seoul and Rome, but not necessarily in the
same order. The ground floor is numbered 1, the floor above it is numbered 2,
and so on, and the topmost floor is numbered 9.
The one who belongs to Seoul stays on the fourth floor. A does not belong to New
York and does not have puppy naming Sheru and Bullet. There are three floors
between the floors on which C and G stay. D stays on a floor immediately above
the I’s floor. The one who has puppy named as Gabru stays on an even numbered
floor. F does not belong to Rome .The one who belongs to London stays on the
topmost floor. The one who has puppy named as Rocky stays immediate below to
the one who has puppy named as Tiger. F has puppy named as Sheru and does not
stay on the ground floor. H belongs to Dubai and stays on an even-numbered floor
and he has puppy named as Rocky. E stays on the second floor and belongs to
Paris. There are three people between the one who has puppy named as Rony and
the one who has puppy named as Bullet. The person who has puppy named as
Rony stays below the person who has puppy named as Bullet. The one who belongs
to Rome stays on the third floor. The one who has puppy named as Jumbo does
not stay on sixth floor. There are two floors between the floors on which the
people who are from Singapore and Dubai stay. The person who has puppy named
as Bullet is from Singapore. C belongs to Bangkok. The one who has puppy named
as Cheetah stays immediate above C. There is one floor between the floors on
which F and G stay. There is one floor between the floors on which the one who
has puppy named as Panther and the one who has puppy named as Cheetah stay. A
stays on an even-numbered floor below the floor on which H stays.

Q1. How many floors are there between the one who has puppy named as Gabru
and the one who has puppy named as Jumbo?
(a) Three
(b) Five
(c) Two
(d) Six
(e) Four

Q2. If B related to Istanbul in the same way as F related to Panther. Which of the
following is D related to, following the same pattern?
(a) Sheru
(b) Lucknow
(c) Bangkok
(d) Dubai
(e) Jumbo

Q3. Which of the following combination is correct for A?


(a) Panther-Seoul
(b) Rony-Bangkok
(c) Gabru-New York
(d) Sheru-Istanbul
(e) Gabru-Istanbul

Q4. Four of the following five are alike in a certain way and hence they form a
group. Which one of the following does not belong to that group?
(a) Bangkok
(b) Bullet
(c) Sheru
(d) Dubai
(e) Tiger
Q5. If all puppies are arranged in alphabetical order from top to lowest floor. Then
the person who stays on fifth floor has which puppy?
(a) Rony
(b) Panther
(c) Jumbo
(d) Rocky
(e) Tiger

PUZZLE-143
Directions (1-5): Study the information and answer the given questions:
Eight persons S, T, U, V, W, X, Y, and Z lives on eight storey building with ground floor
being numbered one and the next floor is number 2 and so on. They have their birth
date on August, May, September, November, July, December, June, and October but
not necessarily in same order.
S lives on even numbered floor above fourth floor. Two people lives between the floors
on which S lives and the one who was born in July. Four people lives between the floors
of U and X. U lives on even numbered floor above X. The person who was born in May
lives on odd numbered floor above the one who born in July and below the floor upon
which S lives. Number of people living between the one who born in July and S, is two
more than the number of persons between the one who was born in May and U. T was
not born in May. As many people live between the one who born in June and October
same as between the one who born in May and person T. The person who was born in
June lives on one of the floor above the one who born in October and lives on even
number floor below 8th floor but not on 4th floor.
Y lives on odd floor immediately above V. Four people lives between the one, who born
in December and November. The person who born in December lives on one of the
floors above the person who born in November. T lives immediately above Z. V was not
born in August. The number of person born between U and the one born in October is
less than 3.

Q1. S lives on which of the following floor ?


(a) First
(b) Fifth
(c) Seventh
(d) Eight
(e) None of these

Q2. T borns in which of the following month?


(a) June
(b) July
(c) October
(d) November
(e) None of these

Q3.How many floor between Z and S?


(a) Three
(b) Four
(c) Two
(d) Five
(e) None of these

Q4.V borns in which of the following month?


(a) July
(b) October
(c) November
(d) September
(e) None of these

Q5. In some way S is related to W and Y is related to V, in the same way T is


related to?
(a) Z (b) Y (c) V (d) X (e) None of these
PUZZLE-144
Directions (1-5): Study the information and answer the given questions:
Eight persons A, B, C, D E, F , G and H all of them faces north direction but not necessarily in
same order. All of them stay in different floors viz. 2nd, 6th, 10th, 15th, 20th, 30th, 31st, and
37th but not necessarily in same order. D sits fifth to the left of the persons who stays
on 20th floor. The person on the 20th floor does not sit at extreme ends. There are two persons
sitting between the one who stays on 20th floor and F. The person staying on 6th floor sits
second to the right of G. G is not an immediate neighbor of D. G does not sit
on 20th floor. Difference between the numerical values for floor numbers of immediate
neighbours of G is 10. The person staying on 10th floor is not an immediate neighbor of F. Two
persons sitting between A and the person staying on 10th floor. Neither F nor G is staying
on 10th floor. More than one person sitting between B and the person staying on 2nd floor. The
floors on which C and H are staying are higher than the floor upon which A staying. C is not an
immediate neighbor of D. One person is sitting between B and the person staying on 31st floor.

Q1. H lives on which floor?


(a) 10th
(b) 37th
(c) 2nd
(d) 30th
(e) None of these

Q2. How many persons sits between A and E?


(a) two
(b) One
(c) three
(d) four
(e) None of these

Q3. C lives on which of the following floor?


(a) 15th
(b) 20th
(c) 6th
(d) 31th
(e) None of these

Q4. Who among following sits immediate left of the person one who lives on 15th floor?
(a) A (b) F (c) D (d) B (e) None of these
Q5. Who among following sits third to right of F?
(a) D
(b) C
(c) A
(d) B
(e) None of these

PUZZLE-145
Directions (6-10): Study the following information and answer the given questions:
Seven teachers I, J, K, L, M, N and O live on separate floors of a 7-floor building.
Ground floor is numbered 1, first floor is numbered 2 and so on. Each of them is
teaching different subjects, viz Hindi, English, Math, Science, Civics, History and
Sanskrit but not necessarily in the same order. Only three teachers live above the
floor on which I lives. Only one teacher lives between I and the one who teaches
History. N lives immediately below the one who teaches English. The one who teaches
English lives on an even-numbered floor. Only three teachers live between the
teachers who teach History and Math. M lives immediately above K. M does not teach
Math. Only two people live between J and the one who teaches Civics. The one who
teaches Civics lives below the floor on which J lives. The one teaches Hindi does not
live immediately above or immediately below J. L does not live immediately above or
immediately below I. O does not teach Science.
Q6. Which of the following is true with respect to O as per the given information?
(a) The one who lives immediately below O ,teaches English.
(b) O lives on floor no. 7
(c) O lives immediately below M.
(d) O lives on the lowermost floor.
(e) O teaches History.

Q7. Who among the following lives on floor no. 3?


(a) The one who teaches Science
(b) The one who teaches Civics
(c) K
(d) O
(e) M
Q8. Who lives on the floor immediately above M?
(a) I
(b) J
(c) L
(d) O
(e) N

Q9. L teaches which of the following subject?


(a) English
(b) History
(c) Math
(d) Civics
(e) Science

Q10. How many teachers live between the floors on which L and the one who
teaches English live?
(a) None
(b) Two
(c) One
(d) More than three
(e) Three
PUZZLE-146
Directions (6-10): Study the following information carefully and answer the given
questions.
A, B, C, D, E, F, G and H are sitting around a square table in such a way that four of
them sit at four corners of the square while four sit in the middle of each of the four
sides. The one who sits at the four corners face the centre of the table while those
who sit in the middle of the sides face outside. Each of them likes a different colour,
viz Red, Blue, Pink, Yellow, Violet, Orange, White and Black colour but not
necessarily in the same order. C sits third to the left of the one who likes Black
colour. The one who likes Black colour faces outside. Only two persons sit between C
and H. The one who likes Red colour sits on the immediate right of H. The one who
likes Violet colour sits second to the right of G. G is neither an immediate neighbour
of H nor of C. G does not like Black colour. Only one person sits between A and the
one who likes Violet colour. D sits on the immediate left of the one who likes Orange
colour. G does not like Orange colour. E likes White colour. E is not an immediate
neighbour of A. The one who likes Blue colour is an immediate neighbour of E. The
one who likes Yellow colour is an immediate neighbour of F.

Q6. What is the position of the one who likes Orange colour with respect to G?
(a) Second to the left
(b) Third to the right
(c) Fourth to the left
(d) Second to the right
(e) Third to the left

Q7. Who among the following represent the immediate neighbours of the one who
likes Violet colour?
(a) B, F
(b) C, E
(c) B, E
(d) D, F
(e) F, H

Q8. Who among the following sits exactly between H and B?


(a) C
(b) The one who likes Blue colour
(c) The one who likes Pink colour
(d) G
(e) A
Q9. Which of the following is true regarding B?
(a) B is one of the immediate neighbours of D.
(b) The one who likes Black colour is an immediate neighbour of B.
(c) B sits second to the left of H.
(d) B likes White colour.
(e) B is an immediate neighbour of the one who likes Red colour.

Q10. Who amongst the following sits diagonally opposite the one who likes Red
colour?
(a) The one who likes Blue colour
(b) D
(c) A
(d) The one who likes Pink colour
(e) The one who likes Yellow colour

PUZZLE-147
Directions (1-5): Study the following information carefully and answer the
questions given below:
A, B, C, D, E, F, G, and H are eight persons and they like different colours viz. Red,
Blue, Pink, Black, White, Brown, Yellow, and Violet but not necessarily in the same
order. Some of them are sitting either facing to centre or opposite to centre. There
are two persons sitting between A and H, who likes Violet colour. G sits second to left
of A. G likes yellow colour and sits not adjacent to H. F likes brown colour and sits
third to left of A. F faces the same direction as A. There are three persons sitting
between F and B, who likes black colour. D likes white colour and sits not adjacent to
B. E likes pink colour and sits third to the right of D. A does not like the blue colour. A
faces D. Both the immediate neighbor A faces the same direction as A( Faces the
same direction means if A faces towards the centre, then both neighbor of A faces
towards the centre and vice-versa). The immediate neighbor of D faces opposite
direction ( Opposite direction means if one nighbour of D faces towards the centre
then other neighbor faces opposite to centre and vice-versa). C sits immediate right
of B, and both faces opposite to centre. The immediate neighbor of E faces opposite
direction(Opposite direction means if one nighbour of E faces towards the centre then
other neighbor faces opposite to centre and vice-versa).

Q1. C likes which of the following colour?


(a) Red
(b) Black
(c) Blue
(d) Violet
(e) None of these

Q2. Who sits third to the right of F?


(a) D
(b) H
(c) C
(d) A
(e) None of these

Q3. Who sits third to the left of B?


(a) G
(b) H
(c) A
(d) C
(e) None of these

Q4. Four of the following five are alike in a certain way based on their seating
positions and so form a group. Which of the following is different from the group?
(a) A
(b) C
(c) F
(d) D
(e) H
Q5. Which of the following statements is/are definitely false?
(a) E is the immediate neighbor of A
(b) G sits immediate right of F.
(c) D is the immediate neighbor of F.
(d) G sits immediate right of E.
(e) A likes red colour

PUZZLE-148
Directions (Q. 11-15): Study the following information and answer the questions
given below.
Eight professors A, B, C, D, E, F, G and H of different subjects viz. Science, Hindi,
English, Math, Civics, Biology, Chemistry and Physics not necessarily in same order are
sitting around a rectangular table. Four professor sits on each middle side of
rectangular table, while others four professors on the corner of table. All of them are
facing towards the center. D is sitting second to the right of the professor, who
teaches Physics. E is sitting third to the left of the professor, who teaches Biology.
Professor F and G are sitting opposite to each other. Professor C is sitting opposite the
professor, who teaches Chemistry. Professor B is sitting opposite the professor, who
teaches Science. The professor who teaches Math is sitting second to the right of the
professor, who teaches Civics and second to the left of A, who is not sitting near the
professor who teaches Chemistry. G is sitting on the smaller side and to the
immediate right of the professor who teaches Chemistry. The professor who teaches
Civics and Math are not on the same side of the table. The professor who teaches
Math is sitting third to the right of the professor who teaches Physics, who is not
sitting opposite to the professor who teaches Hindi. The professor who teaches Math
is third to the left of professor C. F sits second to the left of professor H. The
professor who teaches Hindi sits opposite to the professor who teaches Civics.
11. Who is sitting third to the right of the professor who teaches Biology?
(a)The professor who teaches Civics
(b)A
(c) The professor who teaches Chemistry
(d)None of these
(e) Can’t determine

12. Who is sitting opposite to D?


(a) A
(b) C
(c) The professor who teaches Physics
(d) The professor who teaches Hindi
(e)Can't be determined

13. Who among following is sitting third to left of H?


(a) E
(b) D
(c) C
(d) B
(e) A

14. A teaches which of the following subject?


(a) English
(b) Hindi
(c) Science
(d) Math
(e) Civics

15. Biology is taught by which of the following professor?


(a) B
(b) C
(c) D
(d) E
(e) F
PUZZLE-149

Directions (1-5): Study the following information carefully to answer the given
questions.
Seven cricket players viz. P, Q, R, S, T, V and W played in seven different matches in
a week starting from Monday and ending on Sunday. Some of them are bowlers and
some of them are batsman. Their Run/Wickets in the seven matches are – 50, 5, 4,
16, 8, 4 and 25. No bowler can take more than 10 wickets.
V plays on Thursday. Two matches are played between the matches played by player
V and player P. On the first day of the week, the batsman scored half century played
in the match.
The sum of Runs/Wickets of the player W and Q is equaled to the Runs/Wickets of S.
All the bowlers should be preceded and succeeded by batsman and T is the only
bowler which cannot be succeeded by any one. P is the only batsman which is
followed by another batsman.
Only one match is played between the matches played by the player T and S. The
score of the batsman who played the match on Thursday is the perfect square of the
score of the player who played on Saturday.
The match played by T is not played on the day immediately before or immediately
after the day when the match of player V is played. The match played by R is played
on the day immediately before the day when the match of player W is played. Match
played by S is not played after the match of player Q.

Q1. As per the given arrangement which of the following combination represents
only the people who are bowlers?.
(a) T, V
(b) Q, P, T
(c) V, W, R
(d) P, T
(e) W, T, S
Q2. As per the given arrangement which of the following person represent the one
who was played in between the W and S?
(a) U
(b) P
(c) R
(d) V
(e) W

Q3. Who among the following takes 8 wicket ?


(a) T
(b) W
(c) S
(d) Q
(e) None of these

Q4. Which of the following combinations is correct as per the given arrangement?
(a) Monday – P
(b) Tuesday – W
(c) Wednesday – Q
(d) Thursday – S
(e) Friday – T

Q5. Who amongst the following scored half century?


(a) P
(b) V
(c) U
(d) W
(e) T
PUZZLE-150
Directions (Q. 11-15): Study the following information carefully and answer the
given questions:
Sadhna, Saraswati, Savita, Sima, Sarita, Sonam, Shivi and Siddhi are eight girls sitting
around a circular table. Four of them are facing towards the centre and four of them
are facing away from the centre. All of them like a different subject viz. Hindi,
English, Sanskrit, Math, Physics, Chemistry, Economics and Commerce, but not
necessarily in the same order.
Sarita faces towards the centre and likes Hindi. Both the immediate neighbours of
Sarita face away from the centre and like Sanskrit or Math. Sima faces away from the
centre. Both the immediate neighbours of Sima do not face away from the centre.
Sarita sits third to the right of Sonam, who likes Physics and faces away from the
centre. Savita sits third to the left of Sonam.
The one who likes Sanskrit sits opposits of Sonam. The one who likes English is not
the immediate neighbour of Sonam and faces away from the centre. Sadhna sits
second to the left of Savita and does not like Economics or Commerce. The one who
likes Economics sits between Siddhi and Sonam. Saraswati faces away from the centre
and does not like English.

Q11. Who sits third to right of that girl , who sits opposite to Siddhi?
(a) Shivi
(b) Sonam
(c) Sarita
(d) Saraswati
(e)None of these

Q12. Who among following is sits second to right of Sima?


(a)Saraswati
(b) Sonam
(c) Shivi
(d) Siddhi
(e)None of these

Q13. Savita likes which of the following subjects?


(a) English
(b) Chemistry
(c) Commerce
(d) Economics
(e)None of these
Q14.Sima likes which of the following subject?
(a) Chemistry
(b) English
(c) Economics
(d) Math
(e)None of these

Q15.Who among following sits third to left of Shivi?


(a) Saraswati
(b) Sadhna
(c) Sima
(d) Savita
(e)None of these

PUZZLE-151
Directions (1 – 5): Study the following information carefully and answer the
question given below:
Ten persons A, B, C, D, E, F, G, H, I and J are sitting around a rectangular table and
likes a different colours i.e. Red, Blue, White, Black, Green, Yellow, Brown, Orange,
Violet and Pink but not necessarily in the same order. All of them are sitting at a
rectangular table in such a way that four of them sit at the corners, two each on the
longer sides and one each on the smaller sides, but not necessarily in the same order.
Some of them are facing the centre while the rest are not facing the center. Not more
than two friends sitting together face the same direction.
E sits on the immediate left of D and is not an immediate neighbor of C. A and E face
in the same direction. The one who likes Red colour sits immediate left of the one
who likes Yellow colour. D and G sit diagonally and face the opposite directions. Five
of them face in the same directions. H does not like Yellow and Violet colour. The one
who likes Orange colour sits immediate right of I, who likes Violet colour. The one
who likes Yellow colour sits second to the left of C. Only two among four sitting on
the corners face outward. H and D are sitting on the immediate left and third to the
left of J respectively. A sits on one of the smaller sides and third to the right of F. D
likes Brown colour and sits third to the left of the one who likes Green colour. J sits
immediate right of the one who likes Pink colour. E likes Blue colour and sits second
to the left of the one who likes Black colour. I is not an immediate neighbour of A, B
or F, but sits on the immediate right of C, who is not facing the centre. One of the
four friends sitting on the corner is I. I faces to the centre.

1 Who among following is sits third to right of B?


(a) The one who likes brown colour
(b) E
(c) The one who likes black colour
(d) I
(e) None of these
2. What is the position of H with respect to A?
(a) Immediate right
(b) Second to the right
(c) Fourth to the right
(d) Can’t be determined
(e) None of these

3. A likes which of the following colour?


(a) Green
(b) Pink
(c) Yellow
(d) Red
(e) None of these

4. Who among the following persons like White colour?


(a)A
(b) C
(c) H
(d) J
(e) None of these
5. Who sits third to the left of the one who likes Yellow colour?
(a)The one who likes Orange colour
(b) The one who likes Violet colour
(c) B
(d) F
(e) The one who likes Blue colour

PUZZLE-152
There are 16 persons – B,C,D,E,F,G,H,I,P,Q,R,S,T,U,V and W standing in a square plot. Inside a Square
plot, a square shaped garden is developed. The persons who are standing inside the garden facing
outside. The persons who are standing outside the garden facing inside the centre and likes colours
namely viz., – Red, Blue, Black, Brown, Yellow, Green, Violate and Pink. So all the persons standing in the
inner square faces the persons standing in the outer Square and likes fruits namely viz., – Apple, Orange,
Mango, Grapes, Papaya, Pomegranate, Guava and Banana. G faces the centre and W faces G. D sits
second to the right of G. There are four persons sits between G and E. D is not an immediate neighbour
of E. There are three persons standing between I and E. There are two persons standing between I and
B. B stands exactly between the E and F. F stands to the immediate left of G. There are two persons
standing between W and U. U faces H. T faces outside. There are two persons standing between T and
Q. T faces C. Q stands to the immediate left of W. R, the one who faces B stands exactly between the
persons P and V. P faces E. The one who sits in the corner of the square likes Red. The one who likes Red
sits between the persons who like Black and Blue. The one who likes Blue sits second to the right of the
person who likes Green. Three persons sit between one who likes Black and one who likes Green. Two
persons sit between one who likes Black and one who likes Yellow. Two persons sit between one who
likes Yellow and one who likes Pink. G and F do not like Violate and Yellow respectively. The one who
likes Red faces P. The immediate neighbours of P are the one who likes apple and the one who likes
Grapes. The one who likes Apple faces the one who likes Black. Three persons sit between the one who
likes apple and the one who likes Guava. The immediate neighbours of the person who likes Orange are
the one who likes apple and the one who likes Pomegranate. The one who likes Papaya sits exactly
behind to the one who likes Orange. The one who likes Banana sits exactly behind to the one who likes
Mango. The one who likes Banana faces E.

PUZZLE-153
Directions (1-5): Read the following information carefully and answer the questions given
below.
Five friends G, H, I, J and K of one colony A are playing chess with five other friends namely U,
V, W, X and Y of another colony B. Each friend of colony A has a different profession viz.
Doctor, Engineer, Businessman, Lawyer and Manager but not necessarily in the same order
.Each friend of colony B has also a different profession viz; Fashion designer, Choreographer,
Interior Decorator, Politician and Teacher. Among friends, there are only three females – the
one who is playing with W, the one whose profession is Doctor and the one who is playing with
the person who is a Politician. The female whose profession is Engineer is playing with the
person who is Interior Decorator.
The Person who is a Choreographer is playing with the person whose profession is Doctor .X is
neither a Choreographer nor a Fashion designer. Y is not an Interior Decorator. The person who
is playing with Y is a female but she is not J. K’s profession is Engineer and is not a male. G
and I are not females. G’s profession is Manager and is playing neither with Y nor with the
Choreographer. H’s profession is Lawyer. V is a Fashion designer and he is not playing with G.
The one who is a Fashion designer is not playing with J who is a doctor.
Q1. The one whose profession is Businessman is playing with who among the following
person?
(a)W
(b)Choreographer
(c)Y
(d)Fashion designer
(e)None of these

Q2. Which of the following pairs correctly represents the pair of friends who are playing
together?
(a)IW
(b)GX
(c)JY
(d)WH
(e)None of these

Q3. Who among the following is a Politician?


(a)Y
(b)X
(c)W
(d)U
(e)None of these
Q4. Which of the following represents the group of persons having female friend
who are playing with him?
(a)Y, W,U
(b)X, V, W
(c)R, P, T
(d)Y ,X, V
(e)None of these
Q5. Which of the following is the specialization of X?
(a)Teacher
(b)Politician
(c)Interior Decorator
(d)Fashion designer
(e)None of these
PUZZLE-154
There are eight friends A, B, C, D, E, F, G and H, who live in an eight-storey building. The
ground floor is numbered one and the topmost floor is numbered eight. Each of
them are working in different nationalize bank, viz BOI, BOB, Dena, UBI, CBI, IOB, PNB and
OBC, but not necessarily in the same order. There is only one floor between A and the floor on
which person that works in OBC. The person who works in OBC does not live on floor number
1. D lives just below B. The one who works in BOI live on even-numbered floor and just above
the floor on which person who works in CBI lives. The person who works in IOB lives on an
even-numbered floor but not on the 8th floor. Neither D nor H lives on the 1st floor. Only one
person lives between the one who works in PNB and D. A lives on an odd-numbered floor and E
lives just above A. B lives on the fourth floor. Only two friends live between the one who works
in IOB and A. F lives just below the one who works in CBI. D works neither in CBI nor OBC. The
person who works in Dena bank does not live on an odd-numbered floor. G does not work in
BOB. There are two floors between the floor on which H lives and the floor on which E lives.
Only two persons live between the one who works in UBI and the one who works in DENA
bank.

Q6. Who among the following works in BOB?


(a) D
(b) C
(c) F
(d) E
(e) None of these

Q7. How many persons are there between E and B?


(a) One
(b) Two
(c) Three
(d) Four
(e) None of these

Q8. Who among the following lives on the topmost floor?


(a) The one who works in BOI
(b) The one who works in IOB
(c) The one who works in DENA
(d) The one who works in BOB
(e) None of these

Q9. Which of the following combinations is/are true?


(a) Floor no. 2 – D – UBI
(b) Floor no. 5 – F – OBC
(c) Floor no. 1 – C – BOB
(d) Floor no. 8 – E – BOI
(e) None of these
Q10. A works in which of the following bank ?
(a) BOI
(b) OBC
(c) IOB
(d) CBI
(e) None of these

PUZZLE-155
Directions (1–5): Study the following informa¬tion carefully and answer the questions which
follow.
Eight delegates from different countries, Brunie, Cambodia,Malaysia, Myanmar, Vietnam, Thailand,
Philippines and India are participating in the ASEAN summit in laos. They are A, B, C, D , E, F, G and H
are sitting around a circular table facing to the centre but not necessary in the same order. They all are
the ministers belonging to different departments Defense, Tourism, Nuclear energy, External affairs,
Home affairs , Railway, Finance and Human resource development.
F is seated second to the right of H and they both do not belong to Finance department. G does not work
in tourism or railway department but sits 3rd to the left of the person who belong to Finance department.
C sits immediate left to the person who belongs to Tourism department and sits opposite to one who is
immediate right of H. A is on the 3rd to the left of D and opposite to one who belongs to Finance
department. The person who belongs to Home affairs department is not near to the person who belongs
to Tourism and Finance department. F is near to E. The person belongs to Home affairs department is
3rd to the left of the person who belongs to external affairs department. A and H don't belong to Human
resource department and Railways department. B is not near to the person who belong to Nuclear energy
department . D is seated opposite to one who belongs to Home affairs department.
1. Who belongs to the Defense department?
(a) F (b) G (c) H (d) C (e) None of these

2.What is the position of the person who belongs to External affairs department with respect to
the person who belongs to Human resource development department?
(a) Second to the left
(b) Immediate left
(c) Second to the right
(d) Third to the right
(e) Immediate right

3.The person who belongs to Finance department sits opposite to one who belongs to
______________________ Department.
(a) Human resource
(b) Nuclear energy
(c)Railways
(d) Home affairs
(e) External affairs

4.Which of the following statement is true?


(a) E is seated opposite to C and belongs to Home affairs department.
(b) F is seated opposite to D and belongs to railway department.
(c) A is seated opposite to E and belongs to Nuclear energy department.
(d) D belongs to Tourism department and seated opposite to the person who belongs to Home affairs
department.
(e) None of the above is true.

5.'H' is related to 'Nuclear energy' in the same way as 'C' is related to 'Defense' based on the given
arrangement. Who among the following is ‘F’ related to, following the same pattern?
(a)External affairs
(b) Human resource development
(c)Railways
(d) Finance
(e) Home affairs
PUZZLE-156
Directions (1-5): Study the following information and answer the questions given below:
There are seven persons, namely P, Q, R, S, T, U and V. They go to gym by different car namely Audi,
Toyota, Tata, Ford, Nissan, Hyundai and Fiat, but not necessarily in the same order, starting from
Monday to Sunday (of the same week). T goes to gym on Thursday. Only one person goes
to gym between T and the one who goes to gym by Nissan car. P goes to gym immediately after the one
who goes to gym by Nissan car. Only three people go to gym between P and the one who goes to gym by
Tata car. Only two people go to gym between the one who goes to gym by Tata car and U. The person,
who goes to gym by Fiat car, goes to gym before U, but after Thursday. More than two people go
to gym between the one who goes to gym by Fiat car and R. The one who goes to gym by Audi car goes
to gym immediately before the one who goes to gym by Hyundai car. The one who goes to gym by Ford
car goes to gym immediately before Q. V does not go to gym on Tuesday.

Q1.Who amongst the following goes to gym by Toyota car?


(a)R (b)S (c)P (d)T (e)V

Q2.Four of the following five are alike in a certain way and so form a group. Which of the following
does not belong to that group?
(a)R-Tuesday
(b)T-Monday
(c)Q-Sunday
(d)P-Friday
(e)U-Saturday

Q3.Which of the following is true about S?


(a)S goes to gym immediately after R.
(b)S goes to gym by Tata car.
(c)Only two people go to gym between S and R.
(d)All the given statements are true
(e)S goes to gym on Sunday.

Q4.T has which of the following car?


(a) Tata
(b) Audi
(c) Hyundai
(d) Ford
(e)Fiat

Q5. As per the given arrangement Q is related to the one who goes to gym by Tata car in a certain
way and P is related to the one who goes to gym by Fiat car , in the same way, U is related to?
(a) The one who goes to gym by Toyota car.
(b) The one who goes to gym by Ford car.
(c) The one who goes to gym by Tata car.
(d) The one who goes to gym by Audi car.
(e) None of these.
Days Person Car

Monday R Audi

Tuesday S Hyundai

Wednesday V Tata

Thursday T Ford

Friday Q Fiat

Saturday U Nissan

Sunday P Toyota

PUZZLE-157
Directions (11-15): Study the following information carefully and answer the questions given
below.
There are ten students are sitting in twelve seats in two parallel rows containing five students on each, in
such a way that there is an equal distance between adjacent students. In row 1, A, B, C, D and E are
seated and all of them are facing south, and in row 2, P,Q, R, S and T are sitting and all of them are
facing north. One seat is vacant in each row. Therefore, in the given seating arrangement each member
seated in a row faces another member of the other row. All of them have a different favorite subject i.e.
Hindi, English, Sanskrit, Urdu, Art, Math, Science, Economics, Biology and Physics.
A sits second to left of one, whose favorite subject is Hindi. Either A or the one, whose favorite subject is
Hindi, seats adjacent to the extreme end position. T sits one of the extreme ends of the row. There are
three students sit between T and S, whose favorite subject is English. Immediate neighbor of T faces B.
One of immediate neighbor of B faces R, whose favorite subject is Sanskrit. There are no vacant seats
adjacent to R. C’s favorite subject is Urdu and sits second to left of vacant seat. One of immediate
neighbor of R is Q. Q’s favorite subject is Art. One of immediate neighbor of Q faces D. D’s favorite
subject is Math. The one whose favorite subject is Science sits immediate left of the one whose favorite
subject is Economics. The one whose favorite subject is Biology sits third to left of one whose favorite
subject is Physics.

Q11. Who among following sits on the extreme end of the row?
(a) E,Q
(b) B,P
(c) B,S
(d) D,P
(e) None of these

Q12. Whose favorite subject is Physics?


(a)P
(b)R
(c)Q
(d)T
(e)S

Q13. Who among following sits second to left of T?


(a) S
(b) P
(c) R
(d) Q
(e) None of these

Q14. Who among following sits opposite to C?


(a) P
(b) Q
(c) R
(d) S
(e) T

Q15. Whose favorite subject is Science?


(a) A
(b) B
(c) C
(d) D
(e) E

PUZZLE-158
Directions (1-5): Study the following information carefully to answer the given questions.
There are ten girls who sits on two parallel rows of bench containing five people each, in such a way that
there is an equal distance between adjacent girls, for doing some college work and they like different
mobile phones viz. LG, Redmi note 4, Redmi , Moto x, Lenovo, Samsung, Nokia, MI5, I Phone 6 and
Vivo. In row 1- C, D, E, F, and G are seated and some of them are facing south and some of them are
facing north. In row 2 – M, N, O, P, and Q are seated and some of them are facing south and some of
them are facing north. Therefore in the given seating arrangement, each member seated in a row either
faces another member of the other row or seated behind each other.
The girl who likes Moto x sits to the immediate right of N, who seated exactly in the middle of the row.
One of the immediate neighbors of the girl, who likes Lenovo, sits behind the girl who likes Redmi. P does
not like Redmi. F likes neither Lenovo nor Samsung. M sits immediate right of the girl who likes I Phone 6.
E sits third to the right of the girl who likes Redmi note 4. M does not face C and faces south direction.
The girl who likes LG sits exactly between the girls who like Vivo and Lenovo C likes Lenovo and sits in
the middle of the row 1. Q faces North Direction and sits immediate left of N. Only one girl sit between the
girls who like Redmi and Moto X. M faces one of the immediate neighbors of the girl who likes Redmi
note 4. F faces one of the immediate neighbors of the girl who likes Redmi. Only One girl sits between the
girl who likes I Phone 6 and N who likes Nokia. E sits to the immediate right of the girl who faces P. Only
two people sit between E and G. P likes neither LG nor MI5. The girl who likes Lenovo sits second to the
right of the one who faces North Direction. O sits one of the extreme ends of the line and likes I Phone 6.
C faces the opposite direction to the girl who likes Nokia. F faces north direction and the one who faces F,
faces south direction. D faces north direction.

Q1. Who amongst the following faces the girl D?


(a) The girl who likes LG
(b) F
(c) The girl who likes Lenovo
(d) The girl who likes Moto X
(e) D

Q2. O likes which Mobile phones?


(a) LG
(b) Nokia
(c) I Phone 6
(d) Samsung
(e) Redmi

Q3. Which of the following is true regarding Q?


(a) Q faces south direction.
(b) None of the given options is true
(c) Q likes Redmi
(d) The girl who likes MI5 faces Q
(e) Q is E‘s immediate neighbour

Q4. Which one is the immediate neighbor of F?


(a) O
(b) P
(c) The one who likes Redmi
(d) The one who likes LG
(e) The one who likes Samsung

Q5. Who amongst the following sit at middile of the row 1?


(a) G
(b) D
(c) E
(d) The girls who likes Lenovo
(e) None of these
PUZZLE-159
Directions (1-5) : Study the following information carefully and answer the questions which
follow–
Mr. X, who lives in Bihar has five sons i.e. A, B, C, D and E and they are working in five different
organizations-Google, Bank of America, HSBC Holdings, Bank of China and NetApp but not necessary in
that order. Mr. Y who lives in Punjab has five daughters i.e. P, Q, R, S and T and they are working in five
different organizations-Barclays, Microsoft, Goldman Sachs, Deutsche Bank and Cisco but not necessary
in that order. These five boys are married to these five girls but not necessary in given order.
S is either working in Barclays or Cisco but does not married to one who works in Bank of America. A
does not work in Bank of America or Google but married to one who works in Cisco. The one who works
in Goldman Sachs is married to E who does not work in Google and Bank of America.
The boy who works in Google is either married to S or T. E is not married to P, who works either in
Goldman Sachs or Deutsche Bank. A is working in Netapp and married to S. D does not work in HSBC
Holdings or Bank of America but married to one who works in Microsoft.
C is married to R but does not work with NetApp or Bank of America. R does not married to one who
works in Bank of America. Q is married to one who works in HSBC Holdings.

Q1. Who is married to T?


(a) A (b) B (c) C (d) D (e) E

Q2. Who is the employee of Bank of China?


(a) A (b) B (c) C (d) D (e) E

Q3. In which of the following bank the wife of E is working?


(a) Goldman Sachs (b) Cisco (c) Barclays (d) Microsoft (e) Deutsche Bank

Q4. Who works with Microsoft?


(a) Wife of C (b) P (c) Q (d) T (e) Wife of one who works in Bank of America

Q5. Who is married to one who works with NetApp?


(a) The one who works with Deutsche Bank
(b) The one who works with Barclays
(c) The one who works with Goldman Sachs
(d) The one who works with Cisco
(e) Cannot be determined
PUZZLE-160
Directions (1-5): Read the following information carefully and answer the questions given below:

Six students A, B, C, D, E and F are sitting around a circular table facing towards the center of the table in
a library. They read different comics like Indrajal, Raj, King, Nutan, Champak and Tulsi but not
necessarily in the same order. They are wearing T-shirts of different colours, i.e. white, violet, green, red,
yellow and blue but not necessarily in the same order. The students, who read Indrajal, King and
Champak are neither in white T-shirt nor in Violet. The students who are in green and yellow T-shirts
have neither read Indrajal nor King comics. A is neither in white T-shirt nor on the immediate left of the
one who reads Nutan comics. The only student who is between E and F reads Raj comics. The one who
is on the immediate left side of the student who is in white T-shirt does not read Tulsi comics. D reads
Nutan comics and the colour of his T-shirt is green, he is facing the student who reads Raj comics. The
One who reads Indrajal comics is seated opposite to the student wearing blue T-shirt, while the student
whose T-shirt is of green colour is on the immediate left of the student who reads Champak comics. One
who reads Tulsi comics is on the immediate right of the student in white T-shirt but on the immediate left
of the student who reads King comics. C does not read King comics while F does not read Indrajal
comics.

Q1. Who among the following is in white T-shirt?


(a) A (b) B (c) C (d) E (e) None of these

Q2. The only student, who is between E and D, is wearing T-shirt of the colour?
(a) red
(b) blue
(c) violet
(d) yellow
(e) None of these

Q3. Who among the following has read Champak comics?


(a) A (b) B (c) C (d) E (e) None of these

Q4. Which of the following is correctly matched?


(a) A-Yellow-Nutan
(b) B-Red-King
(c) E-Red-Indrajal
(d) F-Violet-Champak
(e) None of these

Q5. The colour of the T-shirt of the student, who reads Tulsi comics, is
(a) red
(b) yellow
(c) blue
(d) violet
(e) None of these
PUZZLE-161
Directions (Q. 1-5): Study the following information and answer the questions given below.
Eight persons A, B, C, D, E, F, G and H like different mobile phones viz. Apple, Garmin, Vertu, Thuraya,
Foxconn, Pantech, Wiko and Lanix but not necessarily in same order. All of them are sitting around a
rectangular table. Four persons sits on each middle side of rectangular table, while other four persons on
the corner of table. All of them are facing towards the center. D is sitting second to the right of the one,
who likes Lanix mobile. E is sitting third to the left of the one, who likes Pantech mobile. F and G are
sitting opposite to each other. C is sitting opposite to one, who likes Wiko mobile. B is sitting opposite the
one, who likes Apple mobile. The one who likes Thuraya mobile is sitting second to the right of the one,
who likes Foxconn mobile and second to the left of A, who is not sitting near the one, who
likes Wiko mobile. G is sitting on the smaller side and to the immediate right of the one, who
likes Wiko mobile. The persons who like Foxconn and Thuraya mobile are not on the same side of the
table. The one, who likes Thuraya mobile is sitting third to the right of the one ,who likes Lanix mobile,
who is not sitting opposite to the person, who likes Garmin mobile. The one who likes Thuraya mobile is
sitting third to the left of C. F sits second to the left of H. The one who likes Garmin mobile sits opposite to
the one who likes Foxconn mobile.

1. Who is sitting third to the right of the one, who likes Pantech mobile?
(a)The one, who likes Foxconn mobile.
(b)A
(c) The one who likes Wiko mobile.
(d)None of these
(e) Can’t determine

2. Who is sitting opposite to D?


(a) A
(b) C
(c) The one who likes Lanix mobile.
(d) The one who likes Garmin mobile.
(e)Can't be determined
3. Who among following is sitting third to left of H?
(a) E
(b) D
(c) C
(d) B
(e) A

4. A likes which of the following mobile?


(a) Vertu
(b) Garmin
(c) Apple
(d) Thuraya
(e) Foxconn

5. Pantech mobile is like by which of the following one?


(a) B
(b) C
(c) D
(d) E
(e) F

PUZZLE-162
Directions (1-5): Read the following information carefully and answer the questions given below.
Eight family members P,Q,R,S,T,U, V and W are seated in a straight line at an equal distance between
each other, but not necessarily in the same order. There are three generation in the family. Some of them
are facing north and some are facing south. R’s daughter is an immediate neighbour of one who is sitting
at an extreme end of the line. Only three people sit between V and his wife. Q’s daughter sits second to
the right of P’s brother-in-law. R’s granddaughter does not sit at an extreme end of the line. P’s mother
sits on the immediate left of her son. R’s wife is not an immediate neighbour of T’s husband. The
immediate neighbour of T’s brother faces opposite directions.(i.e. If one neighbour faces north then other
faces south and vice versa.) The persons sitting at the extreme ends faces opposite directions.(i.e. If one
person faces north then other faces south and vice versa.) W’s daughter-in-law sits second to the left of
T’s father-in-law. U faces north. U is not an immediate neighbour of S’s aunt. The immediate neighbours
of U faces same directions.(i.e. If one neighbour faces north then other also faces north and if one
neighbour faces south then other also faces south).Both T and Q face a direction opposite to that of
S.(i.e. If S faces north then T and Q faces south and vice-versa.) T is the only daughter of R. W is female
member and U is male member.

Q1. As per the following arrangements, which of the following statements is not true with respect
to P?
(a) P faces south.
(b) P is fourth to the right of Q.
(c) P is 2nd to left of T.
(d) P is between V and W.
(e) None of these

Q2. How many people sit on the left of Q?


(a) One (b) Two (c) Three (d) Four(e) None of these

Q3. What is the position of S with respect to P?


(a) Immediate left (b) Third to left (c) Third to right (d) Fourth to left (e) None of these

Q4. Who among following is grand-mother of S?


(a) T (b) S (c) W (d) R (e) None of these

Q5 Four of the given five are alike in a certain way based on the given arrangement and hence
form a group. Which of them does not belong to that group .?
(a) R
(b) S
(c) P
(d) U
(e) V
PUZZLE-163
Directions (1-5): Read the following information carefully and answer the questions given below:
There are eight persons P, Q, R, S, T, U, V and W are sitting around a circle. Some are facing the center
while some are not. They also like different car like Audi, Maruti, Toyota, Nissan, Ford, Fiat, Skoda and
Hyundai. P is sitting third to right of R. There is one person sitting between P and Q. S is sitting fourth to
right of Q. The person who like Toyota sits second to right of S. P does not like Toyota car. Three persons
sit between the person who like Toyota and Hyundai car. The person who like Audi car sits second to
right of person who like Hyundai car. S does not like Audi car. The person who likes Fiat sits third to right
of person who like Audi car. R likes Fiat car. There is two persons sits between S and T. There is three
persons sits between the person who like Ford and Nissan car. T does not like Ford and Nissan car.
There is one person sits between the person who like Nissan and Maruti car. The person who like Toyota
car sits third to right of person who like Maruti car. The person who like Skoda car sits third to right of
person who like Nissan car. V sits fourth to right of person who like Ford car. U is not immediate neighbor
of V. The person who like Ford car sits third to left of person who like Toyota car and both faces the same
direction.(Same direction means if one faces center then other also faces the center and vice-versa).T
faces opposite the center. P likes the Hyundai car.

Q1. Who among the following like Toyota car?


(a) P (b) Q (c) R (d) T (e) W

Q2.If T is related to R and Q is related to S, in the same way P is related to?


(a) P
(b) Q
(c) R
(d) T
(e) W

Q3. Who among the following sits third to right of person who like Hyundai car?
(a) U
(b) S
(c) V
(d)W
(e) E

Q4. Which of the following is correctly matched?


(a) T-Audi
(b) R-Toyota
(c) S-Skoda
(d)U-Nissan
(e) W-Fiat

Q5. Who sits opposite to person who like Audi car?


(a) U
(b) The person who likes Nissan car.
(c) R
(d) The person who likes Hyundai car.
(e) The person who likes Skoda car.
PUZZLE-164
Directions (1-5): Read the following information carefully and answer the questions given below:
In a recent union budget finance minister Arun Jaitely disclose a budget of 2017. He adequate some
money (crores) in different-different sectors i.e. Digitalization, Agriculture, Defense, Infrastructure,
Social welfare, Science research and Education but not less than 11cr in any sector. Each sector is
headed by different-different minister namely I, J, K, L, M, N and O, but not necessarily in the same order.
Each minister has a different age.
Neither N nor oldest minister is head of Social welfare sector. In defense sector Arun Jaitely adequate
78cr. M is older than K but younger than N. 4th oldest minister is 55 year old and Education sector is
headed by him. 2nd youngest minister is heading Defense sector. In two sectors Arun Jaitely adequate
same money. Total amount adequate by Arun Jaitely is 192cr. K is not youngest minister. The number of
ministers, who is younger and older than O is same. J is the head of that sector for which Arun Jaitely
adequate 35cr. Difference of money allocated for Agriculture sector and Social welfare sector is
20cr. youngest minister is head of Agriculture sector. I is younger than L but older than K. M is
not head of that sector for which Arun Jaitely adequate 78cr. At least three minister younger than I. 5th
youngest minister is not head of Social welfare sector. 3rd oldest minister is not head of Science
Research sector. N is head of that sector for which Arun Jaitely adequate 1cr more than that sector,
which is headed by M. Infrastructure sector is headed by the 7th youngest minister and in that sector for
which Arun Jaitely adequate 22cr. In digitalization sector Arun Jaitely adequate least money.

Q1. Which of the following sectors is headed by minister N?


(a) Defense
(b) Education
(c) Science research
(d) Digitalization
(e) Agriculture
Q2. Which of the following combination is true?
(a) J-Agriculture- 78cr
(b) None of these
(c) O-Science research-15cr
(d) M-Social welfare- 15cr
(e) K- Defense- 22cr

Q3. If L is 13 year older than 4th oldest minister, then what is the possible age of 6th youngest
minister?
(a) 54 year
(b) 63 year
(c) Can’t be determined
(d) More than 68 year
(e) Less than 55 year

Q4. If Social welfare is related to 15cr in the same way as Science research is related to 16cr. Then
which of the following is Digitalization related to, following the same pattern?
(a) 15cr
(b) None of these
(c) 78cr
(d) 22cr
(e) 11cr

Q5. Which of the following minister is head of that sector for which Arun Jaitely adequate 3rd
least amount?
(a) J
(b) N
(c) O
(d) I
(e) L
PUZZLE-165
Directions (1-5): Study the following information carefully and answer the questions.
There are seven students – M, N, O, P, Q, R and S – who like different subjects viz- Hindi, English, Math,
Science, Physics, Chemistry and Biology but not necessarily in the same order. They read on different
days of the week starting from Monday. They also like different sports, viz- Hockey, Cricket, Football,
Tennis, Badminton, Baseball and Kho-Kho but not necessarily in the same order.
N, who likes Baseball, reads on the fourth day of the week but neither Hindi nor Biology. Two students
read between the days on which N and R read and neither of them read on the 1st day of the week.
There is only one student who reads between M and O. But O does not read either on 1st or 3rd day of
the week. O likes Cricket sport and reads Hindi. The one who reads Math on the last day of the week
likes Football sport. P does not read on the day either immediately before or immediately after the day on
which M read, who doesn’t like either Badminton or Hockey or Tennis sport. P does not read science. S
reads immediately after O and he likes Hockey sport. Q does not like Badminton sport and reads
Chemistry. M does not read either Physics or Science. The one who reads English takes class
immediately after the day when Hindi has been read.

Q1. Which of the following combinations is definitely false?


(a)O – Cricket – Hindi
(b)M – Kho-Kho –Biology
(c)Q – Tennis –Chemistry
(d)R – Cricket – Hindi
(e)None of these
Q2.Who among the following reads on the 5th day of the week?
(a)The student who likes Kho-Kho sport
(b)The student who reads Math
(c)The student who reads Hindi
(d)The student who likes Tennis sport
(e)None of these

Q3.Who read Science?


(a)The student who likes Baseball sport
(b)The student who likes Kho-Kho sport
(c)The student who likes Badminton sport
(d) The student who likes Football sport
(e)None of these

Q4.If ‘N’ is related to ‘Cricket’ and ‘S’ is related to ‘Football’ then which of the following is ‘M’
related to?
(a)Badminton
(b)Baseball
(c)Tennis
(d)Kho-Kho
(e)None of these
Q5.Who reads Physics?
(a)The student who reads on fourth day of the week
(b)The student who reads just before Q
(c)The student who reads just after Q
(d)Can’t be determined
(e)None of these

PUZZLE-166
Directions (1-5): Read the following information carefully and answer the questions given below:
There are nine floors building in the area in which nine people are living on different floor separately. The
ground floor is numbered 1 and so on. Nine persons are A, B, C, D, E, F, G, H, and I. They also like
different colours like Red, Blue, Black, White, Yellow, Violet, Pink, Orange and Brown but not necessarily
in the same order.
I lives on the even number floor but not on the eight floor. There is gap of two floor between the I and the
person who likes Orange colour. There is gap of two floor between the person who like Orange and Blue
colour. The person who likes Orange colour lives on top floor. The person who likes Red colour lives on
the odd number floor. E likes Red colour and he does not live immediately above or immediately below
the floor on which I lives. There are gap of two floors between the floor on which E and D live. D likes
Yellow colour. A likes violet colour and he lives on the even number floor. There are gap of two floors
between A and the person who likes White colour. There is gap of one floor between the person who likes
White and Brown colour. B likes brown colour and lives one of the floor above on which the person who
likes White colour lives. C likes Black colour and lives on even number of floor but not on the second
floor. F does not like Pink and Orange colour. G does not like Orange colour.

Q1. Who lives on floor number 3?


(a) E (b) A (c) G (d) D (e) F

Q2.Which of the following is correct?


(a) C-Black-9th floor (b) I-Blue-7th floor (c) E-Red-2nd floor (d) G-Pink-3rd floor (e) G-Orange-9th floor
Q3.F is related to Brown and A is related to Yellow, in the same way I is related to?
(a) White
(b) Orange
(c) Black
(d) Red
(e) None of these

Q4.How many floors are between the floor on which I and E live?
(a) one
(b) two
(c) three
(d) more than three
(e) None of these

Q5.How many floor below on which D lives?


(a) One
(b) two
(c) more than three
(d) three
(e) None of these

Solution (1-5):
PUZZLE-167
Directions (6-10): Study the following information carefully and answer the question given below-
A, B, C, D, E, F, G and H are eight kids sitting around a circular table. Four of them are facing away from
the centre and four of them are facing towards the centre. Each of them like different candies-Hajmola,
Poppins, Kismi, Satmola, Aam Pachan, Rochak, Pan pasand and Chatmola. All of them are holding a
different colour balloon viz. White, Blue, Orange, Pink, Green, Purple, Yellow and Red but not necessarily
in the same order. E faces towards the centre and holds White balloon. Both the immediate neighbors of
E face away from the centre and are holding either Orange and Pink balloon. D faces away from
the center and his favorite candy is Hajmola. Both the immediate neighbours of D do not face away from
the centre. E sits third to the right of F, who has a Green balloon and faces away from the centre. C sits
third to the left of F. The one who has an Orange balloon sits opposite to F. The one who has Blue
balloon is not the immediate neighbour of F and faces away from the centre. A sits second to the left of C
and he have neither Yellow nor Red balloon. The one who has a Yellow balloon sits between H and F. B
faces away from the centre, likes Poppins and does not have a Blue balloon. E’s favourite candy is
Pan pasand. The person who likes Satmola opposite to D. The person having Purple balloon likes kismi.
The person who likes Chatmola is not near to G nor E. C faces the person who likes Rochak.

Q6. Which one of the following related to Purple?


(a) E
(b) F
(c) B
(d) D
(e) A

Q7. Which one of the following related to Orange?


(a) B (b) H (c) F (d) D (e) None of these

Q8. How many persons are there between the one who related to Pink and the one who related to
Orange when counted in anticlockwise direction from the person who related to Pink?
(a) Four
(b) Two
(c) Three
(d) CND
(e) None

Q9. Which one of the following related to Pink?


(a) B
(b) H
(c) F
(d) D
(e) None of these

Q10. Which of the following is B’s position with respect to F?


(a) Fourth to the left
(b) Third to the right
(c) Second to the left
(d) Second to the right
(e) None of these
PUZZLE-168
Directions (Q. 1 – 5): Study the following information carefully and answer the questions
which follow—
A, B, C, D, E and F are six persons who have joined 6 different banks OBC Bank, Punjab National Bank,
Bank of Maharashtra, Bank of India, Bank of Baroda, and State Bank of India by scoring different marks
in the written exam of a maximum of 200 marks. (marks are an integer value) They all are sitting around a
circle facing the centre with equal distance.
C is second to the right of the person who joined Bank of Baroda whose score is 169 marks which were
the third lowest marks out of the 6 students. A is immediate to the left of the person who is opposite to the
person who joined State Bank of India, who is not near to F. B scored the 2nd highest marks and did not
join Bank of India but is 2nd to the left of the person, who scored 174 marks. D is seated opposite to one
who joined Bank of India. D is not near to B. E has not scored the lowest marks. C is immediate to the left
of the one who joined State Bank of India and C scored 170 marks. The person who joined Punjab
National Bank is second to the left of the one who scored 172 marks. The person who joined OBC did not
score the highest marks. One of the six students scored which was a prime number. The lowest scored
mark is 164.

Q1. Who amongst the following is from OBC?


(a) D (b) E (c) C (d) A (e) B

Q2. Four of the following five are alike in a certain way based on the given seating arrangement
and thus form a group, who is the one that does not belong to that group?
(a) A (b) B (c) C (d) D (e) E

Q3. Who is seated between D and the person from Bank of Baroda?
(a) E (b) A (c) C (d) None (e) The person from Bank of Maharashtra
Q4.Which of the following is true regarding the given information?
(a) F is from Bank of Baroda and seated immediate right of the person who is opposite to the person who
joined bank of Maharashtra.
(b) A is from Punjab National Bank and scored 174 marks and is opposite to C
(c) E is from OBC and is to the immediate right of B
(d) C scored 170 marks and is opposite to one who joined Bank of Maharashtra.
(e) The person from Bank of Maharashtra is seated opposite to one who scored 164 marks.

Q5.Which of the following order of marks in descending order is true?


(a) A > B > C > D > E > F
(b) A > B > C > F > E > D
(c) A > B > E > C > F > D
(d) A > B > C > E > F > D
(e) A > B > C > D > F > E
PUZZLE-169
Directions (1-5): Read the following information carefully and answer the questions that follow
Ten persons are sitting in two parallel rows containing five persons each. In row 1, A, B, C, D and E are
sitting and all of them are facing south. In row 2, P, Q, R, S and T are sitting and all of them are facing
north. In the given seating arrangement, each member seated in a row faces another member of the
other row. Moreover, each of them is going in the market and each person eats different types of foods
i.e. Idali, Cutlet, Dosa, Golgappe, Bread roll, Tikki, Noodles, Biryani, Chili potato and Eggroll, but not
necessarily in the same order.
There are only two person’s sit between the one who eats Idali, who sits at an extreme end and E. P, who
sits in the middle of the row, is not an immediate neighbour of Q, who doesn’t eat Eggroll. T is sitting at an
extreme end. E eats Bread roll, sits on the immediate right of the person who eats Cutlet and faces the
immediate neighhour of Q. B is not sitting at the extreme left end. P does not eat Chili potato. There is
only one person between C and D, who eats Idali. S eats Tikki, is an immediate neighbour of the person,
who eats Eggroll and does not face the person, who eats Dosa. R, who eats Golgappe, is an immediate
neighbour of the person who eats Chili potato and the person who eats chili potato faces the immediate
neighbour of the person who eats Bread roll. There are two persons between the person who eats
Noodles and the person who eats Dosa. B does not eat Noodles. T does not eat Biryani.

Q1.A eats which of the following foods?


(a) Biryani
(b) Noodles
(c) Dosa
(d) None of these
(e) Can't be determined

Q2.Who eats Biryani?


(a) R
(b) B
(c) T
(d) P
(e) Can't be determined

Q3.‘Idali’ is related to ' Noodles ' in a certain way, based on their seating positions. Then Eggroll is
related to whom, following the same seating positions?
(a) Biryani
(b) Chili potato
(c) Cutlet
(d) Tikki
(e) Golgappe

Q4.Four of the following five are alike in a certain way based on their seating positions and so
form a group. Which of the following is different from the group?
(a) Bread roll
(b) Noodles
(c) Eggroll
(d) Golgappe
(e) Tikki
Q5.Which of the following statements is/are definitely false?
(a) B eats Dosa.
(b) There are two persons sitting between the person who eats Tikki and the person, who eats Chili
potato.
(c) The person who eats cutlet faces the person who eats Biryani.
(d) The person who eats Chili potato sits opposite the person, who eats Idali.
(e) All are true

PUZZLE-170
Directions (11-15): Study the following information carefully and answer the question given below:
There are eight persons namely- P, Q, R, S, T, U, V and W sitting around a circular table facing the
centre, but not necessarily in the same order. Each person belongs to different country i.e. Australia,
South Africa, Bangladesh, New Zealand, West Indies, Srilanka, India and Pakistan also likes different
kinds of fruits viz.- Mango, Litchi, Apple, Orange, Pineapple, Grapes, Guava and Banana, but not
necessarily in the same order.
Person belongs to India sits second to the right of S, who belongs to neither Srilanka nor West Indies. U
sits second to the right of the one, who belongs to Srilanka. The one who likes Pineapple sits second to
the left of the one, who belongs to South Africa. Two people sit between the one, who belong to West
Indies and Q. V and the one, who belongs to New Zealand are immediate neighbours of each other.
Persons who like Orange and Grapes are adjacent to the one who belongs to New Zealand. Person likes
Orange sits immediate left of the one who belongs to South Africa. Q does not like Guava and Banana.
The one who likes Banana sits second to the right of the one who likes Guava. R and T is immediate
neighbour of each other. Neither R nor T is an immediate neighbour of either Q or the one who belongs to
Srilanka. Q is not belongs to New Zealand. The one who belongs to South Africa sits second to the left of
the one who belongs to Pakistan. Person who likes Apple sits fourth to the right of the one who likes
Litchi. The one who likes Guava is not immediate neighbour of the one who belongs to Srilanka. Neither
R nor T sits third to the left of the person, who belongs to Srilanka. P likes Banana. Only one person sits
between R and the one who belongs to Australia. W sits third to the left of the one who belongs to
Bangladesh. W does not like Apple.

Q11. What is the position of Q with respect to the one, who belongs to South Africa?
(a)Second to the left
(b) Second to the right
(c) Sixth to the left
(d) Third to the right
(e) Both (b) and (c)
Q12. Which of the following fruits does Q like?
(a)Pineapple
(b) Mango
(c) Apple
(d) Litchi
(e) None of these

Q13. Who sits opposite P?


(a)The one who belongs to Srilanka
(b)The one who belongs to South Africa
(c)The one who belongs to New Zealand
(d)The one who belongs to Bangladesh
(e)None of these

Q14. Four of the following five are alike in a certain way and hence they form a group. Which one
of the following does not belong to that group?
(a)S
(b) The one who likes Banana
(c) The one who belongs to South Africa
(d) The one who belongs to New Zealand
(e) The one who likes Orange

Q15. U belongs to which of the following country?


(a)Bangladesh
(b) India
(c) West Indies
(d) South Africa
(e) New Zealand
PUZZLE-171
Directions (1-5): Study the following information and answer the following questions:

A, B, C, D, E, G and I are seven friends who study in Career Power at three different branches namely
Mukharjee Nagar, Dwarka, and Laxmi Nagar such that not less than two friends study in the same
branch. Each friend also has a different favourite subject viz. History, Civics, English, G.K., Reasoning,
Maths and Economics but not necessarily in the same order. Each of the students attends the classes of
his favourite subjects at different time
B attends his classes at 10 AM. G attends his classes at 7 AM. A likes Maths and studies in the
Mukharjee Nagar branch with only one other friend who likes G.K. I studies with two other friends. There
is only one hour gap between the time of class of the persons from Dwarka and the one whose favourite
subject is G.K. Both the friends who study with I like the same subjects, which can be either Reasoning or
English. D studies in the Dwarka branch with only one person and does not like civics. The one whose
favourite subject is Economics takes his class at 8 AM. I attends his class at 8 ‘O clock. The persons
whose favourite subjects are Economics and History do not have their classes at the same time. E
studies with only one friend. The one who likes history does not study in Mukharjee Nagar nor Dwarka
branch. E does not like G.K. subject. C does not like English, Reasoning or Civics. A and the one whose
favourite subject is Civics have their class at 11 AM. C attends his class after D.

Q1. Which combination represents E’s favourite subject and the branch in which he studies?
(a) Civics and Laxmi Nagar
(b) Economics and Mukharjee Nagar
(c) Civics and Dwarka
(d) History and Laxmi Nagar
(e) Economics and Laxmi Nagar

Q2. I attends his class at?


(a) 11 AM
(b) 8 AM
(c) 10 AM
(d) 9 AM
(e) None of these

Q3. Who amongst the following studies in the Laxmi Nagar branch?
(a) G
(b) C
(c) E
(d) D
(e) Either D or B

Q4. Which of the following combinations is definitely correct?


(a) I and Reasoning
(b) G and English
(c) C and G.K.
(d) B and Reasoning
(e) E and Economics
Q5. Which of the following subjects does G like?
(a) Either Maths or G.K.
(b) Either Reasoning or English
(c) Either Reasoning or Civics
(d) Either Reasoning or G.K.
(e) Either Civics or Economics

PUZZLE-172
Directions (1-5): Study the following information carefully and answer the question given below:
Eight employee Q, R, S, T, W, X, Y and Z of the multinational company (MNC) will have to attend the
meeting in the months of August and December but not necessarily in the same order. In each month, the
meeting will conduct on dates 21st, 23th, 27th and 31st of the given month. Only one person will attend
the meeting on these given dates.
Q will attend the meeting on either 27th August or 27th December. There is only one person will attend
the meeting between Q and T. There are two persons will attend the meeting between the T and R. There
is one person will attend the meeting between the R and S. S will attend the meeting on 23th of any
month either December or August. Only one person will attend the meeting between S and W. There are
as many as the person will attend the meeting between W and Y as between Y and Z. X will not attend
the meeting on 21st of August.

Q1.Who among following will attend the meeting on 27th August?


(a) Z
(b) Q
(c) R
(d) Y
(e) S

Q2. How many persons will attend the meeting between R and W?
(a) One
(b) Two
(c) Three
(d) More than three
(e) None of these
Q3. Who among following will attend the meeting on 31st December?
(a) R
(b) Y
(c) S
(d) X
(e) W

Q4. If T is related to Q and R is related to S, in the same way Y is related to?


(a) X
(b) T
(c) Z
(d) Q
(e) R

Q5. X will attend the meeting on which of the following dates?


(a) 23th August
(b) 31st December
(c) 27th August
(d) 31st August
(e) None of these
PUZZLE-173
Directions (1–5): Study the following information carefully to answer the given questions.
In Year 2016, seven persons viz. P, Q, R, S, T, V and W donated some money to a charitable
trust on different months of the year starting from January and ending on November. Some of them are
females and some of them are male. Their contribution,(in rupees) are – 5000, 500, 40, 1600, 80, 40 and
2500. V made his donation in June. Two donations were made between the donations made by V and P.
On the first month of the year, a male made a contribution of 5000 rupees. S did not made his donation
on a month after the donation of Q. None of the given person made his contribution in a month which has
28 days. There are two such cases where the difference in the number of days of the months of two
successive donations is zero. In one such case, the months are consecutive months of the year. The
contribution of Rs 500 was made in November month. The sum of contributions of W and Q is equal to
that of S. Contribution by a female is always preceded and succeeded by the contribution of a male. T is
the only female who cannot be succeeded by any one. P is the only male which is followed by another
male. Only one donation is made between the donations made by T and S. The donation of the male who
donated his money on June is the perfect square of the donation made on August. The donation by T is
not made on the month immediately before or immediately after the month when V made his donation
(considering only those month in which the donations were made. R donated money immediately before
W. Only three persons made their contribution in a month which has 31 days.

Q1. As per the given arrangement which of the following combination represents only the people
who are females?
(a) T, V
(b) Q, P, T
(c) V, W, R
(d) P, T
(e) W, T, S

Q2. As per the given arrangement which of the following person represent the one who was
donated money in between the W and S?
(a) U
(b) P
(c) R
(d) V
(e) W

Q3. Who among the following is a female?


(a) P
(b) R
(c) S
(d) All of the Above
(e) Other than those given as options

Q4. Which of the following combinations is correct as per the given arrangement?
(a) January – P
(b) February – W
(c) April – Q
(d) June – S
(e) July – T
Q5. Who amongst the following is male who donated 2500 Rs?
(a) R
(b) V
(c) U
(d) W
(e) T

PUZZLE-174
Directions (1-5): Study the following information carefully and answer the questions given below:
Eight persons A, B, C, D, E, F, G and H are sitting around a circular table facing the centre. They belong
to different states, viz Punjab, Kerela, Goa, Sikkim, Manipur, Nagaland, Bihar and Telengna, but not
necessarily in the same order. They also like different colours– Red, White and Blue. At least two persons
like one colour.
Only D and E like Red and D sits third to the right of E. A is from Manipur and likes White and sits on the
immediate right of E. F is from Sikkim and likes Blue. H likes the same colour as G. G is third to the left of
F. B sits opposite the person who is from Telengna. C sits exactly opposite the person who belongs to
Punjab. The persons who belongs to Nagaland and Goa like White. G is from Telengna. B is from Goa.
The persons who like White do not sits adjacent to each other. The one who is from Bihar does not like
White or Red. H is not from Nagaland.

Q1. Who among the following is from Kerela?


(a) The one who likes Blue
(b) D
(c) The one who likes White
(d) E
(e) None of these
Q2. Who sits fifth to right of H?
(a) F
(b) C
(c) B
(d) G
(e) A
Q3. E belong to which of the following state?
(a) Nagaland
(b) Kerela
(c) Punjab
(d) Bihar
(e) None of these

Q4. Which of the following combinations is true?


(a) G – White – Telengna
(b) C – Blue – Kerela
(c) D – Red – Punjab
(d) A-Blue- Manipur
(e) None is true

Q5. Which of the following statements is/are true?


(a) G is from Telengna and likes White.
(b) C is from Nagaland and likes White.
(c) F does not like Blue.
(d) H belongs to Kerela and likes Red.
(e) None of these
PUZZLE-175
Eight groups A, B, C, D, E, F, G and H are going forrock climbing in different mountain
ranges, viz. Aravallis,Himalayas, Satpura, Andes, Alps, Rockies, Atlas andAppalachian but
not necessarily in the same order. Thesegroups are also going for rafting in different rivers,
viz.Spree, Hudson, Amazon, Tigris, Tiber, Nile, Danube andThames but not necessarily in
the same order.Group G is going neither to river Danube nor to riverHudson. Neither group
F nor group G is going to raft in theriver Tigris or the river Nile. Group E is going to
theAppalachian mountain range. Group C is going to the Andes mountain range and the
river Tiber. The one who is goingto raft in the Amazon is also going to the
Appalachianmountain range. Group A and B are going for rafting either in the Nile or
Thames. Group D does not want to raft in theTigris and F does not want to raft in the
Hudson. Thegroup which is climbing on the Aravallis is rafting in theDanube and the group
which is climbing on the Atlas israfting in the Hudson. A and G do not climb on the
Rockiesand the Satpura. The group which is climbing on theHimalayas is rafting in the
Thames. Group H and G climbeither on Alps or on the Satpura mountain range.
Name Mountain River
A Himalayas Thames
B Rockies Nile
C Andes Tiber
D Atlas Hudson
E Appalachian Amazon
F Aravallis Danube
G Alps Spree
H Satpura Tigris

PUZZLE-176
PK, SP, RJ, MP, DK, SK and AK are seven shopkeepers of different shops, viz Grocery,
Cosmetics, Clothes,Footwear, Gift items, Watches and Mobile phones. They go to
supermarkets, viz. Big Bazaar, Max, Reliance Trends andEasyDay only on Sunday but not
necessarily in the same order. At least one shopkeeper goes to one supermarket, but no
super market is visited by more than two shopkeepers.SK, who is a shopkeeper of Clothes,
goes alone to Reliance Trends.The one who is a shopkeeper of Footwear does not go to
Big Bazaar. Also, he never goes either with DK or with AK.MP goes to EasyDay with the
person who is the shopkeeper of Mobile phones.RJ goes to Max. AK is not a shopkeeper of
Mobile phones.The one who is the shopkeeper of Footwear goes to the market with the
person who is the shopkeeper of Grocery.The one who is the shopkeeper of Gift items goes
to EasyDay.PK is a shopkeeper of neither Cosmetics nor Mobile phones.The one who is the
shopkeeper of Watches goes to Big Bazaar neither with DK nor with MP.The person who is
the shopkeeper of Cosmetics goes to the supermarket with PK.
Shopkeeper Super Market Shop
PK Big BazaaR Watch
SP Max Grocery/Foodwear
RJ Max Foodwear/Grocery
MP EasyDay Gift Items
DK EasyDay Mobile phones
SK Reliance Trends Clothes
AK Big Bazaar Cosmetics

PUZZLE-177
There are eight actors – Amitabh, Akshay, Shahrukh, Hrithik, Priyanka, Anushka, Diya and
Sonakshi. Out of them four are males and remaining are females. Moreover, each of them
endorses a different brand, vizAirtel, Titan, Hero, Britannia, LUX, Rin, Nokia and Cadbury,
though not necessarily in the same order. All of them are sitting around a circular table,
facing the centre. The person who endorses LUX sits opposite Shahrukh. There are two
persons between Diya and Hrithik, who endorses Nokia. Priyanka, who does not endorse
Rin, sits second to the left of Amitabh, who endorses Airtel. The person opposite Diya
endorses Titan.Shahrukh is not an immediate neighbour of either Amitabh or Priyanka.
Anushka does not endorse LUX. Akshay sits second to the right of Hrithik. Sonakshi sits
third to the right of the person endorsing Titan and she endorses neither Britannia nor
Cadbury.Shahrukh endorses Britannia and the person sitting between Shahrukh and Hrithik
endorses Hero. No three females are sitting together.
PUZZLE-178
T - 20 World Cup is being organized by Bangladesh. Different teams are participating in T-
20 World Cup. There are some new teams which are also participating in this T-20 for the
first time. Seven new teams which are participating for the first time are - Afghanistan,
Nepal, UAE, Bangladesh, Netherlands, Hong Kong and Ireland. They will be playing three
practice matches at different places, namely Mirpur, Chittagong, Sylhet, Dhaka, Begra,
Khulna and Fatullah, but not necessarily in the same order, on different days of the week,
starting from Monday.
• Afghanistan will be playing at Chittagong, but neither on Friday nor on Monday.
• The host country will be playing on Thursday at Begra
• There is a gap of one day between Afghanistan and Bangladesh match
• Hong Kong will be playing the match before Ireland but after Netherlands at Sylhet on
Wednesday
• Ireland will be playing on the last day of the week
• Nepal will be playing at Fatullah
• No match will be played at Mirpur on Monday
• The match at Khulna will be played on the last day of the week
• The match at Dhaka will be played on Tuesday
• No match will be played at Fatullah on Tuesday and Friday
Afghanistan Chittagong Saturday
Bangladesh Begra Thursday
HongKong Sylhet Wednesday
Ireland Khulna Sunday
Nepal Fatullah Monday
UAE Mirpur Friday
Netherlands Dhaka Tuesday
PUZZLE-179
Six friends P, Q, R, S, T and U from six different areas, viz. Rohini, Indirapuram, Dwarka,
Kaushambi, Munirka and Vaishali, go for shopping in six different shopping complexes, viz.
Sahara, GIP, CSM, V3S, Shipra and Select Citywalk, but not necessarily in the same order.
Three different types of transport sare used by them, viz. Car, Bus and Metro, in such a
manner that two persons use the same type of transport but not necessarily in the same
order.
• P uses car and lives in Vaishali but he does not go to CSM and Select Citywalk.
• The person going to Shipra Complex uses car and the one going to GIP uses bus.
• T uses the same type of transport as the person from Dwarka.
• CSM Complex is chosen by the person who uses bus.
• R is neither from Rohini nor from Munirka and he goes to Shipra Complex.
• One of the persons who use Metro goes to Sahara Complex.
• Q is from Kaushambi and goes to GIP. He does not use the same transport as S.
• U is neither from Indirapuram nor from Munirka.
• Select Citywalk and GIP Complex are not visited by the persons who use car.
• The persons from Kaushambi and Rohini are using the same type of transport.

Person Transport Place Shopping Complex

P Car Vaishali V3S

Q Bus Kaushambi GIP

R Car Indirapuram Shipra

S Metro Dwarka Select Citywalk/Sahara

T Metro Munirka Sahara/Select Citywalk

U Bus Rohini CSM


PUZZLE-180
A, C, E, I, O, N, P and S are eight persons working in an MNC. They are sitting around a
circular table in a meeting but not necessarily in the same order. All of them hold a different
post, viz President, MD, GM, CA, HR, Vice President, Chairman and Vice Chairman, but not
necessarily in the same order. C sits third to the right of the Vice President. Only two
persons sit between Vice President and S. The GM and the MD are immediate neighbours.
Neither C nor S is a GM or an MD. The GM is not an immediate neighbour of the Vice
President. The CA sits third to the right of O, who is not the GM. N sits on the immediate
right of the Chairman. The Vice Chairman sits second to the left of A, who is not an
immediate neighbour of S. The Vice Chairman is an immediate neighbour of both the HR
and the CA. C is not the Chairman. I is not an immediate neighbour of C. E is not an
immediate neighbour of the Vice Chairman.
PUZZLE-181
Ten persons are sitting in two rows containing five persons each. In row 1, A, B, C, D and E
are sitting and all of them facing south. In row 2, P, Q, R, S and T are sitting and all of them
are facing north. In the given seating arrangement each member sitting in a row faces
another member of the other row. Moreover, each of them belongs to a different country
Australia, Brazil, Bulgaria, Belgium, Japan, India, Romania, Indonesia, Iran and Russia, but
not necessarily in the same order. There are only two persons sitting between the
Australian, who sits at an extreme end, and E. P, who sits in the middle of the row, is not an
immediate neighbour of Q, who is not a Russian. T is sitting at an extreme end. E, a
Japanese, sits on the immediate right of the person from Brazil and faces the immediate
neighbour of Q. B is not sitting at the extreme left end of the row. P is not from Iran. There is
only one person sitting between C and D, who is from Australia. S is an Indian and is an
immediate neighbour of the Russian and doesn't face the person from Bulgaria. R, who is
from Belgium, is an immediate neighbour of the person from Iran who is facing an
immediate neighbour of the Japanese. There are two persons between the Romanian and
the Bulgarian. B is not from Romania. T is not from Indonesia.
PUZZLE-182
A, B, C, D, E, F, G and H are bachelor courses. They study in 1st year, 2nd year, and 3rd
year. Not more than three students are there in one year. Each of them has a favourite
subject, viz Geography, Physics, English, Marathi, Mathematics, Chemistry, Biology and
Economics, but not necessarily in the same order. B is not in the second year. E and A are
students of in same year but not with B. C and F are students of the same year. The
students who study in the first year do not like Biology and Mathematics. F likes Physics. D
likes Chemistry and is in final year only with H. C does not like Geography. Marathi is the
favourite subject of A. G does not like Biology. The one who studies in the final year likes
English. (Note: Third year is the final year.)
PERSON SUBJECT YEAR
A Marathi II
B Geography I
C Economics I
D Chemistry III
E Biology II
F Physics I
G Maths II
H English III

PUZZLE-183
T,U, V,W, X, Y and Z live on a seven-storey building but not necessarily in the same order.
The lowermost floor of the building is numbered 1, and the topmost floor is numbered 7.
each of them also likes a watch of a different company, viz, Sonata, 'Titan, Casio, Rado,
Fastrack, Optima and Rolex.
T lives on an Odd-numbered floor but not on floor Number 3. The one who likes Fastrack
lives immediately above T. Only two persons live between W and the one who likes
Fastrack. the one who likes Titan lives on one of the odd-numbered floors but above W.
Only three persons live between V and the one who likes Titan. The one who likes Casio
lives immediately above V. The one who likes Sonata lives immediately above the one who
likes Rolex. Z lives on an odd-numbered floor. Only one person lives between U and X. U
lives on one of the floors above X. Neither V nor T likes Rado. X does not like Casio.

Floor Person Watch


7 Z Rado
6 U Fastrack
5 T Titan
4 X Sonata
3 W Rolex
2 Y Casio
1 V Optima
PUZZLE-184
Eight Friends — Romil, Rakesh, Mukesh, Rohit, Rahul, Anupam, Neeraj and Anil — are
seated around a circular table, but not necessarily in the same order. Four of them are
facing the centre and the others are facing outward. They belong to eight different cities —
Bhopal, Patna, Kolkata, Delhi, Gwalior, Ranchi, Chennai and Rajkot but not necessarily in
the same order. Anupam faces the centre and sits third to the right of Rakesh. Rohit
belongs to Kolkata and faces the person who belongs to Ranchi. Neeraj sits third to the
right of Mukesh, who lives in Bhopal. The persons who belong to Delhi and Gwalior are
facing the same direction (inward or outward). Rahul is sitting between the person who
belongs to Kolkata and the one from Rajkot respectively. Romil belongs to Gwalior and
Rakesh belongs to Patna. The person who belongs to Chennai is facing outward and is the
immediate neighbour of the person who belongs to Rajkot. Anil is the immediate neighbour
of the ones who belong to Gwalior and Chennai. Rahul is on the immediate left of Rohit.
Anil sits third to the right of Rahul.
PUZZLE-185
Nine persons P, Q, R, S, T, U, V, W and X live in a building but not necessarily in the same
order. There are nine floors in that building and only one person lives on each floor. Each of
them has a different vehicle — WagonR, Alto, Hyundai, Corola, Fiat, Maruti, Santro, i20 and
i10 but not necessarily in the same order. The ground floor is numbered 1 and the floor
above it is numbered 2 and so on. The topmost floor is number 9. W has Corola and he
lives on an even-numbered floor. P lives on an even-numbered floor that is below the floor
on which W lives. The person who has i20 lives on the fourth floor. T lives on the 2nd floor
and has Hyundai. The person having i10 lives on the 3rd floor. P doesn't have Maruti car.
There are two floors between the floors on which the persons having Santro and Corola
live. R has Alto. There are three floors between R and V. S lives on the floor immediately
above the floor on which X lives. There is one floor between the floors on which U and V
live. U doesn't have i10. The person having WagonR lives on the topmost floor. U doesn't
live on the ground floor.
Person Floor vehicle

Q 9 Wagonr

W 8 Corola

U 7 Maruti

P 6 Fiat

V 5 Santro

S 4 i20

X 3 i10

T 2 Hyundai

R 1 Alto
PUZZLE-186
Directions (1-5): Study the following information carefully and answer the given questions.
There are seven bollywood actors named Aamir Khan, Akshay Kumar, Ajay Devgan,
Shahid Kapoor, Hrithik Roshan, Salman Khan and Shah Rukh Khan. They are born in a
different month of the same year from January to July, but not necessarily in the same
order. No two people are born in the same month. They also acted in different movies
among Dilwale, Sultan, Dangal, Rustom, Drishyam, Krrish and Dhoom. Akshay Kumar has
his birthday in a month which has 30 days.The ones who have their birthday in the month
immediately preceding & succeeding that of Akshay Kumar's acted in Drishyam and Krrish
respectively.Ajay Devgan has his birthday in the month just before that of Akshay
Kumar.Salman Khan has his birthday in a month that has more than 30 days.Dangal is not
acted by Salman Khan or Shahid Kapoor.Hrithik Roshanacted in Sultan. There is only one
person's birthday between Hrithik Roshan and the one who acted in Dilwale.The persons
who acted in Rustom or Dhoom do not have birthdays in successive months. Akshay
Kumarnot acted in Rustom or Dhoom.There are three persons' birthday between Aamir
Khan and the one who acted in Dangal. Aamir Khan is not born in February.The one who
acted in Dhoom has his birthday before the one who acted in Rustom.
1.The actor who acted in Drishyam born in which of the following month?
April
March
May
July
January

2.Aamir Khanacted in which of the following movies?


Dangal
Dilwale
Krrish
Rustom
Sultan

3.How many actors born between Shahid Kapoor and Hrithik Roshan?
Three
Four
Five
Two
None
4.Four of the following persons form a group, which of the following person does not belong to
that group?
Shah Rukh Khan
Salman Khan
Aamir Khan
Shahid Kapoor
Ajay Devgan

5.Which of the following combinations is true?


July – Hrithik Roshan - Sultan
April – Ajay Devgan - Dilwale
May - Aamir Khan - Rustom
January – Salman Khan - Dangal
February – Shahid Kapoor – Dhoom
PUZZLE-187
Directions (6-10): Study the following information carefully and answer the given
questions.
Eight players are stayed in a building from bottom to top floor numbered from 1-8 each
player from a different international cricket team; Australia, South Africa, England, West
Indies, Sri Lanka, Bangladesh, New Zealand and Afghanistan not necessarily in the same
order. Theyplay for different IPL teams; Mumbai Indians, Delhi Daredevils, Royal
Challengers Bangalore, Gujarat Lions, Rising Pune Supergiants, Kolkata Knight Riders,
Sunrisers Hyderabad and Kings XI Punjab not necessarily in the same order. Each
international player plays for one IPL team only. The arrangement is based on the following
rules:
(i) The player from Sri Lankateam is stayed at an even numbered floor
(ii) There are 3 players between the player from Sri Lankateam and the player play for
Kings XI Punjabteam
(iii) The player from West Indiesteam is either at the top or at the bottom floor
(iv) The player who stayed in floor number 2 plays for Mumbai Indiansteam and he is not
from Sri Lankateam
(v) The player plays for Rising Pune Supergiantsteam is stayed between the player from
New Zealandteam and the player plays for Gujarat Lionsteam
(vi) The player from Australiateam is stayed 1floorabove the player from Englandteam
(vii) There are 4 players between the player who plays for Delhi Daredevilsteam and the
player from New Zealandteam
(viii) There are 3 players between the player who plays for Rising Pune Supergiantsteam
and the player who plays for Royal Challengers Bangaloreteam
(ix) The player from Afghanistanteam is stayed between the player who plays for Sunrisers
Hyderabadteam and the player from Bangladeshteam
(x) The player from New Zealandteam does not play for Sunrisers Hyderabadteam.

6.The player from Sri Lanka team is stay in which of the following floor?
8th floor
6th floor
4th floor
2nd floor
None of these

7.How many players are stayed between the player from Australiateam and the player from
West Indies team?
Four
Five
Three
Six
Two
8. Which of the following international team player stayed in 3rd number floor?
Afghanistan
West Indies
Bangladesh
Australia
South Africa
9.Four of the following five are alike in a certain way and hence they form a group. Which one
of the following does not belong to that group?
Delhi Daredevils
Royal Challengers Bangalore
Sunrisers Hyderabad
Rising Pune Supergiants
Kings XI Punjab

10.Which of the following statement is true?


5 – Australia - Rising Pune Supergiants
4 - South Africa -Kings XI Punjab
2 - New Zealand -Mumbai Indians
3 – Bangladesh - Royal Challengers Bangalore
1 - West Indies - Kolkata Knight Riders
PUZZLE-188
Directions (Q.1-5): Study the given information carefully to answer the given question
A show case fixed against the wall of a showroom has six compartments. Each compartment
contains a chain of different e-books vizReasoning, Quantitative Aptitude, English, Computer,
Banking and Current Affairs, but not necessarily in the same order. The Reasoninge-book has
been kept neither on the top nor on the bottom. Only one e-book has been kept between the
Bankinge-book and the Englishe-book. There are two e-books between the Current Affairse-
book and the Reasoninge-book. The Computere-book is just above the Bankinge-book. The
Quantitative Aptitudee-book is below the Reasoninge-book.
1.What is the position of the Quantitative Aptitude e-book when counted from the bottom?
First
Second
Third
Fourth
Cannot be determined

2. What is the position of the Bankinge-book with respect to Current Affairse-book?


Four places above
Four places below
Two places below
Either option a or c
None of these

3. How many e-books are there between the Current Affairse-book and the Quantitative
Aptitudee-book?
None
Three
Four
Either option a or c
None of these

4.How many e-books are there between the Reasoninge-book and the Quantitative
Aptitudee-book?
None
One
Two
Cannot be determined
None of these
5.How many rearrangements are possible based on the given information?
Two
One
Three
Four
None of these

PUZZLE-189

Directions (Q. 6-10): Study the following information carefully and answer the given
questions.
There are seven friends, namely Deepa, Archana, Lakshmi, Divya, Nisha, Shivya and Geeta.
They work in the different banks, ieAxis Bank, HDFC Bank and ICICI Bank. There are at least
two persons in the same bank. Moreover, each of these seven friends has a different salary as
well. No two of them have the same salary. In addition to this, some information is given as
follows.
I. Shivya works in HDFC Bank and earns more than only Geeta.
II. Deepa does not work with Archana and Geeta.
III. Archana stands 4th from the top when arranged in descending order of their salaries.
IV. Nisha is neither from Axis Bank not from the bank where Divya works.
V. Geeta and Lakshmi are in the same bank.
VI. Divya works only with Archana but not in Axis Bank and she earns less than only Lakshmi
and Nisha.
VII. At most three people can work in the same bank.

6.Among the following persons, Deepa works with whom?


Divya
Lakshmi
Nisha
Cannot be determined
None of these

7.Who among the following work in the HDFC Bank?


Shivya, Nisha and Deepa
Shivya, Divya and Archana
Lakshmi, Geeta and Shivya
Shivya, Deepa and Lakshmi
None of these

8.Which of the following is definitely true about Lakshmi?


She works in the HDFC Bank.
She earns the most among all the friends.
She works with Geeta in the ICICI Bank.
All are true
None of these

9.Who amongst them earns the most?


Lakshmi
Nisha
Geeta
Cannot be determined
None of these

10.Which of the following statements is/are false?


Geeta earns more than only Deepa
Nisha works in the ICICI Bank
The Axis Bank has the maximum number of persons working
All are false
None of these
PUZZLE-190
Directions (1-5): Study the following information carefully and answer the given
questions
Seven friends Harish, Laxman, Vinoth, Sanjay, Madhan, Rajesh and Ganeshlikes
different days Sunday, Monday, Tuesday, Wednesday, Thursday, Friday and Saturday.
But not necessarily in the same order. Everybody has different favouritemonths namely
January, February, March, April, May, June and July but not necessarily in the same
order.
I. Harish likes Monday and Sanjay likes Tuesday,
II. Ganesh'sfavouritemonth is July.
III. Laxmanlikes Sunday and his favouritemonth is January.
IV. Rajesh likes Wednesday and Vinoth'sfavouritemonth is March.
V. Madhandoes not like Friday and Rajesh favouritemonth is not April.
VI. The person,whose favourite month is May, likes Thursday
VII. Sanjay and Rajesh favourite month is not February.
VIII. The person who likes Saturday his favouritemonth is July.

1. Which of the following person likes Friday?


Vinoth
Rajesh
Harish
Sanjay
Laxman

2.Ganesh like which of the following days?


Sunday
Saturday
Wednesday
Tuesday
Monday
3. Madhan likes which of the following month?
March
April
May
June
February

4. Four among the following form a group in a certain way. Which of the following
does not belong to Group?
Laxman
Ganesh
Vinoth
Madhan
Sanjay

5. Which of the following statement is true?


Whose favourite month is February, likes Sunday
The one who likes Friday, his favourite month is May
Madhan Likes Saturday
Rajesh favourite month is June
None of these
Name Days Month
Harish Monday February
Laxman Sunday January
Vinoth Friday March
Sanjay Tuesday April
Madhan Thursday May
Rajesh Wednesday June
Ganesh Saturday July
PUZZLE-191

Directions (6-10): Study the following information carefully and answer the given
questions
Eight People – Indran, Kavin, Mani, Gandhi, Ajay, Prasanth, Mishra and Varun live in
eight different floors of building (but not necessarily in the same order). The lowermost
floor of the building is numbered one, the one above that is numbered two, and so on till
the topmost floor is numbered eight. Each one of them also makes awareness about
different days, namely National Technology Day, International Nurses Day, World
Migratory Day, International Day of the Family, World Information Society Day, World
Museum Day, World Metrological Day and Anti-Terrorism Day (but not necessarily in
the same order). Only one person lives between Kavin and the one who makes
awareness about World Information Society Day. Prasanth lives an odd numbered floor
above the floor numbered four. Only three people live between Gandhi and the one who
makes awareness about World Museum Day. Mani lives on one of the odd numbered
floors above the one who makes awareness about World Museum Day. The one who
makes awareness about World Metrological Day lives immediately above Mishra,
Mishramakes awareness about neither World Museum Day nor National Technology
Day. Ajay does not makes awareness about World Metrological Day. Only three people
live between Mishra and Indran. The one who makes awareness aboutAnti-Terrorism
Day lives immediately above the one who makes awareness about International Nurses
Day, but not on the topmost floor. Only one person lives between Prasanth and the one
who makes awareness about World Migratory Day. The number of people living above
Prasanth is same as the number of people living between Prasanth and Gandhi. Only
two people live between Mani and the one who makes awareness about National
Technology Day.

6. On which of the following floor does Varun stay?


3rd
5th
7th
2nd
1st
7.How many floors are there between the floor on which Prasanth stays and the
floor whomakes awareness about International Nurses Day?
None
Three
Two
One
More than three

8. Who amongst the following stays on the top–most floor?


Who makes awareness about International Day of the Family
Mishra
Kavin
Both a and c
None of these

9. Ajaymakes awareness about which of the following days?


World Information Society Day
World Museum Day
National Technology Day
International Nurses Day
World Migratory Day

10. Which of the following statement is true as per the given information?
Varun stays on a floor immediately below the floor on which Mishra stays
Mani stays on a floor immediately above the floor on who makes awareness about
World Museum Day
Indran stays on sixth floor and makes awareness about International Day of the
Family
There are two persons live between Ajay and Kavin
None is true
Floor Person Days

8 Kavin International Day of the Family

7 Mani World Migratory Day

6 Indran World Information Society Day

5 Prasanth World Museum Day

4 Ajay National Technology Day

3 Varun World Metrological Day

2 Mishra Anti-Terrorism Day

1 Gandhi International Nurses Day


PUZZLE-192
Directions (1-5): Study the following information carefully and answer the given
questions.
There are seven national parks namely Jim Corbett, Ranthambore, Kaziranga,
Periyar, Bandipur, Manas and Satpura located in different cities such as Chennai,
Mumbai, Kochi, Hyderabad, Bangalore, Patna & Kolkata, not necessarily in the
same order. Different animals such as, Lion, Tiger, Elephant and Deer were
presented in the park. Depends on the length of the Parks it was listed from 1 to 7
positions, from top to bottom. The following information is known about them.
 Not more than 3 Parks has same animals.
 There's only one park has Deer which belongs to Bangalore and listed in bottom 3
positions and is not Kaziranga Park
 The one belongs to Chennai listed just ahead of Ranthambore Park, which does not
belong to Bangalore.
 The park in which Tiger presented is listed 1st.
 The Parks which listed in 3rd and 4th have same animals and it is not Lion.
 The park located in Mumbai listed in last and park located in Patna listed as 4th and
none of them are has Tiger.
 Manas Park have Lion animal and is not in bottom 3.
 Periyar Park belongs to Hyderabad and Satpura Park belongs to Chennai, both are
not having lion animal and in the list there are two Parks between them.
 The Parks from Kolkata and Mumbai are having same animals.
 Bandipur Park not located in Kochi, Mumbai or Bangalore.

1). Which of the following animal presented in Bandipur Park?


a) Lion
b) Elephant
c) Deer
d) Either Tiger or Lion
e) Either Tiger or Elephant
2). Which of the following park listed on 3rd place from the top?
a) Jim Corbett Park
b) Kaziranga Park
c) Periyar Park
d) Ranthambore Park
e) Satpura Park

3). Manas park located in______________


a) Kolkata
b) Mumbai
c) Bangalore
d) Patna
e) Kochi

4). If ‘Manas’ is related to ‘Ranthambore ’, ‘Periyar ‘ is related to ‘Bandipur’ , then


which following park is ‘Kaziranga’ related to?
a) Bandipur
b) Ranthambore
c) Jim Corbett
d) Satpura
e) Manas

5). Which of the following combinations is true?


a) 3 – Periyar – Hyderabad - Lion
b) 1 – Manas – Kochi -Tiger
c) 2 - Jim Corbett – Bangalore - Deer
d) 7 – Ranthambore – Mumbai - Tiger
e) 4 – Bandipur – Patna – Elephant
PUZZLE-193
Directions (6-10): Study the following information carefully and answer the given
questions.
Six friends - Harish, Mohan, Suresh, Vikram, Ram, and Gopal went to six different
cities namely Mumbai, Bangalore, Hyderabad, Delhi, Chennai and Patna (not
necessarily in the same order) to attend different probationary officers exam. They
exams are IBPS PO, SBI PO, BOB PO, Syndicate Bank PO, Indian Bank PO and
Dena Bank PO. They attend exams from Monday to Saturday (not necessarily in the
same order).
 Harish attends exam in Delhi and he wrote either IBPS PO or Dena Bank PO.
 Ram did not wrote Dena Bank PO exam
 Suresh attends exam on Tuesday and he is not wrote IBPS PO or BOB PO exam.
 The person who wrote the Syndicate Bank PO exam wrote on Saturday and neither
in Mumbai nor in Bangalore.
 Gopal wrote the SBI PO exam in Patna but not on Wednesday.
 Ram attends the exam at Hyderabad and wrote the exam one day before Gopal
 On first day of the week, Mohan attends the exam in Bangalore.
 The person who attends the exam on Thursday is not wrote BOB PO or IBPS PO
exam.
 The person who attends the exam on Thursday is not Gopal.
6). Who wrote the exam in Chennai?
a) Mohan
b) Suresh
c) Ram
d) Vikram
e) Harish

7). Harish wrote the exam on ___________


a) Wednesday
b) Friday
c) Sunday
d) Thursday
e) Monday

8). Which of the following city conduct Indian Bank PO exam?


a) Bangalore
b) Hyderabad
c) Delhi
d) Chennai
e) Mumbai

9). Which of the following exam held on Monday?


a) SBI PO
b) IBPS PO
c) Dena Bank PO
d) Indian Bank PO
e) BOB PO
10). Which of the following combinations is true?
a) a) Ram - Indian Bank PO - Bangalore - Thursday
b) Suresh - Dena Bank PO – Mumbai - Tuesday
c) Mohan –Syndicate Bank PO – Hyderabad - Monday
d) Harish - IBPS PO - Delhi - Friday
e) None of these

PUZZLE-194
Directions (1-5): Study the following information carefully and answer the given
questions.
Eight persons from different states such as Maharashtra, Punjab, Rajasthan,
Haryana, Kerala, Tamil Nadu, Karnataka and Bihar eating 8 different sweets such as
Badam Halwa, Besan laddu, Gulab jamun, Jalebi, Rasgulla, Rava laddu ,Mysore
pak and Kaju katli . And each of them having a mobile in the brand names of
Samsung J2, Samsung J3, Samsung J5 and Samsung J7
 The person who is from Punjab eating Kaju katli as a sweet
 The person who is eating Mysore pak having Samsung J7 mobile
 The third floor stayed person eating Besan laddu and having a mobile in the brand
name of Samsung J2.
 Gulab jamun is eaten by the person who is having Samsung J3 mobile.
 Minimum of three persons stayed below of Bihar state person. And the person who
belongs to Haryana not stayed in the odd numbered floor.
 The mobile’s brand name Samsung J3 and Samsung J7 having same number of
state-men.
 Rajasthan state person stayed two floor below from the person who is belongs to
Punjab.
 The top floor is not occupied by the person from Karnataka.
 The Samsung J5 owned person eating Badam Halwa.
 The persons from Haryana and Punjab having same mobile brand and both of them
not having Samsung J3 mobile and also three persons stayed in between them.
 The persons who are eating Jalebi sweet and Besan laddu are having same mobile
brand with the one who is eating Rasgulla.
 The Samsung J5 owned person stayed in fourth floor. There are four persons
stayed in between of Karnataka state and Haryana state. The lowest floor is
occupied by the Kerala state person.
 Only three persons having one kind of mobile brand name and the person from
Karnataka who is eating Rasgulla is one of them.
 The person from Tamil Nadu and the person who eating Rasgulla are not stayed in
the even numbered floor
 Only one person having Samsung J5 mobile
 The Maharashtran who likes to eat Rava laddu stayed in even numbered floor and
is having same mobile brand name with the person, who is eating Gulab jamun also
stayed in first floor.
 Samsung J7 mobile is owned by the person who is from Haryana.
 The person who eating Kaju katli stayed above the floor of the person who is eating
Mysore pak.
 The person from Tamil Nadu who is eating Besan laddu having same mobile brand
with the state person from Bihar
1). How many persons have Samsung J2 mobiles?
a) One
b) Two
c) Three
d) Four
e) Five

2). Who lives in the sixth floor?


a) The one who is from Punjab
b) The one who is from Haryana
c) The one who is from Karnataka
d) The one who is from Maharashtra
e) The one who is from Bihar

3). Rasgulla sweet eats by which of the following state person?


a) Rajasthan
b) Punjab
c) Haryana
d) Maharashtra
e) Karnataka

4). Which of the following person sits exactly between who are eating Badam Halwa
and Kaju katli?
a) The one who eats Besan laddu sweet
b) The one who eats Mysore pak sweet
c) The one who eats Rava laddu sweet
d) The one who eats Jalebi sweet
e) The one who eats Gulab jamun sweet
5). Which of the following combinations is true?
a) 2 – Gulab jamun – Samsung J7 - Kerala
b) 4 - Badam Halwa - Samsung J5 - Rajasthan
c) 8 – Badam Halwa – Samsung J3 - Kerala
d) 7 - Besan laddu – Samsung J2 - Maharashtra
e) None of these

PUZZLE-195
Directions (6-10): Study the following information carefully and answer the given
questions.
Eight friends Prakash, Arun, Dinesh, Gupta, Vinoth, Ramesh, Karthik and Anand live
on eight different floors of a building but not necessarily in the same order. The
lowermost floor of the building is numbered 1 and the topmost floor of the building is
numbered 8. Each of them likes different months viz, January, February, March,
April, May, June, July and August but not necessarily in the same order.
 The one who likes February lives on an even-numbered floor but not on the topmost
floor
 Only one person lives between Ramesh and the one who likes May.
 Only two persons live between Ramesh and the one who likes February.
 Neither Vinoth nor Dinesh lives on the first floor.
 Only one person lives between Dinesh and the one who likes March.
 Prakash lives just above Ramesh. Only two persons live between Vinoth and
Prakash.
 The one who likes May does not live on floor number one, Arun lives on an even-
numbered floor and just above Dinesh.
 The one who likes April lives on an even numbered floor and lives just above the
person who likes August
 Dinesh does not like May or August. Only two persons live between the one who
likes June and the one who likes January, Gupta does not like July.
 The one who likes June does not live on an odd-numbered floor. Anand lives just
below the one who likes August.

6). How many persons are between the one who likes April and the one who stays
on 4th floor?
a) One
b) Two
c) Three
d) Four
e) Five

7). Karthik likes which of the following month?


a) July
b) March
c) April
d) August
e) January
8). Who lives on fifth floor?
a) Arun
b) Vinoth
c) The one who likes April
d) The one who likes May
e) Both option b and d

9). Which of the following combinations is true?


a) 1 – Gupta - July
b) 4 – Vinoth - February
c) 3 – Ramesh - August
d) 6 – Anand - June
e) 7 – Dinesh - January

10). Four of the following five are alike in a certain way and hence they form a
group. Which one of the following does not belong to that group?
a) Prakash
b) Dinesh
c) Anand
d) Arun
e) Karthik
PUZZLE-196
Directions (1-5): Study the following information carefully and answer the given
questions.
Six friends Ashok, Shivya, Baskar, Lalitha, Gayathri and Ritish stay on a 6 floor
building with the ground floor numbered as floor number 1 and so on. All the friends
have visa for different countries such as; Australia, Russia, USA, Canada, Kuwait
and UK not necessarily in the same order. They also like different Months via;
March, May, June, September, October and December not necessarily in the same
order. The arrangement is based on the following rules:
 The person on the top floor having visa for countries either UK or USA
 The one who likes May lives 2 floors below the one who has Visa for USA
 Gayathri likes December and lives on an even numbered floor
 One who have visa for USA country likes either September or June
 Baskar lives 3 floors below Shivya
 One who lives on 2nd floor has the visa for Canada country and likes neither
December nor May
 Lalitha’s visa country is Kuwait and she lives 1 floor above Baskar
 Baskar does not like Canada
 One who has the visa of Australia country likes October
 One who likes September lives on an odd numbered floor
 Ashok likes June and stays just above the person who likes September
 One who has Kuwait country visa does not like September.

1). Who lives in the second floor?


a) The one who likes March
b) Ritish
c) The one who have Canada country visa
d) All of the above
e) None of these
2). Which of the following country visa and Month like by Baskar?
a) Russia - June
b) Australia - October
c) USA - December
d) UK - May
e) Russia - September

3). Which of the following person like May?


a) Shivya b) Ritish c) Baskar d) Ashok e) Lalitha

4). The one who likes June have which of the following country visa?
a) UK b) USA c) Kuwait d) Russia e) Australia

5). Which of the following combinations is true?


a) 3 – Lalitha – Kuwait - December
b) 1 - Baskar - Canada- October
c) 5 – Shivya – USA - September
d) 6 – Ashok –Kuwait- June
e) None of these
PUZZLE-197
Directions (6-10): Study the following information carefully and answer the given
questions.
Ritaj, Vaishnavi, Divya, Hema, Subha, Shivani, Bhavani, Prathiba, Nithya and Jaya
have some Debit cards and credit cards. Both kinds of cards are 1-10 in number. No
2 persons have the same number of same cards. It is also known that:
 Vaishnavi has 6 debit cards more than Bhavani.
 Vaishnavi has 10 cards in total.
 Jaya has 3 credit cards.
 The total number of credit cards owned by Divya and Nithya is equal to the number
of credit cards that Prathiba has. Divya has less number of credit cards than Nithya.
 Only Prathiba has 18 cards in total such that the number of credit cards he has is
more than his debit cards.
 Only Divya has equal number of debit card and credit cards.
 Only Bhavani has credit cards twice than the debit cards.
 Ritaj has 9 cards in total. He has more credit cards than debit cards.
 The number of debit cards owned by Hema is equal to the number of credit cards
owned by Subha.
 Subha has 8 cards in total.
 Subha and Shivani have 16 credit cards in total.
 Nithya and Jaya have 16 debit cards in total.

6). Who among the following have 10 credit cards?


a) Divya b) Prathiba c) Subha d) Nithya e) Hema

7). Who among the following have 3 debit cards?


a) Prathiba b) Jaya c) Bhavani d) Shivani e) Vaishnavi

8). What is the sum of total number of cards owned by Hema and Nithya together?
a) 25 b) 20 c) 17 d) 22 e) 28
9). If ‘Ritaj’ is related to ‘Hema’, ‘Subha’ is related to ‘Vaishnavi’, then which
following person is ‘Bhavani’ related to?
a) Divya
b) Hema
c) Shivani
d) Ritaj
e) Jaya

10). Which of the following combinations is true?


a) Bhavani – 6 debit cards – 3 credit cards
b) Jaya - 10 debit cards – 8 credit cards
c) Vaishnavi – 1 debit card – 9 credit cards
d) Hema – 7 debit cards – 4 credit cards
e) None of these
PUZZLE-198
Direction (Q.1-5): Study the following information carefully and answer the questions
given below:
Eight persons P, Q, R, S, T, V and W are playing a game of musical chairs and
facing away from the centre. Each person knows a different language, viz Marathi,
Odia, Tamil, Punjabi, Bengali, Hindi, Telugu and English, but not necessarily in the
same order.
 Neither S nor T speaks English.
 R speaks Hindi and sits third to the left of T.
 Those who speak Tamil and Punjabi are immediate neighbours of each other.
Neither the one who speaks Punjabi nor the one who speaks Tamil is an immediate
neighbour of either T or R.
 The one who speaks Punjabi sits on the immediate left of Q, who speaks Odia.
 Those who speak Telugu and English are immediate neighbours of each other.
 P speaks Telugu and is not an immediate neighbour of the one who speaks Hindi.
 V sits third to the left of the one who speaks English.
 Only U sits between the ones who speak Punjabi and Marathi.

1). Who among the following speaks English?


a) R
b) W
c) S
d) V
e) None of these

2). How many persons are there between those who speak Punjabi and Telugu
respectively?
a) None b) One c) Three d) Four e) None of these
3). Who among the following is sitting third to the left of the one who speaks Tamil?
a) The one who speaks Punjabi
b) The one who speaks English
c) The one who speaks Bengali
d) The one who speaks Telugu
e) None of these

4). The one who sits second to the right of W speaks which of the following
languages?
a) Marathi
b) Bengali
c) Telugu
d) Hindi
e) Odia

5). Who among the following sits opposite the one who speaks Odia?
a) P
b) The one who speaks Bengali
c) The one who speaks English
d) V
e) None of these
PUZZLE-199
Directions (Q. 6-10): Study the following information carefully and answer the
questions given below:
Ten persons are sitting in two parallel rows containing five persons in each
row. In row 1— D, E, F, G, and H are sitting and all are facing north. In row 2 — V,
W, X, Y and Z are sitting and all are facing south. Each person also likes a different
TV channel, viz StarPlus, Life OK, Sony, Star Gold, Zee TV, Colors, ETV, SAB TV,
&TV and DD, but not necessarily in the same order.
H sits in the middle of the row and faces the one who likes Zee TV. Only one
person sits between Y and the one who likes Zee TV. Y faces one of the immediate
neighbours of the one who likes Colors. Only two persons sit between E and the one
who likes Colors. E faces the one who likes SAB TV. V faces one of the immediate
neighbours of F. V does not like SAB TV. G is an immediate neighbour of F.
The one who likes Star Gold is an immediate neighbour of V. Z is not an
immediate neighbour of V and faces the One who likes Life OK. X faces the one
who likes &TV. The one who likes DD faces north. V does not face the one who likes
DD. The one who likes Star Plus sits on the immediate right of the one who likes
ETV. Y does not like Star Plus. The one who faces F does not like ETV and G faces
Y.

6). Which of the following pairs represent the people sitting at the extreme ends of
the rows 2?
a) XG
b) YZ
c) WF
d) VD
e) EV
7). Which of the following TV channels does Z like?
a) Star Plus
b) Colors
c) Zee TV
d) ETV
e) SAB TV

8). Which of the following statements is definitely true?


a) F faces X.
b) W likes SAB TV
c) W is an immediate neighbour of V.
d) The one who likes &TV is an immediate neighbour Of V.
e) The one who likes, Sony faces X.

9). Who among the following likes Star Gold?


a) Y
b) V
c) Z
d) X
e) W

10). Who among the following is/are the immediate(s) neighbour of the one who
likes &TV?
a) The one Who likes Sony and G
b) The one who likes ETV and H
c) W and E
d) The one who likes Life OK and H.
e) Z
PUZZLE-200
Directions (Q. 1-5): Study the following information carefully and answer the given
questions.
Ten persons Shami, Kushak, Nanda, Pranav, Akil, Dileep, Jayanth, Ganapati,
Rohan and Vendan are sitting in two rows with five persons in each row. The
persons in row one are facing south and the persons in row two are facing north.
Each person in row one faces a person from the other row. All of them have a watch
of different companies, viz Maxima, Sonata, Matrix, Fossil, Fastrack, Casio, Timex,
Citizen, Titan and Espoir, but not necessarily in the same order.
 The persons who like Fastrack and Casio sit opposite each other.
 Dileep sits opposite to Shami, who likes Maxima.
 The one who likes Sonata sits opposite the one who likes Citizen.
 Vendan is not facing north but sits third to the left of Jayanth, who likes Sonata.
 There is only one person between Kushak and Nanda.
 Akil sits at one of the ends of the row and likes Casio.
 The one who likes Citizen is on the immediate right of Pranav, who does not like
Espoir.
 The persons who like Matrix and Fossil respectively are not facing north.
 Nanda likes Timex. The one who likes Fossil sits opposite the one who is second to
the right of Kushak.
 Rohan does not like Espoir. Akil sits opposite the one who sits second to the left of
the one who likes Matrix.

1). Who have Titan watch?


a) Akil
b) Kushak
c) Pranav
d) Dileep
e) Jayanth

2). Who sits opposite to Kushak?


a) The one who have Maxima watch
b) The one who have Matrix watch
c) Ganapati
d) The one who have Fastrack watch
e) Jayanth

3). Who sits opposite to the one who sits second to the left of Akil?
a) The one who have Matrix watch
b) The one who have Fossil watch
c) The one who have Titan watch
d) The one who have Sonata watch
e) None of these
4). Who sits at the right end in Row II?
a) Ganapati
b) Vendan
c) Jayanth
d) Akil
e) Pranav

5). Who among the following is the immediate neighbour of the one who have
Maxima watch?
a) The one who have Titan watch
b) Nanda
c) Rohan
d) The one who have Sonata watch
e) None of these
PUZZLE-201
Directions (Q. 6-10): Study the following information carefully and answer the given
questions.
Ten Friends are sitting in two parallel rows of six seats each. One seat is vacant in
each row. Bhavya, Janvi, Hemanika, Preeti and Dharini are sitting in row-1 facing
south. Veni, Radha, Charmy, Nisha and Usha are facing north. Each likes a different
brand of Soaps i.e. Hamam, Dove, Lux, Liril, Vivel, Pears, Nivea, Mysore Sandal,
Medimix and Margo. Nisha sits third to the right of Charmy and likes Liril. Only two
people sit between Radha and the vacant seat. Radha does not like Vivel or Lux
Soap. Dharini is not an immediate neighbour of Hemanika. Janvi likes Margo. The
one who likes Lux Soap faces the one who likes Mysore Sandal. The one who likes
Lux sits opposite to the one who sits third right of the person who sits opposite to
Nisha. Hemanika is not an immediate neighbour of Preeti. Usha, who likes neither
Vivel nor Pears, does not face the vacant seat. Neither Nisha nor Charmy sits at any
of the extreme ends of the row. Preeti faces Charmy. Vacant seats are not opposite
to each other. Two seats are there between Hemanika and Janvi, who sits third right
of the one who likes Nivea. The one who likes Medimix Soap faces the one who
likes Liril. The friends who like the Hamam and Mysore Sandal are adjacent to each
other. Vacant seat of row – 1 is not an immediate neighbour of Preeti. Radha sits at
one of the extreme ends of the row. Charmy does not like Hamam and Mysore
Sandal. Vacant seat of row-1 does not face Nisha who doesn’t sit at any of the
extreme ends of the row.

6). How many people will sit between Veni and Radha?
a) One
b) Two
c) Three
d) Four
e) More than four
7). Which of the following brand Usha likes?
a) Margo b) Liril c) Nivea d) Dove e) Medimix

8). Who among the following likes Pears?


a) Usha
b) Nisha
c) Radha
d) Preeti
e) None of these

9). Four among the following form a group in a certain way. Which of the following
does not belong to Group?
a) Dharini, Margo
b) Veni, Lux
c) Radha, Vivel
d) Janvi, Medimix
e) Usha, Liril

10). Which of the following is correctly matched?


a) Preeti – Margo
b) Veni – Vivel
c) Usha – Nivea
d) Radha – Medimix
e) Charmy – Liril
PUZZLE-202
Directions (Q. 1-5): Study the following information and answer the given questions.
Seven people, namely, Pavi, Panav, Pavin, Punit, Poorna, Prithvi and Prajin
like seven different cartoon characters, namely Tweety, Wiene. Superman, Garfield,
Jerry, Ariel and Popeye. Each of them studies in one of the three standards, viz IV,
VIII and XI with at least two of them in a standard.
(Note: None of the information given is necessarily in the same order.)
Only one person studies with Punit in XI standard. Panav studies with the one
who likes Ariel. Punit does not like Ariel. Prajin studies only with the one who likes
Garfield. Neither Panav nor Punit likes Garfield. Pavi likes Popeye. Pavi does not
study with Panav. The one who likes Tweety studies with Pavi. Poorna studies with
the one who likes Jerry. Poorna does not study in IV standard. Neither Panav nor
Prithvi likes Jerry. The one who likes Superman studies with Poorna.

1). Who among the-following likes Wiene?


a) Poorna
b) Prithvi
c) Punit
d) Prajin
e) Panav

2). Which of the following cartoon characters does Pavin like?


a) Superman
b) Ariel
c) Tweety
d) Jerry
e) Garfield

3). Which of the following statements is true?


a) Both Punit and Pavi study in the same standard.
b) Panav studies in IV standard.
c) Pavi studies with the one who likes Wiene
d) More than two people study in the standard in which Prithvi studies.
e) None of the given statements is true

4). Which of the following combinations represents the combination of people


studying in VIII standard?
a) Poorna and the one who likes Wiene
b) Pavi, Prithvi
c) Panav, Pavin, Poorna
d) Panav and the one who likes Tweety
e) None of these

5). Four of the following five are alike in a certain way based on the given
arrangement and hence form a group. Which of the following does not belong to that
group?
a) Prajin, Punit
b) Pavin, Panav
c) Panav, Poorna
d) Pavin, Poorna
e) Prithvi, Prajin

6). Which of the following combinations represents the standard in which Prithvi
studies and the cartoon character she likes'?
a) VIII - Garfield
b) VIII - Superman
c) XI - Tweety
d) IV - Ariel
e) IV - Garfield
Question 1-5

Eight boxes – P, Q, R, S, T, U, V and W are placed one above the another but not necessarily in
the same order.
Three boxes are placed between S and Q. Two boxes are placed between T and Q. Two boxes are
placed between P and W. W is placed immediately below Q. Two boxes are placed between R
and V. Two boxes are placed between P and U. Box V is not placed at the bottom.

1. How many boxes are placed between S and R?

 two
 none
 three
 five
 one

2. Which box is at bottom most position?

 R
 Q
 W
 V
 Can't be determined

3. Which box is placed just above box W?

 S
 P
 R
 V
 Q

4. How many boxes are there between P and R ?

 2
 1
 4
 5
 3

5. Who is the immediate neighbour of box T ?

 S
 Q
 R
 V
 W

Question 6-10
A, B, C, D, E, F and G are seven different boxes of different colours i.e. Black, Orange, Red,
Pink, Yellow,
White and Green but not necessarily in the same order.
Box B is not placed below D. Box which is of Black colour is immediately above A. There are
Only two box between D and the box which is of Black colour and Box D is above the Black
colour box. Box which is of Red colour is above D but not immediately above D. Only three box
are between C and the box which is of Red colour. The box which is of Green colour is
immediately above C. The box which is of Pink colour is immediately above box G. Only one box
is there between B and E. Neither box B nor A is of Yellow colour. A is not of Orange colour.

6. How many boxes are placed between green colored box and F?

 none
 one
 two
 three
 four

7. Four of the following five are alike in a certain way and so form a group. Which one of the
following does not belong to the group?

 Black box – G
 Orange box - D
 Green box – C
 Red box – B
 Yellow box – E

8. Which of the following box is of yellow colour?

 A
 E
 B
 D
 None of these

9. Which box is placed exactly between Black colored box and E?

 green colored box


 orange colored box
 yellow colored box
 pink colored box
 None of these.

10. Which of the following Statements is true with respect to the given information?

 Yellow colored box is placed immediately above Red box


 Black box is the top.
 Orange colored box is placed immediately above D
 Three boxes are placed between red colored box and E.
 None of these.

Solution of Question 1-10


Ans 1-5
Ans 1 d
Ans 2 a
Ans 3 e
Ans 4 e
Ans 5 a
Ans 6-10

Ans 6 c
Ans 7 a
Ans 8 d
Ans 9 d
Ans 10 c

Direction (Q.1-5): Study the following carefully and answer the question given below:
Eight boxes A, B, C, E, F, G, M and N are placed one above the another but not necessarily in
the same order. Each of them has different colours i.e. Black, Orange, Red, Green, Yellow,
Purple, Blue and Brown.
1. There are only three boxes between B and M.
2. Yellow and black coloured boxes are not on the consecutive places.
3. A box which is immediate above yellow coloured box is neither blue nor purple coloured box.
4. G is an orange coloured box but not on the top.
5. There are only two boxes below red coloured box.
6. Blue coloured box is above purple coloured box.
7. Neither M nor F is a purple coloured box.
8. There are only two boxes between orange and black coloured box.
9. Number of boxes between A and F is same as the number of boxes between A and N.
10. Yellow coloured box is one place above box C.
11. Box B and box G are on the consecutive places.
12. Green coloured box is immediate below box M.
13. Orange and Black coloured boxes are above Red coloured box.

Q.1 Which of the following box has blue colour?


(A) B
(B) C
(C) M
(D) G
(E) E

Q.2 If all the boxes arranged in alphabetically order from the top, how many boxes will
remain their original position?
(A) None
(B) 1
(C) 4
(D) 3
(E) 2

Q.3 Which of the following pair is correct?


(A) F - Brown
(B) E - Orange
(C) B - Yellow
(D) A - Black
(E) None of these

Q.4 Box M has which of the following colour?


(A) Blue
(B) Purple
(C) Black
(D) Red
(E) None of these

Q.5 Four of the following five are alike in a certain way and hence form a group. Which of
them does not belong to the group?
(A) E - B
(B) C - M
(C) N - A
(D) B - N
(E) G - N

Answer 1. (E) E
Answer 2. (B) 1
Answer 3. (C) B - Yellow
Answer 4. (E) None of these
Answer 5. (D) B - N

Detailed Explanation:
8 E Blue
7 G Orange
6 B Yellow
5 N Purple
4 C Black
3 A Red
2 M Brown
1 F Green
Step Wise Solution:
1. There are only three boxes between B and M.
12. Green coloured box is immediate below box M.
11. Box B and box G are on the consecutive places.
4. G is an orange coloured box but not on the top.
8. There are only two boxes between orange and black coloured box.
13. Orange and Black coloured boxes are above Red coloured box.
5. There are only two boxes below red coloured box.
2. Yellow and black coloured boxes are not on the consecutive places.
3. A box which is immediate above yellow coloured box is neither blue nor purple coloured box.
10. Yellow coloured box is one place above box C.
9. Number of boxes between A and F is same as the number of boxes between A and N.
6. Blue coloured box is above purple coloured box.
7. Neither M nor F is a purple coloured box.

Directions (1-5)Read the given information carefully and answer the given questions.

There are five units i.e 1,2,3,4 and 5 .Each unit has a different height. Also each unit contains
books and boxes. Unit 2 is above Unit 1 and unit 3 is above Unit 2 and so on. Every unit belongs
to different city i.e Beijing, Paris, London, Sydney and Zurich. The total height of all five units is
equal to 252 ft.
• Total height of unit is equal to the total height of books plus total height of boxes in each unit.
• Height of books is not equal to the height of boxes, unless specified.
The books belongs to London is an even unit. The total height of Unit 1 is 75 ft. Sydney is not a
Unit 1.The total height of unit, which belongs to Sydney, is 55 ft. There is only one gap between
London and Paris. The height of books and height of boxes in unit 3 are equal. The height of
books in Unit 2 is not less than 30 ft. The books is Unit 4 are four more than books, which is in
Unit 3.The total height of London is not 37 ft. The total height of unit, which contains 37 ft is not
taking the place which is immediately above a unit, which contains 20 ft more than the unit,
which belongs to Sydney. The height of boxes in Unit 2 is 23 ft. Unit belongs to Zurich does not
contain the equal height of books and boxes. The total height of Unit 2 is an odd number and
height is more than 50 ft and less than 55 ft. Unit 1 contains 23 ft height of boxes more than Unit
4 contains height of boxes and Unit 5 contains 7 ft height of boxes less than Unit 1 contains.
1)What is the total height in Unit 3 ?
A)37 ft
B)32 ft
C)53 ft
D)75 ft
E)80 ft

2)Unit 3 belongs to which city ?


A)paris
B)Zurich
C)Sydney
D)Beijing
E)London

3)If’Sydney’ is related to 37 ft in the same way as Bejing is elated to 53 ft. Which of the following
is ‘Paris’ related to, following the same pattern ?
A)53 ft
B)37 ft
C)75 ft
D)32 ft
E)30 ft

4)Four of the following five are alike in a certain way ad hence they form a group . Which one of
the following does not belong to that group ?
A)Sydney
B)32 ft
C)75 ft
D)paris
E)37 ft

5)What is the eight of box in Unit 4 ?


A)23 ft
B)17 ft
C)27 ft
D)40 ft
E)20 ft

6)Which unit contains 30 ft book ?


A)Unit 2
B)Unit 3
C)Unit 1
D)Unit 5
E)Unit 4

New pattern Box Puzzle Questions : Answers with Explanations


1.B)
2.D)
3.D)
4.B)
5.B)
6.A)

Unit Total height(252) Box Book City


5 55 27 28 Sydney
4 37 17 20 Paris
3 32 16 16 Beijing
2 53 23 30 London
1 75 40 35 Zurich

Directions (1-5)Read the given information carefully and answer the given questions.

Seven boxes-A,B,C,D,E,F and G are kept one above the other, but not necessarily in the same
order. Each Box has a different number Viz. 11,14,15,17,18,19 and 22 but not necessarily in the
same order.
Only. three boxes are kept between G and box number 19. Only two boxes are kept between G
and B . B is kept at one of the positions below box number 19. Only one box is kept between B
and box number 14. E is kept immediately below box number 22.E is kept at one of the places
above box number 19. There is only one box between E and the box having number less than E .
E’s box number is neither 17 or 18 .Only two boxes are kept between box number 15 and F .
Thee difference between F and the box immediately below it is less than four .C is not the
topmost box. C’s box number is not 4 . Only two boxes are kept between C and A.

• 1)What is the number of box C ?


A)15
B)19
C)22
D)18
E)11

• 2)How many boxes are kept between E and box number 14 ?


A)3
B)1
C)2
D)More than 4
E)5

• 3)What is the position of D in the given stack of boxes ?


A)Third form the top
B)Fifth from the top
C)First from the top
D)Third from the bottom
E)Fourth from the bottom

• 4)Which of the following boxes is kept immediately above A ?


A)B
B)Box number 5
C)Box number 7
D)G
E)Box number 4

• 5)Four of the following five are alike in a certain way and hence form a group . Which of the
following does not belong to the group ?
A)B-4
B)E-2
C)C-7
D)D-9
E)A-8

• 6)Which of the following boxes is numbered 8 ?


A)A
B)B
C)F
D)G
E)D

Box Type Reasoning Puzzle Questions : Answers


1.B
2.A
3.C
4.C
5.E
6.D

Box Number
D 22
E 15
C 19
B 11
F 17
A 14
G 18

Directions (Q. 1-5): Study the given information carefully to answer the given question.
P, Q, R, S, T, U, and V are seven different boxes of different colors i.e. Black, Orange, Red,
Pink, Yellow, White, and Green but not necessarily in the same order.
Box Q is not placed below S. Box which is of Black color is immediately above P. There are only
two boxes between S and the box which is of Black color and Box S is above the Black color box.
The box which is of Red color is above S but not immediately above S. Only three boxes are
between R and the box which is of Red color. The box which is of Green color is immediately
above R. The box which is of Pink color is immediately above box V. Only one box is there
between Q and T. Neither box Q nor P is of Yellow color. P is not of Orange color.

Q: 1 How many boxes are placed between the green colored box and U?
(a) One (b) Two (c) Three
(d) Four (e) None

Q: 2 Four of the following five are alike in a certain way and so form a group. Which one of
the following does not belong to the group?
(a) Black box – V (b) Orange box – S (c) Green box – R
(d)Red box – Q (e) Yellow box – T

Q: 3 Which of the following box is of yellow color?


(a) P (b) Q (c) R
(d) S (e) T

Q: 4 Which box is placed exactly between the Black colored box and T?
(a) Yellow box (b) Orange box (c) Green box
(d) Pink box (e) None of these

Q: 5 Which of the following Statements is true with respect to the given information?
(a) Yellow colored box is placed immediately above Red box
(b) Black box is the top
(c) Orange colored box is placed immediately above S
(d) Three boxes are placed between red colored box and T
(e) None of these
Sol: 1-5:
Box Colour
U Red
Q Orange
S Yellow
T Green
R Pink
V Black
P White

1. (b)
2. (a)
3. (d)
4. (d)
5. (c)

Directions (1-5): Study the information and answer the given questions:
There are eight boxes viz. A, B, C, D, E, F, G and H. All the box are arranged from top to
bottom.
There are four box between C and H. There are two box between H and A. There are three box
between A and F. There are two box between F and B. There is only one box between B and D.
The box D is not placed immediately above or immediately below the F. There are gap of two
box between E and G. The box E is placed one of the above on H but not on the top.

Q1. Which of the following box is at the bottom?


(a) G
(b) B
(c) C
(d) D
(e) E

Q2. Which of the following box is immediately above the D?


(a) B
(b) E
(c) H
(d) C
(e) A
Q3. How many boxes are placed between the box D and H ?
(a) Four
(b) Three
(c) Two
(d) One
(e) None of these

Q4. How many boxes are placed below the box E?


(a) One
(b) Two
(c) Three
(d) No one.
(e) Four

Q5. If C is related to B and A is related to H in certain way, then D is related to ?


(a) B
(b) E
(c) H
(d) C
(e) A

Solution (1-5):

S1. Ans.(a)
Sol.

S2. Ans.(d)
Sol.

S3. Ans.(b)
Sol.

S4. Ans.(c)
Sol.

S5. Ans.(b)
Sol.

Directions (1 – 5): Study the following information to answer the given questions:
There are 8 boxes which are colored with different colors – white, pink, green, blue, yellow, red,
orange and brown. Each of them contains a number written on them from 1 to 8 according to
which they are placed one above the other. The box containing number 1 is at the lowest place.
Each box has different sweets namely – Rasgulla, Laddu, Peda, Nankhatai, Jalebi, Gulab Jamun,
Petha and Burfi but not necessarily in the same order.
Only one box is there between pink box and the one which contains Jalebi. Number 4 is written
on the brown box. The number on red colored box is odd and is placed somewhere above the box
numbered four.
There are three boxes between blue box and the one which contains Gulab Jamun. The number
written on Green box is odd and is placed above the box which contains Gulab Jamun.The box
which contains Petha is placed immediately above orange colored box, which contains neither
Gulab jamun nor Rasgulla. The yellow box does not contain Petha. There are three boxes placed
between orange and white boxes. The box which contains Burfi is placed immediately above the
one which contains Laddu, but not at the topmost position. Only one box is there between red
colored box and the one which contains Peda. The number of boxes placed above red colored
box is same as the number of boxes placed between blue and red colored boxes. Only one box is
there between the one which contains Burfi and brown colored box. Two boxes are placed
between green colored box and the one which contains Rasgulla.

Which of the following Statements is true with respect to the given information?
A) Orange colored box is placed immediately above the one which contains Rasgula
B) Yellow colored box is placed immediately above green box
C) Three boxes are placed between red colored box and the one which contains Burfi.
D) Blue box contains Jalebi.
E) Number 7 is written on green colored box

Answer
Option E
Arrangement:

Which box is placed exactly between brown colored box and the one which contains Laddu?
A) yellow and red colored boxes
B) orange and blue colored boxes
C) pink and white colored boxes
D) green and red colored boxes
E) yellow and white colored boxes
Answer
Option E

Which of the following sweets does yellow box contain?


A) Burfi
B) Laddu
C) Peda
D) Jalebi
E) Petha
Answer
Option C

Four of the following five are alike in a certain way and so form a group. Which
one of the following does not belong to the group?
A. orange box – Nankhatai
B. green box – Gulab jamun
C. red box – Jalebi
D. brown box – Burfi
E. blue box – Peda
Answer
Option C

How many boxes are placed between green colored box and the one which contains Peda?
A) None
B) One
C) Two
D) Three
E) More than three
Answer
Option D

Directions (6 – 10): Study the following information to answer the given questions:
There are 7 boxes which are colored with different colors – White, Blue, Yellow, Green, Red,
Brown and Pink. They are placed one above the other. And contain different flavored cupcakes –
Mint Oreo, Chocolate, Blueberry, Mango, Lemon, Apple Pie and Cranberry, but not necessarily
in the same order.
Further information is as follows:
(i) Boxes containing Either Mango or Cranberry cupcake is on the top or first place respectively.
(ii) The brown box is placed on 5th place from bottom.
(iii) The boxes containing Chocolate and Apple Pie cupcakes are placed consecutively.
(iv) White and Red colored boxes contain Mint Oreo and Lemon cupcakes respectively.
(v) Yellow and Brown boxes are placed at consecutive places.
(vi) Blue box, which contains Chocolate cupcake, and White box have one box in between them.
(vii) Yellow box, which contains Blueberry cupcake is placed just above the box containing
Apple Pie cupcake.

Box containing which cupcake is immediate next to box containing Lemon cupcake?
A) Chocolate
B) Mint Oreo
C) Either (A) or (B)
D) Blueberry
E) Mango
Answer
Option C

Brown colored box contains which flavored cupcake?


A) Apple Pie
B) Lemon
C) Chocolate
D) Either (A) or (B)
E) Cranberry
Answer
Option A

If the box at the lowest place is colored Green, then which colored box contains Mango
cupcake?
A) Green
B) White
C) Red
D) Cannot be determined
E) None of these
Answer
Option D

The box containing Blueberry cupcake is at which position from below?


A) Second
B) Third
C) Sixth
D) Fourth
E) Fifth
Answer
Option C

If the topmost box contains Cranberry cupcake, then how many boxes are placed between
Chocolate and Cranberry cupcakes?
A) Two
B) Four
C) One
D) Three
E) None of these
Answer
Option A
Directions (1 – 3): Answer the questions on the basis of the information given below.
8 boxes – A, B, C, D, E, F, G and H are placed one above the another but not necessarily in the
same order.
Three boxes are placed between D and B. Two boxes are placed between E and B. Two boxes
are placed between A and H. H is placed immediately below B. Two boxes are placed between C
and G. Two boxes are placed between A and F.
How many boxes are placed between D and C?
A) Two
B) None
C) Three
D) Five
E) One
Answer
Option A
Arrangement:
Three boxes are placed between D and B. Two boxes are placed between E and B. H is placed
immediately below B. 4 possibilities:
– – – E
– – – –
– – – –
D D B B
E – H H
– – – –
– – E –
B B D D
H H
– –
– E
Two boxes are placed between A and H. Two boxes are placed between A and F.
– – – E
– – A A
F – – –
D D B B
E – H H
A A – –
– – E –
B B D D
H H
– –
– E
Only 1st possibility is possible. And now Two boxes are placed between C and G.
So from top to bottom – F > D > E > A > C/G > B > H > C/G

If C is placed above G, which box is at bottom most position?


A) B
B) C
C) H
D) G
E) Cannot be determined
Answer
Option D

Which box is placed just above box H?


A) A
B) D
C) G
D) B
E) C
Answer
Option D

Directions (4 – 6): Answer the questions on the basis of the information given below.
8 boxes – A, B, C, D, E, F, G and H are placed one above the another but not necessarily in the
same order.
Two boxes are placed between F and E. F is placed above E. One box is placed between F and
G. Three boxes are placed between A and H. A is placed immediately below F. Two boxes are
placed between C and H. B is placed somewhere above D.
How many boxes are placed between F and H?
A) Four
B) None
C) Three
D) Five
E) One
Answer
Option A
Arrangement:
Two boxes are placed between F and E. F is placed above E. One box is placed between F and
G. A is placed immediately below F.
– G
– –
F F
A A
G –
E E
Three boxes are placed between A and H. Two boxes are placed between C and H. 1st possibility
cancels out
. H H
. – G G
. – – –
F F F F
A A A A
G G C –
E E E E
– –
H H
1st, 2nd, 4th possibility cancels out.
B is placed somewhere above D.. So from top to bottom – G > B > F > A > C > E > D > H

Which box is placed at top?


A) B
B) C
C) H
D) G
E) Cannot be determined
Answer
Option D

Which box is placed just below box A?


A) E
B) D
C) G
D) B
E) C
Answer
Option E

Directions (7 – 10): Answer the questions on the basis of the information given below.
8 boxes – A, B, C, D, E, F, G and H are placed one above the another but not necessarily in the
same order.
There are four boxes placed between D and G. Two boxes are placed between B and G. Number
of boxes between A and G is same as between H and B. A is placed above G. Two boxes are
placed between A and H. C is placed just above G. There are at least 2 boxes between E and B.
Which box is at top most position?
A) C
B) A
C) E
D) H
E) None of these
Answer
Option D
Arrangement:
4 between D and G. and 2 between B and G.C just above G. So 2 possibilities as:
– C
D G
– –
B –
– B
C –
G D
Now — Number of boxes between A and G is same as between H and B. A is placed above G.
Two boxes are placed between A and H. Gives
H A
– C
D G
A H
B –
– B
C –
G D
Now: There are at least 2 boxes between E and B.. So 2nd possibility cancels out
From top to bottom – H > W > D > A > B > F > C > G

How many boxes are between boxes E and A?


A) None
B) Three
C) One
D) Five
E) Four
Answer
Option C

Which box is placed just above box B?


A) A
B) C
C) E
D) H
E) D
Answer
Option A

How many boxes are below box F?


A) Three
B) Four
C) None
D) Two
E) Six
Answer
Option D

Eight boxes are arranged in a stack from bottom to top numbered from 1-8 each having a
different colour; green, black, blue, white, yellow, orange, red and pink not necessarily in the
same order. They contain a different food item; Milky bar, Lollypop, Ice cream, Pizza, Butter
chicken, Curd rice, Grill chicken and Bovonto not necessarily in the same order. The
arrangement is based on the following rules:
• The yellow box is kept at an even numbered position
• There are 3 boxes between the yellow box and the box with Bovonto
• White box is either at the top or at the bottom of the stack
• Box number 2 has Milky bar and it is not yellow in colour
• Box with the Butter chicken is kept between the box having red colour and the box having Pizza
• The Green coloured box is kept 2 boxes above the blue box
• There are 4 boxes between the box with Lollypop and the red coloured box
• There are 3 boxes between the box with Butter chicken and the box with Ice cream
• Pink box is between the box with Grill chicken and the orange coloured box
• The red box does not have Grill chicken.

(1). Which of the following is the correct combination of number: colour: food item?
a) 8: yellow: Butter chicken b) 6: red: Curd rice c) 5: green: Grill chicken d) 4: blue: Pizza e) 3:
orange: Lollypop

(2). Four of the following bear a certain relationship hence form a group, which of the following
does not belong not belong to that group?
a) Pizza b) Curd rice c) Bovonto d) Ice cream e) Milky bar
(3). If green is related to Pizza and red is related to Butter chicken then which of the following
food items is related to blue?
a) Curd rice b) Grill chicken c) Bovonto d) Ice cream e) Lollypop

(4). Which of the following food items does the black and orange box contains?
a) Milky bar, Lollypop b) Milky bar, Ice cream c) Ice cream, Bovonto d) Bovonto, Pizza e) Curd
rice, Grill chicken
(5). How many boxes are kept between the green box and the box having Curd rice?
a) None b) One c) Two d) Three e) Four
Directions (1 – 5): Answer the questions on 3. Who is sitting exactly between Karan and
the basis of the information given below. the one who likes strawberry?
There are eight friends – Ritika, Sohail, Karan, A) Riya
Saurabh, Neha, Shivam, Riya and Shreya. They B) Saurabh
have different cars – Wagon-R, I20, Ertiga, C) Sohail
Nissan, Alto, Santro, Audi and Suzuki, but not D) Ritika
necessarily in the same order. They like E) None of these
different fruits – apple, strawberry, banana, 4. Which of the following is true regarding
melon, papaya, mango, orange and grapes but Shreya?
not necessarily in the same order. A) She is sitting second to right of Karan
They all are sitting around a circular table with B) She is facing inside.
equal people facing inside and outside. C) She is facing Riya.
The one having Nissan is sitting third to left of D) She does not like Papaya
one having Wagon-R whose name is not Neha E) None of these is correct
and does not like banana. The one having 5. Which of the following pair is incorrect?
Ertiga is facing outside and sitting between the A) I20 – Riya – Banana
ones having Audi and Santro who like mango B) Santro – Ritika – mango
and papaya respectively. Sohail is sitting to the C) Alto – Sohail – Melon
immediate left of the one having Wagon-R who D) I20 – Sohail – grapes
does not like melon and grapes. Shreya is E) Suzuki – Neha – grapes
sitting second to right of the one who likes
mango. The one who likes grapes is facing Directions (6 – 10): Study the following
inside and sitting second to left of the one who information to answer the given questions:
likes melon and he is also sitting third to right There are nine members in the family i.e. A, B,
of Ritika. C, D, E, F, G, H and I. Each person likes different
The one having Ertiga is sitting second to right colors viz. Brown, Green, Black, Pink, Blue, Red,
of the one having Nissan whose name is Karan Yellow, Orange and White. All members of the
and likes apple. Shivam who likes strawberry is family have a relation with A which can be
neither Wagon-R nor Suzuki. Ritika is not defined as Father, Mother, Brother, Sister, Wife,
sitting to the immediate left of Karan. Both the Daughter, Son, and Brother-in-law but not in
neighbors of the one having I20 are facing the same order. They are seating around a
inside. The one having Santro is facing inside. circular table but not necessarily in the same
The ones having I20 and Ertiga face same order. Some of them are facing towards the
direction. The one having Alto is to the center while others are facing away from the
immediate right of Riya and he is also sitting center.
second to right of the one having Suzuki. The A’s brother is seating on the immediate right of
one who likes orange is sitting second to left of the brother of A’s sister and they both face the
the one having Audi. same direction. I is brother of A’s daughter. I’s
father likes Green color. H has a sister. The
1. The one who likes melon is mother of A’s son is E and sits second to the left
A) Sohail of the married son of A’s father. F’s daughter-
B) Saurabh in-law likes White color. A is facing away from
C) Ritika the center. F is a male. A’s father sits third to
D) Shreya the right A’s daughter. Either I or H sits third to
E) Neha the left of the mother of C, but both I and H face
the same direction.
2. Who is sitting second to right of the one A’s brother and sister likes Black and Brown
having I20? color respectively. G is not father of H. In two
A) the one having Ertiga generations, the members of the same
B) Shreya generation are sitting immediate next to each
C) the one having Nissan other and face the same direction. A’s father
D) the one who likes orange does not like Orange or Yellow color. F’s
E) Cannot be determined.
grandson likes Pink color. B is a female and of a same direction are not adjacent to each other.
the same generation as of A. G sits on the They have different sports hobbies –
immediate left of the granddaughter of A’s Basketball, Tennis, Chess, Hockey, Cricket,
father. The one, who is the wife of F likes Football, Kabaddi, and Badminton, but not
Yellow color. A’s brother-in-law neither likes necessarily in the same order. They are from
Orange color nor blue color. A’s father doesn’t different states – Punjab, UP, Haryana, J&K,
sit immediate next to his children. G is facing Rajasthan, Gujarat, Jharkhand and Delhi, but
the centre and is second to the right of A. B sits not necessarily in the same order.
second to the left of F. No three members G is sitting second to the left of the one who is
seating together can face the same direction. from Delhi. The one who plays cricket is
opposite the one who is from UP and they are
6. Who sits third to the right of A’s not facing each other. C is from Gujarat and
mother? plays Chess. The one from Haryana is second to
A) B the right of G. A who plays Hockey is third to
B) H’s brother-in-law the right of B who is from Punjab and both are
C) A’s brother-in-law facing same direction. The one who plays
D) E’s husband Football is immediate to the right of one who is
E) Both B and C from Delhi. G is adjacent to the one who plays
Cricket and the one from UP does not play
7. Who among the following sits exactly Football. The one from Jharkhand plays
between A and his wife? Badminton and is opposite to C and he faces
A) F back of C. E who plays Kabaddi is third to the
B) A’ sister right of one from Delhi. F is sitting second to
C) A’s daughter the left of G. E is sitting opposite the one who
D) D plays Cricket. The one from J&K faces inside
E) None of these and is fourth to left of D who is from Rajasthan.
8. Four of the following five are alike in a H is sitting to the immediate right of the one
certain way so form a group. Which of the who plays Tennis.
following does not belong to the group?
A) B 11. H plays which of the following games?
B) A A) Cricket
C) A’s mother B) Hockey
D) A’’s daughter C) Badminton
E) E D) Football
9. Which of the following persons is facing E) Cannot be determined
towards the center? 12. Who is third to the left of F?
A) B, G, F, I A) E
B) A, G, D, I B) The one who is from Rajasthan
C) G, D, A’s father, G’s wife C) The one who is from Haryana
D) D, G, B, C D) The one who plays Chess
E) None of these E) The one who plays Cricket.
10. Which of the following colors does H like? 13. Which of the following pairs of persons is
A) blue facing the same direction?
B) red A) C, G
C) pink B) A, D
D) brown C) A, G
E) black D) F, H
E) None from above is facing same direction
Directions (11 – 15): Study the following 14. Which of the following triplet is correct as
information to answer the given questions: per given arrangement?
Eight friends – A, B, C, D, E, F, G and H are A) H – Football – Jharkhand
sitting around a circular table in which half are B) F – Basketball – UP
facing outside and other half inside. All 4 facing C) E – Hockey – Haryana
D) G – Badminton – Jharkhand floor. The person belongs to Maharashtra does
E) G – Hockey – Jharkhand not live below the person who belongs to
15. Which of the following is correct with Punjab. Ankul does not like Yellow color.
respect to the arrangement?
A) G is facing outside. 16. Who stays on 8th floor?
B) E is sitting second to right of D. A) Charu
C) H is sitting second to the left of E. B) The one studying BCom
D) There are three persons between B and H. C) The one who likes Purple color
E) None of the above is correct. D) The one who likes White color
E) The one from Haryana
Directions (16 – 20): Answer the questions
on the basis of the information given below. 17. How many floors are there between the ones
Nine people Ankul, Priyal, Seema, Abhi, Rajat, who like green color and who is from
Charu, Reema, Anjali and Gaurav stay in a Maharashtra?
building (floors numbered 1 to 9). They are A) None
studying different courses – BSc and BCom. B) Two
Four of them are studying BCom and remaining C) Four
persons are studying BSc. All of them belong to D) One
a different state viz – Haryana, Punjab, Assam, E) None of these
Nagaland, Telangana, Kerala, Jharkhand, 18. Who is staying 4 floors above the one from
Maharashtra and Bihar, but not necessarily in Assam?
the same order.. Each of them also likes a A) The one from Kerala
different color – Orange, Grey, Pink, Purple, B) The one who likes purple color
White, Blue, Green, Red and Yellow, again but C) The one from Telangana
not necessarily in the same order. D) Ankul
E) The one who likes grey color
There is one floor between the floors on which 19. Four of the following five are alike in a
Charu, who likes Grey color and Reema stay. certain way and forms a group. Find the one
Abhi, who likes Blue, stays on a floor who does not belong to the group?
immediately above Gaurav’s floor, who likes A) Priyal
Green color. Neither Ankul nor Seema belongs B) The one from Telangana
to Telangana. Seema does not belong to C) The who likes red color
Haryana and likes Red color. The one who D) The one from Nagaland
belongs to Jharkhand stays on the fourth floor E) Gaurav
but is not studying BSc. There are two floors 20. Which of the following is correct
between the floors on which the people who combination as per given arrangement?
belong to Kerala and Nagaland stay. Rajat stays A) Anjali – Nagaland – Grey – BSc
on the second floor and belongs to Assam. B) Ankul – Telangana – Grey – BCom
There are three floors between the floor on C) Reema – Kerala – Pink – BSc
which Seema, who is studying BCom and D) Seema – Assam – Red – BCom
Reema stay, who does not belong to Bihar and E) Priyal – Haryana – Orange – BSc
likes Pink color. The person who belongs to
Telangana and Haryana are studying same Directions (21 – 25): Answer the questions
course. Ankul is studying BSc and lives on an on the basis of the information given below.
even numbered floor which is below the floor There are 12 professors – A, B, C, D, Q, R, S, and
on which Anjali stays, who likes purple color. T. They have conferences in different months –
The one who belongs to Bihar stays on the January, March, April, June, August, and
third floor and is not studying BCom. The one October. The conferences are on dates 6th or
who belongs to Haryana stays on the topmost 21st of a given month. Each conference is
floor and likes Orange color. The person attended by some number of people. If the
belongs to Kerala and Nagaland are studying month contains odd number of days then
same course. Anjali belongs to Nagaland and is number of people attending the conference is
studying BSc and lives on an even numbered odd. Like conference in March (31 days) was
attended by odd number of people and D) 49
conference in April (30 days) was attended by E) 44
even number of people. 22. Which of the following pair had conferences
in August?
A had conference on June 6. Q’s conference was A) U, T
before A. Q’s conference was attended by 55 B) D, T
people and he had conference on 21 st of a C) P, E
month. There was one conference between Q’s D) D, F
and S’s conference. The number of people E) C, A
attending S’s conference was 6 less than the 23. Who had conference on 21st June?
number of people who attended Q’s conference. A) S
There were 3 conferences between D’s B) B
conference and the conference attended by 26 C) R
people. D had conference before the conference D) Other than those given in options
attended by 26 people. D’s conference was not E) A
in January. There were 2 conferences between 24. Which of the following is true with respect to
T’s conference and the conference attended by the given arrangement?
26 people. The number of people who attended A) There are 3 conferences between the
T’s conference was greater than the number of conferences of A and F
people who attended S’s conference but less B) The total number of people attending the
than the number of people who attended Q’s conferences of B and R is 44
conference. The number of people who C) The conference of U is in October.
attended T’s conference was a multiple of 3. E D) The total number of people attending the
and P had conference in same month but not conferences of F and R is 34
April. There was one conference between the E) A and C have conferences in same month
conferences of E and U. There were 2 25. How many conferences are there which are
conferences between F’s conference and the attended by less number of people than the
conference attended by 16 people. F had conference of A if conference of C was
conference before the conference attended by attended by 28 people?
16 people. The total number of people who A) One
attended the conferences of U and Q was twice B) Three
the total number of people who attended the C) Seven
conferences of A and B. Conference of C and D) Four
that attended by 18 people was in same month. E) Six
The total number of people who attended the
conferences of E and P was 20. The number of Directions (26 – 30): Answer the questions
people who attended E’s conference was on the basis of the information given below.
greater than the number of people who There are nine members in a family – P, Q, R, S,
attended the conference of F and P. The T, U, V, W, X. They all are sitting in a straight
number of people who attended the conference line from left to right. All of them have a
of T was greater than the number of people relationship with P. Some of them are facing
who attended the conference of B by 24. The North and others are facing South. The
one who had conference in March had more following information is known about them:
number of people attending it than B but less
than 30. There were more than 3 conferences T who is the father of P is sitting to the
between the conferences of R and S. (5 immediate of V’s wife. S is neither P’s mother
questions) nor V’s wife. P’s wife is sitting immediately left
21. The total number of people who of Q who is sitting at 5th position from left end
attended the conferences of D and F is? of row. R is sitting fourth to the right of P’s
A) 43 father. P is sitting 3rd to the right of S. Mother
B) 50 and brother of P are facing the same direction.
C) 47 Both of them are sitting together with one of
them at an extreme end of the line.
There is one person sitting between P and V
and they both are facing opposite directions. W
who is P’s sister is sitting at third position from
the right end. She is sitting facing North
direction. Daughter of P is sitting 2nd to the
right of the mother of P. X is sitting facing south
direction which is opposite of what P’s
daughter is facing. The immediate neighbors of
P’s daughter are facing same direction as X.
Neither P’s brother nor his sister is sitting to
the immediate side of P’s daughter. From nine
members, one is son of P.

26. Who is sitting second to right of R’s


father?
A) Q’s mother
B) U
C) W’s brother
D) R’s brother
E) T’s grandson

27. Who is sitting third to left of P’s son?


A) P’s mother
B) R’s brother
C) P’s daughter
D) V’s father
E) P’s sister-in-law
28. Who are the immediate neighbors of P’s
father?
A) S, V’s father
B) P’s sister, X
C) P’s wife, W
D) V’s wife, S
E) None of the above
29. Which of the following relates the true
relationship with Q?
A) Q is P’s wife
B) Q is V’s niece
C) Q is W’s grandson
D) Q is R’s sister
E) None of these
30. Who are sitting at an extreme end of line?
A) P’s wife, P’s sister-in-law
B) T, W’s mother
C) R’s father, X
D) P’s brother, W’s sister-in-law
E) Cannot be determined
PUZZLES

Puzzle – 1:

Salim, Jack, Bob, Neil, David, Mike and Krish own different brands of car among Honda, Jaguar, Audi, Suzuki, BMW,
Ford and Toyota, not necessarily in the given order. Each of the seven men plays exactly one sport among Tennis,
Badminton andCricket.

1. Mike owns a Ford and doesn’t play Tennis.


2. Neil owns an Audi and plays Tennis.
3. Salim doesn’t play Tennis.
4. Jack doesn’t own a Toyota.
5. Krish is the only Badminton player apart from Jack.
6. David owns a Suzuki and doesn’t play Tennis.
7. Neither the Jaguar owner nor the BMW owner plays Cricket.
8. The Toyota owner plays Badminton.
9. Bob doesn’t own a BMW.

Solution:

Man Car Sport


Salim Honda Cricket
Jack BMW Badminton
Bob Jaguar Tennis
Neil Audi Tennis
David Suzuki Cricket
Mike Ford Cricket
Krish Toyota Badminton
Puzzle – 2:

There are seven friends – Brian, Paul, Qasim, Rustam, Sam, Tom and Winston – each of whom likes a different subject among
Chemistry, Geography, Physics, Math, Sociology, History and Biology. They are ranked 1 to 7 in their final exams, with 1 being the
best rank and 7 the worst.
1. Qasim likes Sociology, and he stands above Tom but below Winston.
2. Tom gets the worst rank, and he likes neither Geography nor History.
3. Sam likes Physics, and he stands above both Qasim and Winston but below Paul.
4. Rustam gets Rank 2, and he likes neither Chemistry nor Math.
5. Paul likes Biology, and he hasn’t secured the best rank.
6. The student who likes Math gets the best rank.
7. Rustam doesn’t likeGeography.

Solution:
Rank Name Subject
1 Brian Maths
2 Rustam History
3 Paul Biology
4 Sam Physics
5 Winston Geography
6 Qasim Sociology
7 Tom Chemistry

Puzzle – 3:
Akash, Barun, Chetan, Dheeraj and Gaurav are married to Paula, Brinda, Linda, Garima and Ketki, not necessarily in this order. All these
couples have a child each. The names of the five children are Kapol, Dave, Chris, Raman and Nazar, in no particular order.
1. Gaurav is neither married to Ketki nor is the father of Dave.
2. Neither Brinda nor Garima is Dave’s mother.
3. Ketki is the mother of Kapol.
4. Chetan is the husband of Brinda.
5. Akash isn’t married to Linda.
6. Barun is the father of Chris.
7. Raman’s mother isPaula.
PUZZLES

Solution:

Man Wife Child


Akash Ketki Kapol
Barun Garima Chris
Chetan Brinda Nazar
Dheeraj Linda Dave
Gaurav Paula Raman

Puzzle – 4:

Aman, Maria, Joshua, Kevin, Saurabh and Deepika have come to participate in an inter-school sports meet held in Chennai. They play
different sports and belong to different cities. The six sports are Cricket, Tennis, Hockey, Football, Badminton and Volleyball, and the
six cities are Chandigarh, Pune, Kolkata, Mumbai, Bhopal and Delhi.

1. Kevin does not belong to Kolkata.


2. The student who is from Mumbai does not play Volleyball.
3. Saurabh belongs to neither Mumbai nor Kolkata.
4. Aman is fromChandigarh.
5. Neither Joshua nor Kevin plays Tennis.
6. The student who plays Hockey belongs to Bhopal.
7. Maria plays Volleyball.
8. Deepika, who plays neither Cricket nor Tennis, belongs to Delhi.
9. The student who is from Kolkata plays Badminton.

Solution:

Student City Sport


Aman Chandigarh Tennis
Saurabh Bhopal Hockey
Maria Pune Volleyball
Deepika Delhi Football
Joshua Kolkata Badminton
Kevin Mumbai Cricket

Puzzle – 5:

Subhash Kapoor, Dibakar Kumar, David Ghai, Sooraj Advani, Nikhil Bajpai and Vinod Bhatt are six renowned film- makers who specialize in
six different genres – Comedy, Thriller, Satire, Horror, Action and Drama – and belong to six different cities – Delhi, Hyderabad, Mumbai,
Bengaluru, Chennai and Kolkata – not necessarily in the same order.

1. Dibakar Kumar belongs to Kolkata.


2. The film-maker who specializes in Action doesn't belong to Kolkata.
3. Sooraj Advani specializes in Thriller and doesn’t belong to Delhi.
4. The film-maker who specializes in Drama belongs to Bengaluru.
5. David Ghai doesn’t belong to Bengaluru.
6. Subhash Kapoor belongs to Mumbai, and he specializes in neither Horror nor Action.
7. Vinod Bhatt doesn't belong to Delhi and doesn’t specialize in Drama.
8. The film-maker who specializes in Satire belongs to Hyderabad.

Solution:

Film Maker City Genre


D.K Kolkata Horror
S.A Chennai Thriller
S.K Mumbai Comedy
N.B Bengaluru Drama
PUZZLES

V.B Hyderabad Satire


D.G Delhi Action

Puzzle – 6:

Aston, Karamvir, Lalit, Phil, Dracs and Maxie, who represent six different companies – Microsoft, Google, GE, Unilever, IBM and
Nestle – are sitting at a circular table, not necessarily in the given order, facing the centre. Each man iswearinga shirtofadifferent colour
amongWhite,Pink, Red, Brown,Blueand Black, againinnoparticular order.
1. The Microsoft’s representative is wearing neither a black nor a brown shirt.
2. The Google’s representative is not wearing a black shirt.
3. Karamvir is neither from Unilever nor wearing a red shirt.
4. The only person sitting between Dracs and Maxie is wearing a pink shirt.
5. Maxie is wearing a white shirt and he is sitting to the immediate left of the person who is wearing a red shirt.
6. The person in black shirt is sitting at a position diametrically opposite that of the representative of Nestle.
7. GE’s representative is sitting to the immediate left of the person who is wearing a brown shirt.
8. The person wearing a white shirt is sitting to the immediate right of Unilever’s representative.
9. Philiswearinga blue shirt and heisfrom GE. Heissittingat a position diametrically opposite that oftheperson in pink shirt.
10. Aston is not wearing a red shirt.

Solution:

Puzzle – 7:

Four Indians – Nathan, Bill, Jason and Rahul – and four Australians – Arif, Taskin, Michael and Sachin – are sitting around a circular table.
Two Indians and two Australians are facing the centre of the table, while the rest four persons have their backs towards the
centre.
1. Jason is sitting at the second position to the left of Michael.
2. Arif is sitting at the third position to the left of Taskin.
3. Nathan is the third person to the right of Rahul, who is facing the centre.
4. Bill and Sachin are sitting at diametrically opposite positions and facing each other. None of them is sitting next to Rahul or Arif.
5. No three Australians are sitting together.
6. Michael is not sitting next to Rahul.

Solution:
PUZZLES

Puzzle – 8:

The top eight ranks in a city-level inter-school debate competition are bagged by the students of top three city schools – APS, KCM
and VLP. The eight toppers are Ira, Kavya, Charvi, Sana, Jiya, Riya, Tanya and Myra, in no particular order. Three of them study in KCM,
three in APS and two in VLP.
1. The girl who gets Rank 1 does not study in VLP.
2. Jiya’s rank is better than that of Kavya.
3. The girl who gets Rank 8 does not study in KCM.
4. Sana studies inAPS.
5. Riya’s rank is better than that of both Ira and Sana, but worse than that of both Myra and Kavya.
6. Tanya studies neither in KCM nor in VLP.
7. Jiya, who does not study in KCM, gets Rank 2.
8. Tanya’s rank is worse than that of Sana but better than that of Ira.
9. Myra, who gets Rank 4, studies in APS.
10. Kavya does not study in VLP.

Solution:

Rank Name School


1 Charvi KCM
2 Jiya VLP
3 Kavya KCM
4 Myra APS
5 Riya KCM
6 Sana APS
7 Tanya APS
8 Ira VLP

Puzzle – 9:

Six students – John, Julian, Justin, Jack, Jacob and Joseph – study six different subjects – Math, Geography, Biology, Chemistry,
Economics and History – and play six different games – Football, Hockey, Cricket, Tennis, Baseball and Golf – not necessarily in the
given order. Two of them are Chinese, two Indian and two Korean.

1. Jacob plays Football and studies History.


2. Jack has the same nationality as the student who plays Tennis.
3. Both Geography and History are studied by Chinese students.
4. Julian studies Chemistry, and doesn’t play Golf.
5. One of the two Korean students studies Economics.
6. Justin and Jacob have different nationalities.
7. The student who plays Golf doesn’t study Biology.
8. Biology is studied by neither of the two Indians.
9. The two students who play Football and Cricket have the same nationality.
10. One of the two Indians studies Chemistry.
PUZZLES

11. John is Indian and plays Hockey.


Solution:

Name Sport Subject Nationality


Jacob Football History Chinese
Joseph Cricket Geography Chinese
John Hockey Math Indian
Julian Baseball Chemistry Indian
Jack Golf Economics Korean
Justin Tennis Biology Korean

Puzzle – 10:

An Indo-Australian Cricket committee comprises eight persons – James, Peter, Roger, Nathan, Brendon, Ricky, Shikhar and Rohit –
who are sitting in a row in the given order, from left to right, all facing the North direction. Half the committee members are
Australians. There are three Secretaries, two Joint Secretaries, two Vice Presidents and a President in the committee.

1. Each Secretary and the President always sit next to at least one Joint Secretary.
2. No two Indians are sitting next to each other.
3. James is notIndian.
4. One of the Secretaries is sitting at one of the ends.
5. TheIndianVicePresidentsitstotheimmediaterightoftheAustralianVicePresidentwho isnotsittingatanyof the ends.
6. Exactly one Vice President and exactly one Joint Secretary are Australian.
7. No two Secretaries are sitting next to each other.
8. Exactly two Secretaries are Indian.

Solution:

Name James Peter Roger Nathon Brendon Ricky Shikhar Rohit


Nationality Aus Ind Aus Ind Aus Ind Aus Ind
Post S JS P S JS S VP VP

Puzzle – 11:

Kali, Gati, Fiza, Pari, Avni, Neha, Diya and Teji are eight friends all of whom have different heights. They are standing in two rows of
four girls each, with four facing the North direction and four facing South, as shown in the figure given below.

1. Teji is shorter than Kali.


2. The tallest girl among the eight is neither standing at any of the ends nor facing North.
3. Avni is shorter than Gati and is facing South.
4. Pari is the shortest among the eight girls and is standing directly opposite Neha who is facing South.
5. Diya is taller than Fiza but shorter than Teji.
6. Kali is standing at the second position to the right of Diya.
7. Neha is shorter than only two girls, Avni and Gati.
8. Teji is standing directly opposite Fiza who is facing South.
9. Diya is standing at the third position to the left of Pari.
Solution:

A F G N Facing South
D T K P Facing North

P<F<D<T<K<N<A<G

Puzzle – 12:

Seven friends – Rio, Pat, Omi, Zak, Sam, Leo and Tom – had to open an office each and decided to do it in the same building. The
building has five floors numbered 1 to 5 (lowest to highest) and each floor hosts the office of at least one of the seven friends.

1. The number of friends with office on Floor-5 is different from that on Floor-2.
PUZZLES

2. Tom’s office is on an odd-numbered floor.


3. The offices of Sam and Omi are on the same floor, which is immediately above Leo’s floor.
4. No floor has got more than two offices.
5. Rio’s office is immediately above that of Pat.
6. The number of friends with office above Zak’s floor is the same as that below Zak’s floor.

Solution:

Floor.No Offices Names


5 2 Sam, Omi
4 1 Leo
3 1 Zak
2 1 Rio
1 2 Tom, Pat

Puzzle – 13:

Rohit, Brendon, Posco, Mark and Shahid are lawyer, architect, doctor, waiter and engineer by profession, not necessarily in the
same order. They live in different cities among Delhi, Chicago, London, New York and Kolkata, and drive cars of different colours
among black, red, yellow, white and grey, but not necessarily in the given order. Each of them works for either a Korean or a
Japanese company.

1. The engineer lives in Delhi.


2. Both the architect and the person whose drives a white car work for a Japanese company.
3. Posco is neither an architect nor lives in London.
4. Mark is a lawyer and works for a Japanese company.
5. Brendon works for a Korean company.
6. All three – the person who drives a grey car, the person who lives in London, and the one who lives in New York
– work for a Korean company.
7. The architect drives a yellow car.
8. Rohit drives neither a yellow nor a grey car.
9. Neither Brendon nor Rohit is a waiter by profession.
10. Shahid doesn’t live in Chicago.
11. The doctor doesn’t drive a red car.

Solution:

Name Company Profession City Car Colour


Shahid Japanese Architect Kolkata Yellow
Mark Japanese Lawyer Chicago White
Brendon Korean Engineer Delhi Grey
Rohit Korean Doctor London Black
Posco Korean Waiter New York Red
Puzzle – 14:

India played seven T20 Cricket matches against Pakistan, Sri Lanka, New Zealand, South Africa, Australia, England and West Indies, one
against each team. The matches were held in seven different cities – Chennai, Mumbai, Kolkata, Bengaluru, Delhi, Jaipur and Nagpur
– on seven consecutive days, starting on a Monday and ending on the following Sunday.

1. India played exactly one match between the matches against Sri Lanka and Australia.
2. The match against South Africa was not held on Monday.
3. The match against England was not held in Delhi.
4. The match held in Nagpur happened before the match held in Chennai.
5. India played exactly one match between the match held in Chennai and the match against West Indies.
6. The match against New Zealand was not held in Chennai.
7. The match against England was held a day after the match against New Zealand, and a day before the match against West Indies.
8. The match held in Mumbai happened a day before the match held in Delhi.
9. The match against Australia was held in Bengaluru on Friday.
10. The match held in Kolkata was on Monday.
PUZZLES

Solution:

Day Monday Tuesday Wednesday Thursday Friday Saturday Sunday


Opponent Pakistan NZ England WI Australia SA Sri Lanka
City Kolkata Nagpur Mumbai Delhi Bengaluru Chennai Jaipur

Puzzle – 15:

Merkel, Rajan, Kerry, Ghani, Jindal, Trump, Stark and Blair are eight diplomats who represent eight different countries – Brazil,
China, France, India, Russia, Spain, UK and USA – not necessarily in the given order. The eight diplomats are attending a conference to
discuss some important issues concerning world peace. They are sitting at a square-shaped table, two on each side, as depicted in
the figure given below.

1. Jindal represents UK and is sitting at the third position to the left of Trump who represents Brazil.
2. Jindal and Ghani are sitting on the same side of the table.
3. Blair represents Russia.
4. Merkel, who represents France, is sitting between Trump and Kerry.
5. Rajan and Jindal are immediate neighbours.
6. Kerry represents USA and is sitting at the second position to the left of Rajan who represents India.
7. The person who represents China is a neighbour of Ghani.
Solution

Puzzle – 16:

Akhil, Binay, Chetan, Deepu, Ekant, Faizal, Garvit and Himesh are sitting around a circular table, with half of them facing the centre of
the table and the rest facing away from the table, in the opposite direction. Their favourite colours are black, pink, blue, red, green,
yellow, white and orange, not necessarily in the same order.
1. Binay, whose favourite colour is green, is facing away from the table.
2. The favourite colours of the two immediate neighbours of Chetan are green and black respectively.
3. Deepu, whose favourite colour is white, is sitting at the third position to the left of Binay.
4. Garvit is facing the centre of the table and his favourite colour is pink.
5. Akhil is sitting at the third position to the left of Faizal, whose favourite colour is not orange, and who is facing away from the
table.
6. Ekant, whose favourite colour is yellow, is facing away from the table.
7. Himesh is not an immediate neighbour of the person whose favourite colour is white.
8. The person whose favourite colour is black is facing away from the table.
9. Both the immediate neighbours of the person whose favourite colour is yellow are facing the centre of the table.
10. Akhil, whose favourite colour is red, is sitting at the second position to the left of Chetan who is facing the centre of thetable.
Solution:
PUZZLES

Puzzle – 17:

Boon, Mac, Neo, Om, Pete, Rick, Sid and Tony belong to different countries among Australia, Brazil, Croatia, Egypt, France, Holland,
Ireland and Sweden, not necessarily in the given order. They are sitting at a circular table facing the centre of the table.

1. Tony is not an immediate neighbour of the person from Holland.


2. Only one person sits between the persons from Sweden and Holland.
3. The person from France sits at the third position to the left of the person from Ireland.
4. Exactly two persons are sitting between Om and the person from Holland.
5. Neo and Boon are sitting beside each other.
6. Only one person sits between Sid and the person from Holland.
7. The person from Australia sits to the immediate left of the person from Sweden.
8. Tony is not from Ireland, Australia or Croatia.
9. Mac is sitting at the second position to the right of the person from Ireland.
10. Boon and Sid are not immediate neighbours.
11. Sid sits to the immediate right of Tony.
12. The person from France is sitting at the second place to the right of Rick.
13. Om is from Brazil.
Solution:

Puzzle – 18:
PUZZLES

Twelve persons are sitting at a rectangular table, six along each of the two longer sides, as shown in the figure given
below. Neymar, Messi, Cafu, Zidane, Henry and Gerrard are facing the North direction, and Kaka, Best, Pele, Suarez,
Iniesta and Ronaldo are facing South.

1. Cafu is not sitting next to Henry.


2. Kaka sits at the third position to the right of Suarez.
3. Gerrard is not sitting directly opposite Kaka.
4. Henry is sitting directly opposite Suarez.
5. Gerrard is not sitting at any of the corners of the table.
6. Exactly two persons are sitting between Pele and Iniesta.
7. Neymar is not sitting next to Cafu.
8. Iniesta is not sitting directly opposite Gerrard.
9. Pele is sitting at one of the corners of the table.
10. Exactly two persons are sitting between Cafu and Messi.
11. Gerrard is not sitting next to Henry.
12. Neither Kaka nor Suarez sits at any of the corners of the table.
13. Best is not sitting directly opposite Gerrard.
Solution:

P K R I S B Facing South
C Z G M H N Facing North
Puzzle – 19:

Eight friends Michael, Saina, Jackson, Fawad, Britney, Hong, Rahul and Anand are reading different books sitting around a circular table,
four of them facing the centre of the table and the rest away from the table. The eight books are The Shining, Jane Eyre, Hamlet,
Animal Farm, AtlasShrugged, JurassicPark,Twilight and The Alchemist.

1. The person reading Animal Farm is sitting at the second position to the left of the person who is reading The Shining.
2. Jackson is reading Atlas Shrugged and sitting at the third position to the right of Britney.
3. HongisreadingHamlet,andsittingatapositiondiametricallyoppositethatofMichaelwhoisfacingawayfrom the table.
4. Britney is reading The Alchemist.
5. Fawad is sitting at the second position to the right of Britney, and reading Animal Farm.
6. Both Hong and Jackson are facing away from the table.
7. Michael is not reading Jurassic Park.
8. Saina is sitting at the second position to the right of Rahul.
9. Rahul, who is reading Twilight, is sitting at the third position to the right of Hong and facing away from the table.

Solution:

Puzzle – 20:
PUZZLES

Eight girls – Lauren, Olivia, Ruby, Katie, Daisy, Maya, Nicole and Alice – are sitting at a circular table, facing the centre, but not
necessarily in the same order. All of them are Android fans; however, each of them uses a different version of the mobile
operating system among Froyo, Honeycomb, Cupcake, Kitkat, Gingerbread, Lollipop, Eclair andDonut.

1) Lauren sits at the third position to the right of the girl who uses Donut.
2) Nicole is not an immediate neighbour of the girl who uses Froyo.
3) Olivia is not an immediate neighbour of the girl who uses Kitkat.
4) The girls who use Cupcake and Eclair are immediate neighbours.
5) Lauren doesn’t useHoneycomb.
6) Exactly two girls are sitting between Alice and the Donut user.
7) Olivia does not use Eclair.
8) The Eclair user is not an immediate neighbour of the Donut user.
9) The Froyo user sits at the second position to the left of Daisy.
10) Daisy is not an immediate neighbour of Alice.
11) The Froyo user is the only person sitting between the Gingerbread and Kitkat users.
12) The Gingerbread user sits at the third position to the right of Olivia.
13) Ruby sits to the immediate right of the Honeycomb user.
14) Maya and Lauren are not immediate neighbours.
15) Neither Lauren nor Alice uses Cupcake or Eclair.

Solution:

Puzzle – 21:

Praveen, Pallavi, Prabhu, Pavani, Prem, Pooja, and Priya are seven teachers working in four institutes namely Ganga, Yamuna,
Narmada and Godavari. These institutes are operated by the group of seven teachers mentioned above. The institutes are open only on
Sunday. At least one teacher visits the coaching institute on each Sunday but no institute is visited by more than two teachers. Each
teacher teaches different subjects. Subjects are: Hindi, English, Maths, Physic, History, Biology and Sanskrit. The names of the subjects
are not necessarily in the same order as the names of the teachers. Now study some additional clues.
 Pooja, who teaches Sanskrit, visits alone the institute Yamuna.
 The one who teaches Maths does not visit the institute Ganga. also, he never visits with Prem or Priya.
 Pallavi visit the institute Narmada,she does not teach Maths.
 Prabhu visit the institute Narmada. Priya does not teaches Physics. The one who teaches Maths visits the institute with the
person who teaches Hindi.
 The one who teaches History visits the institute Godavari.
 Praveen teachesneither English norPhysics.Allthepersons followthe sameroutine onevery Sunday.
 Pavani visits the institute Godavari with the person who teaches Physics.
PUZZLES

Solution:

Ganga Yamuna Narmada Godavari


Priya(English) Pooja(Sanskrit) Pallavi(Hindi) Pavani(History)
Praveen (Biology) – Prabhu (Maths) Prem(Physics)

Puzzle – 22:

Aparna, Aruna, Arya, Ashwini and Anu are five women sitting in a line facing south – while Aditya, Akash, Anand, Ashok and Akshay are
five men sitting in a second line parallel to the first line and are facing North. All of them are residents of a five storey building. There
are two people living one each floor of the building.

1. Aruna, who is sitting to the immediate left of Ashwini, is facing Akshay, who lives on the floor below Aruna.
2. Arya, who lives on the same floor as Akshay, is on one of the extreme ends of her line.
3. Akash who lives on the topmost floor is on one of the extreme ends of his line.
4. Anu sits opposite to Anand and both live on the same floor, which is two floors below Akash.
5. Anand’sneighbourAditya lives on the same floor as Aruna and he facesArya who liveson the floor below him.
6. Ashwini lives on the same floor as Akash.
7. Akshay is on the immediate left of Akash who faces Aparna, who lives on the floor below Akash.

Solution:

Arya(I) Anu(III) Ashwini(V) Aruna(II) Aparna(IV)


Aditya(II) Anand(III) Ashok(IV) Akshay(I) Akash(V)

Puzzle – 23:

Six friends– Kalash, Kalyan, Kamal, Kannan, Kapil and Kiran- work in different banks- SBI, BOB, BOI, CBI, SBT and PNB – and they are located
on six different floors of a building, numbered 1 to 6. They drive six different cars Baleno, Swift, Alto, Kwid, Ritz and Ford, in no
particular order. Each bank is closed on a different day from Monday to Saturday.

1. Kalyan drives Ritz and his bank, BOB is on the 6th floor of the building.
2. Kapil and Kalash have banks on even numbered floors which are closed on Saturday and Monday.
3. SBI is on the lowest floor and it is closed on Tuesday.
4. CBI bank is on the floor next to Kalyan and it is closed on Thursday.
5. The manager of PNB drives a Ford and has leave on Monday.
6. Kiran drives a Kwid and he has only one neighbour.
7. One of the banks on even numbered floors has leave on Friday.
8. Kannan is located two floors below Kamal and he drives an Alto.
9. Kapil, who is not a neighbour of Kamal, drives a Baleno and has leave on Saturday. SBT is on an even numbered floor.

Solution:

Floor Person Car Bank Day


6 Kalyan Ritz BOB Friday
5 Kamal Swift CBI Thursday
4 Kalash Ford PNB Monday
3 Kannan Alto BOI Wednesday
2 Kapil Baleno SBT Saturday
1 Kiran Kwid SBI Tuesday

Puzzle – 24:

Eight friends – Ganesh, Gaurav, George, Giri, Gokul, Gowri, Gopal, and Gayathri – go to a Sports Complex on different days of the
week, from Monday to Friday. They take part in eight different games – Football, Basketball, Cricket, Badminton, Swimming, Archery,
PUZZLES

Table Tennis and Hockey, not necessarily in any order. Two of them walk to the complex, two go by cycle, two by bike
and two by car.

1. Gowri goes by bike on Tuesday along with GANESH who plays Table Tennis.
2. Gayathri and Gaurav travel by car on different days after the day on which GANESH goes to the complex.
3. Gokul plays Basketball on the day exactly between Gayathri and Gaurav.
4. GANESH and Gokul go by cycle.
5. George alone goes for Archery on the day before Gowri.
6. Football and Cricket practice is held on Friday.
7. Gayathri plays Badminton.
8. Gopal, who plays cricket and Giri, who plays Hockey, walk to the sports complex.
9. Giri goes to the complex on the same day as Gokul.
Solution:

Monday George Bike Archery


Tuesday Gouri Bike Swimming
Ganesh Cycle Tabletennis
Wednesday Gayathri Car Badminton
Thursday Giri Walk Hockey
Gokul Cycle Basketball
Friday Gaurav Car Football
Gopal Walk Cricket

Puzzle – 25:

Nine people, A, B, C, D, E, F, G, H and I stay in a building, but not necessarily in the same order. The building has nine floors and only one
person stays on one floor. All of them own one car each, and each car is of a different colour, i.e. blue, grey, white, black, yellow,
green, red, orange and pink, but not necessarily in the same order. The ground floor is numbered 1, the floor above it is numbered 2,
and so on, and the topmost floor is numbered 9.

1. H owns a black-coloured car and stays on an even numbered floor.


2. A stays on any even-numbered floor below the floor on which H stays.
3. The one who owns an orange- coloured car stays on the fourth floor.
4. E stays on the second floor and owns a white-coloured car.
5. The one who owns a pink-coloured car stays on the third floor.
6. A does not own a green-coloured car.
7. There are two floors between the floors on which the people owning the red and the black-coloured cars stay.
8. C owns a grey-coloured car.
9. There are three floors between the floors on which C and G stay.
10. D stays on a floor immediately above I’s floor.
11. There is one floor between the floors on which F and G stay.
12. F does not own the pink- coloured car.
13. The one who owns the blue car stays on the topmost floor.
14. F does not stay on the ground floor.
Solution:

Person Floor Colour of car


A 6 Yellow
B 9 Blue
C 1 Grey
D 4 Orange
E 2 White
F 7 Green
G 5 Red
H 8 Black
PUZZLES

I 3 Pink
PRESIDENCY CAREERPOINT
200 Circular Arrangements ques
Set 1 Set 2
Directions (Q. 1–5): Study the following information carefully and Directions (1-5): Study the following information to answer the given
answer the questions given below: questions.
Cetking students Varun, Udit and Tarun are seated in a circle facing
Eight Cetking students A, B, C, D, E, F, G and H are sitting around a the centre. Akash, Bharti and Charu are also seated in the same circle
circular table for playing a game. Four of them A, B, E and F are not but two of them are not facing the centre (facing opposite direction of
facing the centre. D is third to the left of H, who is fourth to the right the centre.) Varun is second to the left of Charu. Udit is second to the
of E. A is third to the right of B, who is third to the left of D. F is second right of Akash. Bharti is third to the left of Tarun. Charu is second to
to the left of C. the right of Tarun. Akash is seated next to Varun.

1. How many persons sit between A and B (if the counting start from B 1. Which of the following are not facing the centre?
in CW direction)? (1) Bharti-Akash
1) None 2) One 3) Two 4) Three 5) Other than given options (2) Charu-Akash
(3) Bharti-Charu
2. Who among the following sits third to the left of E? (4) Cannot be determined
1) G 2) A 3) H 4) D 5) B (5) None of these

3. What is the position of G with respect to F? 2. Which of the following is Tarun’s Position with respect to Bharti?
1) Third to the right 2) Third to the left (1) Third to the right
3) Second to the right 4) Cannot be determined (2) Second to the right
5) Other than given options (3) Third to the left
(4) Third either to the right or to the left
4. Who among the following is the neighbour of F and C? (5) None of these
1) E 2) H 3) A 4) B 5) Other than given options
3. Which of the following is Varun’s position with respect to Charu?
5. Four of the following five are alike in a certain way and hence form (1) Second to the right
a group. Find the one that does not belong to that group. (2) Third to the left
1) EC 2) AG 3) DE 4) HA 5) FB (3) Fourth to the right
(4) Fourth to the left
(5) Cannot be determined

4. Which of the following is true regarding the seating arrangement?


(1) Akash, Bharti and Charu are seated adjacent
(2) Varun, Udit and Tarun are seated adjacent
(3) There are two persons whose seating arrangement cannot be
ascertained
(4) Those not facing the centre are seated adjacent
(5) there are only two persons seated between Varun and Tarun

5. Which of the following is Akash’s position with respect to Udit?


(1) Second to the left
(2) Second to the right
(3) Third to the right
(4) Cannot be determined
(5) None of these

1. 3
2. 2 Answer
3. 1 1-3 2-4 3-3 4-4 5-1
4. 4
5. 5
200 Circular Arrangements ques
Set 3
Directions: Study the information carefully and answer the given 5. Who amongst the following is immediate right of Jack?
questions: (a) Monty (b) Kunal (c) Queen (d) Nikhil (e) None of the above

Eight Cetking students Jack, Kunal, Leon, Monty, Nikhil, Ober, Prince (b) They face opposite directions
and Queen are sitting around a circular table but not necessarily in the Answer:(e)
same order. Some of them are facing the centre and some of them Answer:(a)
are facing outside (ie in a direction opposite the centre.) Facing the Answer:(b)
same direction means if one person faces the centre then the other Answer:(d)
person also faces the centre and if one person faces outward then the Answer:(a)
other person also faces outward. Facing the opposite direction means
if one person faces the centre then the other person faces outward
and vice versa. SET 4
Immediate neighbours facing the same direction means if one Directions (1-5): Read the information given below and then answer
neighbour faces the centre then the other neighbour also faces the the questions that follow:
centre and if one neighbour faces outward then the other neighbour Eight Cetking students A, B, C, D, E, F, G and H are sitting around a
also faces outward. Immediate neighbour faces the opposite direction circle in such a way that three of them are facing outside and rest of
means if one neighbour faces the centre then the other neighbour them are facing the centre.
faces outward and vice versa. E is 2nd to the left of H, who is facing outside. D is second to the left
• Only one person sits between Kunal and Ober. Queen sits third to of F and third to the right of E, who is facing the centre. B is third to
the right of Ober. the left of G and faces outside. G is third to the left of A, who is not an
• Monty sits on the immediate right of Queen. Queen faces outward. immediate neighbor of E and F. The friends facing outside are not
• Leon sits second to the left of Prince. Prince is not an immediate adjacent.
neighbour of Ober.
• Leon faces a direction opposite that of Ober. The immediate 1. Which of the following group is facing outside?
neighbours of Leon face opposite direction. (1) BHE (2) CBH (3) CDH (4) cannot be determined (5) None
• Jack sits third to the left of Nikhil. Jack is not an immediate
neighbour of neither Prince nor Kunal. 2. What is D’s position with respect to A?
• Monty and Jack face the direction same as that of Nikhil. (1) Immediate right (2) 3rd to the left
(3) Opposite (4) 2nd to the right
1. Four of the following five are alike in a certain way based on the (5) Immediate left
directions they are forming and so form a group. Which is the one
that does not belong to that group? 3. Who is 2nd to the right of B?
(a) Kunal, Leon (b) Queen, Monty (1) H (2) G (3) C (4) A (5) D
(c) Prince, Queen (d) Nikhil, Jack
(e) Queen, Leon 4. If all the friends change their respective position to immediate right
place then who are still facing inside the centre?
2. Who among the following sit exactly between Queen and Ober (1) A and E (2) F and D (3) C and G (4) A and D (5) G and A
when counted from the right of Queen?
(a) Prince, Jack (b) Monty, Prince 5. Which of the following is true about the given information?
(c) Leon, Kunal (d) Nikhil, Leon (1) C and F are facing each other
(e) Jack, Monty (2) E is third to the left of F
(3) C is third to the left of B
3. Which of the following is not true regarding Kunal as per the given (4) G and A are two places away from each other.
arrangement? (5) None of these
(a) Nikhil is an immediate neighbour of Kunal.
(b) None of the given options is true
(c) Only three people sit between Kunal and Monty.
(d) Leon sits on the immediate left of Kunal.
(e) Kunal faces outside.

4. What is Leon’s position with respect to Nikhil?


(a) Immediate right (b) Third to the right
(c) Second to the right (d) Third to the left
(e) Immediate left

4. Who amongst the following are immediate neighbours of Prince?


(a) Monty, Nikhil (b) Kunal, Monty
(c) Jack, Queen (d) Nikhil, Kunal
(e) Queen, Nikhil
200 Circular Arrangements ques
SET 5 SET 6
Directions (11-15): Study the following information carefully and Directions: Study the following information carefully and answer the
answer the questions given below: questions given below:

Eight Cetking students Sameer, Sally, Salina, Sadie, Suchi, Shanky, Eight Cetking students Pintu , Queen, Rekha, Suman, Trilok, Uber,
Shane and Smith are sitting around a circular table. Three of them are Vivan and Wasim are sitting around a square table. Four of them are
facing the centre. sitting in the middle of the sides and facing outward from the centre.
Suchi is not an immediate neighbor of Sadie or Salina. The one who Rest are sitting in the corners of the square and facing the centre.
sits exactly between Sadie and Shanky is not facing the centre. Shane Wasim is sitting in a corner and is an immediate neighbour of Pintu
is third to the right of Sameer and is facing the centre. Salina is third and Rekha. Suman sits fourth to the right of Pintu and is not facing the
to the left of Sadie and both are facing the centre. Shanky is centre. Trilok is second to the left of Wasim and is an immediate
immediate neighbor of Suchi but not the neighbor of Sameer. Sally neighbour of Vivan. Uber is not the neighbor of Rekha.
does not sit second to the right of Shane.
1. How many persons sit between Rekha and Vivan?
Q11. Who among the following is second to the right of Sally? (a) None (b) One (c) Two (d) Three (e) None of the above
(a) Suchi (b) Sameer (c) Salina (d) Shane
(e) Other than given as options 2. Who among the following sits third to the right of Suman?
(a) Trilok (b) Wasim (c) Pintu (d) Uber (e) None of the above
Q12. Who among the following sits opposite Salina?
(a) Sally (b) Sameer (c) Shane (d) Smith (e) Shanky 3. Which of the following groups sits between Trilok and Queen if
counted from right of Trilok?
Q13. Who among the following sits third to the left of Salina? (a) Rekha, Wasim, Pintu (b) Suman, Pintu, Wasim
(a) Sadie (b) Suchi (c) Shanky (d) Shane (e) None of the above (c) Uber, Vivan, Suman (d) Rekha, Vivan, Wasim
(e) None of the above
Q14. What is the position of Shanky with respect to Sally? Answer: (d)
(a) Third to the right (b) Second to the left Answer: (b)
(c) Immediate left (d) Immediate right Answer: (a)
(e) None of the above

Q15. Who among the following are immediate neighbours of Sameer?


(a) Sadie, Smith (b) Sadie, Sally (c) Salina, Smith
(d) Salina, Suchi (e) None of the above

S11. Ans.(b)
S12. Ans.(a)
S13. Ans.(c)
S14. Ans.(c)
S15. Ans.(a)
200 Circular Arrangements ques
SET 7 SET 8
Directions (Q.1-5): Read the information given below and then answer Directions (Q. 1-6): Study the information given below and answer the
the questions given below: given questions.
Eight Cetking students A, B, C, D, E, F, G and H are sitting in a circle,
Eight Cetking students Aero, Boom, Chris, Dale, Elina, Francos, Gayle but not necessarily in the same order. Four of them are facing outside
and Hippa are sitting around a circle in such a way that three of them and four of them are facing the centre.
are facing outside and rest of them are facing the centre. E faces outside, Both the immediate neighbours of E face the centre.
Elina is 2nd to the left of Hippa, who is facing outside. Dale is second H sits second to the right of E. B sits third to the left of E.
to the left of Francos and third to the right of Elina, who is facing the D faces the centre. Both the immediate neighbours of D face outside.
centre. Boom is third to the left of Gayle and faces outside. Gayle is G sits second to the left of A. B sits third to the right of H.
third to the left of Aero, who is not an immediate neighbour of Elina F is an immediate neighbour of D. C is an immediate neighbour of G.
and Francos. The friends facing outside are not adjacent. D is not an immediate neighbour of B.

1. Which of the following group is facing outside? 1. Who amongst the following sits on the immediate right of H?
(a) Boom,Hippa,Elina 1) A 2) D 3) C 4) G 5) None of these
(b) Chris,Boom,Hippa
(c) Chris,Dale,Hippa 2. Who amongst the following sits third to the right of A?
(d) cannot be determined 1) D 2) E 3) F 4) A 5) None of these
(e) None of these
3. Four of the following five are alike in a certain way, based on the
2. What is Dale’s position with respect to Aero? information given above and so form a group. Which is the one that
(a) Immediate right does not belong to that group?
(b) 3rd to the left 1) HA 2) FH 3) GC 4) AD 5) AE
(c) Opposite
(d) 2nd to the right 4. If all the people are made to sit in an alphabetical order, in
(e) Immediate left clockwise direction, starting from A, the position of whom amongst
the following remains the same (excluding A)?
3. Who is 2nd to the right of Boom? 1) E 2) F 3) C 4) G 5) None of these.
(a) Hippa
(b) Gayle 5. How many people are seated between A and C (counting clockwise
(c) Chris from A)?
(d) Aero 1) Two 2) Four 3) None 4) One 5) Three
(e) Dale
6. Who amongst the following sits exactly between F and C (and is
4. If all the friends change their respective position to immediate right also their neighbour)?
place then who are still facing inside the centre? 1) E 2) B 3) G 4) A 5) None of these
(a) Aero and Elina
(b) Francos and Dale 1). 4
(c) Francos and Gayle 2). 3
(d) Aero and Dale 3). 3
(e) Gayle and Aero 4). 2
5). 1
5. Which of the following is true about the given information? 6). 2
(a) Chris and Francos are facing each other.
(b) Elina is third to the left of Francos.
(c) Chris is third to the left of Boom.
(d) Gayle and Aero are two places away from each other, if counted in
clock wise direction from Aero.
(e) None of these.

B
A
D
E
D
200 Circular Arrangements ques
SET 9 SET 10
Cetking students K, L, M, P, Q, R, S and T are sitting around a square Directions (1-6) Study the following information to answer the
table in such a way that four of them sit at four corners of the square questions.
while four sit in the middle of each of the four sides. The ones who sit Eight Cetking students A, B, C, D, E, F, G and H are sitting in a circle,
at the four corners face outside while those who sit in the middle of not necessarily in the same order. Four of them are facing outside and
the sides face the centre of the table. P sits third to the right of S. S four of them are facing the centre. E faces outside &, both the
faces the centre. Q sits third to the left of M. M does not sit in the immediate neighbours of E face the centre. H sits second to the right
middle of the sides. Only one person sits between Q and R. R is not an of E. B sits third to the left of E.
immediate neighbour of M. T faces the centre. K is not an immediate D faces the centre. Both the immediate neighbours of D face
neighbour of R. outside. G sits second to the left of A. B sits third to the right of H. F is
an immediate neighbour D. C is an immediate neighbour of G. D is not
1. What is position of M with respect to L? an immediate neighbour of B.
a) Third to the right
b) M and L sit diagonally opposite to each other 1. Who amongst the following sits to the immediate rights of H?
c) Second to the right (a) A (b) D
d) Second to the left (c) C (d) G
e) Fifth to the right (e) None of these.

2. Who sits exactly between Q and R? 2. Who amongst the following sits third to the right of A?
a) T b) P c) K d) M e) S and K (a) D (b) E
(c) F (d) A
3. Which of the following pairs represents the persons seated in the (e) None of these.
middle of the sides who face each other?
a) S, Q b) K, L c) M, P d) R, T e) T, Q 3. If all the people are made to sit in alphabetical order, in clockwise
direction, starting from A , the position of whom amongst the
4. Who among the following sit between R and K when counted in following remains the same (excluding A)
anti-clockwise direction from K? (a) E (b) F
a) No one sits between R and K as R and K are immediate neighbours (c) C (d) G
of each other (e) None of these.
b) S, P and L c) P and Q d) L and R e) M, S and T
4. Four of the following five are alike in a certain way, based on the
5. If K is made to face the opposite direction, who would sit to his information given above and so form a group. Which is the one that
immediate right? does not belong to that group?
a) R b) Q c) P d) T e) S (a) HA (b) FH
(c) GC (d) DE
6. Four of the following five are alike in a certain way and so form a (e) AE
group. Which is the one that does not belong to that group?
a) L 5. Who amongst the following sits exactly between F and C (and also
b) M their neighbour)?
c) K (a) E (b) B
d) P (c) G (d) A
e) R (e) None of these.

Answers: 6. How many people are seated between A and C (counting


1) D clockwise from A)?
2) B (a) Two (b) One
3) E (c) None (d) three
4) C (e) Four.
5) B
6) E ANSWERS:
1. (d)
2. (c)
3. (b)
4. (b)
5. (b)
6. (a)
200 Circular Arrangements ques
SET 11 SET 12
Question (1-5) Directions:-Study the following information carefully Question (6-10) Directions: Study the following information carefully
and answer the questions given below: and answer the questions given below:
Cetking students A, B, C, D, E, F, G and H are sitting around a square Eight Cetking students are sitting around a circular table, viz Z, Y, X, W,
table in such a way that four of them sit at four corners of the square V, U, T and S, but not
while four sit in the middle of each of the four sides. The ones who sit necessarily in the same order. Four of them are not facing the centre.
at the four corners face the centre while those who sit in the middle W is facing the center and is third to the right of Z, who is third to the
of the sides face outside. Two females sit in the middle of the sides right of U, who is facing the center. X-is fourth to the left of T, S is
and two at the corners. A sits second to the left of G. G sits in the third to the right of V, who is not a neighbour of T. V sits second to the
middle of one of the sides. C sits fourth to the right of his wife and his right of X. Y is on the immediate right of T.
wife is not an immediate neighbour of A or G. B sits third to right of
her husband. B does not sit at any of the corners. Only D sits between 6) What is S's position with respect to Y?
B and H. H is husband of A. E is a male. A) 2nd to the left.
B) 3rd to the left.
1. Which of the following is true with respect to given seating C) 2nd to the right.
arrangement? D) Either (A) or (C).
A) No two males are immediate neighbours of each other E) None.
B) G and H face each other in the seating arrangement
C) A sits in the centre of one of the sides of the square table 7) Who is second to the left of V?
D) F is a male and sits diagonally opposite to E A) W
E) E and D are immediate neighbours of each other B) X
C) Y
2. Who amongst the following is B’s husband? D) T
A) C E) None of these
B) G
C) F 8) Which of the following pairs are neighbours of T?
D) E A) W, Y
E) Can't be determined B) U, S
C) Z, W
3. How many people sit between B and C when counted in anti- clock D) Can't be determined
wise direction from B ? E) None of these
A) None
B) One 9) Four of the following five are alike in a certain way and so form a
C) Two group. Which is the one that does not belong to that group?
D) Three A) W
E) Four B) Y
C) V
4. Who amongst the following is the wife of C? D) Z
A) D E) U
B) F
C) B 10) Which of the following statements is/are true?
D) G A) Z is not facing the center.
E) Can’t be determined B) Y is second to the right of W.
C) T is facing the center.
5. What is the position of E with respect to C? D) Both (a) and (b) are true.
A) Immediately to the right E) None of these.
B) Second to the left
C) Third to the right Answers & Explanation
D) Immediately to the left 6) C
E) Second to the right 7) D
8) A
Solution 9) B
1. C 10)E
2. D
3. C
4. A
5. E
200 Circular Arrangements ques
SET 13 SET 14
Directions (Q. 1–6): Study the following information carefully and Directions (6-10): Study the following information carefully and
answer the questions given below: answer the given questions:
Eight Cetking students P, Q, R, S, T, V, W and Y are sitting around a
There are nine Cetking students viz. P, Q, R, S, T, U, V, W and Z are square table in such a way that four of them sit at four corners of the
sitting around a circular table. Four of them are facing away from the square while four sit in the middle of each of the four sides. The ones
centre while five are facing towards the centre. R is not the immediate who sit at the four corners face the centre while those who sit in the
neighbour of either T or U. Z is fifth to the left of W, who is immediate middle of the sides face outside.
left of two people T and Q simultaneously. Z is exactly in the middle P who faces the centre sits third to the right of V. T, who faces the
between P and R. W, Q and T sit together but one of them facing in centre, is not an immediate neighbor of V. Only one person sits
different direction (either inward or outward) from others two U is between V and W. S sits second to the right of Q. Q faces the centre. R
facing away from the centre, who is second to the right of both Z and is not an immediate neighbor of P.
W. V is fourth to the left of U and facing away from the centre. T is
immediate right to U. 6. Who sits second to the left of Q?
1. Who among the following sits second to the left of Z? (a) V
1) S 2) V 3) Q 4) T 5) Other than given options (b) P
(c) T
2. How many people sit between S and V counting clockwise from S? (d) Y
1) One 2) Two 3) None 4) Four 5) More than four (e) Cannot be determined

3. What is the position of R with respect to T? 7. What is the position of T with respect to V?
1) Third to the left 2) Fourth to the right 3) Fifth to the left (a) 4th to the left
4) Both 2 and 3 5) Other than given options (b) 2nd to the left
(c) 3rd to the left
4. Four of the following five are alike in a certain way and so form a (d) 3rd to the right
group. Which is the one that does not belong to that group? (e) 2nd to the right
1) SV 2) WQ 3) PU 4) RZ 5) TW
8. Four of the following five are alike in a certain way and so form a
5. Who among the following sits exactly between Q and V? group. Which if the one that does not belong to that group?
1) S 2) T 3) R 4) They are neighbour to each other (a) R
5) Other than given options (b) W
(c) V
6. Who among the following is at equal distance from P, if counting is (d) S
done either anticlockwise or clockwise? (e) Y
1) Q 2) S 3) W 4) None 5) Other than given options
9. Which of the following will come in place of the question mark
based upon the given seating arrangement?
WP TR QW RS ?
(a) YT
(b) VY
(c) VQ
(d) PY
(e) QV

10. Which of the following is true regarding R?


(a) R is an immediate neighbor of V
(b) R faces centre
(c) R sits exactly between T and S
(d) Q sits third to the left of R
(e) None of These

6. b
1. 2 7. c
2. 3 8. d
3. 4 9. a
4. 5 10. c
5. 1
6. 4
200 Circular Arrangements ques
SET 15 SET 16
Study the following information carefully and answer the given Directions (1-6) Study the following information and answer the given
questions: questions:
Eight Cetking students I, J, K, L, M, N, O and P are sitting around a
circular table but not necessarily in the same order. Three of them are Eight Cetking students – A, B , C, D, E, F, G and H are sitting around a
facing outward while five are facing towards the centre. circular table not necessarily in the same order. Three of three of
M is third to the right of K. N is sitting third to the left of M. Three them are facing outside (opposite to the centre) while five are facing
persons are sitting between N and J. O is sitting third to the right of N, the centre. There are equal number of males and females in the
who is not facing the centre. L is sitting third to the right of I, who is group.
not facing the centre. C is facing the centre, E is sitting third to the right of C. F is sitting third
to the left of E. Three persons are sitting between F and B. The
1. Who sits between N and I? immediate niegbours of B are females. G is sitting third to the right of
1) J F. D is sitting third to the right of A. A is not an immediate neighbor of
2) K E. The immediate neighbours of E are males and facing the centre.
3) M The immediate neighbours of D are females and face outside. The one
4) O sitting third to the left of B is male. No female is an immediate
5) None of these neigbour of G.

2. Who among the following is second to the right of J? 1. Who is sitting second to the right of E?
1) L (1) C (2) B
2) K (3) G (4) H
3) M (5) None of these
4) N
5) None of these 2. How many persons are sitting between H and C when confused
from the left side of H?
3. Which of the following statements is/are true with respect to P? (1) One (2) Two
1) P is opposite of O. (3) Three (4) Four
2) P is fourth to the right of O. (5) More than four
3) P is fourth to the left of O.
4) All are true 3. Which of the following statements are true regarding H?
5) None of these (1) The one who is second to the right of H is female
(2) H is facing the centre
4. Which of the following groups represents the group, of those facing (3) H is a male
away from the centre? (4) The immediate neighbors of H is facing outside
1) IKN (5) None is true
2) KNL
3) IJK 4. What is D;s position with respect to G?
4) LMN (1) Third to the left
5) None of these (2) Third to the right
(3) Second to the left
5. What is the position of K with respect to P? (4) Second to the right
1) K is third to the right of P. (5) None of these
2) K is second to the left of P.
3) K is third to the left of P.
4) K is in front of P. 1. 3
5) None of these 2. 2
3. 1
Answers 4. 2
1- 5 2- 2 3- 4 4- 1 5- 3
200 Circular Arrangements ques
SET 17 SET 18
Directions(3-7): Study the information carefully and answer the Directions (6-10): Study the following information carefully and
questions. answer the questions given below:
Cetking students L, M, N, O, P, Q, R and S are sitting around a circular Eight Cetking students Aman, Bhavana, Chandan, Harsh, Gita, Farukh,
area at equal distances between each other, but not necessarily in the Deepa and Ela are sitting around a circular table. Some of them are
same order. Some of the people are facing the centre while some face not facing the centre. No more than two friends sitting together face
outside (i.e., in a direction opposite to the centre).S sits fourth to left the same direction.
of P. O sits second to right of S. S faces outside. L sits third to right of Note: Facing the same directions means if one person faces the centre
R. R is not an immediate neighbour of O. L is not an immediate then the other also faces the centre and if one person faces outward
neighbour of P. Both the immediate neighbours of Q face the centre. then other also faces outward. Facing opposite directions means if
Both the immediate neighbours of M face opposite directions (i.e., if one person faces the centre then the other person faces outward and
one person faces the centre then the other faces outside and vice- vice versa.
versa). Both the immediate neighbours of S face same direction as 0 Bhavana is not an immediate neighbor of Aman or Chandan. The one
(i.e., if 0 faces the centre then both the immediate neighbours of S who sits exactly between Aman and Chandan sits third to Ela and
also faces the centre and vice-versa) immediate neighbours of P face a faces outward. The immediate neighbours of Farukh face the same
direction opposite to that of P (i.e., if P faces the centre then both his direction.
immediate neighbours face outside and vice-versa). The immediate neighbours of Bhavana face opposite directions.
Chandan sits second to the left of Aman. Gita sits second to the right
3.) What is R's position with respect to O? of Bhavana. Deepa is an immediate neighbour of Gita and faces the
1) Third to the right centre. Deepa or Harsh is not an immediate neighbor of Aman. Aman
2) Fifth to the left and Bhavana face opposite directions. Harsh sits opposite Gita and
3) Second to the left the both are faces opposite directions.
4) Third to the left
5) Fourth to the right Q6. Who among the following are immediate neighbours of Farukh?
(a) Ela and Harsh (b) Gita and Aman (c) Chandan and Harsh
4.) Four of the following are a like in a certain way based on the given (d) Chandan and Aman (e) None of the above
seating arrangement and so form a group. Which is the one that does
not belong to that group? Q7. Who among the following sits on the immediate left of Harsh?
1) N (a) Chandan (b) Ela (c) Deepa (d) Bhavana (e) None of the above
2) O
3) M Q8. Four of the following five are alike in a certain way and hence
4) P form a group. Which is the one that does not belong to that group?
5) L (a) Ela and Gita (b) Bhavana and Chandan
(c) Chandan and Farukh (d) Aman and Ela
5.) Which of the following is true regarding N as per the given seating (e) Harsh and Gita
arrangement?
1) N sits to immediate right of M Q9. Who among the following sits third to the right of Deepa?
2) N faces outside (a) Chandan (b) Harsh (c) Farukh (d) Ela (e) None of the above
3) Only two people sit between N and Q
4) L sits second to right of N Q10. What is the position of Farukh with respect to Harsh?
5) S is one of the immediate neighbours of N (a) Second to the right (b) Third to the left
(c) Second to the left (d) Fourth to the right
6.) Who is sitting to immediate left of P? (e) None of the above
1) R 2) M 3) O 4) S 5) Q

7.) How many people in the given arrangement face outside?


1) One 2) Three 3) Five 4) Four 5) Two

03. 5
04. 3
05. 5
06. 2
07. 4

1 D 2 B 3A 4B 5A
200 Circular Arrangements ques
SET 19 SET 20
Directions (7 – 11): Study the following information carefully and Directions (Q.1-5): Study the following information carefully and
answer the questions given below: answer the given questions: Cetking students Dwayne, Elaya, Francois,
Houston and Indra are seated in a circle facing the centre. Alastair,
Eight Cetking students Prince, Queen, Ricky, Smith, Tim, Unholy, Ben and Chris are also seated in the same circle but two of them are
Vivian and Watson are sitting around a circular table. Some are not not facing the centre. Francois is second to the left of Chris. Elaya is
facing the centre while some are facing the centre. third to the right of Alastair. Ben is third to the left of Dwayne, who is
Vivian sits third to the right of Queen, Tim sits third to the right of immediate neighbour of Houston and Indra. Chris is second the right
Smith. The immediate neighbors of Prince are facing the centre. of Dwayne and third to the right of Ben.
Watson sits second to the left of Smith and Prince sits second to the
left of Unholy, who is facing the centre. Prince is an immediate 1. Which of the following pairs is not facing the centre?
neighbor of neither Queen nor Vivian. Unholy is not an immediate 1. Ben, Chris
neighbor of Vivian. Tim and the one who sits opposite Tim are not 2. Alastair, Chris
facing the centre. The one who is on the immediate left of Tim is not 3. Dwayne, Elaya
facing the centre while the immediate neighbor of Watson is facing 4. Ben, Dwayne
outward. 5. None of the above

7. How many persons are not facing the centre? 2. What is the position of Dwayne with respect of Francois?
(a) Two (b) Three (c) Five (d) Four (e) None of these 1. Second to the right
2. Third to the left
8. Who among the following sits on the immediate left of Tim? 3. Fifth to the right
(a) Ricky (b) Queen (c) Vivian (d) Prince (e) None of these 4. Can’t be determined
5. None of the above
9. Who among the following are immediate neighbors of Queen?
(a) Unholy, Smith (b) Unholy, Tim (c) Tim, Ricky 3. Who among the following is second to the left of Ben?
(d) Watson, Vivian (e) None of these 1. Alastair
2. Houston
10. Who among the following sits second to the left of Vivian? 3. Indra
(a) Tim (b) Watson (c) Prince (d) Unholy (e) None of these 4. Either Houston or Indra
5. None of the above
11. Four of the following five are alike in a certain way and hence form
a group. Which is the one that does not belong to that group? 4. If Houston is on the immediate right of Elaya then what is
(a) Vivian, Ricky Houston’s position with respect of Chris?
(b) Tim, Prince 1. Third to the left
(c) Queen, Unholy 2. Third to the right
(d) Smith, Prince 3. Immediate right
(e) Ricky, Prince 4. Can’t be determined
5. None of the above

5. Which of the following is/are the possible positions(s) of Indra with


respect to Alastair?
1. Fourth to the right
2. Second to the left
3. Fourth to the left
4. Only (a) and (c)
5. All the above

Solution: 1A 2E 3D 4B 5E

D
A
B
C
D
200 Circular Arrangements ques
SET 21 SET 22
Directions (Q.6-10): Study the following information and answer the Directions (Q.6 – 10): Study the following information carefully and
questions that follow: Eight Cetking students Alisha, Bisht, Charan, answer the questions given below.
Dravid, Esha, Farah, Gautam and Hitesh in a school sports competition
participated in a game in which they were sitting around a circular Eight Cetking students Suchit, Sameer, Shivam, Shubham, Sneha,
ring. The seats of the ring are not directed towards the centre. All the Swati, Somya and Shruti are sitting around a circular table. Three
eight students are in four groups I, II, III and IV, i.e. two students in friends facing outside the centre while, five friends are facing towards
each group, but not necessarily in the same order. These students are the centre. Sameer sits third to the right of Swati. Shubham is not an
from different sport houses, viz Maharishi, Vyas, Aryabhatt, Vashistha, immediate neighbour of Shruti. Shivam sits third to the left of
Shankaracharya, Balmiki, Dhruv and Dayanand. • No two students of Shubham, who is second to the right of Suchit. Two friends are sitting
the same group are sitting adjacent to each other except those of between Shruti and Sneha and two are sitting between Swati and
group III. Students from group IV are sitting opposite each other. • Sneha. Shivam is second to the left of Somya, who is facing the centre.
Dravid is neither in Dayanand nor in Aryabhatt house. • The student Sameer is facing the centre. Sneha is not opposite Swati or Shivam.
from Dhruv house is sitting on the immediate right of the student Shivam faces the centre whereas Shruti faces outside the centre.
from Dayanand house. • Charan, who is in Vashistha house, is a group
I. She is sitting on the immediate right of Farah, who is in group III. • 6. Who sits fifth to the left of Shruti?
Farah is not in Aryabhatt house and she has also participated in other (a) Shivam
sports. • Bisht from Vyas house is neither in group IV nor in group I or (b) Swati
II. • Bisht is sitting opposite Esha. Only Balmiki participant Alisha is (c) Sneha
sitting between Dhruv participant Esha and the Shankaracharya (d) Suchit
participant. • Both the students of group II are sitting adjacent to (e) None of these
students of group IV.
7. Which of the following statements is not true?
6. Who is in Maharishi house? (a) Shruti is sitting third to the right of Shubham.
1 Gautam 2 Farah 3 Hitesh 4 can’t say 5 None of the above (b) Sneha is sitting opposite Suchit.
(c) Swati is sitting third to the left of Sneha.
7. ‘Hitesh’ is in which house? (d) Somya is sitting between Shubham and Sneha.
1 Dayanand 2 Shankaracharya 3 Maharishi 4 Can’t say (e) All are true
5 None of the above
8. Who is sitting on the immediate right of Suchit?
8. Who among the following students are in group IV? (a) Sameer
1 Alisha and Bisht 2 Alisha and Charan (b) Somya
3 Gautam and Hitesh 4 Data inadequate (c) Sneha
5 None of the above (d) Shruti
(e) None of these
9. The student from which house is sitting opposite the Vashistha
house participant? 9. Who is sitting opposite Somya?
1 Dayanand 2 Shankaracharya 3 Dhruv 4 Can’t say (a) Sameer
5 None of the above (b) Sneha
(c) Shruti
10. Which of the following statements is false? (d) Shivam
1 The Balmiki representative is sitting opposite the Maharishi house (e) Shubham
representative
2 The Vyas house representative is sitting opposite the Dhruv house 10. Which group of three friends is facing outside the circle?
representative (a) Suchit, Shruti and Somya
3 Hitesh and Gautam are sitting opposite each other (b) Sameer, Sneha and Shruti
4 The two girl-participants are sitting opposite each other (c) Suchit, Sneha and Swati
5 None of the above (d)Sneha, Suchit and Shruti
(e) None of these
Solution 1B 2D 3C 4B 5D

6-e
7-e
8-e
9-c
10-d
200 Circular Arrangements ques
SET 23 (b) JN
Directions (1-6): Study the following information carefully and answer (c) RN
the questions given below: (d) QK
(e) MS
Ten Cetking students J, K, L, M, N, O, P, Q, R and S are sitting at a
rectangular table in such a way that four of them sit at the corners, Q6. How many persons are not facing the centre?
two each on the longer sides and one each on the smaller sides, but (a) Three
not necessarily in the same order. Some of them are facing the centre (b) Four
while the rest are not facing the centre. Not more than two friends (c) Five
sitting together face the same direction. (d) Six
(e) None of the above
J sits on one of the smaller sides and third to the right of O.
R is not an immediate neighbor of J, K or O, but sits on the immediate
right of L, who is not facing the centre.
Only two among four sitting on the corners face outward.
Q and M are sitting on the immediate left and third to the left of S
respectively.
One of the four friends sitting on the corner is R.
N sits on the immediate left of M and is not an immediate neighbor of
L. J and N face the same direction.
M and P sit diagonally and face opposite directions.
Five of them face the same direction.
Note: Facing the same direction means if one person faces the centre
then the other person also faces the centre and if one person faces
outward then the other person also faces outward. Facing the
opposite direction means if one person faces the centre then the S1. Ans.(b)
other person faces outward and vice versa. S2. Ans.(e)
S3. Ans.(c)
Q1. Which of the following groups sit on the corners of the S4. Ans.(b)
rectangular table? S5. Ans.(d)
(a) MRQS S6. Ans.(c)
(b) SMRP
(c) QMPR SET 24
(d) QMPL Directions (6-10): Study the following information carefully and
(e) None of the above answer the questions given below:

Q2. Who among the following sits third to the right of N? Cetking students S, T, U, V, W, X, Y and Z are sitting around a circular
(a) P table, but not necessarily in the same order. Some of them are facing
(b) Q outward.
(c) J Note: Facing the same direction means that if one is facing the centre
(d) S then the other person is also facing the centre and if one is facing
(e) None of the above outward then other person is also facing outward.
Facing opposite directions means that if one is facing the centre then
Q3. What is the position of M with respect to J? the other person is facing outward and vice versa.
(a) Immediate right W and Z face opposite directions. One of the immediate neighbours of
(b) Second to the right W faces the centre. Z sits second to the right of T, who faces the
(c) Fourth to the right centre. X sits second to the right of W, who sits third to the left of Z. U
(d) Can’t be determined sits on the immediate left of X. V sits second to the left of Y, who faces
(e) None of the above the same direction as W. Y is not an immediate neighbour of T. The
immediate neighbours of Z face opposite directions. S is not facing the
Q4. Who among the following sits opposite Q? centre.
(a) R
(b) K Q6. How many persons in the given arrangement face outward?
(c) M (a) Two
(d) N (b) Three
(e) None of the above (c) Four
(d) Five
Q5. Four of the following five are alike in a certain way and hence (e) None of the above
form a group. Which is the one that does not belong to that group?
(a) OL Q7. Who among the following is second to the right of S?
200 Circular Arrangements ques
(a) U Q8. Who among the following is an immediate neighbour of D and F?
(b) V (a) G
(c) Z (b) C
(d) W (c) B
(e) None of the above (d) A
(e) None of the above
Q8. What is the position of V with respect to U?
(a) Second to the left Q9. Who among the following sits third to the left of E?
(b) Third to the left (a) A
(c) Immediate right (b) H
(d) Third to the right (c) F
(e) None of the above (d) B
(e) None of the above
Q9. Who among the following is fifth to the right of V?
(a) S Q10. Four of the following five are like in a certain way and hence
(b) V form a group. Which is the one than does not belong to that group?
(c) W (a) F, B
(d) Can’t be determined (b) D, E
(e) None of the above (c) G, H
(d) D, G
Q10. Which of the following statements is/are true? (e) None of the above
(a) V sits on the immediate left of T.
(b) Y sits third to the right of U.
(c) X sits opposite Y.
(d) All are true
(e) None of the above

SET 25
Directions (6-10): Study the following information carefully and
answer the questions given below:
Cetking students A, B, C, D, E, F, G and H are sitting around a circular
table. Three of them are not facing the centre. H is not an immediate
neighbour of F or G. G is third to the right of F and one of them is not
facing the centre. D is not an immediate neighbor of H or C. A sits
second to the left of H, who is not facing the centre. B sits third to the
left of C, and one of them is facing the centre but both of them are
not immediate neighbours of G. F sits second to the left of C, who is
facing the same direction as G.

Q6. Which of the following groups is not facing the centre?


(a) F, H, D
(b) C, H, G
(c) B, C, H
(d) A, F, H
(e) None of the above

Q7. Who among the following sits third to the right of C?


(a) G
(b) B
(c) E
(d) F
(e) None of the above
200 Circular Arrangements ques
Advance Sets
SET 1
Directions (Q.1-7): Cetking students Johnny, Pacino, Denzel, Russel,
Jolie, Cruise, Robert and Charlie are sitting around a square table in
such a way that four of them sit at four corners of the square while
four sit in the middle of each of the four sides. The one who sit at the
four corners face the centre of the table while those who sit in the
middle of the sides face outside. Each one of them likes a different
subject viz. Mathematics, Hindi, English, Biology, Chemistry, Physics,
History and Geography. (None of the information given is necessary in
the same order).

Denzel sits third to left of the one who likes Geography. The one who Answer: (a)
likes Geography faces outside. Only two people sit between Denzel Answer: (c)
and Charlie. The one who likes Mathematics sits to immediate right of Answer: (e)
Charlie. The one who likes Chemistry sits second to the right of Answer: (e)
Robert. Robert is neither an immediate neighbour of Charlie nor Answer: (e)
Denzel. Robert does not like Geography, only one person sits between Answer: (a)
Johnny and the one who likes Chemistry. Russel sits to immediate left Answer: (b)
of the one who likes Physics. Robert does not like Physics. Jolie likes
History. Jolie is not an immediate neighbour of Johnny. The one who SET 2
likes Hindi is an immediate neighbour of Jolie. The one who likes Directions: Study the following information carefully and answer the
Biology is an immediate neighbour of Cruise. questions given below:
Eight Cetking students S, R, N, L, M, T, O and P are sitting in a circle
Q1. Who amongst the following sits diagonally opposite the one who facing the centre. All eight belong to different professions ___
likes Mathematics? reporter, doctor, cricketer, teacher, accountant, shopkeeper, painter
(a) The one who likes Hindi (b) Russel (c) Johnny and supervisor. They are not necessarily seated in the mentioned
(d) The one who likes English (e) The one who likes Biology order.
M is sitting third to the left of O. The doctor is to the immediate right
Q2. Who amongst the following represents the immediate neighbour of M and M is not a reporter. R is sitting fourth to the right of P.
of the one who likes Chemistry? Neither R nor P is an immediate neighbour of M. T is a teacher and is
(a) Pacino, Cruise (b) Denzel, Jolie (c) Pacino, Jolie sitting third to the right of the doctor. The shopkeeper is sitting
(d) Russel, Cruise (e) Cruise, Charlie second to the left of the teacher. The painter is sitting second to the
left of M. S the cricketer is sitting exactly between T and P. The
Q3. Who amongst the following sits exactly between Charie and accountant is sitting second to the right of the cricketer. N is sitting
Pacino? third to the left of T.
(a) Denzel (b) The one who likes Hindi (c) The one who likes English
(d) Robert (e) Johnny 6. Who amongst the following is a reporter?
1)O 2)L 3)N 4)R 5)None of these
Q4. Which of the following is true regarding Pacino?
(a) Pacino is one of the immediate neighbours of Russel 7. What is S’s position with respect to R?
(b) The one who likes Geography is an immediate neighbour of Pacino 1)Third to the right 2)Second to the right
(c) Pacino sits second to left of Charlie 3)Third to the left 4)second to the left
(d) Pacino likes History 5)Fourth to the right
(e) Pacino is an immediate neighbour of the one who likes
Mathematics 8. How many people are sitting between P and N when counted in an
anti-clockwise direction from N?
Q5. What is the position of the one who likes Physics with respected 1)One 2)Two 3)Three 4)Four 5)None
to Robert?
(a) Second to the left (b) Third to the right (c) Fourth to the left 9. Four of the following five pairs alike in a certain way based on their
(d) Second to the right (e) Third to the left positions in the above arrangement and so form a group. Which of the
following does not belong to the group?
Q6. Which of the following subjects does Russel likes? 1)Teacher – Painter 2)Supervisor - Shopkeeper
(a) Biology (b) Mathematics (c) Hindi (d) Chemistry (e) English 3)Cricketer – Reporter 4)Doctor - Accountant
5)Shopkeeper – Doctor
Q7. Who amongst the following likes Geography?
(a) Pacino (b) Cruise (c) Charlie (d) Johnny (e) Russel 10. Which one of the following statements is false according to the
above mentioned arrangement?
1)N is to the immediate right of the supervisor
2)The cricketer is third to the right of the shopkeeper
200 Circular Arrangements ques
3)The doctor is sitting exactly between the supervisor and the (c) Cloth (d) Book
accountant (e) None of the above
4)L is neither a teacher nor a supervisor
5)There are only three people between S and N Q5. If D and F interchange their places then who among the following
is on the immediate left of G?
Answers (a) B (b) D (c) H (d) F (e) None of the above
6. 1
7. 1
8. 4
9. 5
10.3

SET 3 1. D 2. B 3. A 4. C 5. D
Directions (1-5): Study the following information carefully and answer
the questions given below: SET 4
Direction : Study the following information and answer the questions
Eight Cetking students A, B, C, D, E, F, G and H are sitting around a given below :
circular table but not necessarily in the same order. Some of them are
facing outward. They are working at four stalls, viz Cloth, Food, Four Maruti Suzuki's cars - Ertigo,Celerio, Swift and Wagon'r and four
Footwear and Book, in a trade fair. Two persons are working at each Hyundai's cars - I-10, Verna, I-20 and Santa Fe owned by Cetking
stall. students are on a display in a circular parking. two Maruti Suzuki's
Note: Facing the same direction means if one person faces the centre cars and two Hyundai's cars are not facing the centre of the auto expo
then the other person also faces the centre and if one person faces centre. Swift is placed second to the left of Verna, which is not placed
outward then the other person also faces outward. immediate next to Celerio. I-10 is third to the left of Santa Fe and one
Facing opposite directions means if one person faces the centre then of them is not facing centre of the auto expo centre. Ertigo is third to
the other person faces outward and vice versa. the right of Celerio, which is facing the centre of the auto expo centre.
I-20 and Wagon'r are facing each other and both are not placed
• C sits third to the left of H, who works at the Footwear stall, and immediate next to Celerio or Santa Fe. No three Hyundai's car's are
both are facing the same direction. placed together.
• G sits on the immediate right of B, who works at the Cloth stall.
• C and B are not facing the same direction but C is an immediate 1. Which of the following Car is placed between Celerio and Wagon'r ?
neighbor of E, who is fourth to the left of G 1) Swift 2) Verna 3) Santa Fe 4) I - 10 5) None of these
• E and G both are facing opposite directions but both work at the
same stall. 2. How many cars are placed between I-10 and Verna ?
• Those who work at the Cloth stall sit adjacent to each other but face 1) Three 2) Two 3) One 4) Can't be determined 5) None of these
opposite directions.
• The person, who work at the Food stall sit opposite each other. 3. Which Car is displayed third to the left of Ertigo ?
• The immediate neighbour of E are not facing outward. 1) Celerio 2) Wagon'r 3) I-20 4) I-10 5) None of these
• a person who works at the Footwear stall is an immediate neighbour
of the both persons who work at the Book stall. 4. Which of the following groups of four Cars are not facing the
• D and F are immediate neighbour of H. centre?
• D is not facing the centre and works at the Book stall. 1) Swift, Wagon'r, Ertigo and Verna 2) Verna, I-10, Ertigo and Wagon'r
• The one who is on the immediate left of F is not facing the centre. F 3) Ertigo, Verna. I-10 and Swift 4) I-20, I-10, Santa Fe and Celerio
sits second to the right of C. 5) None of these

Q1. Who among the following work at the Footwear stall? 5. Who is placed between I-20 and Santa Fe ?
(a) D and F (b) H and F (c) G and C (d) C and H 1) I-10 2) Ertigo 3) Swift 4) Verna 5) None of these
(e) None of the above

Q2. Who among the following sits on the immediate right of the
person who works at the Food stall?
(a) B (b) D (c) A (d) F (e) H

Q3. How many persons are facing outward?


(a) Two (b) Three (c) Four (d) Can’t be determined
(e) None of the above

Q4. A works at which of the following stalls?


(a) Either Cloth or Footwear (b) Either Book or Food
200 Circular Arrangements ques
SET 5
Directions (1-6): Study the following information carefully and answer
the questions given below.

Eight Cetking students J, K, L, M, N, O, P and Q are sitting around a


circular table. Each of them has a laptop of a different company, viz
Dell, Lenovo, HP, Sony, Acer, Apple, Asus and Toshiba but not
necessarily in the same order. Four of them are not facing the centre.
The one who has Acer is on the immediate left of M, who does not
have Asus. M is third to the right of Q. P is fourth to the left of O.
Neither P nor O is an immediate neighbor of M. L has HP and sits
exactly between J and O. L is facing the centre and is to the right of
both J and O. N is third to the left J, who has Toshiba.
J sits third to the left of the one who has Acer. The one who has
1. 2 Lenovo sits second to the left of the one who has Toshiba. The one
2. 5 who has Dell sits second to the right of M. The one who has Apple sits
3. 2 second to the right of the person who has HP. K does not like Acer. J
4. 2 faces the centre.
5. 1
Q1. Who among the following has the laptop of Sony?
(a) M (b) O (c) Q (d) L (e) None of the above

Q2. Who among the following sits third to the right of the one who
has the laptop of Asus?
(a) N (b) M (c) O (d) L (e) None of the above

Q3. Who among the following sits third to the left of Q?


(a) L (b) K (c) O (d) M (e) None of the above

Q4. Four of the following five are alike in a certain way and so form a
group. Which is the one that does not belong to that group?
(a) OK (b) LN (c) JM (d) MN (e) LQ

Q5. Who among the following sits opposite N?


(a) The one who has Toshiba (b) The one who has Apple
(c) The one who has HP (d) The one who has Sony
(e) None of the above

Q6. Which of the following statements is/are true?


(a) The one who has Asus is not facing the centre.
(b) O has Dell and he is facing the centre.
(c) The one who has Apple is on the immediate right of M.
(d) Only (a) and (c) are true
(e) None of the above
200 Circular Arrangements ques
SET 6 6.Which of the following subjects does D like?
Directions (Q. 1 – 7): Study the following information carefully and 1) Biology
answer the given questions. 2) Mathematics
A, B, C, D, E, F, G and H are sitting around a square table in such a way 3) Hindi
that four of them sit at four corners of the square while four sit in the 4) Chemistry
middle of each of the foul sides. The ones who sit at four corners face 5) English
the centre of the table while those who sit in the middle of the sides
outside. Each of them likes a different subject, viz Hindi, Mathematics, 7. Who among the following likes Geography?
English, Biology, Physics, Chemistry, History and Geography. (Not 1) B
necessarily in the same order.) 2) F
3) H
C sits third to the left of the one who likes Geography. The one who 4) A
likes Geography faces outside. Only two persons sit between C and H. 5) D
The one who likes Mathematics sits on the immediate right of H. The
one who likes Chemistry sits second to the right of G. G is neither an Hints: For Ques(1-7)
immediate neighbour of H nor of C. G does not like Geography. Only
one person sits between A and the one who likes Chemistry. D sits on 1. 1
the immediate left of the one who likes Physics. G does not like 2. 3
Physics. E likes History. E is not an immediate neighbour of A. The one 3. 5
who likes Hindi is an immediate neighbour of E. The one who likes 4. 5
Biology is an immediate neighbour of F. 5. 2
6. 1
1. Who amongst the following sits diagonally opposite the one who 7. 2
likes Mathematics?
1) The one who likes Hindi
2) D
3) A
4) The one who likes English
5) The one who likes Biology

2. Who among the following represent the immediate neighbours of


the one who likes Chemistry?
1) B, F
2) C, F
3) B, E
4) D, F
5) F, H

3. Who among the following sits exactly between H and B?


1) C
2) The one who likes Hindi
3) The one who likes English
4) G
5) A

4. Which of the following is true regarding B?


1) B is one of the immediate neighbours of D.
2) The one who likes Geography is an immediate neighbour of B.
3) B sits second to the left of H.
4) B likes History
5) B is an immediate neighbour of the one who likes Mathematics.

5. What is the position of the one who likes Physics with respect to
G?
1) Second to the left
2) Third to the right
3) Fourth to the left
4) Second to the right
5) Third to the left
200 Circular Arrangements ques
SET 7 E
Eight family members of a Cetking student S, T, U, V, W, X, Y and Z are
sitting around a circular table but not necessarily in the same order.
Some of them are females and some are males. All of them are
related to each other in the same way ot the other. Some of them are
facing the centre while some are facing outside (i.e opposite to the
centre). Only two people sit between T and W. T faces the centre. X
sits second to the right of T. W is the wife of S. No females is an
immediate neighbour of W. U is not an immediate neighbour of T. U is
the daughter of W. Both the immediate neighbours of U face the
centre. Only three people sit between S and U’s brother. X is not the
brther of U. Neither S nor U’s brother is an immediate neighbour of X.
Z, the wife of T, sits to the immediate left of V. Both Y and S face a
direction opposite to that of U (i.e if U faces the centre then both Y
and S face outside and vice-versa). U’s husband sits second to the left
of Y. T’s father sits to the immediate right of W. T sits second to the
right of S’s father. Both the immediate neighbours of X are females.
1. How many people sit between T and S’s father when counted from
the right of T?
a) Four
b) Three
c) None
d) One
e) Two

2. Who amongst the following sits exactly between Y and W when


counted from the left of Y?
a) T
b) X
c) S
d) z
e) U

3. Which of the following statements regarding T is definitely true?


a) X and Z are immediate neighbours of T.
b) T sits second to the left of X.
c) T is the son of S.
d) None of the given options is correct.
e) V is the father of T.

4. Who amongst the following faces outside (i.e opposite to the


centre)?
a) W
b) V
c) U
d) Z
e) T

5. If it is given that Y is married to X, then what is the position of T


with respect to Y’s daughter-in-law?
a) Third to the right
b) Second to the right
c) Immediate right
d) Second to the left
e) Third to the left

D
D
C
C
200 Circular Arrangements ques
SET 8
Directions (6-11): Study the following information carefully and
answer the questions given below:

A, B, C, D, E, F and G are seven family members of a Cetking student


are sitting around a circular table facing the centre. There are two
married couples, two sons and two daughters in the family. D sits
third to the right of her grandson. G is son of C and father of B. F sits
second to the right of her mother but is not an immediate neighbor of
her father. E sits third to the left of her daughter but is not an
immediate neighbor of her husband. D is grandmother of A. A is not
granddaughter of C, who is an immediate neighbor of his wife.

Q6. How many females are there in the family?


(a) Two
(b) Three
(c) Four
(d) Can’t be determined
(e) None of the above

Q7. Who among the following are parents of A?


(a) F, D
(b) G, E C
(c) C, D B
(d) E, F A
(e) None of the above C
D
Q8. How is F related of E? D
(a) Sister-in-law
(b) Son
(c) Daughter
(d) Sister
(e) None of the above

Q9. Who among the following is third to the right of D?


(a) F
(b) A
(c) G
(d) E
(e) None of the above

Q10. Who among the following are immediate neighbours of F?


(a) G, D
(b) A, B
(c) D, C
(d) B, G
(e) None of these

Q11. Which of the following statements is/are true?


(a) G is mother of A
(b) F is granddaughter of D
(c) B is sister of F
(d) F is aunt of A
(e) None of the above
200 Circular Arrangements ques
SET 9
Directions: Study the following information carefully and answer the
questions given below:

Eight Cetking students Q, R, S, T, V, W, Y and Z are sitting around a


circular table, facing the centre. There are three males and five
females in the group of friends. No two males are immediate neigh
bours of each other.
•V sits second to the right of his wife.
•S sits third to the right of V.
•W sits second to the right of her husband Z. Z is not an immediar
neighbour of V's wife.
•T is a male and Y is not an immediate neighbour of V.
•R sits second to the right of Q.

1.What is the position of T with respect to Z?


1) Second to the left 2) Immediately to the right
3) Third to the left 4) Second to the right
5) Third to the right

2.Which of the following statements regarding S is definitely correct?


1) S is one of the male members of the group.
2) Both the immediate neighbours of S are females.
3) S sits third to the left of T.
4) W is an immediate neighbour of S.
5) S sits second to the right of Q.

3.Who amongst the following is V's wife?


1) Q 2) Y 3) R 4) T 5) None of these

4.Who amongst the following has a male sitting to the immediate left
and the right?
1) Y 2) R 3) Q 4) S 5) None of these

5.Which of the following is not true regarding T?


1) T is an immediate neighbour of Z's wife.
2) No male is an immediate neighbour of T.
3) Q sits second to right of T.
4) The one who sits third to the left of T is a male.
5) All are true

6.Which of the following pairs represents the immediate neighbours


of T
1) RQ 2) WZ 3) YV 4) WY 5) None of these

7.How many people sit between V and S when counted in anti-


clockwise direction?
1) None 2) One 3) Two 4) Three 5) Four

8.Who amongst the following sits exactly between V and Y?


1) Q 2) W 3) R 4) T 5) Z

9.Who amongst the following is Z's wife?


1) S 2) W 3) Y 4) T 5) None of these

10.Who Sit's Opposite to V?


1) Wife of Z 2) Wife of R 3) Wife of T 4) Wife of Q 5) None of these

You might also like